You are on page 1of 343

Board Exam Question (Smiley + Star) + 2007

FAMILY MEDICINE

FAMILY MEDICINE
(75) MULTIPLE CHOICE QUESTIONS / TYPE I Select the correct answers to the following questions!!! ...each qestion may have more than one correct answer. FM-1. Renal calcification is a possible complication of: A) medullary cystic kidney disease B) renal tuberculosis C) sarcoidosis D) sickle cell anemia E) secondary hyperparathyroidism FM-4. Factors causing a susceptibility to urinary tract infect include: A) urinary tract obstruction B) diabetes mellitus C) hyperkalemia D) prolonged tetracycline therapy E) pregnancy FM-5. The medical history of a 45-year-old male reveals episodes of vertigo and loss of consciousness associated with sweating. Possible causes of his symptoms include: A) hyperventilation B) hyperglycemia C) Zollinger-Ellison syndrome D) pheochromocytoma E) paroxysmal tachycardia FM-6. Possible causes of hematemesis include: A) salicylate administration B) an oral iron supplement overdose C) severe burn injury D) Menetrier's disease (giant hypertrophic gastritis) E) feeding via a nasogastric tube FM-7. The use of which of the following should be avoided in patients receiving monoamino-oxidase inhibitor therapy: A) cheese B) imipramine (Melipramin) C) phentolamine (Regitin) D) pethidine (Dolargan) DrRognerud@www.medfever.com

Answer: ABC

Answer: ABE

Answer: ADE

Answer: ABCD

Answer: ABD

1 / 343

Board Exam Question (Smiley + Star) + 2007 E) sulphonamides FM-8. Which of the following conditions are usually associated with purpura? A) Henoch-Schonlein syndrome B) hepatic cirrhosis C) systemic lupus erythematosus D) Addison's disease E) Raynaud's phenomenon FM-9. Bone density is markedly increased in: A) osteopetrosis B) Paget's disease of the bone C) following the intake of a large amount of fluoride D) hyperparathyroidism E) renal osteodystrophy FM-10. Which of the following conditions is associated with a male-type distribution of hair in females? A) myxedema B) true hermaphroditism C) Laurence-Moon-Biedl syndrome D) Stein-Leventhal.syndrome (polycystic ovary) E) Cushing's syndrome FM-11. In which of the following conditions can central cyanosis be detected? A) methemoglobinemia B) ventilation-perfusion mismatch C) pulmonary arteriovenous fistula D) heatstroke E) heavy physical exercise FM-12. Which of-the following statements about delirium tremens are correct? A) chlormethiazole (Heminevrin) is suitable for treatment B) acoustic hallucinations are common C) visual hallucinations are common D) elecroshock therapy is indicated in severe cases E) the condition may be fatal FM-13. In which of the following conditions can renal failure be fatal? A) hypernephroma B) systemic lupus erythematosus (SLE) C) Weil's disease D) renal tuberculosis E) accelerated hypertension DrRognerud@www.medfever.com

FAMILY MEDICINE

Answer: ABC

Answer: ABC

Answer: DE

Answer: ABC

Answer: ABCE

Answer: BCE

2 / 343

Board Exam Question (Smiley + Star) + 2007 FM-14. Penicillin administration is the appropriate therapy in which of the following complications of syphilis? A) meningitis B) aneurysm of the aorta C) interstitial keratitis D) condyloma latum E) the generalized paralysis of psychotic patients FM-19. Bilateral parotid gland enlargement is a symptom of A) Mikulicz's syndrome B) infectious mononucleosis C) mumps D) brucellosis E) sarcoidosis FM-20. Which of the following statements about Conn's syndrome are correct? A) a high aldosterone level in the serum is a characteristic finding B) the plasma renin activity is elevated C) the associated hypertension is malignant in 10% of cases D) hyperkalemia is a common complication E) operative therapy is usually recommended FM-21. Recognized causes of nodular hepatomegaly include: A) postnecrotic cirrhosis B) primary biliary cirrhosis C) syphilis of the liver D) Weil's disease (Leptospira icterohemorrhagica) E) carcinomatous metastases of the liver FM-22. Which of the following symptoms are associated with Turner's syndrome? A) infantilism B) congenital abnormalities of the external genitalia C) an atrial septal defect D) retinitis pigmentosa E) a short stature FM-23. Symptoms characteristic of an acute exacerbation of ulcerative colitis include: A) the development of anemia B) the occurrence of vertigo following sulfasalazine therapy C) macroscopically detected blood in the feces D) the development of generalized eruptions E) an increased erythrocyte sedimentation rate DrRognerud@www.medfever.com

FAMILY MEDICINE

Answer: AD

Answer: ACE

Answer: AE

Answer: ACE

Answer: ABE

Answer: ACE

3 / 343

Board Exam Question (Smiley + Star) + 2007 FM-25. Which of the following statements about congenital hypertrophic pyloric stenosis are correct? A) it is more frequent in females B) there is an increased likelihood for any offspring to be similarly afflicted C) any vomit almost never contains bile D) the pyloric ring is rarely palpable E) the majority of patients require surgical therapy FM-27. Which of the following conditions is associated with true hematuria? A) urinary tract tuberculosis B) acute pyelonephritis C) acute cystitis D) malignant hypertension E) renal infarction FM-30. Primary optic nerve atrophy is a recognized complication of: A) glaucoma B) disseminated sclerosis C) Paget's disease of the skull D) neurosyphilis E) ethambutol (Sural) therapy FM-55. Which of the following statements about gastric secretion are correct? A) approximately 250 ml gastric juice is secreted daily B) gelatinase is a normal constituent of gastric juice C) the stomach is capable of producing both acidic or alkaline secretions D) if the rate of production is slow, the Na+ concentration is high E) only the glands of the pyloric region are capable of secreting mucus FM-57. Transient amnesia is possible: A) following head trauma B) in patients suffering from epilepsy C) in cerebrovascular insufficiency D) in Alzheimer's disease E) in patients with a temporal lobe tumor FM-68. Characteristics of Korsakoff.s syndrome include: A) polyneuritis which is detected in all cases B) a clear consciousness C) impaired judgement DrRognerud@www.medfever.com

FAMILY MEDICINE

Answer: C

Answer: ACE

Answer: BDE

Answer: BCD

Answer: ABCDE

Answer: BCE

4 / 343

Board Exam Question (Smiley + Star) + 2007 D) morphological abnormalities in the hypothalamus E) confabulation FM-76. An opening snap is detected in: A) mitral stenosis developing as a consequence of rheumatic carditis B) congenital mitral stenosis C) mitral insufficiency associated with a rigid posterior but a normal anterior cusp D) the presence of a myxoma in the left atrium E) severe aortic insufficiency FM-78. Possible therapeutic interventions in thyrotoxic crisis include: A) a high dose of dexamethasone B) that the patient must be kept warm C) the administration of beta-blockers D) immediate propylthiouracil treatment E) the administration of iodine FM-79. Specific indications for dialysis include: A) a serum potassium concentration of 7.4 mmol/l (7.4 mEq/1) B) a blood pH of 7.2 C) a blood urea concentration of 63 mmol/l (378 mg/ 100 ml) D) pericarditis E) strong lumbar pain FM-82. Complications of massive irradiation include: A) an increased prevalence of leukemia B) a thrombocytopenia developing within 4 days C) an increased prevalence of visceral malignancies D) leukemoid reactions in some patients. E) hemolytic anemia FM-86. Which of the following statements about rubella during pregnancy are correct? A) in the first trimester, it is usually associated with a higher risk of fetal developmental abnormalities B) it causes cardiac anomalies in the newborn C) it causes deafness in the newborn D) retinopathy is a well known complication E) the virus is easily isolated from the throat of the newborn even if there is no other evidence for the disease FM-88. Recognized causes of recurrent pneumonia include: A) chronic alcoholism DrRognerud@www.medfever.com

FAMILY MEDICINE

Answer: ABCD

Answer: CDE

Answer: ACD

Answer: ABCD

Answer: ABCDE

Answer: ABDE

5 / 343

Board Exam Question (Smiley + Star) + 2007 B) multiple myeloma C) hereditary spherocytosis D) esophageal lesions E) allergic bronchopulmonary aspergillosis FM-89. Barbiturates, if continuously administered can: A) contribute to the development of convulsions B) induce physical dependence C) cause relaxation of skeletal muscles D) cause parkinsonism E) cause ataxia FM-107. In acute pyelonephritis: A) a common symptom is shaking chills B) vomiting is a possible complaint C) the absence of any lumbar pain excludes the diagnosis D) an intravenous pyelogram is necessary for the diagnosis E) to confirm the diagnosis, a hemoculture should routinely be made FM-112. The prevalence of suicide is higher in: A) depressive psychosis B) psychopathic patients C) obsessive-compulsive neurosis D) chronic alcoholism E) epilepsy FM-119. The complications of diphtheria include: A) a paralysis of the diaphragm B) airway obstruction C) bronchopulmonary diphtheria D) glossopharyngeal neuritis E) subacute sclerozing panencephalitis FM-120. Tetanus: A) is caused by an anaerobic gram-positive rod B) bacterium is shed into the soil through the intestine of infected animals C) may cause carpopedal spasm D) infected patients frequently exhibit mental confusion at admittance E) urinary retention and constipation may develop FM-122. Characteristics of a Trichinella spiralis infection include: A) prodromal diarrhea in adults DrRognerud@www.medfever.com

FAMILY MEDICINE

Answer: BCE

Answer: AB

Answer: ABD

Answer: ABCD

Answer: ABE

Answer: ABCDE

6 / 343

Board Exam Question (Smiley + Star) + 2007 B) severe muscular pain C) periorbital edema D) subconjunctival hemorrhages E) patchy subungual hemorrhages FM-123. Myopathy is typically associated with: A) excessive alcohol ingestion B) McArdle's disease (phosphorylase deficiency) C) strychnine poisoning D) Guillain-Barre syndrome E) hypothyroidism FM-125. Bullous skin lesions are observed in which of the following conditions? A) herpetiform dermatitis B) a barbiturate overdose C) Albright's disease D) ataxia-teleangiectasia E) pemphigoid FM-128. Which of the following findings can be detected in Hodgkin's disease: A) a normochromic, normocytic anemia B) an increased erythrocyte sedimentation rate C) leukocytosis D) thrombocytosis E) eosinophilia FM-129. Trichomoniasis: A) causes perinatal intertrigo B) is sexually transmitted C) causes severe systemic symptoms D) may be completely symptomless E) the disease of the newborns is transmitted from the mother FM-130. Symptoms of a ventricular septal defect include: A) an elevated jugular vein pressure, even in the absence of cardiac failure B) a pansystolic murmur over the apex C) a systolic ejection murmur D) a Graham-Steel sound, which is a very common finding following the development of pulmonary hypertension E) a fixed, widely split second heart sound FM-131. Infective endocarditis is rarely associated with: A) combined mitral valvular disease DrRognerud@www.medfever.com

FAMILY MEDICINE

Answer: ABE

Answer: ABE

Answer: ABCDE

Answer: ABD

Answer: C

Answer: DE

7 / 343

Board Exam Question (Smiley + Star) + 2007 B) a patent ductus arteriosus C) cogenital bicuspid aortic stenosis D) an atrial septal defect E) advanced mitral stenosis FM-136. Nicotinamide deficiency may cause: A) high output cardiac failure B) dementia C) glossitis D) sensory polyneuropathy E) dermatitis FM-138. Which of the following findings would suggest a benign rather than a malignant paraproteinemia? A) Bence-Jones proteinuria B) the IgG level is higher than 2 g/ 100 ml C) there is an elevated paraprotein level D) no skeletal abnormalities can be detected E) a 2-year symptomless period FM-139. Which of the following statements about nitroglycerin are correct? A) only topical application is effective in Raynaud's disease B) it causes paroxysmal nocturnal dyspnea as a side-effect C) it relieves the pain caused by diffuse esophageal spasms D) it might relieve pain in biliary colic E) it alleviates the symptoms of bronchial asthma FM-140. Which of the following statements concerning doxorubicin (Adriamycin) therapy are correct? A) the drug is applied intramuscularly B) the drug causes myelosuppression C) the drug causes cardiomyopathy D) the drug is successful in the treatment of primary hepatocellular carcinoma E) the drug should be administered daily, for a period of 21 days FM-141. A classic type migraine is characterized by which of the following? A) it cannot be diagnosed if there are no prodromal symptoms B) it shows a gradual progression C) a homonymous hemianopsia is present D) there is edema of the papilla E) frequently occuring acoustic hallucinations FM-142. Progressive spinal muscular atrophy of infancy is associated with: DrRognerud@www.medfever.com

FAMILY MEDICINE

Answer: BCE

Answer: DE

Answer: ACD

Answer: BC

Answer: BC

Answer: ABD 8 / 343

Board Exam Question (Smiley + Star) + 2007 A) severe general syncope B) fasciculation of the tongue C) a loss of function of the spinothalamic tract D) spontaneous fibrillation revealed by electromyography E) normal tendon reflexes FM-144. In pyloric stenosis of infancy: A) there is an autosomal dominant imheritance pattern B) vomiting occurs during the first week of life C) the vomit is frequently tinged with bile D) an abdominal tumor is nearly always palpable E) if left untreated complications during adulthood commonly develop FM-149. Which of the following observations help to differentiate neurosis from psychosis? A) neurotic patients characteristically disclaim reality B) endogenous experiences cause excitation in neurosis C) real illusions can occur in neurosis D) associative function is not affected in neurosis E) the 'ego' is intact in neurotic patients FM-151. Plague: A) is transmitted by droplet infection B) usually does not cause fever C) causes painful enlargement of the lymph nodes D) causes characterictic circular erythematous skin lesions E) responds to high dose penicillin therapy FM-152. In amebic dysentery: A) symptoms might be similar to those observed in duodenal ulcer B) alternating diarrhea and constipation suggest an underlying carcinoma C) intestinal movements are associated with a characteristic sweet odor D) amebiasis of the liver is a rare complication E) metronidazole (Klion) is the therapeutic drug of first choice FM-158. Infectious mononucleosis is associated with: A) periorbital swelling B) generalized lymphadenopathy C) jaundice in the majority of patients D) petechiae on the palate E) pruritus

FAMILY MEDICINE

Answer: D

Answer: DE

Answer: AC

Answer: AE

Answer: ABD

DrRognerud@www.medfever.com

9 / 343

Board Exam Question (Smiley + Star) + 2007 FM-164. ECG abnormalities characteristic for acute rheumatic fever include: A) a short PR interval B) non-paroxysmal AV nodal tachycardia C) a third degree AV block D) a long QT interval E) tall, asymmetric T waves observed in the precordial leads FM-165. Which of the following statements relating to aortic regurgitation are correct? A) an early diastolic murmur revealed by auscultation during acute rheumatic valvulitis is usually transient B) angina pectoris is a more frequent complication than in aortic stenosis C) a progression of the condition usually results in accentuation of the murmur D) an accentuated first sound differentiates an Austin-Flint murmur from organic mitral stenosis E) an accentuated presystolic murmur may be detected without an associated mitral stenosis FM-168. Characteristics of rheumatic polymyalgia include: A) an onset which occurs in young adulthood B) painful muscles and restriction of movements are characteristic C) that it may be associated with temporal arteritis D) a red blood cell sedimentation rate which is typically normal E) characteristic abnormalities revealed by electrmyography FM-169. Bilateral pleural effusion is observed in: A) pleural mesothelioma B) sytemic lupus erythematosus C) miliary tuberculosis D) carcinomatous lymphangitis E) sarcoidosis FM-171. Classical symptoms of multiple sclerosis include: A) paresthesia B) retrobulbar neuritis C) a loss of position and vibration sensation D) diplopia E) a steady progression FM-186. In chronic lymphocytic leukemia: A) an absolute lymphocytosis is observed B) thrombocytosis is observed DrRognerud@www.medfever.com

FAMILY MEDICINE

Answer: BD

Answer: E

Answer: BC

Answer: BCD

Answer: ABCD

Answer: AE

10 / 343

Board Exam Question (Smiley + Star) + 2007 C) splenomegaly is always present D) the development of an acute blastic crisis is possible E) Coombs positivity is possible FM-187. Which of the following findings would suggest a thrombocyte defect rather than a coagulation abnormality? A) hemarthrosis B) bleeding from superficial abrasions C) immediate and strong bleeding from tooth extraction D) bleeding of the mucous membranes E) menorrhagia FM-188. In classic type hemophilia: A) the inheritance pattern is autosomal recessive B) every son in the offspring of a male patient is affected C) 50% of daughters of the carrier females become carriers themselves D) hemarthrosis is a common manifestation E) spontaneous hemorrhaging in the brain frequently develops FM-192. Symptoms of rickets of infancy include: A) muscular hypertrophy B) sweating of the skin of the head C) craniotabes D) coxa vara E) an early closing of the fontanelles FM-196. Which of the following statement about a bicuspid aortic valve is correct? A) coarctation of the aorta is sometimes associated with this anomaly B) calcification of the abnormal valve is rare C) infectious endocarditis is extremely rare D) incompetence of the valve is more frequent than stenosis of the valve E) this anomaly is frequently associated with Turner's syndrome FM-198. An accentuated first heart sound is audible in which of the following conditions? A) complete heart block B) severe mitral regurgitation C) mitral stenosis D) acute myocarditis E) pulmonary embolism FM-199. Characteristic physical symptoms of pneumothorax include: A) percussion over the affected side reveals dullness DrRognerud@www.medfever.com

FAMILY MEDICINE

Answer: BCD

Answer: CD

Answer: BCD

Answer: A

Answer: CE

Answer: BDE

11 / 343

Board Exam Question (Smiley + Star) + 2007 B) auscultation over the affected side reveals weaker respiratory sounds C) end-rspiratory crepitations are detected D) the mediastinum is shifted towards the opposite side E) that there are decreased respiratory movements on the affected side FM-200. Adequate procedures in the treatment of asthmatic crisis include: A) the inhalation of 40% oxygen if the PaC02 is elevated B) sedation with pethidine (Dolargan) if the patient is agitated C) the intravenous administration of hydrocortisone D) salbutamol inhalation E) water deprivation for the prevention of cardiac insufficiency FM-201. Findings and parameters which help to differentiate chronic bronchitis from emphysema are: A) prolonged expiration and rales B) the PaC02 C) the diffusion capacity D) the Pa02 E) eosinophilia FM-202. Predisposing factors to thromboembolic disorders are: A) carcinoma of the pancreas B) a type O blood group C) oral contraceptives D) obesity E) myocardial infarction FM-214. Urinary calcium loss is increased in: A) osteoporosis B) osteomalacia C) primary hyperparathyroidism D) secondary hyperparathyroidism E) sarcoidosis FM-216. Recognized alterations in the ECG produced by hyperkalemia include: A) prominent U waves B) the lack of P waves C) wide QRS complexes D) ventricular tachycardia E) a depression of the ST segment FM-217. Disadvantages of a preterm delivery include: A) the increased frequency of an intracranial hemorrhage in the DrRognerud@www.medfever.com

FAMILY MEDICINE

Answer: C

Answer: ABCD

Answer: ACDE

Answer: CE

Answer: BCD

Answer: ACE

12 / 343

Board Exam Question (Smiley + Star) + 2007 newborn B) inadequate sweating leading to hyperthermia C) an immature respiratory center D) an insufficient vitamin B12 level which causes anemia E) an increased susceptibility to infections FM-223. Characteristics of delirium tremens include: A) marked drowsiness B) a gradual onset C) visual hallucinations D) bradycardia E) illusions FM-224. Case Study: A 3-year-old child loses his appetite and subsequently refuses food. Possible causes include: A) an early onset of schizophrenia B) negative behavior C) daydreaming D) anorexia nervosa E) the parents have spoiled the child FM-233. Clinical symptoms of cretinism are: A) obesity B) goiter C) spastic diplegia D) deafness E) mental deficiency FM-240. In measles: A) a morbilliform erythema is observed B) a suboccipital lymphadenopathy is continuously present C) lymphopenia is a common complication D) arthritis is a possible complication E) frequent relapses are observed FM-241. Herpes simplex infection: A) is commonly associated with carcinoma of the uterus B) may cause Kaposi's varicelliform eruptions C) may cause keratoconjunctivitis D) may cause subacute sclerosing panencephalitis E) may cause acute gingivostomatitis

FAMILY MEDICINE

Answer: CE

Answer: BCE

Answer: CDE

Answer: ABD

Answer: BCE

DrRognerud@www.medfever.com

13 / 343

Board Exam Question (Smiley + Star) + 2007

PUBLIC HEALTH

PUBLIC HEALTH
(78) SINGLE CHOICE QUESTIONS Select the single best response to each of the following questions!!! PBH-2. The most frequently used indicator for the quality of life is: A) the life expectancy rate at birth B) the life expectancy rate without chronic disease and disability C) the life expectancy rate at 60 years of age D) the raw mortality rate PBH-5. Reliable indicators of a quality-life include: A) the amount of the gross domestic product B) the unemployment rate C) the raw mortality rate D) crime statistics E) leisure utilization statistics PBH-8. The term "descriptive epidemiology" means: A) the obseivationanddescription of phenomena prwailing in the population B) the research and testing of associations and correlations C) the application of experimental instruments for the description of phenomena D) the establishment and testing of the zero-hypothesis PBH-10. The term "perinatal mortality" means: A) the number of fatalities among live-born neonates during the first 6 days of life B) the number of fatalities among live-born neonates during the first 27 days of life C) the number of stillbirths plus the number of fatalities among live-born neonates during the first 6 days of life D) the number of stillbirths PBH-13. The leading diseases causing disability are: A) psychiatric conditions B) malignancies C) musculoskeletal diseases D) cardiovascular diseases PBH-15. All of the following are considered as deviant behavior, EXCEPT. DrRognerud@www.medfever.com

Answer: B

Answer: C

Answer: A

Answer: C

Answer: D

Answer: D 14 / 343

Board Exam Question (Smiley + Star) + 2007 A) alcoholism B) suicide C) drug abuse D) accidents PBH-18. All of the following are characteristic features of drug dependence, EXCEPT: A) anxiety and nervous behavior B) excessive smoking C) weight gain D) poor personal hygiene PBH-19. Which of the following is used as health indicators of a population? A) demographic parameters B) somatometric parameters C) epidemiologic parameters D) all of the above E) none of the above PBH-21. A pyramid shaped age distribution tree is characteristic of countries with: A) an increasing population B) a decreasing population C) a stagnant population PBH-22. Where is the World Health Organization's principle headquarters? A) London B) Geneva C) New York D) Vienna PBH-24. The initiative "Health for all" means: A) the achievement of a health level that ensures complete physical, mental, and social well being B) the achievement of a health level that is based on the rights and responsibilities of the individual as well as the society C) the achievement of a health level that ensures complete physical, mental, and economic well being D) the provision of a health level that ensures a socially and financially productive life for every individual PBH-26. The term "lethality" means: A)the intensity of a disease B) the ratio of fatal cases among patients with the given disease C) the mortality rate of a given disease DrRognerud@www.medfever.com

PUBLIC HEALTH

Answer: C

Answer: D

Answer: A

Answer: B

Answer: D

Answer: B

15 / 343

Board Exam Question (Smiley + Star) + 2007 D) the morbidity conditions of a given disease PBH-29. Epidemiology can be regarded as the study of A) the etiology of human diseases B) the incidence and causes of human death C) the distribution of human diseases and the incidence of their determining factors D) the functional parameters of the health care delivery system PBH-30. Which of the following statements regarding the infant mortality rate is correct? A) the numerator contains the number of neonates who died during the first month of life B) this is the ratio of neonatal deaths during the first week of life per thousand live births C) this is the ratio of infant deaths during the first year of life per thousand live births PBH-38. According to the results of the "KOMOV Study", all of the following belong to the 3 most frequent ICD main groups in Hungary, EXCEPT: A) skin and connective tissue diseases B) upper respiratory tract and respiratory system diseases C) cardiovascular diseases D) gastrointestinal diseases PBH-45. All of the following are valid statements regarding the significance of hypertension, EXCEPT: A) it is the most prevalent disease in the main group of cardiovascular disorders B) absenteeism from work associated with cardiovascular disorders is mainly due to hypertension C) it is a prevalent cause of death in the mortality statistics D) hypertension is extremely demanding on nursing and health care facilities PBH-48. All of the following are risk factors for hypertension, EXCEPT: A) a high bodyweight B) a hereditary disposition C) an excessive intake of sodium D) the regular use of alcohol E) smoking PBH-51. All of the following factors influence the impact of diabetes mellitus on public health, EXCEPT: DrRognerud@www.medfever.com

PUBLIC HEALTH

Answer: C

Answer: C

Answer: A

Answer: C

Answer: E

Answer: A

16 / 343

Board Exam Question (Smiley + Star) + 2007 A) insulin dependent (type I) diabetes mellitus decreases the average life expectancy by 15% B) the prevalence of disability is 2-3 times higher than in the normal population C) the prevalence of blindness is 10 times higher than in the total population D) the prevalence of limb amputations is 20-30 times higher than in the normal population PBH-52. All of the following are valid statements regarding diabetes mellitus, EXCEPT: A) the prevalence of diabetes depends on the diagnostic criteria which are applied B) the prevalence of diabetes increases with advancing age C) the prevalence of diabetes is higher in males D) adult onset diabetes is called type II, non-insulin dependent diabetes PBH-57. Risk factors for the development of a myocardial infarction include all of the following, EXCEPT: A) an excess bodyweight B) an elevated cholesterol level C) a lack of exercise D) alcohol abuse E) an A-type personality F) smoking PBH-65. Primary hepatocellular carcinoma may develop in: A) hepatitis E B) hepatitis D C) hepatitis C D) hepatitis B PBH-66. The application of gamma-globulin (immunoglobulin) currently manufactured in Hungary is effective for post-exposure protection in: A) a hepatitis A infection B) a hepatitis B infection C) a hepatitis C infection D) a hepatitis D infection E) none of the above PBH-68. In Hungary, the mortality rate resulting from malignant disease is highest in tumors of the: A) stomach B) large intestine C) rectum DrRognerud@www.medfever.com

PUBLIC HEALTH

Answer: C

Answer: D

Answer: D

Answer: A

Answer: D

17 / 343

Board Exam Question (Smiley + Star) + 2007 D) lung PBH-77. Which of the following nosocomial infections is the most common in Hungary? A) pneumonia B) surgical wound infections C) septic conditions D) urinary tract infections PBH-79. Which of the following hospital departments is afflicted by the highest relative incidence of nosocomial infections? A) hospital nurseries B) intensive care units C) departments of surgery D) departments of urology PBH-91. Case Study: Several patients present at your office simultaneously, with symptoms occurring 2-3 hours after a meal. Their complaints include malaise, nausea and vomiting although they have no fever. Which of the following food-poisonings is the likely cause of this condition? A) botulism B) salmonellosis C) staphylococcal food poisoning (caused by S. aureus) D) Clostridium perfringens food poisoning PBH-92. Which of the following may cause cancer of the urinary bladder? A) cadmium B) mercury C) aniline dyes D) lead PBH-96. In which of the following types of food poisonings should a blood sample be sent to the laboratory for examination? A) staphylococcal food poisoning B) botulism C) campylobacteriosis D) salmonellosis PBH-100. Initial hearing-loss resulting from exposure to noise is detectable at a freqency of: A) 1000 Hz B) 2000 Hz C) 3000 Hz DrRognerud@www.medfever.com

PUBLIC HEALTH

Answer: D

Answer: B

Answer: C

Answer: C

Answer: B

Answer: D

18 / 343

Board Exam Question (Smiley + Star) + 2007 D) 4000 Hz PBH-102. Contraindications to post-exposure active immunization for rabies include(s): A) pregnancy B) rheumatoid arthritis C) active tuberculosis D) all of the above E) none of the above PBH-103. The recent fecal contamination of drinking water is suggested by a high concentration of. A) chloride B) nitrate C) sulphate D) ammonia E) nitrite PBH-104. Hemangiosarcoma may develop as a result of prolonged exposure to: A) asbestos B) polyvinylchloride (PVC) C) beryllium D) chlorinated hydrocarbons PBH-109. A child with polio should be isolated: A) at home B) in the hospital (in the department of infectious diseases) C) no isolation is necessary PBH-116. The length of the incubation period of anthrax is: A) several hours B) 2 weeks C) 3-4 days D) 21 days PBH-118. Single Choice Question The pathogen of Q fever is: A) Coxiella burneth B) Chlamydia sp. C) Miyagawanella sp. D) viruses PBH-120. A high concentration of detergents is characteristic of: DrRognerud@www.medfever.com

PUBLIC HEALTH

Answer: E

Answer: D

Answer: B

Answer: B

Answer: C

Answer: A

Answer: C 19 / 343

Board Exam Question (Smiley + Star) + 2007 A) industrial sewage B) agricultural sewage C) domestic sewage D) sewage of health-care institutions PBH-121. Leukemia may develop insidiously with prolonged exposure to: A) methylbromide B) benzene C) trichlorethylene D) carbon tetrachloride PBH-123. Organic phosphate esters: A) inhibit the activity of phosphophosphorylase B) inhibit the activity of acetylcholinesterase C) inhibit the activity of malate dehydrogenase D) stimulate the activity of pseudocholinesterase PBH-130. Streptococcus pyogenes is the pathogen of: A) a puerperal infection B) diarrhea outbreaks in hospitals C) pemphigus D) Lyell's syndrome (toxic epidermal necrolysis) PBH-137. The reservoir of yellow fever is/are: A) infected monkeys and humans B) mosquitoes C) tsetse fly D) rat PBH-139. Measures important for the prevention of toxoplasmosis include: A) the hygenic control of domestic dogs B) avoiding the ingestion of undercooked meat C) avoiding the ingestion of game meat D) supplementing the diet of pregnant women with meat PBH-140. The presence of proglottides in the gastrointestinal tract is a characteristic feature of. A) echinococcosis (hydatid disease) B) taeniasis (beef tapeworm infection) C) hymenolepidosis (dwarf tapeworm infection) D) toxoplasmosis PBH-149. In infectious mononucleosis, preventive measures include that: DrRognerud@www.medfever.com

PUBLIC HEALTH

Answer: B

Answer: B

Answer: A

Answer: A

Answer: B

Answer: B

Answer: D 20 / 343

Board Exam Question (Smiley + Star) + 2007 A) the patient should be isolated B) laboratory testing is mandatory C) reporting is not required D) continuous disinfection is necessary PBH-163. The positivity of the Gruber-Widal reaction for typhoid fever: A) is of diagnostic value when the antigen titers are 1:200 or greater B) is of diagnostic value when the antigen titers are 1:50 or greater C) is reliable in the range of 1:100 to 1:3200 D) is the sole important feature as the antibody titers are irrelevant PBH-179. Components of the chemotherapeutic index include: A) the toxic dose B) the DL50 value C) both the tolerated and the toxic dose D) both the tolerated and the curative dose PBH-188. Serum hepatitis corresponds to: A) hepatitis A B) hepatitis B C) non-A non-B hepatitis D) chronic aggressive hepatitis PBH-211. Vitamin A deficiency results in: A) keratomalacia B) polyneuritis C) anemia D) ossification disorders PBH-212. Rachitis tarda is characteristic in: A) the adult age B) the neonatal age C) puberty D) advanced age PBH-218. The additive effects of two different poisons having entered the organism is referred to as: A) interaction B) antagonism C) summation D) synergism PBH-221. Considering the low success rate of therapy in affected cancer patients DrRognerud@www.medfever.com

PUBLIC HEALTH

Answer: B

Answer: D

Answer: B

Answer: A

Answer: C

Answer: C

Answer: C 21 / 343

Board Exam Question (Smiley + Star) + 2007 already detected by screening, primary prevention is particularly important in: A) skin cancer B) breast cancer C) bronchial cancer D) rectal cancer E) cervical cancer PBH-229. The diversity of the surface antigens of the HIV virus is caused by frequent changes of the nucleotides in the: A) env gene sequence B) vif gene sequence C) gag gene sequence D) tat gene sequence PBH-230. Viral surface antigens are: A) proteins B) polysaccharides C) lipopolysaccharides D) glycoproteins PBH-233. Human pathogens of tuberculosis include: A) Mycobacterium tuberculosis hominis (in 99% of cases) B) Mycobacterium tuberculosis bovis (in 3% of cases) C) Mycobacterium tuberculosis hominis (in 97% of cases) D) Mycobacterium brevis (in 3% of cases) PBH-236. A proper epidemiologic control measure for measles: A) the occurrence of the infection should be reported to the local infectious disease control centre B) laboratory testing is not mandatory C) continuous disinfection is unnecessary D) isolation is unnecessary after clinical reovery PBH-237. A proper epidemiologic control measure for rubella: A) isolation of the patient from pregnant women is unnecessary B) continuous disinfection is unnecessary C) laboratory testing is mandatory D) rubella infections and any cases of the congenital rubella syndrome should be reported PBH-238. Which of the following plays the most important role in warm weather? A) the sympathetic vegetative system B) the parasympathetic vegetative system DrRognerud@www.medfever.com

PUBLIC HEALTH

Answer: A

Answer: D

Answer: A

Answer: B

Answer: D

Answer: A

22 / 343

Board Exam Question (Smiley + Star) + 2007 C) the appendages of the skin D) the respiratory system PBH-240. The primary portal of entry for tricresyl-phosphate is the: A) skin B) mucous membranes C) lung D) secretory organs PBH-242. The length of the incubation period of gonorrhoea is: A) 2 days B) 14 days C) 3-7 days D) several hours PBH-252. Provided that patient compliance is high, which of the following conditions can be treated? A) lung cancer B) bronchial cancer C) cancer of the colon D) breast cancer E) cervical cancer PBH-269. All of the following are principal causes of indoor nosocomial infections, EXCEPT: A) mycobacteria B) Escherichia coli C) Staphylococcus aureus D) Aspergillus flavus

PUBLIC HEALTH

Answer: A

Answer: C

Answer: E

Answer: B

MULTIPLE CHOICE QUESTIONS WITH KEY ANSWERS / TYPE II Every question or incomplete statement has only one answer in the following combinations: A) if the answers 1, 2, and 3 are true B) if the answers 1 and 3 are true C) if the answers 2 and 4 are true D) if only the answer 4 is true E) if all the four answers are true Select one of these key combinations!!! PBH-289. Environmental noise is an etiologic factor in the development of 1) neurosis 2) hypertension 3) hearing loss 4) Raynaud's syndrome DrRognerud@www.medfever.com

Answer: A

23 / 343

Board Exam Question (Smiley + Star) + 2007 PBH-295. Protective immunization against hepatitis B is recommended for: 1) the personnel of infectious disease departments 2) the personnel of hemodialysis units 3) dentists 4) laboratory staff PBH-315. In patients with an ovalbumin allergy in their medical history, the application of viral vaccines is: 1) absolutely contraindicated 2) not contraindicated in patients over the age of 6 years 3) appropriate if antihistamines are administered simultaneously 4) possible, depending on the results of the skin test RELATION ANALYSIS In the following questions determine if the statement in the first half of the sentence and the explanation in the second half of the sentence are true and if a causal relationship exists between them. Select the single correct version from the five possible combinations: A) both the statement and the explanation are true and a causal relationship exists between them; B) both the statement and the explanation are true but there is no causal relationship between them; C) the statement is true, but the explanation is false; D) the statement is false, but the explanation itself is true; E) both the statement and the explanation are false. PBH-353. Smokers are better protected against the development of Alzheimer's disease than non-smokers because the influence of nicotine on dopamine metabolism is highly preventive. PBH-380. Polycyclic hydrocarbon constituents of tobacco smoke are pluripotent carcinogens because these substances form adducts in the blood. PBH-385. Air pollution has dramatic effects on human health because its influence is more significant than that of soil contamination. PBH-402. The effects of chemical (environmental) carcinogens are usually manifested during the process of multiphasic carcinogenesis because they only cause malignancy after a decade-long exposure. PBH-434. The control of environmental conditions reduces the incidence of DrRognerud@www.medfever.com

PUBLIC HEALTH

Answer: E

Answer: D

Answer: C

Answer: B

Answer: B

Answer: B

Answer: A 24 / 343

Board Exam Question (Smiley + Star) + 2007 malignant disease because environmental factors are responsible for the development of malignant disease in at least 10% of cases. PBH-439. Stroke can be prevented by appropriately scheduled screening and intervention because factors other than dietary ones are also involved in its etiology. PBH-456. The HIV virus is destroyed outside the body because it is extremely sensitive to the changes of temperature and pH. PBH-501. The consumption of fish is deleterious to health because water pollutants may accumulate in the fish liver. PBH-505. Vitamin A has a protective effect against malignancy because crystalline vitamin A has a toxic effect on tumor cells.

PUBLIC HEALTH

Answer: A

Answer: A

Answer: D

Answer: C

ASSOCIATION QUESTIONS Associate the following terms/statements marked by the letters A, B, C... with the corresponding statements/terms marked by and in the order given by the figures 1, 2, 3... ...for example: 1-C, 2-B, 3-A, 4-D. Put the answer as C, B, A, D! (Note: Different statements can be associated with the same terms!!!) PBH-536. Associate the following term(s) with their corresponding statement(s)! A) Sensitivity B) Specificity C) Predictive value D) Validity E) Relative risk 1) indicates that the method or test in fact measures the targeted parameter 2) has a negative value in healthy individuals 3) has a positive value in ill individuals 4) indicates the risk of developing the disease of a given case if the individual exposes himself to the effects of certain risk factors 5) indicates the prevalence of patients among the cases indicated positive by this method PBH-537. Associate the following term(s) with their corresponding statement(s)! A) Case-control study B) Cohort study C) Cross-sectional study 1) starts with an unstratified sample 2) analyzes the incidence of risk factors in patient and control groups 3) examines both exposed and unexposed groups 4) none of the members of the examined groups have the disease DrRognerud@www.medfever.com

Answer: DBAEC

Answer: CABBACAB

25 / 343

Board Exam Question (Smiley + Star) + 2007 5) analyzes the incidence of risk factors in two groups 6) analyzes the presence of illness and risk factors in every individual 7) it is also called a retrospective study 8) it is also called a follow-up study PBH-538. Associate the following term(s) with their corresponding statement(s)! A) Lethality B) Mortality C) Age-specific mortality D) Infant death-rate E) None of the above 1) the ratio of infant mortality during the first year of life per 1000 live births 2) the most common populational measure of mortality 3) indicates the life-threatening nature of the disease 4) measures the mortality of certain age groups 5) indicates the mortality of populations with different agedistribution PBH-539. Associate the following term(s) with their corresponding statement(s)! A) Life expectancy at birth B) Probable life expectancy C) Normal life expectancy D) Average life expectancy E) None of the above 1) the number of deaths per year 2) the average of the age of the members of a population alive at a given time 3) the age characteristic of most of the individuals deceased during the year 4) the possible length of life in years for a neonate, assuming that mortality conditions will not change 5) the period during which the number of individuals born in the same year is halved PBH-541. Associate the following term(s) with their corresponding statement(s)! A) Mechanical biological concept B) Functional physiological concept C) Corticovisceral pathology D) Psychosomatic concept 1) the environment in its completeness is manifested by the complexity of social conditions 2) concentrates on the method of processing social effects 3) regards both health and disease as purely biological phenomena 4) its essential principle is the regulative coordination of functions; disordered coordination results in the development of lesions

PUBLIC HEALTH

Answer: DBACE

Answer: EDCAB

Answer: CDAB

DrRognerud@www.medfever.com

26 / 343

Board Exam Question (Smiley + Star) + 2007 PBH-548. Associate the following term(s) with their corresponding statement(s)! A) Hard risk factor B) Soft risk factor 1) blood pressure 2) cholesterol level 3) lifestyle 4) family conditions 5) diabetes mellitus

PUBLIC HEALTH

Answer: AABBA

DrRognerud@www.medfever.com

27 / 343

Board Exam Question (Smiley + Star) + 2007

PSYCHIATRY

PSYCHIATRY
(134) TRUE-FALSE TYPE QUESTIONS Put T for true statements and F for false statements!!! PSY-1. During family therapy, a more advisable approach is to tell each family member that they can change for the better, rather than to stigmatize them. PSY-2. In order to be more effective with a particular course of therapy, it is advisable not to have any preconceptions that can govern our therapeutic attempts. PSY-4. In a partially separated family with children who areneglecting their duties, there is still a risk for strong loyalties among the divided family members. PSY-5. It is advisable to stay neutral rather than to becomeauthorative over the family during family psychotherapy. SINGLE CHOICE QUESTIONS Select the single best response to each of the following questions!!! PSY-6. A doctor, who smokes, asserts that smoking is beneficial because it inhibits the development of obesity is using which of the following "defense" mechanisms? A) dissociation B) intellectualization C) rationalization D) reaction formation E) projection PSY-7. The most accepted hypothesis explaining the biological basis for schizophrenia is: A) the transmethylation hypothesis B) the double bind hypothesis C) the serotonin hypothesis D) the dopamine hypothesis E) the endogenous opiate hypothesis

Answer: T

Answer: T

Answer: T

Answer: T

Answer: C

Answer: D

DrRognerud@www.medfever.com

28 / 343

Board Exam Question (Smiley + Star) + 2007 PSY-8. A disorientation to time is characteristic of. A) Korsakoff s syndrome B) an acute schizophrenic episode C) hypomania D) depressive psychosis E) agoraphobia PSY-10. In attempting to teach a child to accomplish a new task one must: A) reinforce the child immediately following completion of the task B) reinforce the child, with a slight delay, following completion of the task C) reinforce the child, after a marked delay, following completion of the task PSY-11. When a 5-year-old child "throws a fit", he/she is usually punished but at times the child gets what he/she wants. These fits are most likely to: A) become less frequent B) gradually cease C) become continuous PSY-13. To properly develop a good behavior in a child, it is advisable to: A) punish the child B) reward the child C) both of the above D) none of the above PSY-15. In attempting to reinforce the behavior of a child, it is advisable to: A) punish the child B) praise the child C) both of the above D) none of the above PSY-16. Punishment is effective if: A) it does not generate aversion towards the punishing person B) it does not result in an escape reaction C) it decreases the necessity of further punishment D) it does not reinforce an aggressive behavior E) all of the above PSY-17. Punishment is effective if it is applied: A) immediately B) with a slight delay C) with marked delay DrRognerud@www.medfever.com

PSYCHIATRY

Answer: A

Answer: A

Answer: C

Answer: B

Answer: B

Answer: E

Answer: A

29 / 343

Board Exam Question (Smiley + Star) + 2007 D) none of the above PSY-18. In the following example, the best way to reinforce a child's behavior is to tell him/her: 1) "Go to bed, I'll tell you a tale!" 2) "I'll beat you if you don't go straight to bedl" 3) "I'm happy that you've put your pyjamas on!" 4) "If you don't go to bed, you can't have breakfast in the morning!" A) answers (1),( 2), and (3) are correct B) answers (1) and (3) are correct C) answers (2) and (4) are correct D) all of the above PSY-21. Important mechanisms in the development of the "ego" include: A) condensation B) rationalization C) defense D) identification PSY-25. Contraindications to lithium administration include: A) an administration in combination with chlorpromazine (Thorazine) B) the presence of a renal disease C) any occurrence of the symptoms of schizophrenia D) the presence of depression E) an administration in combination with imipramine (Tofranil) PSY-26. "Suggestion" as a form of a psychotherapy is used in: A) conversion disorder B) child psychiatry C) patients with a low IQ D) all of the above E) none of the above PSY-27. Perception without corresponding environmental stimuli is: A) a hallucination B) an illusion C) a delusion D) derealization E) depersonalization PSY-28. The occurrence of which of the following symptoms would allow differentiation between delirium and dementia? A) an impaired judgment DrRognerud@www.medfever.com

PSYCHIATRY

Answer: B

Answer: D

Answer: B

Answer: D

Answer: A

Answer: C

30 / 343

Board Exam Question (Smiley + Star) + 2007 B) a memory deficit C) an impaired consciousness D) an impaired process of thinking E) disorientation PSY-29. A loss of remote memory is a typical symptom of delirium tremens A) delirium tremens B) senile dementia C) schizophrenia D) Korsakoffs syndrome E) hysteria PSY-31. Early in the psychiatric interview, it is important for the physician to; A) inform the patient of the fee B) obtain details of any past psychiatric illnesses C) let patients talk about what is bothering them D) obtain information about the patient's mood E) record the family history PSY-36. A "projection" mechanism is most characteristic for which of the following personality disorders? A) an anancastic personality B) a schizoid personality C) a hypomanic personality D) a paranoid personality E) an antisocial personality PSY-38. Anxiety is a common symptom of all the following conditions, EXCEPT. A) hypoglycemia B) hypothyroidism C) pheochromocytoma D) porphyria E) hypocalcemia PSY-40. A "borderline personality disorder" is characterized by all of the following symptoms, EXCEPT: A) impulsivity and an unpredictable behavior B) identity disturbances C) mood instability D) withdrawal from social activity E) recurrent suicidal gestures and short psychotic episodes PSY-42. Case Study: A 56-year-old male presents with symptoms of irritability and disinterest DrRognerud@www.medfever.com

PSYCHIATRY

Answer: D

Answer: C

Answer: D

Answer: B

Answer: D

Answer: D

31 / 343

Board Exam Question (Smiley + Star) + 2007 in his daily activities. At times, he is confused and forgetful. His gait is unsteady. The deep tendon reflexes are diminished. He frequently experiences tingling in his legs. The most likely diagnosis is: A) hypothyroidism B) a cerebellar tumor C) multiple sclerosis D) a vitamin B12 deficiency E) presenile dementia PSY-43. Bleuler's symptoms of schizophrenia (the four A's) include all of the following EXCEPT: A) ambivalence B) affective flattening C) apathy D) autism E) loose associations PSY-44. Case Study: A 25-year-old patient complains of hearing voices speaking about him and threatening him. The patient regards them as real and suffers from them. The most likely diagnosis is: A) schizophrenia B) alcoholic hallucinations C) dementia D) hysteria E) debility PSY-47. Case Study: A 23-year-old woman complains of becoming occasionally anxious. These occurences are associated with tachycardia and excessive sweating. The condition usually develops in the morning. Which of the following tests has to be performed urgently? A) thyroid function tests B) toxicological screening C) determination of the serum sodium level D) determination of the blood glucose level E) determination of the serum ammonia level PSY-51. Case Study: A middle-aged waiter was admitted to the psychiatric ward in a drunken state. While in the ward his behavior became bizarre and he gradually became became disoriented to place and time. He sometimes acted as if he was taking orders or serving dishes. On examination he usually misinterpretated the antecendents and circumstances of his admission. He was unable to recall his answers to simple questions after a few minutes. He denies any DrRognerud@www.medfever.com

PSYCHIATRY

Answer: C

Answer: B

Answer: D

Answer: D

32 / 343

Board Exam Question (Smiley + Star) + 2007 hallucinations or delusions. The patient has had no prior psychiatric disorders. His relatives haven't found anything extraordinary about his behavior. The most likely diagnosis is: A) alcohol withdrawal syndrome (delirium tremens) B) acute paranoid schizophrenia C) alcoholic hallucinations D) alcohol amnestic syndrome (Korsakof's syndrome) E) manic phase (of bipolar disorder) PSY-54. The psychotherapy of schizophrenic patients includes all of the following, EXCEPT: A) a warm, open relationship aiming to promote the patient's selfesteem and educating the patient about his/her disease B) a supportive psychotherapy that focuses on resolving the problems of the patient in his/her everyday life C) setting limits on the patient's behavior, including the consequences of his/her violent actions D) encouraging socialization in order to build more extensive social relationships E) encouraging the patient to express his/her anger and hostility as much as possible in the therapeutic relationship in order to reduce the intensity of these emotions outside the consulting office PSY-58. Case Study: A 25-year-old female was brought to the hospital ward by ambulance. Upon examination she was febrile, confused, and a bizarre posture was observed. The results of blood and cerebrospinal fluid tests were normal. The patient was diagnosed as schizophrenic and is currently on chlorpromazine (Hibernal). Which is the most likely cause of her current symptoms? A) an acute dystonic reaction B) akathisia C) tardive dyskinesia D) a malignant neuroleptic syndrome E) an allergic reaction to chlorpromazine PSY-59. Statements which are characteristic for the various psychotherapeutic methods include all of the following, EXCEPT: A) these methods aim to relieve anxiety and to improve social integration B) the theories concerning the application of the methods are specific for each of the psychiatric disorders C) these focus on childhood events and experiences D) impulsivness and resistance develop between the physician and the patient E) these methods all have a therapeutic aim and elicit a learning process in the patient DrRognerud@www.medfever.com

PSYCHIATRY

Answer: E

Answer: D

Answer: C

33 / 343

Board Exam Question (Smiley + Star) + 2007 PSY-60. Which of the following tests is important for the differential diagnosis of organic and psychogenic impotence? A) monitoring of the serum gonadotropine levels over 24 hours B) nasopharyngeal EEG during sexual stimulation C) night-time erections D) projective tests E) monitoring of any alterations in the testosterone levels PSY-61. Negative symptoms of schizophrenia include all of the following, EXCEPT: A) flat affect B) auditory hallucinations C) lack of motivation and initiative D) anhedonia E) poverty of thought content PSY-63. Hallucinations are symptoms of: A) mood disorders B) mental disorders C) thought disorders D) abnormal perception E). disorders of concentration PSY-64. Delusion is characterized by all of the following statements, EXCEPT: A) delusion is a belief that does not correspond to the experiences of the individual B) delusions are common symptoms of schizophrenia C) delusions are possible symptoms of affective disorders D) delusions may be symptoms of an organic mental disorder E) delusions may be eliminated by logical explanations PSY-65. Which of the following is the most common cause of the cessation of sexual activity in married couples? A) aging B) marital discord C) physical illness D) cultural prohibition E) depression PSY-68. Case Study: A young man develops an irrepressible urge to wash his hair several times a day which he explains as a means of protection against infection from others. He assures everyone that he feels well but he becomes extremely anxious if he cannot wash his hair. The most probable diagnosis is: DrRognerud@www.medfever.com

PSYCHIATRY

Answer: C

Answer: B

Answer: D

Answer: E

Answer: B

Answer: B

34 / 343

Board Exam Question (Smiley + Star) + 2007 A) automatism B) compulsive personality disorder C) hypochondriasis D) compulsive thoughts E) phobia PSY-69. Unconscious emotions generated by a physician during psychotherapy are best described by the term: A) projection B) impulse transmission C) acting out D) identification E) introjection PSY-73. Based on the results of psychiatric epidemiological studies, the most common psychiatric disorder among the general population is: A) depression B) schizophrenia C) alcoholism D) phobias E) dementia PSY-74. The ratio of psychiatric disturbances among patients who seek evaluation for somatic diseases is: A) 10% B) 20% C) 33% D) 50% E) 90% PSY-78. Case Study: A 20-year-old man is admitted to the hospital. He developed hallucinations and delusions of persecution three weeks ago. He is currently agitated. Possible diagnoses, based on the DSM-III-R, include all of the following, EXCEPT: A) brief reactive psychosis B) organic mental disorder C) borderline personality disorder D) schizophrenia E) schizophreniform disorder PSY-79. The most important reason for monitoring the serum lithium level is: A) to check on the patient's compliance B) because the toxic dose is very close to the therapeutic level C) because lithium is rapidly excreted from the body DrRognerud@www.medfever.com

PSYCHIATRY

Answer: B

Answer: C

Answer: C

Answer: D

Answer: B

35 / 343

Board Exam Question (Smiley + Star) + 2007 D) because lithium is a salt, rather than a drug E) none of the above PSY-83. Catatonic motor disorder is best defined as: A) a marked hyperactivity which is commonly violent and aimless B) a generalized muscle rigidity C) waxy flexibility D) stupor or mutism, without an organic cause E) a severe psychomotor disturbance which cannot be attributed to an organic cause PSY-84. Drugs contraindicated in acute alcohol intoxication include all of the following, EXCEPT: A) diazepam (Seduxen) B) phenobarbital (Sevenal) C) disulfiram (Anticol, Antaethyl) D) glutethimide (Noxyron) E) haloperidol PSY-85. Common complications of alcoholism include: A) cerebral damage B) gastritis C) suicide D) polyneuropathy E) all of the above PSY-87. Organic mental syndromes include all of the following, EXCEPT: A) delirium B) dementia C) amnestic syndromes D) paranoid disorders E) organic hallucinosis PSY-88. Diseases that lead to mental retardation and require genetic counselling include all of the following, EXCEPT: A) Tay-Sachs disease B) galactosemia C) phenylketonuria D) Down's syndrome E) cerebral paralysis (Little's disease)

PSYCHIATRY

Answer: E

Answer: E

Answer: E

Answer: D

Answer: E

MULTIPLE CHOICE QUESTIONS WITH KEY ANSWERS / TYPE II Every question or incomplete statement has only one answer in the following combinations: A) if the answers 1, 2, and 3 are true DrRognerud@www.medfever.com 36 / 343

Board Exam Question (Smiley + Star) + 2007 B) if the answers 1 and 3 are true C) if the answers 2 and 4 are true D) if only the answer 4 is true E) if all the four answers are true Select one of these key combinations!!! PSY-90. Symptoms of a perception disorder include: 1) hallucination 2) depersonalization 3) illusion 4) perseveration PSY-92. Sleep patterns characteristic for major (endogenous) depression include: 1) frequent nightmares followed by awakening 2) waking up too early 3) a marked prolongation of the 3rd and 4th phases (delta phase) of sleep 4) a markedly decreased ratio of the REM phases PSY-94. Functions of the ego include: 1) the regulation of intrapsychic conflicts 2) the regulation of instincts 3) reality testing 4) developing relationships with objects PSY-95. A complete psychic evaluation should include inquiries about any: 1) suicidal gestures 2) homicidal ideas 3) delusional thinking 4) hallucinations PSY-97. Case Study: A 35-year-old man complains of hearing voices at night, telling him that he is a bad and guilty person. Having no other available information, which of the following conditions would you consider ? 1) schizophreniform disorder 2) personality disorder 3) hypnagogic hallucinations 4) organic hallucinations PSY-99. Characteristics of a conversion (histrionic) disorder include: 1) mimicking an organic disease 2) expression of a psychiological conflict 3) seeking other people's attention in order to gain their support DrRognerud@www.medfever.com

PSYCHIATRY

Answer: A

Answer: C

Answer: E

Answer: E

Answer: E

Answer: A

37 / 343

Board Exam Question (Smiley + Star) + 2007 4) the patient's ability to control his/her symptoms voluntarily PSY-104. Which of the following statements concerning suicide are correct? 1) the ratio of suicides with a fatal outcome decreases with age 2) women die from suicide more often than men 3) a suicide can reliably be predicted by certain clinical features 4) a conversation with the patient about his/her ideas of suicide may protect the patient from being a potential victim PSY-105. Psychotropic drugs that can cause addiction include: 1) benzodiazepines 2) amphetamines 3) meprobamate 4) butyrophenones PSY-106. Case Study: A 62-year-old man seeks evaluation for weakness, a loss of initiative, a loss of weight, and abdominal discomfort. He appears to be depressed. Possible diagnoses can include: 1) dementia 2) pain killer abuse 3) pancreatic carcinoma 4) hyperthyroidism PSY-107. Case Study: A 36-year-old man is brought to the emergency room by the police. He has been caught speeding on the highway at night without his headlights on. On examination he is agitated and belligerent. He warned the physician and the policemen that he has "friends" in high places whom he is currently in contact with and that the policemen who have incarcerated him will be punished. Diagnostic possibilities can include: 1) hyperthyroidism 2) arsenic intoxication 3) amphetamine overdose 4) Addison's disease PSY-109. Case Study: A 67-year-old man is brought to the emergency room by the police for exposing himself in the nude to schoolchildren. There is no history of similar events in the past. Possible causes of this behavior include: 1) a petit mal seizure 2) Alzheimer's disease 3) digitalis intoxication 4) an intracranial tumor

PSYCHIATRY

Answer: D

Answer: A

Answer: A

Answer: B

Answer: C

DrRognerud@www.medfever.com

38 / 343

Board Exam Question (Smiley + Star) + 2007 PSY-111. Which of the following agents have an important role in the therapy of alcohol withdrawal delirium? 1) meprobamate and benzodiazepines 2) vitamin B complex 3) chlormethiazole (Heminevrin) 4) potassium and magnesium ions PSY-114. Visual hallucinations are characteristic for which of the following conditions? 1) acute alcohol abuse 2) (Korsakof's syndrome 3) alcohol hallucinations 4) delirium tremens PSY-117. If someone is said to be disoriented, they are most likely not to know: 1) the date 2) where they are 3) the time . 4) some famous people PSY-120. Which of the following statements concerning the risk of fatal suicide are correct? 1) women are at a higher risk than men 2) the risk for the patients above the age of 65 is higher than for those between 25-35-years-old 3) the incidence of suicide is higher during times of war 4) alcohol addicts are at a higher risk for suicide PSY-121. Possible causes of organic anxiety disorders include: 1) the withdrawal of sedatives or sleeping pills 2) pheochromocytoma 3) an excessive use of caffeine 4) hypoparathyroidism PSY-125. Some undesirable complications of a somatization disorder (chronic neurosis) include: 1) the excessive use of drugs 2) secondary iatrogenic complications of invasive diagnostic interventions 3) an excessive dependence on health care 4) a frequent change of physicians

PSYCHIATRY

Answer: E

Answer: D

Answer: A

Answer: C

Answer: A

Answer: E

DrRognerud@www.medfever.com

39 / 343

Board Exam Question (Smiley + Star) + 2007 PSY-126. How does malingering differ from somatization disorders? 1) it occurs more rarely 2) this disease is more susceptible to therapy 3) malingering is not a psychiatric disorder 4) it may involve the abnormality of several organs PSY-136. The anticholinergic side-effects of tricyclic antidepressants that are fi equently observed, especially during the therapy of elderly patients include: 1) tachycardia 2) constipation 3) the retention of urine 4) blurred vision PSY-137. Case Study: A 53-year-old woman underwent an operation for a fracture of the neck of the femur. On the second postoperative day she became agitated and incooperative. On the third day she was noted to have hallucinations and addressed the nurses by the names of her own children. Possible causes of her symptoms include: 1) alcohol withdrawal 2) intravenously administered penicillin 3) sepsis 4) general anesthesia PSY-139. Tricyclic antidepressants and MAO inhibitors are effective for the treatment of 1) bulimia 2) compulsive personality disorder 3) anancastic disorder 4) atypical depression PSY-144. Common complications of alcoholism include: 1) cerebral damage 2) gastritis 3) hypertension 4) suicide PSY-145. Drugs that can cause dependence include: 1) benzodiazepines 2) antihistamines 3) barbiturates 4) tricyclic and tetracyclic antidepressants

PSYCHIATRY Answer: A

Answer: E

Answer: A

Answer: E

Answer: E

Answer: B

DrRognerud@www.medfever.com

40 / 343

Board Exam Question (Smiley + Star) + 2007 SINGLE CHOICE QUESTIONS Select the single best response to each of the following questions!!! PSY-147. The most typical example of a simple phobia is a fear of: A) heights B) public transportation C) dogs D) being in crowds E) social situations PSY-148. Case Study: A 22-year-old patient, during an interview, recalled that she had seen two doves sitting on the window sill, which she recognized as a future sign of an important event that would take place in her life in two weeks time. This symptom should be regarded as a(n): A) illusion B) hallucination C) delusion D) neologism E) incoherence PSY-149. The most important process in the development of the ego is: A) identification B) projection C) reaction formation D) regression E) repression PSY-150. Emotional reactions towards the physician, which reflect recent experiences and relationships outside of the therapeutic setting, may be defined as: A) acting out B) fixation C) free associations D) impulse transmission E) anxiety PSY-153. A characteristic defense mechanism involved in paranoid symptom formation is: A) reality denial B) conversion C) projection D) isolation E) acting out

PSYCHIATRY

Answer: A

Answer: C

Answer: A

Answer: D

Answer: C

DrRognerud@www.medfever.com

41 / 343

Board Exam Question (Smiley + Star) + 2007 PSY-155. Case Study A 29-year-old woman is brought to the emergency room by her husband. The woman complains of a sharp, intensive pain on the left side of her chest, accompanied by shortness of breath and palpitations. She fears that she had a heart attack. The results of her physical examination and blood tests are normal. The pain ceased after a few hours of observation and she was released. 3.155/ 1. Similar situations have occured previously, although an organic cause has never been demonstrated. What is the most likely diagnosis? A) histrionic conversion reaction B) malingering C) anancastic neurosis (panic disorder) D) hypochondriasis E) compulsive personality disorder 3.155/2. During these episodes of discomfort the patient talks about herself as if being an independent observer. This is an example of A) derealization B) depersonalization C) illusions D) hallucinations E) alienation PSY-158. characteristics of conversion disorders include all of the following, EXCEPT: A) their incidence in children is equal in both sexes B) the symptoms are involuntary C) their incidence is decreasing D) the symptoms correspond to the pathophysiology of the disorders E) they are more frequenly diagnosed in women by midadolescence PSY-160. Which of the following food constituents has to be avoided when prescribing a diet for patients treated with monoamino-oxidase inhibitors? A) cholesterol B) choline C) lactose D) tryptophan E) tyramine PSY-165. The most frequent type of schizophrenia among hospital admissions is: A) the autistic type B) the catatonic type C) the hebephrenic type D) the paranoid type E) the undifferentiated type DrRognerud@www.medfever.com

PSYCHIATRY

Answer: CB

Answer: D

Answer: E

Answer: D

42 / 343

Board Exam Question (Smiley + Star) + 2007 PSY-167. The leading symptom of affective disorders is a disturbance of A) concentration and cognitive functions B) mood C) association and the thought process D) initiatives and psychomotility E) perception PSY-169. Which of the following focal organic mental disorders is characterized by a loss of initiative? A) temporal lobe syndrome B) injury of the frontal convexity C) injury of the frontal base D) Korsakoffs syndrome E) diencephalic syndrome PSY-171. The risk for developing schizophrenia in a sister of a schizophrenic male child is: A) 70% B) 40% C) 25% D) 12% E) 1% PSY-174. Which of the following symptoms is indicative of barbiturate intoxication, rather than drug withdrawal? A) confusion B) nystagmus C) postural hypotension D) disorientation E) agitation PSY-177. Case Study: A 43-year-old woman has been found unconscious in her garage. The car was running and the door to the garage was closed. Upon examination she is confused. The most likely cause of her confusion is: A) lead poisoning B) hypoxia C) hypoglycemia D) gasoline inhalation E) none of the above PSY-178. A deficiency of which of the following vitamins is an important factor in the etiology of Korsakoff s syndrome? DrRognerud@www.medfever.com

PSYCHIATRY

Answer: B

Answer: B

Answer: D

Answer: B

Answer: B

Answer: D

43 / 343

Board Exam Question (Smiley + Star) + 2007 A) vitamin B6 B) folic acid C) nicotinamide D) vitamin B 1 E) vitamin B12 PSY-179. Case Study: An 8-year-old girl, in a febrile state, assumes that the curtain in her bedroom window is moving and an animal is trying to come into the room through the window. This symptom is a typical example of: A) delusion B) fantasy C) hallucination D) illusion E) phobia PSY-180. Case study: A physician asks an elderly male patient what he had for supper the previous day. The patient asserts that he had his christmas dinner together with his wife and children. This is late June now and the patient's wife died three years ago. Nobody visited the patient the previous day. The patient's reply is characteristic of: A) circumstantiality B) confabulation C) deja vu D) a flight of ideas E) an illusion PSY-181. Case Study: A 30-year-old man complains of impotence. He thinks that strangers on the street are laughing at him. He is sure that they know about his problem and that they are probably responsible for the development of his condition. This complaint should be regarded as a sign of: A) concreteness of thought B) delusions of reference C) imaginativeness D) decline of affect E) somatic delusions PSY-182. Which of the following is a best example for a double bind? A) Mary's parents want her to wait to get married until she finishes high school B) John's parents encourage him to go to high school but want him to decide about his own life C) Joe's parents encourage him to go to high school but they dissuade his sister from the same thing DrRognerud@www.medfever.com

PSYCHIATRY

Answer: D

Answer: B

Answer: B

Answer: D

44 / 343

Board Exam Question (Smiley + Star) + 2007 D) Frank's parents encourage him to apply to a high school but frequently remind him of the financial sacrifices his education requires from the family E) Sophie's parents encourage her to apply to a high school but recommend to her to work for a few years first in order to earn the costs of her education PSY-183. The psychic structure which regulates the conflicts between unconscious drives and the reality is the: A) ego B) ego-ideal C) id D) preconscious E) superego PSY-186. The description: "attributing one's own unacceptable motives and emotions to someone else" best characterizes: A) fantasy B) splitting C) regression D) projection E) identification PSY-188. Borderline personality disorder is characterized by all of the following, EXCEPT: A) severe impulsiveness and unpredictable behavior B) disturbances of identity C) emotional lability D) withdrawal from social relations E) recurrent suicidal gestures and short psychotic episodes PSY-189. Violent behavior is most characteristic for which of the following conditions? A) bipolar disease; manic type B) anancastic neurosis C) melancholia D) somatoform disorder E) compulsive personality disorder PSY-190. Case Study: A 27-year-old woman is brought to the emergency room complaining of shortness of breath, dizziness, and a tingling in her extremities. Careful examination fails to discover any organic abnormalities. Which of the following is the most likely cause of her symptoms? A) situational reaction DrRognerud@www.medfever.com

PSYCHIATRY

Answer: A

Answer: D

Answer: D

Answer: A

Answer: D

45 / 343

Board Exam Question (Smiley + Star) + 2007 B) endogenous anxiety C) caffeine abuse D) hyperventilation syndrome E) post-traumatic stress disorder PSY-193. Case Study: A middle-aged man becomes ill with Parkinson's disease. The prescribed medication fails to improve his motor abnormalities. His mood is depressed. He says he has lost his relish for life and the only thing he does is sleep. The first steps of managing this patient include: A) to admit him to a psychiatric ward with respect to a possibility of suicide B) reassuring the patient that the prescribed medications are effective in Parkinson's disease C) a discussion with the patient about his troubles and inquiry about any suicidal ideations D) referring the patient to a neurologist or a psychiatrist saying, "let them hear his story" E) scheduling frequent therapeutic settings and calling the attention of the family members to the possibility of suicide PSY-194. Case Study: A 41-year-old man complains that life does not give him what he wants. He feels disappointed and unhappy. He was depressed for a while after his girlfriend left him 8 years ago. What is the most likely diagnosis? A) psychogenic depression B) schizophrenia C) bipolar disorder; depressive phase D) dysthymia (neurotic depression) E) cyclothymia PSY-201. Schizophrenia is characterized by all of the following symptoms, EXCEPT: A) incoherence of thoughts B) bizarre delusions C) auditory hallucinations D) Korsakoff's syndrome E) parathymia PSY-203. The use of which of the following substances is most commonly associated with violent behavior? A) heroin B) cocaine C) amphetamines D) steroids DrRognerud@www.medfever.com

PSYCHIATRY

Answer: C

Answer: D

Answer: D

Answer: E

46 / 343

Board Exam Question (Smiley + Star) + 2007 E) alcohol PSY-205. Common features of delirium and dementia include all of the following, EXCEPT: A) impaired remote memory B) distorted thought process C) cognitive impairment D) EEG abnormalities E) organic pathology PSY-206. Characteristics of delirium tremens include all of the following, EXCEPT: A) an introductory grand mal seizure B) auditory hallucinations associated with clear thoughts and proper orientation C) tremors and sweating D) blackouts E) disorientation PSY-208. Dementia is characterized by all of the following statements, EXCEPT: A) demented patients are often depressed B) the ability to generalize from past experiences and to recognize the relationship between similar situations is impaired C) an early feature is an inability to recall events from the distant past D) demented patients may experience hallucinations E) Creutzfeldt-Jakob disease is a dementia caused by a slow virus infection

PSYCHIATRY

Answer: C

Answer: B

Answer: C

MULTIPLE CHOICE QUESTIONS WITH KEY ANSWERS / TYPE II Every question or incomplete statement has only one answer in the following combinations: A) if the answers 1, 2, and 3 are true B) if the answers 1 and 3 are true C) if the answers 2 and 4 are true D) if only the answer 4 is true E) if all the four answers are true Select one of these key combinations!!! PSY-209. Intelligence tests have which of the following characteristics? 1) they compare the performance of an individual as compared to a large group 2) they are influenced by culture 3) they do not measure an individual's entire intellectual capacity 4) they define an IQ of 100 as average

Answer: E

DrRognerud@www.medfever.com

47 / 343

Board Exam Question (Smiley + Star) + 2007 PSY-213. Risk factors for a patient's violent behavior in a physician's office include: 1) a history of manic disease , 2) a history of suicide attempts 3) alcohol abuse 4) head trauma PSY-215. Symptoms usually present in somatization disorder include: 1) dysmenorrhea 2) palpitations 3) anxiety 4) nausea PSY-218. Which of the following statements concerning social deprivation are correct? 1) it may be associated with severe mental retardation 2) it may be associated with a severe personality disorder 3) it may be experimentally modelled in animals 4) it frequently occurs in poorly organized hospital wards PSY-220. A Characteristics of neurotic depression include: 1) recurrent short hypomanic episodes 2) a sustained, low-level intensity of mood 3) unresponsiveness to therapy 4) a lack of psychotic symptoms PSY-225. A manic state is characterized by which of the following symptoms? 1) diffuseness and self-assurance 2) holothymic hallucinations and delusions of grandeur 3) increased activity and a decreased need for sleep 4) specific precipitating causes preceding the onset of symptoms PSY-227. Delirium tremens, in its initial phase, may be effectively prevented by: 1) meprobamate 2) benzodiazepines 3) chlomethiazole (Heminevrin) 4) barbiturates PSY-228. Characteristics of alcohol dependence include: 1) the need for drinking every day in order to maintain one's performance 2) the need to increase the amount consumed to elicit the same effect 3) tremor, sweating, and disorientation developing after two days DrRognerud@www.medfever.com

PSYCHIATRY Answer: E

Answer: E

Answer: E

Answer: C

Answer: A

Answer: A

Answer: E

48 / 343

Board Exam Question (Smiley + Star) + 2007 of abstinence 4) two or more blackouts during an acute alcohol abuse period PSY-229. Conditions which increase the risk of subdural hemorrhage include: 1) hypertension 2) advanced age 3) atherosclerosis 4) alcoholism PSY-231. The side-effects of tricyclic antidepressants include: 1) hypertension 2) dry mouth 3) diarrhea 4) blurred vision PSY-233. Which of the following statements concerning juvenile suicide are FALSE? 1) the prevalence of juvenile suicide attempts has increased 2) impulsive patients are at a higher risk 3) the leading cause of death among adolescents is suicide 4) child abuse is usually not associated with suicide PSY-234. Correct statements about illusions include: 1) they are elicited by an environmental stimulus and negatively affect sensory discrimination 2) they are a misperception of an existing environmental stimulus 3) they are more frequently observed in organic mental disorders than in functional psychiatric illnesses 4) they are not always associated with psychoses PSY-238. Characteristics of personality disorders include: 1) a gradual flattening of the thought process 2) a normal sense of reality 3) delusions, observed over a long period 4) the possible occurrence of psychotic episodes PSY-241. Antidepressant-type therapy may be of use in which of the following conditions? 1) bulimia 2) affective disorders 3) compulsive personality disorder 4) anancastic disorder PSY-242. Primary (psychologic and sociopsychologic) disease advantages DrRognerud@www.medfever.com

PSYCHIATRY

Answer: C

Answer: C

Answer: D

Answer: E

Answer: C

Answer: E

Answer: A 49 / 343

Board Exam Question (Smiley + Star) + 2007 include which of the following? 1) the disease elicits attention and care in the environment 2) affective conflicts are repressed from consciousness 3) the disease satisfies an unconscious need for dependency 4) the patient is given all the excuses a society can offer PSY-245. Delusions are best defined as false considerations which: 1) persist for a long time despite being obviously unrealistic 2) originate from the misinterpretation of existing external stimuli 3) appear to be real to the individual 4) are pathognostic for schizophrenia PSY-246. Case Study: A 17-year-old boy is brought to the emergency room by his father. The father reported that his son had taken three tablets of diazepam (Seduxen) in a suicide attempt. The boy minimized the episode, saying that he was just upset about school. The father became angry at his son for making such a fuss over nothing. The nurses started making jokes about the three diazepam "suicide". The father was impatient to take his son back home. Both were resistant to a psychiatric evaluation. 3.246/ 1. Since the overdose was not life threatening, the most appropriate treatment at this time would include: 1) calling other family members to the hospital 2) reporting the event to the boy's school and obtaining information about his academic problems 3) encouraging the father and son to stay and to be interviewed individually 4) encouraging the father to keep an eye on his son 3.246/2. Additional therapeutic approaches at this time would include: 1) initiating antidepressant therapy 2) warning the father to hide any medications kept at home 3) giving the son an excuse from school for a few days 4) suggesting a family consultation to relieve any present tensions at home 3.246/3. The son remained silent and the father insisted on going home. The following day the boy was found dead from a self-inflicted gunshot. This case represents common errors in evaluating a suicide attempt, including: 1) not adequately evaluating the son's emotions 2) not appreciating what the first suicide attempt meant to the son and his father 3) not assessing adequately the father's capacity to support his son 4) not hospitalizing the patient, even if it was against his will

PSYCHIATRY

Answer: A

Answer: BDA

DrRognerud@www.medfever.com

50 / 343

Board Exam Question (Smiley + Star) + 2007 PSY-248. Conditions that may be associated with the catatonic syndrome (rigidity, mutism, catalepsia, waxy flexibility) include: 1) an affective disorder 2) viral encephalitis 3) hypnosis 4) schizophrenia PSY-249. Uncommon side-effects of tricyclic antidepressant therapy include which of the following? 1) a dry mouth 2) tremor 3) constipation 4) extrapyramidal movement disorders PSY-250. Which of the following somatic diseases may be associated with a depression syndrome? 1) pancreatic carcinoma 2) hypertension 3) hypothyroidism 4) peptic ulcer disease PSY-251. Which of the following medications are capable of eliciting mania? 1) amphetamines 2) tricyclic antidepressants 3) corticosteroids 4) reserpine PSY-255. Atherosclerotic (multi-infarct) dementias are characterized by: 1) associated internal and neurologic symptoms 2) numerous malacic foci in the brain 3) a focal loss of memory 4) a dominant inheritance pattern PSY-257. Case Study: A 45-year-old man is admitted to the surgical ward. When interviewed by the nurse he states that he is married, is a father of three boys, and that he lives with his family. He had previously told his physician that he had been living with his father since his girlfriend and her two children had left him. When asked about these contradictions he became confused. Conditions that are likely to account for the patient's confabulation include: 1) Korsakoff's syndrome 2) diabetes mellitus 3) presenile dementia 4) Addison's disease DrRognerud@www.medfever.com

PSYCHIATRY

Answer: E

Answer: D

Answer: B

Answer: A

Answer: A

Answer: B

51 / 343

Board Exam Question (Smiley + Star) + 2007 PSY-259. Wernicke's encephalopathy is characterized by: 1) a sudden onset 2) nystagmus and ophthalmoplegia 3) a somnolent state 4) pathologic changes in the mamillary body MULTIPLE CHOICE QUESTIONS / TYPE I Select the correct answers to the following questions!!! ...each qestion may have more than one correct answer. PSY-262. Which of the following conditions are associated with mutism? A) alcohol withdrawal B) conversion neurosis C) catatonic schizophrenia D) depression E) Ganser's syndrome PSY-263. Echolaha is characteristic for: A) catatonic schizophrenia B) anorexia nervosa C) Alzheimer's disease D) infantile autism E) petit mal epilepsy PSY-264. Characteristic symptoms of schizophrenia include: A) compulsive thoughts B) progressive dementia C) depersonalization D) waking up early in the morning E) thought withdrawal PSY-265. Which of the following symptoms or findings suggest a poor prognosis in acute schizophrenic psychosis? A) an IQ above the average B) flatness of affect C) an abrupt onset D) a normal premorbid personality E) marked thought disorder PSY-267. Which of the following statements is characteristic for tardive dyskinesia? A) recent phenothiazine therapy is usually found in the history B) the intramuscular injection of benztropine rapidly relieves the DrRognerud@www.medfever.com

PSYCHIATRY

Answer: E

Answer: BCD

Answer: ACD

Answer: CE

Answer: BE

Answer: CE

52 / 343

Board Exam Question (Smiley + Star) + 2007 symptoms C) grimacing is typical D) intentional tremor is diagnostic for the condition E) the administration of phenothiazine may precipitate the attack PSY-268. Grandious delusions may occur in which of the following conditions? A) schizophrenia B) frontal lobe tumor C) manic syndrome D) compulsive neurosis E) amphetamine intoxication PSY-269. Characteristic symptoms of acute manic psychosis include: A) lack of insight B) flight of ideas C) confabulation D) distractibility E) depression in the family history PSY-270. Characteristic symptoms of depression include: A) diminished concentration B) hallucinations C) hypochondriasis D) delusions of persecution E) weight loss

PSYCHIATRY

Answer: ABCE

Answer: BDE

Answer: ACE

DrRognerud@www.medfever.com

53 / 343

Board Exam Question (Smiley + Star) + 2007

PEDIATRICS

PEDIATRICS
(139) SINGLE CHOICE QUESTIONS Select the single best response to each of the following questions!!! PED-2. All of the following statements regarding amoebiasis (Entamoeba hystolitica infection) are correct, EXCEPT: A) the disease is transmitted by the ingestion of cysts B) digestive enzymes release trophozoites from the cysts C) bacteria of the intestinal flora also contribute to the intestinal damage caused by the trophozoits D) the intestinal lesions are superficial E) hepatic abscesses area common complication PED-3. All of the following statements regarding maple syrup urine disease are correct, EXCEPT: A) newborns are symptomless at birth B) early symptoms include difficulties of feeding, irregular respiration and a weak Moro reflex C) spasms are rarely seen D) the time of the onset of symptoms is the third to fifth day after birth E) the disease is associated with a branched-chain ketoaciduria PED-6. All of the following statements concerning congenital adrenal hyperplasia are correct, EXCEPT: A) cortisol production is elevated in the second week following birth B) the overproduction of androgens causes masculinization of the external genitals in females C) the acute sodium deficiency adrenal crisis is due to deficient aldosterone production D) these infants have a decreased appetite and somatic growth is impaired E) an excessive sodium deficiency causes severe water loss and dehydration PED-7. 7 All of the following statements about the adrenogenital syndrome are correct, EXCEPT: A) androgen hypersecretion in the adrenal medulla causes virilization and increased protein anabolism B) virilizing adrenal tumors are rarely palpable, but they do dislocate DrRognerud@www.medfever.com

Answer: D

Answer: C

Answer: A

Answer: C

54 / 343

Board Exam Question (Smiley + Star) + 2007 the kidney C) the urinary 17-KS levels are decreased D) virilizing adrenal tumors do not cause excessive cortisol production E) the skeletal muscles are well developed in both boys and girls PED-8. The most likely cause of goiter in a newborn is: A) the maternal consumption of goitrogenic substances (foods) B) congenital hypothyroidism C) a severe peroxidase defect D) an abnormality of the thyrolingual duct E) congenital thyroiditis PED-9. All of the following statements about heart sounds in children are correct, EXCEPT: A) a third heart sound is commonly detected during childhood B) the second heart sound is generated by the closing of the semilunar valves C) the intensity of the first heart sound decreases if the ejection fraction increases D) a fourth heart sound can only be heard if ventricular ejection is impeded E) the origin of normal heart sounds is obscure PED-10. Which of the following would differentiate most clearly between a venous hum and the murmur caused by a patent ductus arteriosus? A) the site of auscultation B) auscultation during systole and diastole C) a venous murmur is always soft D) altering the patient's head position diminishes or accentuates the intensity of the murmur E) the intensity of the murmur is accentuated during exercise PED-12. Rectal prolapse is a possible complication of all of the following conditions, EXCEPT: A) cystic fibrosis B) severe malnutrition C) whooping cough D) chronic dysentery E) enterobiasis PED-13. All of the following statements about congenital obstruction of the upper gastrointestinal tract are correct, EXCEPT: A) vomiting might become continuous even without feeding B) the obstruction is frequently associated with polyhydramnios C) in the initial phases of the obstruction, meconium can pass DrRognerud@www.medfever.com

PEDIATRICS

Answer: A

Answer: C

Answer: D

Answer: E

Answer: D

55 / 343

Board Exam Question (Smiley + Star) + 2007 D) the vomit is always stained with bile PED-21. The main cause of neonatal mortality is: A) septicemia B) bleeding C) pulmonary hyaline membrane disease with resorptional atelectasis D) bronchopneumonia E) cerebral malformation PED-25. Which of the following statements regarding hypertrophic pyloric stenosis is valid? A) the disease is manifested during the first few days of life B) metabolic acidosis is characteristic C) the disease is manifested during the first 4-6 weeks of life D) bilious vomit is a characteristic finding PED-26. Which of the following is a rare symptom of acute appendicitis? A) epigastric pain radiating to the right lower quadrant B) nausea, vomiting and moderate fever C) diarrhea D) leukocytosis (the white blood cell count is approximately 14,000 G/1) E) tenderness at McBurney's point PED-27. A transient prolongation of the PR interval on the ECG is detected in: A) acute rheumatic fever B) digitalis therapy C) massage of the carotid sinus D) all of the above E) none of the above PED-29. The most frequent cause of urinary tract infections in a young girl is: A) bacterla which ascend through the urethra and the lower urinary tract B) bacteria penetrating from adjacent organs via the lymphatic vessels C) direct extension from adjacent organs D) hematogenic dissemination PED-31. A cerebral abscess most freqently occurs in the bacterral meningitis that is caused by: A) Meningococcus B) Pneumococcus C) Streptococcus D) Haemophilus influenzae

PEDIATRICS

Answer: C

Answer: C

Answer: C

Answer: D

Answer: A

Answer: B

DrRognerud@www.medfever.com

56 / 343

Board Exam Question (Smiley + Star) + 2007 PED-32. Characteristics of hypertonic dehydration in infancy include all of the following, EXCEPT: A) a loss of consciousness B) less decreased skin turgor relative to the severity of the dehydration C) abdomen is lardaceous to touch D) serum sodium concentration is above 145 mmol/1 E) central nervous system complications , are relatively frequent PED-33. The inheritance patterns of the muscular dystrophies are markedly different. The inheritance of progressive muscular dystrophy is: A) sex-linked recessive B) autosomal dominant C) autosomal recessive D) sex-linked dominant E) autosomal recessive and sex-linked recessive PED-34. All of the following statements about a child with encopresis are correct, EXCEPT: A) the prevalence of encopresis in the general child population is 1-3% B) the prevalence of encopresis among children with psychiatric abnormalities is 5-7% C) the etiologic background is a sensitivity to gluten D) it is more common during daytime than in the night E) impactation of the stool and constipation are frequently associated, up to 80-95% of all cases PED-35. All of the following conditions can be diagnosed prenatally, EXCEPT: A) arginino-succinic acidemia B) a cystathionine synthase defect C) Hunter's syndrome D) the Lesch-Nyhan syndrome E) the Guillain-Barr syndrome PED-38. Bleeding, due to thrombocytopenia occurs if the platelet count is less than: A) 150,000 /ul B) 100,000 /ul C) 80,000 /ul D) 50,000 /ul E) 20,000 /ul PED-41. Possible causes of a fever of unknown origin include all of the following, EXCEPT: DrRognerud@www.medfever.com

PEDIATRICS

Answer: C

Answer: A

Answer: C

Answer: E

Answer: D

Answer: D

57 / 343

Board Exam Question (Smiley + Star) + 2007 A) pyelonephritis B) subacute bacterial endocarditis C) Salmonella enteritis D) hypothyroidism E) thyroiditis PED-43. In which case is the newborn considered live-born? A) if the body mass exceeds 750 g B) if the body mass at birth exceeds 500 g C) if the heart beats and the newborn breathes and moves D) if any of the life functions are detectable after birth PED-45. Which of the following age groups does the term 'post-neonatal mortality' refer to? A) 6-11 months B) 3-11 months C) 28-364 days D) 7-364 days PED-47. The number of the human haploid chromosomes is: A) 22 B) 23 C) 44 D) 46 E) 48 PED-48. Case Study: The parents of a child who is suffering from a metabolic disease, characterized by an autosomal recessive inheritance pattern, ask for your advice. They intend to have a second baby. Which of the following considerations is INCORRECT? A) the phenotype of 75% of the potential offspring will be normal B) the genotype of 25% of the potential offspring will be normal C) the genotype of 50% of the potential offspring will be normal D) 50% of the offspring will be heterozygous PED-49. All of the following statements concerning a sex-linked recessive inheritance pattern are correct, EXCEPT: A) the disease only affects boys B) the father of the diseased sons might be affected C) only girls can be carriers D) the diseased boys might have diseased grandfathers E) not all of the daughters of a carrier mother will be carriers

PEDIATRICS

Answer: D

Answer: C

Answer: B

Answer: C

Answer: B

DrRognerud@www.medfever.com

58 / 343

Board Exam Question (Smiley + Star) + 2007 PED-56. Case Study: A 2-week-old infant is brought to your office. Two days ago the baby vomited. The baby also has diarrhea. The mother tells you that the infant's Appetite is good, and he voids large amounts of urine (the nappy is always wet). On observation the infant is alert. What is the most likely diagnosis? A) sepsis B) gastroenteritis C) intestinal malrotation D) volvulus of the small intestine E) overfeeding PED-57. How much is the energy requirement of a 4-week-old, normally developing infant? A) 100-150 kcal/kg/day B) 200-250 kcal/kg/day C) 300-350 kcal/kg/day PED-58. Case Study: A breast-fed baby is developing well. The baby's mother has no problem with lactation. At what age would you introduce vegetable puree to the diet? A) at the age of 6 weeks. B) at the age of 6 months C) at the age of 9 months D) at the age of 11 months PED-61. Case Study: The mother of a 2-week-old baby has no or very little breast milk. Which of the following preparations would you tell the mother to use? A) pasteurized mother's milk B) 50% diluted cow's milk C) Isomil (soy-based formula) PED-62. What is the protein requirement of a 2-month-old infant? A) 1 g/kg/day B) 2.5 g/kg/day C) 10 g/kg/day D) 4.5 g/kg/day PED-64. How much is the daily vitamin D requirement of a 6-month-old healthy infant on an average mixed diet? A) 2.5 .g/day B) 10 .g/day C) 5 .g/day DrRognerud@www.medfever.com Answer: E

PEDIATRICS

Answer: A

Answer: B

Answer: A

Answer: B

Answer: B

59 / 343

Board Exam Question (Smiley + Star) + 2007 D) 15 .g/day PED-74. Case Study: A 4-month-old baby boy, born at term with 3,400 g birth weight, currently weighs 6,000 g. What is your opinion? A) the body weight of the infant is normal B) the infant is overweight C) the infant's nourishment is deficient PED-77. The most common cause of obesity in childhood is: A) adrenal cortex hyperfunction B) hypothyroidism C) corticosteroid therapy D) overeating, the lack of physical activity E) hyperthyroidism F) adiposogenital dystrophy (Fr.hlich's syndrome) PED-83. What is the optimal initial therapy in case of moderately severe dehydration? A) the transfusion of whole blood B) the transfusion of packed red blood cells C) the infusion of plasma D) the infusion of 0.9% sodium chloride solution E) the infusion of half-isotonic sodium chloride-dextrose solution F) the infusion of 10% dextrose solution G) the infusion of Rheomacrodex PED-84. Case Study: A 12-month-old infant is admitted to the department with complaints of diarrhea and exsiccosis for the last three days. During a general examination a week before admission the infant was found healthy and weighed 10 kg. For the last few days the body temperature has been 39C and the baby had stool 10-12 times daily. The infant did not void urine during the last 18 hours. Current examination reveals dry skin with markedly decreased turgor, and hollow eyes. What is to be done first? A) hemoculture; a complete and differential blood cell count B) urinalysis: cultivation, electrolytes and specific gravity C) blood is drawn for the determination of ions and urea nitrogen; 20 ml/kg half-isotonic sodium chloride-dextrose solution is administered intravenously, and a continuous infusion is started D) bacteriology, a reduction test and a determination of the fat content of the stool PED-85. All of the following statements concerning the clinical symptoms of the DrRognerud@www.medfever.com

PEDIATRICS

Answer: A

Answer: D

Answer: E

Answer: C

Answer: C 60 / 343

Board Exam Question (Smiley + Star) + 2007 conditions characterized by fluid and electrolyte loss are correct, EXCEPT: A) the turgor is decreased, the extremities are cold and the heart rate is elevated in a moderately severe isotonic dehydration B) loss of consciousness develops in severe hyponatremic dehydration C) marked lethargy develops in a moderately severe hypernatremic dehydration D) abdominal distension and muscular weakness develop in acute hypokalemia E) during prolonged vomiting the concentrating capacity of the kidney decreases, polyuria develops and peripheral paresis may occur PED-86. All of the following statements concerning isotonic dehydration are correct, EXCEPT: A) 80% of the dehydration states developing during childhood are isotonic B) possible causes include diarrhea, the sequestration of fluid in ileus, excessive sweating, edema formation C) the heart rate increases, the blood pressure increases, turgor is decreased, the extremities are cool D) the application of an oral rehydration solution is suitable in mild cases E) the rate of the infusion of fluid is constant from the beginning F) 2% glucose in an oral rehydration solution is optimal for the absorption of sodium PED-89. All of the following conditions may be associated with hypokalemia, EXCEPT: A) hyperaldosteronism (Conn's syndrome) B) post-acidotic syndrome C) Bartter's syndrome D) acute renal failure PED-100. All of the following statements concerning the newborn of a diabetic mother are correct, EXCEPT: A) most of these newborns, are overweight relative to the gestational age B) hypoglycemia, developing soon after birth, is common C) tachypnea frequently occurs D) the prevalence of hyaline membrane disease among these infants is the same as in a control group of the same gestational age PED-101. Which newborn is the least prone to hypoglycemia? A) a premature newborn with a low body weight relative to the gestational age B) a premature newborn with normal body weight relative to the gestational age DrRognerud@www.medfever.com

PEDIATRICS

Answer: E

Answer: D

Answer: D

Answer: B

61 / 343

Board Exam Question (Smiley + Star) + 2007 C) a neonate born at term, with intrauterine retardation D) the newborn of a diabetic mother PED-103. All of the following statements concerning pneumothorax in a neonate are correct, EXCEPT: A) the major causes are neonatal hypoxia and the respiration therapy indicated in hypoxia B) in case of valvular pneumothorax, the lung on the affected side is compressed and shock may develop C) transillumination is an important step in the physical examination D) any respiration therapy should be discontinued immediately after the detection of a pneumothorax E) the application of continuous aspiration is necessary in case of a severe pneumothorax PED-107. All of the following statements concerning IRDS (hyaline membrane disease) are correct, EXCEPT: A) it is manifested within a few hours following birth B) tachypnea, jugular and intercostal retraction are observed C) the observed expiratory grunting is a result of the closing of the glottis, which aims to keep the alveoli open D) the development of a pneumo-bronchogram on the chest x-ray verifies IRDS E) during the third stage, the diaphragm-lung and the heart-lung borders are obscure on the chest x-ray PED-109. All of the following statements concerning pneumonia of a newborn are correct, EXCEPT: A) it may develop during the fetal period, during delivery, or postnatally B) an early rupture of the amnion plus maternal infections predispose this condition C) a group B streptococcus infection is a frequent cause D) symptoms of the condition are similar to those in IRDS E) an elevated or, occasionally, very low white blood cell count, associated with a low platelet count are indicative of pneumonia, rather than IRDS F) a strong elevation of the serum IgG level is indicative for a connatal pneumonia PED-111. Possible causes of the development of pathologic hemolysis associated with hyperbilimbinemia in the newborn include all of the following, EXCEPT: A) bacterial or viral sepsis B) erythroblastosis C) ABO incompatibility D) vitamin K deficiency DrRognerud@www.medfever.com

PEDIATRICS

Answer: D

Answer: D

Answer: F

Answer: D

62 / 343

Board Exam Question (Smiley + Star) + 2007 E) red cell enzyme abnormalities F) congenital abnormalities of the red blood cells PED-112. All of the following statements concerning the anemia of neonatal hemolysic disease are correct, EXCEPT: A) the lower limit of the reference range of the hemoglobin concentration of a newborn during the first days is 8.5 mmol/1(14 g/dl) B) the lower limit of the reference range of the hemoglobin concentration of a 6-week-old infant is 6.5 mmol/1(10 g/dl) C) the late anemia caused by an incompatibility develops 5-6 weeks after the blood exchange D) during blood exchange irregular antibodies reach the organism, this is the cause of the late anemia PED-116. Meconium ileus is a possible early symptom of: A) pyloric stenosis B) Hirschsprung's disease C) cystic fibrosis D) intestinal perforation PED-117. Which of the following is the most likely cause of a bilious vomit of a newborn? A) esophageal atresia B) pyloric stenosis C) achalasia D) volvulus of the small intestine PED-122. Case Study: A 6-hour-old newborn exhibits tachypnea and dyspnea and vomits repeatedly. The body temperature of the newborn is normal. The physical examination reveals a tympanic resonance over the left side of the chest-, no respiratory sounds are audible over this side. The surface of the abdomen is concave. What is the most likely cause of this condition? A) IRDS B) neonatal sepsis C) left-sided pneumonia D) a diaphragmatic hernia on the left side E) pneumothorax F) congenital valvular disease PED-124. Indications of neonatal antibiotic prophylaxis include all of the following, EXCEPT: A) a peripartal maternal infection B) an early rupture of the amnion C) a fetid amniotic fluid DrRognerud@www.medfever.com

PEDIATRICS

Answer: D

Answer: C

Answer: D

Answer: D

Answer: E

63 / 343

Board Exam Question (Smiley + Star) + 2007 D) a prolonged and difficult delivery E) a blood transfusion F) a blood exchange G) catheterization of the umbilical vessels PED-127. All of the following statements concerning necrotizing enterocolitis (NEC) are correct, EXCEPT: A) the disease is primarily manifested between the ages of 6-12 months B) breastfeeding increases the resistance for developing NEC C) an intestinal pneumatosis is of diagnostic value D) bloody stool is observed in the majority of cases E) in case of the failure of conservative therapy, an operation is indicated PED-128. The occurrence of which of the following symptoms is the least likely to occur in neonatal sepsis? A) fever B) the refusal of food C) jaundice D) lethargy E) irritability PED-131. What is the most likely acid-base abnormality in congenital hypertrophic pyloric stenosis? A) respiratory alkalosis B) none, as these is a normal acid-base status C) metabolic acidosis D) metabolic alkalosis E) respiratory acidosis PED-132. Which of the following statements concerning the therapy of congenital hypertrophic pyloric stenosis is correct? A) a pyloromyotomy is indicated immediately B) after normalization of the acid-base and electrolyte balance, a pyloromyotomy has to be performed in each case C) gradually increased doses of atropine are administered to relieve pyloric spasm D) with frequent feeding and low quantities of food the newborn survives the critical period, the pyloric passage inreases with time PED-133. Case Study: A 3-month-old infant is brought to you with a 5-week history of diarrhea. No microorganism has been demonstrated by fecal bacteriology. Furthermore, the administration of several antibacterial drugs failed to normalize the condition. The infant has been on an average diet containing mixed food since the age of 6 weeks. The possible causes of this condition include all of the following, EXCEPT: DrRognerud@www.medfever.com

PEDIATRICS

Answer: A

Answer: A

Answer: D

Answer: B

Answer: E

64 / 343

Board Exam Question (Smiley + Star) + 2007 A) cow s milk protein intolerance B) a lactase deficiency following a bacterial diarrhea C) the so-called infantile, intractable diarrhea of unknown origin D) a change of the GI flora caused by the administration of antibiotics E) a diarrhea caused by a viral infection PED-134. Which of the following findings would support the diagnosis of a suspected mucoviscidosis (cystic fibrosis)? A) a decreased absorption of xylose B) a decreased vital capacity C) a decreased biliary pigment content of the duodenal juice D) steatorrhea E) the chloride concentration of the sweat is above 60 mmol/1 F) cor pulmonale PED-135. Which of the following statements concerning celiac disease is correct? A) an intestinal biopsy revealing subtotal atrophy of the villi in a patient on an average diet proves celiac disease B) a gluten-free diet is introduced following the detection of subtotal atrophy of the villi, and a biopsy is repeated 2 months following the relief of symptoms; if the histology is still abnormal, then celiac disease is excluded C) a xylose absorption test is necessary for the diagnosis D) the diagnosis is only reliable if the biopsy is repeated 1 year after the restriction of gluten and the histology is normal and then, 6 months after the re-introduction of gluten, the biopsy preparation reveals the characteristic abnormalities again PED-138. Which of the following tests is the most informative in the recognition of appendicitis in childhood? A) the white blood cell count and the red blood cell sedimentation rate B) a native abdominal x-ray C) an abdominal ultrasound D) the repeated physical examination of the abdomen E) a rectal digital examination PED-142. All of the following are possible causes of mechanical ileus, EXCEPT: A) meconium plug B) volvulus C) invagination D) Meckel's diverticulum E) ulcerative colitis PED-143. Recognized causes of paralytic ileus include all of the following, EXCEPT: DrRognerud@www.medfever.com

PEDIATRICS

Answer: E

Answer: D

Answer: D

Answer: E

Answer: E 65 / 343

Board Exam Question (Smiley + Star) + 2007 A) an abdominal operation B) a toxic Infection C) renal colic D) perforation peritonitis E) intestinal helminthiasis F) pneumonia G) hypokalemia PED-154. All of the following conditions are associated with a systolic murmur, EXCEPT: A) anemia B) hyprthyroidism C) mitral incompetence D) aortic incompetence E) ventricular septal defect F) fever PED-156. If the vascular tracings of the lungs are decreased on a chest x-ray of a cyanotic newborn, which of the following congenital valvular diseases is a possible cause? A) complete transposition of the great vessels B) complete transposition of the pulmonary veins C) pulmonary atresia PED-160. All of the following statements concerning rheumatic polyarthritis are correct, EXCEPT: A) it usually affects the great joints B) the affected joints are swollen, warm, and movements are painful C) it is associated with a high fever D) joint deformities develop during healing if the appropriate treatment is not introduced in time E) the hip joint may also be affected PED-169. All of the following statements concerning tachycardias are correct, EXCEPT: A) possible causes of sinus tachycardia include fever, hyperthyroidism and the consumption of caffeine B) possible causes of acute supraventricular tachycardia include fever, carditis and ephedrine administration C) chronic supraventricular paroxysmal tachycardia is always associated with valvular disease D) atrial fibrillation is usually associated with heart disease E) possible causes of ventricular fibrillation include digitalis toxicity and cardiomyopathy

PEDIATRICS

Answer: D

Answer: C

Answer: D

Answer: C

DrRognerud@www.medfever.com

66 / 343

Board Exam Question (Smiley + Star) + 2007 PED-171. All of the following statements concerning bradycardias are correct, EXCEPT: A) the possible causes of acute bradycardia include an atrioventricular block due to hypokalemia, an increased intracranial pressure, an increased vagal tone, hypoxia and hypothermia B) the possible causes of chronic bradycardia include beta-blocker therapy, previous cardac surgery and regular sporting activity C) a Mobitz II type second degree AV-block usually develops in a previously healthy heart D) connatal atrioventricular block is a possible cause of connatal bradycardia E) interventions used for the management of bradycardia include the administration of atropine, ephedrine, isoprenaline and pacemaker-therapy PED-181. All of the following statements about retropharyngeal abscesses are correct, EXCEPT: A) they occur as a complication of a purulent pharyngitis B) they inhibit swallowing C) they might cause laryngeal edema D) the position of the head is similar to that observed in meningism E) it is advisable to wait for the spontaneous opening of these abscesses instead of performing an operation F) laryngoscopy is indispensable for the diagnosis G) a painful swelling of the lymph nodes is detected on both sides of the mandible PED-185. Case Study: The family history of a 2-year-old child reveals, like both his parents, Allergic rhinitis caused by ragweed. The child also has had pseudocroup (thymic asthma) twice. Is an allergological examination indicated in this case? A) yes, because there is a positive family history for allergy B) no, because there is no relationship between ragweed allergy and pseudocroup, plus sensitivity for other allergens cannot be usually demonstrated C) yes, because pseudocroup is a forerunner of bronchial asthma D) yes, because desensitization with the demonstrated allergen can be used to prevent the development of a pseudocroup PED-201. Which of the following statements about interstitial plasmocytic pneumonia is correct? A) bronchial respiratory sounds are heard over the lung fields B) crepitations are heard over the entire lung C) it is manifested in newborns with a low birth weight, at the age of 3-8 weeks D) the onset is after the age of 4 months DrRognerud@www.medfever.com Answer: C

PEDIATRICS

Answer: E

Answer: B

Answer: C

67 / 343

Board Exam Question (Smiley + Star) + 2007 E) the patients exhibit a hoarse cough PED-204. Symptoms of bronchiectasis include all of the following, EXCEPT: A) the recurrence of pneumonia affecting the same area B) voluminous secretion of the airways C) a loss of appetite-, growth retardation D) cyanosis due to the circulatory abnormality E) clubbing of the fingers PED-205. All of the following statements about the diagnosis and the therapy of bronchiectasis are correct, EXCEPT: A) bronchoscopy and bronchograpy are necessary for the precise diagnosis B) a postural drainage, nursing home care and the vigorous treatment of infections is indicated in mild cases C) an associated chronic sinusitis must be cured D) the regular administration of gamma-globulin is important in all cases E) in cases when bronchiectasis affects only 1-2 lobes or in prolonged cases a lobectomy is indicated PED-207. Which of the following immunoglobulin classes is able to penetrate the placenta? A) IgG B) IgM C) IgA D) IgE E) IgD PED-210. All of the following interventions cause an iatrogenic immune defect, EXCEPT: A) prolonged treatment with corticosteroids B) cytostatic therapy C) BCG vaccination D) splenectomy PED-217. Case Study: An 18-month-old infant is brought to your office. The parents tell you that the infant has always been pale; the infant's appetite is changing; and they report recurrent upper respiratory infections. The laboratory results are as follows: hemoglobin: 4.7 mmol/1; hematocrit: 25%; serum iron: 6 .mol/1; total iron binding capacity (TIBC): 78 .mol/1; reticulocyte count: 3%o; platelet count: 200 G/1. What is the most likely diagnosis? A) minor beta thalassemia DrRognerud@www.medfever.com

PEDIATRICS

Answer: D

Answer: D

Answer: A

Answer: C

Answer: C

68 / 343

Board Exam Question (Smiley + Star) + 2007 B) aplastic crisis of a hemolytic anemia C) iron deficiency anemia D) initial phase of a malignancy E) sideroachrestic anemia PED-223. Case Study: Which of the following is the most important therapeutic intervention in familial spherocytosis of a 6-year-old child, who frequently becomes anemic? A) a transfusion of packed red blood cells B) a splenectomy C) the prolonged administration of steroids D) immunosuppressive therapy E) iron replacement therapy PED-231. Case Study: You are examining an 18-month-old boy. The parents have noted the occurrence of large hematomas on the skin following mild traumas during the last6 months. The results of the laboratory tests are not yet ready, when theparents inform you that the child's finger started to bleed again, two hoursafter the puncture. What is the most likely diagnosis? A) thrombocytopenia B) von Willebrand's disease C) hemophilia D) vasculopathy E) thrombocytopathy PED-236. Case Study: An 8-year-old, febrile child is admitted to the pediatric department. Large lymph nodes are palpable on both sides of the neck. The examination reveals pharyngitis and hepatosplenomegaly. No symptoms of anemia or bleeding are observed. Atypical mononuclear cells are seen in the peripheral blood smear. What is the most likely diagnosis? A) leukemia B) toxoplasmosis C) lymphoma D) infectious mononucleosis E) cytomegalovirus infection PED-251. When is it recommended to start the therapy of congenital hypothyroidism? A) immediately after ablactation, when the infant does not receive thyroid hormone with the milk any longer B) when the serum TSH level starts to elevate C) as soon as possible, having received the result of the neonatal screening test, because any hesitation could markedly worsen the prognosis DrRognerud@www.medfever.com

PEDIATRICS

Answer: B

Answer: C

Answer: D

Answer: C

69 / 343

Board Exam Question (Smiley + Star) + 2007 D) if the result of the neonatal screening test is positive and the heart rate is less than 80/min PED-259. Case Study: A treated diabetic child loses consciousness. Glucose is found in the urine. What are your considerations before any further interventions? A) the possibility of hypoglycemia is considered unlikely B) the patient is likely to have a hyperosmolar coma because the respiration is not acidotic C) the blood glucose level should be checked with a finger stick and attempts should be made to clarify the antecendents of the attack PED-260. Case Study: A 13-year-old girl with treated diabetes is brought to you because of a loss of consciousness. Which of the following should you check immediately? A) if a respiratory abnormality is present, or if skin is cold and dry B) if the patient received an insulin dose in the normal time C) if the patient missed a meal D) if an additional acute disease is present E) if the patient has experienced emotional stress F) if the patient experienced some type of head trauma G) if some type of drug intoxication is present H) all of the above PED-261. Case Study: A 13-year-old girl with treated diabetes is brought to you because of a loss of consciousness. The time of the last insulin injection is unknown. Her skin is dry; respiration is more frequent and deeper. Which of the following do you recommend? A) the girl should be taken home and the prescribed insulin dose administered B) to measure her blood glucose and having the result, decide to refer the patient to the hospital or release her home C) refer the patient to a hospital PED-264. What has to be done in case of recurrent pyuria? A) specific drug therapy B) morphology and function of the kidneys plus the lower and upper urinary tract should be examined C) a concentration test; followed by the examinations in point (B) above PED-266. All of the following statements concerning acute poststreptococcal glomerulonephritis are correct, EXCEPT: DrRognerud@www.medfever.com

PEDIATRICS

Answer: C

Answer: H

Answer: C

Answer: B

Answer: B

70 / 343

Board Exam Question (Smiley + Star) + 2007 A) it can be prevented with the early antibiotic therapy of streptococcal infections of the upper respiratory tract and the skin B) exacerbation of an acute glomerulonephritis is frequent in the course of a chronic nephritis C) the prognosis of the disease in childhood is usually good D) a hypertensive encephalopathy may be the first symptom of the disease PED-267. Possible complications of acute poststreptococcal glomerulonephritis include all of the following, EXCEPT: A) hyperkalemia B) hypermagnesiemia C) encephalopathy D) pulmonary edema E) anuria PED-270. All of the following statements concerning idiopathic nephrotic syndrome are correct, EXCEPT: A) the development of edema is caused by hypoproteinemia B) an increased reabsorption of sodium is an additional cause for the edema formation C) diarrhea is a possible complication of the condition D) serum lipid levels are modestly decreased PED-271. All of the following statements concerning idiopathic nephrotic syndrome are correct, EXCEPT: A) the onset is usually between 1-6 years of age B) the histology reveals a "minimal change" process C) the early development of renal failure is characteristic D) patients usually do not exhibit hypertension E) hyperlipidemia is usually present PED-272. Which of the following drugs is used first during the therapy of the "minimal change" form of the nephrotic syndrome of childhood? A) prednisone B) chlorambucil C) cyclophosphamide D) prednisone + chlorambucil E) prednisone + cyclophosphamide PED-277. Which of the following statements concerrning hypercalciuria is FALSE? A) primary hyperparathyroidism causes hypercalciuria B) idiopathic hypercalciuria is a frequent cause of hematuria C) in the renal form, even if the calcium content of the diet is low, hypercalciuria can be detected DrRognerud@www.medfever.com

PEDIATRICS

Answer: B

Answer: D

Answer: C

Answer: A

Answer: E

71 / 343

Board Exam Question (Smiley + Star) + 2007 D) a low calcium content diet diminishes the absorptive form E) the most efficient therapy is the administration of a high dose of vitamin D3 F) hydrochlorothiazide (Hypothiazid) can decrease calcium excretion in renal hypercalciuria PED-304. What is the most important therapeutic step in a strongly suspected diphtheria if diphtheria antitoxin is not detectable in the serum? A) antibiotic administration B) corticosteroid administration C) diphtheria antitoxin administration D) high dose vitamin B1supplementation PED-306. Specify the pathogenic agent of the epidemic form of scarlet fever: A) Clostridium difficile B) Staphylococcus aureus C) Streptococcus pneumoniae D) Branhamella catarrhalis E) Streptococcus pyogenes F) Staphylococcus epidermidis PED-307. Possible complications of scarlet fever include all of the following, EXCEPT: A) submandibular lymphadenitis B) otitis media C) subacute sclerosing panencephalitis D) acute glomerulonephritis E) peritonsillar abscess F) rheumatic fever PED-308. Possible complications of measles include all of the following, EXCEPT: A) conjunctivitis B) rhinitis C) tracheobronchitis D) lamellar desquamation E) Koplile's spots F) maculous eruptions PED-310. Specify the type of eruption characteristic for measles: A) vesiculo-pustulous eruptions B) confluent maculous eruptions C) confluent eruptions consisting of punctual elements D) erythema annulare E) eruptions consisting of isolated maculo-papulous elements F) sharply delineated, diffuse erythema

PEDIATRICS

Answer: C

Answer: E

Answer: C

Answer: D

Answer: B

DrRognerud@www.medfever.com

72 / 343

Board Exam Question (Smiley + Star) + 2007 PED-323. Which of the following statements about toxoplasmosis is correct? A) if the newborn of a mother has congenital toxoplasmosis, the next child of the mother is very likely to have the same disease B) maternal infection during the first trimester is less dangerous than in the third trimester C) a toxoplasma infection during pregnancy is usually symptomatic D) the majority of acute maternal diseases do not cause congenital infection E) aquired toxoplasmosis causes cerebral paresis PED-338. What is the correct sequence of the steps of resuscitation? A) providing open airways; cardiac massage; administration of drugs B) mouth to mouth breathing; cardiac massage; administration of drugs and infusions; ECG control C) providing open airways; mouth to mouth breathing; cardiac massage; administration of drugs and infusions; ECG control D) cardiac massage; mouth to mouth breathing, drug administration PED-339. Which is the most straight-forward method to ensure ventilation of the lung in a state of clinical death? A) the administration of oxygen via a nasal-pharyngeal tube B) thoracic compression C) following aspiration of the airways, the simultaneous application of mouth to mouth breathing and cardiac massage D) the administration of drugs which stimulate respiration and cardiac function E) artificial maintenance of the circulation PED-345. Possible causes of anaphylaxic shock include all of the following, EXCEPT: A) penicillin therapy B) heterologous sera C) a wasp bite D) the intracutaneous administration of Tuberculin E) the intravenous administration of contrast media during an xray examination PED-346. The therapy of anaphylaxic shock includes all of the following steps, EXCEPT: A) epinephrine (Tonogen), sc. oriv. B) corticosteroid iv. C) Rheomacrodex infusion D) aminophylline E) beta-blockers F) norepinephrine DrRognerud@www.medfever.com

PEDIATRICS

Answer: B

Answer: C

Answer: C

Answer: D

Answer: E

73 / 343

Board Exam Question (Smiley + Star) + 2007 G) oxygen PED-355. All of the following statements concerning ethmoiditis are correct, EXCEPT: A) its most frequent cause is a Staphylococcus aureus infection B) the disease is relatively mild C) it is associated with palpebral edema, flushing of the face and fever D) it might involve the orbit E) in the initial phase, it might be misdiagnosed as conjunctivitis PED-359. What should be done to a coughing child with a suspected foreign body in the airways? A) the child should be held upside down by the feet until any existing foreign body drops out B) cough suppressants and antibiotics should be administered, if the cough is relieved, then the foreign body has been excluded C) cough relief, sedation, observation; if the cough persists, a chest x-ray is necessary D) a chest x-ray, as soon as possible, to visualize the foreign body E) following the emegency physical examination, the child must be sent to a specialized department where the pproper diagnosis and appropri ate care can be undertaken PED-360. What is the therapy of an acute purulent otitis media associated with fever in infancy? A) myringotomy, administration of antibiotics, and nasal drops B) a poultice should be applied to relieve the pain C) antibiotic ear drops D) infrared exposure of the ear, nasal drops, and the relief of fever E) to prevent complications, hospital admission is advisable in each case, F) otitis media is a secondary disease, only the underlying disease has to be treated PED-364. All of the following statements concerning superficial fungal infections are correct, EXCEPT: A) microsporiasis and trichophytiasis are superficial fungal infections B) Microsporium and Trichophyton cause red, scaly and round lesions C) if the hair or nails are involved, griseofulvin is the treatment of choice D) tinea pedis occurs most often in prepuberal children PED-372. Case Study: A 3-year-old boy is brought to your office by his mother. The mother DrRognerud@www.medfever.com

PEDIATRICS

Answer: B

Answer: E

Answer: A

Answer: D

Answer: A

74 / 343

Board Exam Question (Smiley + Star) + 2007 says that the child developed a soar throat and fever every 3rd-4th week ever since he has attended nursery school. The physical examination revealed modestly enlarged tonsils. What should be done? A) cultivation of the pharyngeal discharge; if it is negative, the mother should be reassured that such symptoms commonly develop during the first few months of attending a nursery school B) order a complete laboratory check-up C) a gamma globulin injection should be given regularly, (once a month) D) penicillin tablets should be given for 10 days as a prophylaxis E) a tonsillectomy is indicated F) an extensive immunological examination is indicated PED-376. Herpetic gingivostomatitis is not associated with: A) fever B) swelling of the gingiva C) lymphadenopathy D) ulceration of the oral mucosa E) the development of papulous eruptions PED-392. Case Study: The physical examination of a newborn reveals respiratory insufficiency, repleted cervical veins, low blood pressure, tympanic resonance on one side of the chest, weak respiratory sounds and subcutaneous emphysema on the same side. The most likely diagnosis is: A) hyaline membrane disease B) staphylococcal pneumonia C) pneumothorax and pneumomediastinum D) primary atelectasis E) diaphragmatic hernia PED-400. Case Study: A 1-year-old girl cannot sit down without assistance. She is able to take hold of objects with one hand but is unable to put them into the other hand. She cannot climb or stand up. She can pronounce a few words, but shows no interest for pictures. She does not respond very well to external stimuli. What is your opinion about the somato-mental maturity of the child? A) severe mental retardation B) her maturity corresponds to the age of 11 months C) her somatic maturity corresponds to the age of 6-8 months; a moderate mental retardation is detected D) her maturity corresponds to her age-group; no abnormality is detected PED-404. Which of the following is an absolute contraindication of breastfeeding? A) erythroblastosis fetalis DrRognerud@www.medfever.com

PEDIATRICS

Answer: E

Answer: E

Answer: C

Answer: E

75 / 343

Board Exam Question (Smiley + Star) + 2007 B) crater nipple C) mastitis D) smoking E) phenylketonuria PED-408. The clinical symptoms of rickets include all of the following, EXCEPT: A) craniotabes B) "rachitic rosary" C) swelling of the wrists and ankles D) slow development of the motor system E) conjunctivitis PED-409. The daily vitamin D requirement of a developing infant is: A) 100 IU B) 400 IU C) 600 IU D) 1,000 IU E) 50 IU

PEDIATRICS

Answer: E

Answer: B

MULTIPLE CHOICE QUESTIONS WITH KEY ANSWERS / TYPE II Every question or incomplete statement has only one answer in the following combinations: A) if the answers 1, 2, and 3 are true B) if the answers 1 and 3 are true C) if the answers 2 and 4 are true D) if only the answer 4 is true E) if all the four answers are true Select one of these key combinations!!! PED-419. It is important to realize that the technique of resuscitation is different in children and adult patients. Which of the following considerations concerning the anatomy of the infant or child may be important during endotracheal intubation? 1) the epiglottis is shorter in the infant, and it has a U shape, whereas in the adult it is longer and flat 2) the larynx is situated more anteriorly and cranially in infants and children than in the adult 3) the angle of the epiglottis and the vocal cords is more sharp in infants and children 4) the position of the heart is considerably higher in infants (it is usually behind the mid-sternum), whereas in the adult it is found behind the lower third of the sternum PED-420. Which of the following statements about the indications of operation in congenital heart diseases are correct? 1) in the Tetralogy of Fallot an operation is indicated in each case DrRognerud@www.medfever.com

Answer: A

Answer: A

76 / 343

Board Exam Question (Smiley + Star) + 2007 2) the most suitable time for the surgical correction of tricuspidal atresia is after the age of 10 years 3) the optimal time for the correction of coarctation of the aorta is 2-5 years of age, except for cases with "critically severe coarctation of the aorta" 4) the optimal time for the surgery of a large ventricular septal defect is at the age of 8-10 years PED-426. Which of the following statements are correct concerning growth during the first year of life? 1) by the end of the 6th month, the body weight is twice as large, by the end of the first year the body weight is three times that of the birth weight 2) the body height increases by approximately 25 cm 3) the head circumference increases by 12 cm 4) most mature infants regain their birth weight by the 2nd week PED-428. Which of the following drugs can be administered to the mother during lactation, without any risk? 1) digitalis 2) antithyroid drugs 3) insulin 4) most of the drugs used for the treatment of malignancies PED-439. Which of the following statements relate to acute poststreptococcal glomerulonephritis? 1) unexpectedly urine becomes dark, a mild edema and a decreased urine volume is detected 2) proper management of the streptococcal pharyngitis decreases the frequency of nephritis by half 3) a strong hematuria usually relieves during the first week, but microscopic hematuria may persist for two months 4) complete healing is predictable if the child survives the first period PED-440. Characteristics of Klinefelter's syndrome include: 1) delayed puberty 2) gynecomastia 3) mental retardation and psychic abnormalities 4) aortic stenosis

PEDIATRICS

Answer: E

Answer: B

Answer: E

Answer: A

CASE STUDIES Answer the multiple task questions (simple choice and multiple choice with/without key answers; relation analysis etc.) as they are related to each case study!!! PED-445. Case Study DrRognerud@www.medfever.com Answer: BCDE 77 / 343

Board Exam Question (Smiley + Star) + 2007 A 12-year-old girl's mother has repeatedly complained about her daughter's "recurrent infections". The attending family doctor reassures her that despite the various laboratory analyses, physical examinations etc. he has performed, he has found no evidence of any type of infections. He subsequently plans to perform allergological studies. 4.445/1. Histamine: 1) is a blocking antibody, it is formed upon desensitization 2) contributes to the development of anaphylaxis 3) contributes to the development of urticaria 4) causes angioedema 5) is the initial immune response to an infection A) (1), (4), and (5) are correct B) (2) and (3) are correct C) (1) and (5) are correct D) only (1) is correct E) (2) and (4) are correct F) (1) and (3) are correct G) all of the above H) none of the above 4.445/2. 1) is a blocking antibody, it is formed upon desensitization 2) contributes to the development of anaphylaxis 3) contributes to the development of urticaria 4) causes angioedema 5) is the initial immune response to an infection A) (1), (4), and (5) are correct B) (2) and (3) are correct C) (1) and (5) are correct D) only (1) is correct E) (2) and (4) are correct F) (1) and (3) are correct G) all of the above H) none of the above 4.445/3. 1) is a blocking antibody, it is formed upon desensitization 2) contributes to the development of anaphylaxis 3) contributes to the development of urticaria 4) causes angioedema 5) is the initial immune response to an infection A) (1), (4), and (5) are correct B) (2) and (3) are correct C) (1) and (5) are correct D) only (1) is correct E) (2) and (4) are correct F) (1) and (3) are correct G) all of the above H) none of the above 4.445/4. Bradykinin: 1) is a blocking antibody, it is formed upon desensitization 2) contributes to the development of anaphylaxis DrRognerud@www.medfever.com

PEDIATRICS

78 / 343

Board Exam Question (Smiley + Star) + 2007 3) contributes to the development of urticaria 4) causes angioedema 5) is the initial immune response to an infection A) (1), (4), and (5) are correct B) (2) and (3) are correct C) (1) and (5) are correct D) only (1) is correct E) (2) and (4) are correct F) (1) and (3) are correct G) all of the above H) none of the above PED-446. Case Study 4-year-old boy suddenly develops shaking chills and a high fever. He has a headache, he vomits repeatedly, and occipital stiffness is detected. Dermographism of the skin is increased and small pinhead size petechiae are observed on the entire body surface. A loss of consci ousness and circulatory failure develops within hours. 4.446/1. The presumable diagnosis is: A) mushroom poisoning B) organic solvent intoxication C) Waterhouse-Friderichsen syndrome (meningococcal meningitis and septicemia) D) encephalitis E) hemophilia 4.446/2. The most important test for the exact diagnosis is: A) examination of the gastric lavage fluid B) a skull x-ray C) a hemostasis evaluation D) examination of the cerebrospinal fluid 4.446/3. The appropriate therapy during the acute phase is: A) the administration of atropine B) a gastric lavage C) transportation to a hospital urgently; the administration of antibiotics based on the results of the cerebrospinal fluid tests D) the administration of fresh frozen plasma E) the relief of fever and observation in a hospital PED-447. Case Study A 4-year-old girl complained of abdominal pain the day before the examination. She vomited once during the night. Her face is pale and perioral cyanosis is seen. She has dyspnea and cough. Physical examination: the stool is normal and the abdomen is bloated. The liver exceeds the costal arch by 2 fingers. The spleen is not palpable. Respiratory rate: 4l/min; heart rate: 115/min; blood pressure: 100/60 DrRognerud@www.medfever.com

PEDIATRICS

Answer: CDC

Answer: CCBD

79 / 343

Board Exam Question (Smiley + Star) + 2007 mmHg; body temperature: 38.6C. The pharyngeal mucosa is moderately hyperemic. 1-2 "pea-sized" lymph nodes under the chin, and one "bean-sized" lymph node in the left inguinal region are palpable. Heart sounds are clear and normal. Percussion reveals dullness over an area of 10 cm in diameter below the right scapula. Loud, bronchial respiratory sounds are audible over this area. Diaphragmatic movements are normal. No meningeal symptoms are present. The child is weak and fatigued. The skin shows no alterations. 4.447/1. The diagnosis based on the physical examination is: A) right-sided pleuropneumonia B) influenza C) right-sided lobar pneumonia, with peritonitis as a complication D) acute lymphoblastic leukemia (ALL) E) acute appendicitis 4.447/2. All of the following supplementary tests are indicated, EXCEPT: A) the red blood cell sedimentation rate B) complete differential and blood cell counts C) examination of the vulvar smear D) a chest x-ray E) hepatic functional tests 4.447/3. The most likely causative microorganism of this affliction is: A) Staphylococcus aureus B) Streptococcus pneumoniae C) adenovirus D) cytomegalovirus E) Epstein-Barr virus 4.447/4. Which therapy would you choose first? A) thoracocentesis B) chloramphenicol (Chlorocid) C) amidazophen D) ampicillin E) penicillin (Maripen) PED-449. Case Study An 8-year-old boy, two weeks after developing pharyngitis, develops palpebral edema. He also complains of headaches and vertigo. 4.449/1. Which of the following questions should be asked from the parents of the patient? 1) Did the child suffer from enuresis? 2) Did the child complain of tingling micturition? 3) Did they note any smoke-colored urine? 4) Did the urine volume increase? 5) Did the urine volume decrease? A) (1), (2), and (3) are correct B) (1) and (3) are correct DrRognerud@www.medfever.com Answer: CACBD

PEDIATRICS

80 / 343

Board Exam Question (Smiley + Star) + 2007 C) (2) and (4) are correct D) only (4) is correct E) all of the above 4.449/2. What tests would you order? 1) measurement of the blood pressure; qualitative tests and sediment examination of fresh urine 2) bacteriology of the urine collected with a catheter 3) antistreptilysine titer, bacteriology of the pharyngeal discharge 4) daily, precise body weight, and urine volume determination 5) intravenous urography A) (1), (2), and (3) are correct B) (1) and (3) are correct C) (2) and (4) are correct D) only (4) is correct E) all of the above 4.449/3. The examination reveals hematuria (10-100 RBC/HPF), proteinuria (1 g/m2/day) and an elevated serum creatinine concentration. What is your diagnosis? A) idiopathic nephrotic syndrome B) nephrolithiasis C) acute glomerulonephritis D) acute pyelonephritis E) isolated hematuria 4.449/4. Which of the following findings is not coherent witha diagnosis ofthe nephrotic syndrome? A) proteinuria B) hematuria C) edema D) an increase in body weight E) a transiently decreased urine volume 4.449/5. Based on the diagnosis you presumed in question (3), what is therapy required? 1) azathioprine 2) prednisone (2mg/kg/day) 3) penicillin 4) cyclophoshamide (5mg/kg/day) 5) a low salt and protein content diet A) only (1) is correct B) only (2) is correct C) (2) and (4) are correct D) (3) and (5) are correct E) all of the above PED-450. Case Study A 4-week-old boy is brought to your office. His body temperature is no mal, and he weighs 3,500 g. The infant has vomited after each meal for DrRognerud@www.medfever.com

PEDIATRICS

Answer: CCABCE

81 / 343

Board Exam Question (Smiley + Star) + 2007 3the last 5-6 days. No somatic growth has been seen during the last week. 4.450/ 1. Which of the following questions have to be asked to complete the history? A) Is the quantity of the vomit large? B) Does the infant vomit during feeding? C) Did the parents note projectile vomiting or bilious vomit? D) Does the infant accept mother's milk 4.450/2. What is the most likely result of the examination? A) it is unrevealing B) cyanotic skin C) resistance may possibly be detected on the right side of the navel, peristaltic waves in the epigastrium D) muscular hypotonia 4.450/3. Which of the following laboratory results is most likely to be positive in the condition? A) metabolic alkalosis B) negative urinalysis C) a normal serum sodium concentration D) negative renal functional tests E) normal blood cell and differential counts 4.450/4. Which of the following tests is useful to confirm the diagnosis? A) pneumoencephalography B) barium-swallow x-ray C) native abdominal x-ray D) basal and stimulated acid secretion of the stomach E) intravenous urography 4.450/5. Based on the above findings, the presumable diagnosis is: A) salt-losing adrenogenital syndrome B) aerophagia C) hypertrophied pyloric stenosis D) lack of belching after breastfeeding E) inflammation of the central nervous system (meningitis, encephalitis) 4.450/6. The therapy indicated in this condition is: A) the administration of deoxycorticosterone acetate (DOCA) B) a gastric lavage C) frequent feeding with small quantities D) the administration of spasmolytics E) operation PED-452. Case Study A 6-year-old girl is brought to your office. She voids turbid, dark red colored urine and complains of dull lumbar pain. Her blood pressure is 155/95 mmHg. Currently, the pharyngeal structures are normal. She had pharyngitis associated with fever 3 weeks ago. DrRognerud@www.medfever.com Answer: DCBC

PEDIATRICS

82 / 343

Board Exam Question (Smiley + Star) + 2007 4.452/1. A particularly imortant aspect of the physical examination is: A) the neurologic status B) auscultation of the lungs C) a palpable resistance, which might possibly be present in the abdomen D) the detection of palpebral edema 4.452/2. The most likely diagnosis is: A) rheumatoid arthritis B) nephrolithiasis C) glomerulonephritis D) renal tumor 4.452/3. The most important additional test is: A) a measurement of serum creatinine level B) a urinalysis C) a chest x-ray D) a native abdominal x-ray E) an ECG 4.452/4. Which of the following is the most important therapeutic intervention following admission to hospital? A) the administration of spasmolytics B) the administration of antibiotics C) a salt-and protein-restricted diet D) the administration of large quantities of fluid PED-455. Case Study A 6-year-old boy has been complaining of a sore throat and swelling of his knees and ankles for a few days. He is admitted to a hospital because of severe epigastric pain and blood in the stool. At admission: the ankles and feet are moderately swollen and painful. Purpura is observed around the ankles and the extensor surface of the legs. These are hemontiVc papules, surrounded by normal skin. The abdominal examination reveals abnormalities. Amoderate anemia and leukocytosis are detected. Urinalysis: microscopic hematuria. The blood pressure is normal. Antistreptolysin titer. 800 U. 4.455/1. The most likely diagnosis is: A) rheumatoid arthritis B) ulcerative colitis C) Schonlein-Henoch purpura D) rheumatic fever E) acute glomerulonephritis 4.455/2. The thrombocyte count found to be normal is of diagnostic value in the following conditions: 1) rheumatic fever 2) rheumatoid arthritis 3) acute glomerulonephritis DrRognerud@www.medfever.com Answer: CDCEB

PEDIATRICS

83 / 343

Board Exam Question (Smiley + Star) + 2007 4) Schonlein-Henoch purpura A) (1), (2), and (3) are correct B) (1) and (3) are correct C) (2) and (4) are correct D) only (4) is correct E) all of the above 4.455/3. The diagnosis, confirmed by the data, indicates penicillin administration, because the disease is caused by staphylococcus. A) both the statement and the explanation are true and a causal relationship exists between them B) both the statement and the explanation are true but there is no causal relationship between them C) the statement is true, but the explanation is false D) the statement is false, but the explanation itself is true E) both the statement and the explanation are false 4.455/4. Hematuria rarely occurs during the early phase of the disease because renal lesions do not develop either in the early or in the late phase of the disease. A) both the statement and the explanation are true and a causal relationship exists between them B) both the statement and the explanation are true but there is no causal relationship between them C) the statement is true, but the explanation is false D) the statement is false, but the explanation itself is true E) both the statement and the explanation are false 4.455/5. The disease sometimes heals leaving residual symptoms because streptococcus may have a role in the development of the disease. A) both the statement and the explanation are true, and a causal relationship exists between them B) both the statement and the explanation are true, but there is no causal relationship between them C) the statement is true, but the explanation is false D) the statement is false, but the explanation itself is true E) both the statement and the explanation are false PED-456. Case Study A 3-month-old infant living on a farm has been subfebrile for one week. He subsequently receives antibiotic therapy for an upper respiratory tract infection. The physician also detects cyanosis during his examination and immediately sends the baby to a hospital. On admission: the baby's general status is satisfactory, the diffuse cyanosis is more marked on the extremities. The infant is agitated and has tachypnea and tachycardia but no neurologic or internal organ abnormality is detected during the physical examination. The body temperature is normal. No abnormalities are seen on the chest x-ray. 4.456/1. Which of the following conditions are associated with cyanosis in the DrRognerud@www.medfever.com Answer: ACCBD

PEDIATRICS

84 / 343

Board Exam Question (Smiley + Star) + 2007 infancy? 1) congenital cardiac malformation with a right to left shunt 2) bronchopneumonia 3) methemoglobinemia 4) cerebral arteriovenous fistula A) (1), (2), and (3) are correct B) (1) and (3) are correct C) (2) and (4) are correct D) only (4) is correct E) all of the above 4.456/2. The most likely diagnosis is: A) bronchopneumonia B) congenital methemoglobinemia C) acquired methemoglobinemia D) cerebral arteriovenous fistula E) congenital cardiac malformation with a right to left shunt 4.456/3. Bronchopneumonia is excluded because bronchopneumonia in the infancy can always be screened by the physical examination. A) both the statement and the explanation are true and a causal relationship exists between them B) both the statement and the explanation are true but there is no causal relationship between them C) the statement is true, but the explanation is false D) the statement is false, but the explanation itself is true E) both the statement and the explanation are false 4.456/4. Congenital cardiac malformations are not always associated with cyanosis because cyanosis is a sign of the accumulation of reduced hemoglobin. A) both the statement and the explanation are true and a causal relationship exists between them B) both the statement and the explanation are true but there is no causal relationship between them C) the statement is true, but the explanation is false D) the statement is false, but the explanation itself is true E) both the statement and the explanation are false 4.456/5. Acquired methemoglobinemia is a rare condition, because numerous exogenous factors are capable of causing methemoglobinemia. A) both the statement and the explanation are true and a causal relationship exists between them B) both the statement and the explanation are true but there is no causal relationship between them C) the statement is true, but the explanation is false D) the statement is false, but the explanation itself is true E) both the statement and the explanation are false PED-459. Case Study DrRognerud@www.medfever.com Answer: ACCBC

PEDIATRICS

85 / 343

Board Exam Question (Smiley + Star) + 2007 A 6-year-old boy is brought to your office. Since 4-5 days ago, small eruptions occurring on the entire body surface and blueish spots of 2-10 cm in diameter in the lower extremities have been observed. The boy's mother has not detected any alteration in the general state of the child, his appetite and activity have been normal, although two weeks ago the child had been febrile for a few days. Physical examination: is unrevealing except for the skin lesions. Liver and spleen are not palpable, lymph nodes are of normal size. On the trunk and especially on the legs, (primarily on the areas exposed to traumas) numerous ecchymoses, and pinhead sized petechiae on the entire body are observed. Laboratory analysis: Hemoglobin: 6.9 mmol/l; hematocrit: 33%; WBC: 8,500/mm3; thrombocyte count: 10,000/mm3; reticulocyte: 0.2%; lymphocyte: 43%. Urinalysis: negative: Serum electrolytes, blood urea nitrogen and creatinine are within the reference range. 4.459/1. Which of the following questions is the most relevant to this case? 1) What drugs was the child taking before the onset of the symptoms? 2) Has the child recently been abroad? 3) Does the child consume too much milk? 4) Do any of the family members have hemorrhagic diathesis? A) (1) and (4) are correct B) (2), (3), and (4) are correct C) (1), (3), and (4) are correct D) all of the above E) none of the above 4.459/2. Which of the following examinations do you consider necessary? 1) the detection of antinuclear factor 2) a bone marrow aspiration 3) a determination of the bleeding time 4) a determination of the prothrombin time (PT) and the partial thromboplastin time (PTT) 5) a determination of the salicylic acid concentration in the blood A) (2), (3),. (4), and (5) are correct B) (1), (3), and (5) are correct C) (1), (2), and (4) are correct D) all of the above E) none of the above 4.459/3. Further laboratory results are: ANF (antinuclear factor) negative, PT: 12.3 s (control: 12 s), PTT: 32.7 s (norm.: 40 s). Bone marrow aspiration: normal structure and cellularity. The number of megakaryocytes is normal. What is the most likely diagnosis? A) Schonlein-Henoch purpura B) acute lymphoblastic leukemia (ALL) C) idiopathic thrombocytopenic purpura (ITP) D) hemolytic uremic syndrome (HUS) E) von Willebrand's disease F) spanked child syndrome 4.459/4. DrRognerud@www.medfever.com

PEDIATRICS

86 / 343

Board Exam Question (Smiley + Star) + 2007 Which are the possible complications of the condition? 1) intracranial hemorrhage 2) epistaxis 3) hematuria 4) gastrointestinal bleeding 5) aplastic anemia A) (1), (3), (4), and (5) are correct B) (1), (2), (3), and (4) are correct C) (3) and (5) are correct D) all of the above E) none of the above 4.459/5. What would you tell to the child's parents concerning the prognosis of this condition? A) all patients suffering from this disease would recover within 2 weeks B) chronic thrombocytopenia is expectable in each case C) some patients recover spontaneously D) in 40-50% of cases acute lymphoblastic leukemia develops within one year following the thrombocytopenic phase PED-460. Case Study An 18-month-old child is brought to your office by ambulance. The parents report that the child, while playing in the garden unattended, suddenly started to cough gaspingly, and the skin of the child turned blue. This cough lasted for 3 minutes and then the cyanosis ceased. Momentarily the child is symptomless. 4.460/1. Following are the parents' responses to your questions. Which of these is the most important for you? A) the child often drinks from a dug well B) the parents' previous child died because of valvular heart disease C) 2 weeks ago the child had pneumonia D) the child ate walnuts before the attack E) the elder brother of the child has measles 4.460/2. Which of the following diagnostic tests do you order? A) ECG B) chest x-ray film C) chest x-ray transillumination D) observation only, because the child is symptomless E) send for the elder brother having the measles 4.460/3. Which of the following tests do you additionally order? A) bronchoscopy B) bronchography C) ECG monitoring D) swallowing test E) repeated measurements of pC02 and P02 4.460/4. DrRognerud@www.medfever.com Answer: DCAE

PEDIATRICS

87 / 343

Board Exam Question (Smiley + Star) + 2007 Which of the following therapeutic interventions do you choose? A) the administration of methylene blue and vitamin C B) the administration of digitalis C) the administration of diuretics D) observation only E) the administration of antibiotics MULTIPLE CHOICE QUESTIONS / TYPE I Select the correct answers to the following questions!!! ...each qestion may have more than one correct answer. PED-470. Which of the following statements concerning difficulties in breastfeeding are FALSE? A) real hypogalactia is a frequent cause B) the mother should not squeeze her breasts during the first few days following feeding C) in case of a maternal puerperal psychosis ablactation is indicated D) before breastfeeding, a flat nipple should be elevated with a breast aspirator E) an infant with a cleft palate should be nourished with squeezed milk from the mother PED-489. Which of the following congenital cardiac malformations are associated with cyanosis? A) ventricular septal defect B) atrial septal defect C) Tetralogy of Fallot D) transposition of the great arteries PED-500. An 8-month-old infant develops a mild iron deficiency. Which foods do you recommend? A) spinach B) meat pure C) liver D) dairy products PED-539. Which of the following are the dominant symptoms of the prolonged presence of a foreign body in the nose? A) pain B) bleeding from the nose C) serous nasal discharge D) purulent nasal discharge E) lacrimation on the same side F) obstruction

PEDIATRICS

Answer: AB

Answer: CD

Answer: ABC

Answer: DF

DrRognerud@www.medfever.com

88 / 343

Board Exam Question (Smiley + Star) + 2007 PED-629. Symptoms of hypovolemic shock include all of the following, EXCEPT: A) pallor and cold extremities B) agitation, followed by drowsiness C) tachycardia D) a low central venous pressure E) cardiac enlargement F) acidosis G) a decreased arterio-venous oxygen difference PED-630. Which of the following steps of the first aid of a burn injury are INCORRECT? A) immediate cooling with cold water, even with the clothes on B) clothes covering the burned area have to be removed after cooking C) cooking oil is spread over the wounds and it is covered with a sterile bandage D) the wound has to be powdered with vulnerary powder and covered with a sterile bandage E) the wound is covered loosely with sterile gaze F) pain relievers and sedatives are administered G) the patient is immediately referred to a burn centre PED-640. The eradication of which of the following diseases has been made possible primarily by the introduction of a vaccination? A) tuberculosis B) abdominal typhus C) smallpox D) hepatitis A E) influenza F) measles Answer: EG

PEDIATRICS

Answer: CD

Answer: ACF

ASSOCIATION QUESTIONS Associate the following terms/statements marked by the letters A, B, C... with the corresponding statements/terms marked by and in the order given by the figures 1, 2, 3... ...for example: 1-C, 2-B, 3-A, 4-D. Put the answer as C, B, A, D! (Note: Different statements can be associated with the same terms!!!) PED-644. Associate the following term(s) with their corresponding statement(s)! A) Cyanides B) Narcotics C) Both of the above D) None of the above 1) methylene blue is an antagonist or chelator of these poisons 2) atropine sulphate is an antagonist or chelator of these poisons 3) naloxone (Narcan) is an antagonist or chelator of these poisons 4) amylnitrite, sodium nitrite and sodium thiosulphate are antagonists or chelators of these poisons DrRognerud@www.medfever.com Answer: DDBAB

89 / 343

Board Exam Question (Smiley + Star) + 2007 5) the antidote to these poisons may have to be given repeatedly to elicit the effect PED-646. Associate the following term(s) with their corresponding statement(s)! A) Trichophyton tonsurans B) Microsporum cants C) Both of the above D) None of the above 1) it causes a mycosis of the hairy skin of the head 2) it causes a fungal vaginitis following puberty 3) it is usually fluorescent under Wood's lamp 4) it causes vaginal itching and a watery discharge 5) short, fragmented hair is strongly suggestive of the infection PED-648. Associate the following term(s) with their corresponding statement(s)! A) Vitamin A) deficiency B) Vitamin B1 deficiency C) Vitamin B6 deficiency D) Vitamin C deficiency E) Vitamin D deficiency F) Vitamin K deficiency 1) spasms may occur 2) gingival hemorrhage 3) sluggish tendon reflexes 4) dry and hyperkeratotic skin PED-649. Associate the following statement(s) with their corresponding term(s)! ... in terms of dietary treatmentfor the disorder A) a diet containing medium chain fatty acids B) oral supplementation of zinc sulphate C) a gluten-free diet D) a flour and starch-free diet E) a galactose (lactose-) free diet F) a lactose (sucrose-) free diet 1) Celiac disease 2) Congenital sucrose-isomaltase defect 3) Intestinal lymphangiectasis 4) Galactosemia 5) Postenteritic malabsorption 6) Acrodermatitis enteropathica PED-654. Associate the following statement(s) with their corresponding term(s)! A) vesicular-pustular eruptions B) dense, tiny macules C) lens-sized, confluent maculous eruptions D) a butterfly shaped facial flush and garland-like eruptions on DrRognerud@www.medfever.com

PEDIATRICS

Answer: CDBDC

Answer: CDBA

Answer: CDAEFB

Answer: BACAED

90 / 343

Board Exam Question (Smiley + Star) + 2007 the extremities E) usually isolated, lens-sized, maculous eruptions 1) Scarlet fever 2) Chickenpox 3) Measles 4) Herpes zoster 5) German measles 6) Erythema infectiosum PED-659. Associate the following term(s) with their corresponding statement(s)! A) Vitamin A deficiency B) Vitamin B deficiency C) Vitamin C deficiency D) Vitamin D deficiency E) Vitamin E deficiency 1) the deficiency syndrome characterized by peripheral neuropathy 2) the deficiency syndrome characterized by hemorrhagic diathesis 3) the deficiency syndrome characterized by abnormalities of bone formation 4) the deficiency syndrome characterized by Bitot's spots 5) the deficiency syndrome is more frequent in newborns with a low birth weight PED-663. Associate the following term(s) with their corresponding statement(s)! A) Hepatitis A virus B) Hepatitis B virus C) Hepatitis C virus D) Hepatitis D virus E) Hepatitis E virus 1) it is usually acquired parenterally 2) it is actually a parasitic RNA virus 3) it has caused a high mortality rate in pregnant women 4) antibody of this virus appears within 1-4 weeks of clinical symptoms 5) transmission is by the fecal-oral route 6) antibodies to this newly isolated virus may not appear for up to six months 7) this DNA virus is referred to as the Dane particle PED-664. Associate the following term(s) with their corresponding statement(s)! ... in terms of a latent iron deficiency A) Normal value B) Decreased value C) Elevated value 1) hemoglobin DrRognerud@www.medfever.com

PEDIATRICS

Answer: BCDAE

Answer: BDEAACB

Answer: ABCBABCB

91 / 343

Board Exam Question (Smiley + Star) + 2007 2) serum iron 3) total iron binding capacity (TIBC) 4) iron saturation of transferrin 5) hematocrit 6) serum ferritin 7) iron absorption (no malabsorption is present) 8) number of sideroblasts in the bone marrow

PEDIATRICS

DrRognerud@www.medfever.com

92 / 343

Board Exam Question (Smiley + Star) + 2007

OBSTETRICS & GYNECOLOGY

OBSTETRICS & GYNECOLOGY


(302) SINGLE CHOICE QUESTIONS Select the single best response to each of the following questions!!! OBG-1. The most prevalent cause of maternal mortality is: A) toxemia B) infection C) cardiac disease D) hemorrhage E) diabetes OBG-4. In a normal pregnancy, carbohydrate metabolism changes as follows: A) glucose tolerance is reduced B) there is a tendency for the development of glycosuria C) the glomerular filtration rate of glucose is increased D) all of the above E) none of the above OBG-5. An abnormal hemorrhage complicating a delivery occurs most freqently:. A) in the third stage of labor B) during the development of a hematoma which causes the placenta to separate C) in the first stage of labor D) at the time of complete cervical dilation E) in the first hour following the delivery of the placenta OBG-6. Perinatal care of the neonate should include: A) the removal of the vernix B) intubation C) oxygen administration D) removal of the mucus from the mouth and pharynx of the neonate . E) all of the above OBG-7. The first step in the care of a patient with eclampsia is: A) phlebotomy B) termination of the pregnancy C) the transfusion of blood D) correction of the hemoconcentration (volume depletion) E) the intravenous administration of diazepam

Answer: D

Answer: D

Answer: E

Answer: D

Answer: E

DrRognerud@www.medfever.com

93 / 343

Board Exam Question (Smiley + Star) + 2007 OBG-8. Prior to the surgical extraction of the placenta: A) any shock must be completely corrected B) blood for any required transfusion should be made available C) any performed blood transfusions must have been completed before the extraction D) all of the above E) none of the above OBG-9. Currently, the most effective contraceptive method is the: A) oral (hormonal) contraceptive B) condom C) cervical diaphragm D) calendar rhythm method (periodic abstinence) E) intrauterine device OBG-13. The correct gynecologic history should contain the following data: A) menstrual history B) previous deliveries C) family history D) previous diseases E) all of the above OBG-16. Case Study: A 39-year-old nulliparous woman presents with a one-week delay of her last expected menstrual period. The patient has never taken any contraceptives and was married 6 months ago. She has always had a regular menstrual cycle. On examination, the cervix is soft and of a bluish-purple color and the adnexal structures are not palpable. Which of the following methods is suitable for the earliest possible recognition of pregnancy? A) .-hCG radioimmunoassay B) ultrasonography C) a measurement of the basal body temperature D) "palm leaf' arborization (ferning) of the cervical mucus E) progesterone withdrawal OBG-18. Case Study: You are attending to a 36 year-old gravida in the 8 the week of gestation (the patient had 6 pregnancies and 5 deliveries previously). The patient is concerned about delivering a baby with congenital abnormalities. In which week of the pregnancy should amniocentesis be performed? A) immediately (in the 8th week) B) in the 10th week C) in the 12th week D) in the 15th week DrRognerud@www.medfever.com

OBSTETRICS & GYNECOLOGY

Answer: B

Answer: A

Answer: E

Answer: A

Answer: D

94 / 343

Board Exam Question (Smiley + Star) + 2007 E) in the 24th week OBG-19. Case Study: A 41-year-old multiparous (7 pregnancies ,7 deliveries) woman is undergoing a vaginal hysterectomy and reconstructive surgery for uterovaginal prolapse. She has been taking oral contraceptives for the last 10 years. Which of the following potential complications is the most likely associated with contraceptive use? A) pneumonia B) pulmonary embolism C) retinal detachment D) periorbital cellulitis E) ileus OBG-21. Case Study: A 29-year-old multiparous woman (4 pregnancies, 4 deliveries) undergoes laparoscopic fulguration of the oviducts. Thirty-six hours after the operation, the patient begins to complain about abdominal pain and nausea. Her body temperature is 38.3 C and slight abdominal distention is palpated. The most likely diagnosis is: A) pelvic inflammation B) hemorrhage from the uterine tube C) thermal injury to the gut D) a perforating injury to the gut resulting from a stab wound E) tubal abortion OBG-23. Case Study: In a 26-year-old pregnant woman, uterine growth stops abruptly in the 4th month of gestation. The uterus begins to involute but 6 weeks later a spontaneous abortion has still not occurred. Which of the following parameters should be primarily monitored? A) the hematocrit and hemoglobin levels B) the fibrinogen level C) the blood urea nitrogen (BUN) level D) the serum creatinine level E) the bilirubin level OBG-34. The most common defect in the adrenogenital syndrome (congenital adrenal hyperplasia) is: A) none, as it is an idiopathic disorder B) an 11-hydroxylase deficiency C) a 17-hydroxylase deficiency D) a 21-hydroxylase deficiency E) 3-.-ol-dehydrogenase deficiency

OBSTETRICS & GYNECOLOGY

Answer: B

Answer: C

Answer: B

Answer: D

DrRognerud@www.medfever.com

95 / 343

Board Exam Question (Smiley + Star) + 2007 OBG-35. Which of the following cells produces follicle-stimulating hormone (FSH)? A) the chromophobic cells of the anterior pitutary B) the basophilic cells of the anterior pituitary C) the acidophilic cells of the anterior pituitary D) the internal thecal cells E) none of the above OBG-39. Which of the following methods is appropriate for the detection of pregnancy at the earliest time possible? A) a pelvic examination B) a progesterone level determination C) the erythrocyte sedimentation rate D) the erythrocyte agglutination-inhibition test E) a measurement of the hCG .-subunit serum levels OBG-40. Following exposure to excessive heat, the previously normal sperm count will begin to reduce within: A) a day B) 7 days C) 10 days D) 75 days E) 300 days OBG-41. The average volume of menstrual blood loss is: A) 5 to 10 ml B) 10 to 15 ml C) 25 to 50 ml D) 150 to 200 ml E) 250 to 300 ml OBG-47. All the following are contraindications to nursing, EXCEPT: A) a maternal hepatitis B infection B) a surgical reduction of the breast with autotransplantation of the nipple C) breast engorgement D) lithium carbonate therapy of the mother E) tetracycline therapy of the mother OBG-51. Maternal mortality reflects the number of maternal deaths during the reproductive process per: A) 1000 deliveries B) 10,000 deliveries C) 100,000 deliveries D) 10,000 live births DrRognerud@www.medfever.com

OBSTETRICS & GYNECOLOGY Answer: B

Answer: E

Answer: D

Answer: C

Answer: C

Answer: E

96 / 343

Board Exam Question (Smiley + Star) + 2007 E) 100,000 live births OBG-52. Case Study: A 23-year-old woman (2 pregnancies, 2 deliveries) presents with bloody vaginal discharge persisting for 7 days after delivery. The patient should be reassured that bloody puerperal discharge normally lasts for: A) 2 days B) 5 days C) 8 days D) 11 days E) 14 days OBG-60. Case Study: A 46-year-old woman experiences the following symptoms: depression, hot flushes, nocturnal sweating and recurrent headaches. The clinical evaluation denotes anovulation. The most likely diagnosis is: A) psychosomatic disorder B) depression and mania C) uremia D) tuberculosis E) menopause (climacteric) OBG-61. Which of the following describes the chronologic sequence of pubertal events correctly? A) accelerated growth, breast budding, pubarche, menarche B) accelerated growth, pubarche, breast budding, menarche C) breast budding, pubarche, menarche, accelerated growth D) pubarche, breast budding, accelerated growth, menarche OBG-62. During oral contraceptive use, unexpected pregnancy most often develops due to: A) midcycle breakthrough ovulation B) frequent sexual intercourse C) the inappropriate use of oral contraceptives D) reduced gastrointestinal absorption of the contraceptives E) the development of antibodies OBG-63. All of the following are appropriate for terminating a pregnancy in the second trimester, EXCEPT: A) dilation and curettage B) the administration of prostaglandin-E 2vaginal suppositories C) the intra-amniotic administration of oxytocin D) dilation and oxytocin infusion E) the intra-amniotic administration of 30% urea

OBSTETRICS & GYNECOLOGY

Answer: E

Answer: E

Answer: A

Answer: C

Answer: C

DrRognerud@www.medfever.com

97 / 343

Board Exam Question (Smiley + Star) + 2007 OBG-65. Vacuum-curettage has all the following advantages over conventional dilatation and curettage, EXCEPT: A) it is less time-consuming B) it can be performed on outpatients C) it carries a lower risk of injury to the uterus D) it is also applicable in the termination of pregnancies approaching the end of the second trimester OBG-69. All of the following statements are valid regarding polyhydramnios, EXCEPT: A)acute polyhydramnios is a frequent cause of premature delivery occurring before the 28th week of pregnancy B) polyhydramnios is associated with congenital abnormalities in 20% of cases C) edema is common, particularly of the lower extremities and of the vulva D) polyhydramnios is associated with ureteral obstruction in almost 50% of cases E) it can be complicated by the premature separation of the placenta, uterine dysfunction and postpartum hemorrhage OBG-70. All of the following statements are valid regarding progesterone production during pregnancy, EXCEPT: A) during the first 10 weeks of pregnancy, the corpus luteum is the primary source of progesterone B) following the 12th week of gestation, the placenta is the primary source of progesterone C) maternal cholesterol is a major precursor of the progesterone synthetized by the placenta D) progesterone levels rise abruptly after death of the fetus E) progesterone is an essential substrate of maternal cholesterol synthesis OBG-74. Which of the following statements regarding suppurative mastitis is valid? A) the symptoms usually present in the 10th postpartum week B) herpesvirus is the most common causitive pathogen C) the most common cause of masititis is poor personal hygiene of the mother D) the pathogen can never be cultured from breast milk E) the source of infection is almost invariably the nose and throat of the nursing infant OBG-75. The anteroposterior diameter of the pelvic rim is the shortest along the: A) interspinous diameter B) anatomical conjugate DrRognerud@www.medfever.com

OBSTETRICS & GYNECOLOGY

Answer: D

Answer: D

Answer: D

Answer: E

Answer: D

98 / 343

Board Exam Question (Smiley + Star) + 2007 C) diagonal conjugate D) obstetric conjugate E) none of the above OBG-79. Case Study: A 19-year-old woman is referred to the emergency room for a sudden loss of consciousness at her job. The examination reveals slight vaginal bleeding; the abdomen is distended and diffusely tender. The patient complains of shoulder-and abdominal pain. Body temperature: 36.4 C; heart-rate: 120/min; blood-pressure: 96/50 mmHg. Which of the following diagnostic procedures should be performed to verify the tentative diagnosis established by evaluating the available clinical data? A) a pregnancy test B) posterior colpotomy C) dilation and curettage D) diagnostic puncture of the cul-de-sac E) hysteroscopy OBG-85. Which of the following statements regarding placenta previa is valid? A) the incidence of this condition decreases with advancing maternal age B) the incidence of this condition is not influenced by previous deliveries or abortions C) the inital bleeding is painless and seldom causes death D) vaginal surgery is the therapy of choice E) immediate vaginal examination is mandatory when placenta previa is suspected OBG-86. Viremia as well as the presence of rubella virus in the pharynx of an infected individual is related to the appearance of the characteristic skin rash by: A) their occurrence 5-7 days before the development of the rash B) their occurrence 1-2 days before the development of the rash C) their simultaneous occurrence with the rash D) their occurrence 1-2 days after the development of the rash E) there is no correlation between their occurrence and the development of the rash OBG-87. Which of the following is a potential, associated risk in patients developing eclampsia during their first pregnancy? A) diabetes mellitus B) chronic hypertension C) habitual abortion D) chronic liver disease E) delivery of a dead fetus in the third trimester of pregnancy

OBSTETRICS & GYNECOLOGY

Answer: D

Answer: C

Answer: A

Answer: B

DrRognerud@www.medfever.com

99 / 343

Board Exam Question (Smiley + Star) + 2007 OBG-90. All df the following statements regarding polyhydramnios are valid, EXCEPT: A) therapeutic amniocentesis is indicated solely for the alleviation of maternal distress B) in polyhydramnios, the incidence of major congenital abnormalities is 20% C) occasionally, it can be treated safely and effectively with diuretics as well as restricting the intake of water and dietary salt D) polyhydramnios is associated with an increased incidence of premature separation of the placenta, uterine dysfunction and post-partum bleeding E) the rapid removal of amniotic fluid is contraindicated OBG-92. What is the earliest time when a multiple pregnancy can be detected by ultrasonography? A) between the 4th and 6th week of gestation B) between the 8th and 10th week of gestation C) between the 14th and 15th week of gestation D) between the 15th and 16th week of gestation E) between the 15th and 16th week of gestation OBG-96. In the second trimester of pregnancy, ultrasonography is suitable for: A) the measurement of fetal dimensions B) intrauterine detection of congenital abnormalities C) the assessment of fetal position and presentation D) all of the above E) only answers (A) and (B) are true OBG-97. Which of the following fetal abnormalities are detectable by ultrasonography? A) anencephaly B) hydrocephalus C) large abdominal neoplasms D) all of the above E) only answers (A) and (B) are true OBG-98. Which of the following conditions should be suspected if the abdominal circumference is greater than normal? A) multiple pregnancy B) polyhydramnios C) large fetus D) all of the above E) ptotic abdomen

OBSTETRICS & GYNECOLOGY Answer: C

Answer: B

Answer: E

Answer: D

Answer: D

DrRognerud@www.medfever.com

100 / 343

Board Exam Question (Smiley + Star) + 2007 OBG-100. From which week of pregnancy does the fetal skull show with full certainty, on ultrasonography? A) 6 weeks B) 7-8 weeks C) 12-14 weeks D) 16-18 weeks E) 18-20 weeks OBG-102. On ultrasonography, the diameter of which of the following fetal organs yields valuable information on the growth-rate of the fetus? A) the biparietal diameter of the skull (BPD) B) the diameter of the thorax C) the diameter of the abdomen D) all the above if assessed simultaneously E) only answers (A) and (B) are true OBG-103. In which of the following conditions is the measurement of the placental thickness important? A) diabetes B) Rh-incompatibility C) polyhydramnios D) all of the above E) only answers (A) and (B) are true OBG-105. Which of the following radiologic procedures is contraindicated during pregnancy? A) chest x-ray B) chest fluoroscopy C) therapeutic irradiation D) radiography of the pelvis E) fluoroscopy of the pelvis OBG-106. From which week of pregnancy is the radiolucency of fetal bones detectable on radiography? A) 8 weeks B) 16 weeks C) 20 weeks D) 24 weeks E) 28 weeks OBG-107. Which of the following congenital abnormalities is detectable by radiograph? A) anencephaly B) hydrocephalus DrRognerud@www.medfever.com

OBSTETRICS & GYNECOLOGY

Answer: C

Answer: D

Answer: E

Answer: C

Answer: B

Answer: E

101 / 343

Board Exam Question (Smiley + Star) + 2007 C) gross abnormalities of the extremities D) syphilitic osteochondritis E) all of the above OBG-112. What is hysterosalpingography used for? A) for the diagnosis of ectopic pregnancy B) to assess the patency of the Fallopian tubes and detect the morphologic abnormalities of the uterine cavity C) for the diagnosis of ovarian neoplasms D) to assess the motility of the Fallopian tubes E) to measure the size of the ovaries OBG-113. Which of the following describes Nagele's method for estimating the duration of a pregnancy? A) 9 months + 3 days starting from the last day of the last regular menses B) 9 months + 7 days starting from the first day of the last regular menses C) 9 months starting from the first day of the last regular menses D) 9 months + 7 days starting from the time of conception E) 5 months starting from the time when fetal motion is detected OBG-114. Nagele's method for estimating the duration of a pregnancy takes the following under consideration: A) the date of ovulation B) the date of conception C) the first day of the last menses D) the last day of the last regular menses E) the date when fetal motion is first detected OBG-115. The duration of a normal pregnancy from the first day of the last regular menses is: A) 266 days B) 280 days C) 300 days D) 310 days E) 320 days OBG-116. The average duration of a normal pregnancy from the day of conception is: A) 200 days B) 266 days C) 300 days D) 310 days E) 320 days

OBSTETRICS & GYNECOLOGY

Answer: B

Answer: B

Answer: C

Answer: B

Answer: B

DrRognerud@www.medfever.com

102 / 343

Board Exam Question (Smiley + Star) + 2007 OBG-117. Which of the following is taken into consideration when the term of delivery is calculated? A) Nagele's method for estimating the duration of the pregnancy B) the date when fetal motion is first detected C) the ascension rate of the uterine fundus D) the date when the uterine fundus descends E) all of the above OBG-118. Which of the following tests should be performed at each follow-up visit during pregnancy? A) urinalysis B) blood pressure measurement C) measurement of body weight D) all of the above E) only answers (A) and (B) are true OBG-119. Which of the following tests is unnecessary during the first trimester of pregnancy? A) a urinalysis B) blood pressure measurements C) measurements of the body weight D) measurements of the abdominal circumference E) vaginal examinations OBG-120. Which of the following is not a routine test at follow-up visits during pregnancy? A) serologic tests for syphilis (STS) B) hematocrit measurements C) measurements of the hemoglobin level D) urinalysis E) liver function tests OBG-121. Which of the following urinary parameters is/are mandatory when being tested at follow-up visits during pregnancy? A) the presence of any pus B) the glucose level C) the concentration of protein D). all of the above E) only answers (A) and (C) are true OBG-122. The optimal monthly gain of body weight during pregnancy is: A) 0.5-0.6 kg B) 1.0-1.5 kg C) 1.5-2.0 kg DrRognerud@www.medfever.com

OBSTETRICS & GYNECOLOGY

Answer: E

Answer: D

Answer: D

Answer: E

Answer: D

Answer: B

103 / 343

Board Exam Question (Smiley + Star) + 2007 D) 2.0-2.5 kg E) 2.5-3.0 kg OBG-125. The daily fat requirement of pregnant woman is as high as: A) 60-80 g/day B) 100-150 g/day C) 150-200 g/day D) 200-300 g/day E) 300-350 g/day OBG-126. The proper method for preparing the nipples for breastfeeding is: A) washing the nipples with soap every morning and evening B) massage of the nipple and the areola C) only answers (A) and (B) are true D) no preparation is necessary OBG-128. Which of the following drugs is contraindicated during pregnancy? A) coumarins B) oral antidiabetic agents C) actinomycin D D) cytotoxic agents E) all of the above OBG-129. Which of the following drugs is contraindicated during pregnancy? A) thalidomide B) methimazole C) vitamin K in high doses D) all of the above E) only answers (A) and (B) are true OBG-131. Which of the following drugs should not be administered during pregnancy? A) streptomycin B) sulfonamides C) carbutamide D) all of the above E) only answers (A) and (C) are true OBG-132. In which weeks of pregnancy is ultrasonography recommended for monitoring the condition and development of the fetus? A) on weeks 8 and 32 B) on weeks 24 and 32 C) on weeks 8, 24 and 38 D) on weeks 8, 24 and 32 DrRognerud@www.medfever.com

OBSTETRICS & GYNECOLOGY

Answer: A

Answer: C

Answer: E

Answer: D

Answer: D

Answer: E

104 / 343

Board Exam Question (Smiley + Star) + 2007 E)on weeks 8, 18, 28 and 32 OBG-133. What is the influence of pregnancy on epilepsy? A) seizure threshold is lower B) seizure threshold is higher C) there is no relation between pregnancy and seizure threshold OBG-149. Why is it contraindicated to let pregnancy develop to full term in patients with heart valve implants? A) because the risk of congestive heart failure is high B) because the rejection of the implant is common during pregnancy C) because the associated permanent anticoagulant therapy carries the risk of abortion, intrauterine fetal death and bleeding D) because thrombus formation is common despite ongoing anticoagulant therapy OBG-152. Should a mother with active tuberculosis nurse her baby? A) no B) yes C) only if she has been receiving antituberculotic chemotherapy during her pregnancy OBG-153. What is the effect of pregnancy on bronchial asthma? A) it has no influence B) the condition of the patient improves C) the condition of the patient deteriorates D) its effects are inconsequential OBG-162. Which of the following drugs will you not prescribe for constipation associated with pregnancy? A) sennoside A+B (Tisasen A + B) B) phenolphthalein C) bisacodyl (Videx) D) drastic cathartics E) osmotic laxatives OBG-163. Which of the following factors facilitate the development of choletithaiassis during pregnancy? A)a high serum cholesterol level B)reduced muscle tone of the gall bladder C)constipation and reduced excursions of the diaphragm D)all of the above E)only answers (B) and (C) are true

OBSTETRICS & GYNECOLOGY

Answer: A

Answer: C

Answer: A

Answer: D

Answer: D

Answer: D

DrRognerud@www.medfever.com

105 / 343

Board Exam Question (Smiley + Star) + 2007 OBG-164. What should be done if significant, recurrent asymptomatic Bacteriuria develops during pregnancy ? A)identification of the pathogen by cultures B)antibiotic susceptibility testing C)antibiotic therapy D)all of above E)no intervention is necessary OBG-165. What is the possible route for pathogens in renal infections developing during pregnancy? A) an ascending infection originating from the urinary bladder B) the hematogenous spread from distant foci of infection C) the lymphogenic spread from the intestines D) all of the above E) only answers (A) and C) are true OBG-166. Which of the following conditions predisposes to pyelonephritis becoming apparent during pregnancy? A) diabetes B) toxemia C) acute pyelonephritis in the past medical history D) infectious diseases E) all of the above OBG-167. In mild anemia during pregnancy, the serum hemoglobin is in the range of A) 150-430 g/1 B) 110-130 g/1 C) 90-110 g/l D) 80-90 g/1 OBG-168. In moderately severe anemia during pregnancy, the serum hemoglobin is in the range of: A) 130-150 g/l B) 110-130 g/l C) 90-110 g/1 D) 80-90 g/1 OBG-171. In moderately severe anemia during pregnancy, the serum hemoglobin is in the range of: A) 8.0-9.27 mmo/1 B) 6.8-8.0 mmo/I C) 5.6-6.8 mmo/I D) 0.6 mmo/1

OBSTETRICS & GYNECOLOGY

Answer: D

Answer: D

Answer: E

Answer: B

Answer: C

Answer: C

DrRognerud@www.medfever.com

106 / 343

Board Exam Question (Smiley + Star) + 2007 OBG-172. In severe anemia during pregnancy, the serum hemoglobin is in the range of: A) 8.0-9.27 mmo/1 B) 6.8-8.0 mmo/1 C) 5.6-6.8 mmo/I D) 0.6 mmo/1 OBG-181. The complications of toxoplasmosis developing during pregnancy include: A) abortion B) premature delivery C) intrauterine fetal death D) all of the above E) none of the above OBG-183. In listeriosis, the pathogen can be cultured from the: A) blood B) cerebrospinal fluid C) feces D) urine E) all of the above OBG-188. Fetal affects of syphilis include: A) intrauterine fetal death B) abortion C) premature birth D) all of the above E) none of the above OBG-189. In pregnancy, Treponema pallidum can penetrate the placenta in week: A) 6 B) 12 C) 20 D) 28 E) 36 OBG-190. In which of the following cases is the performance of serologic tests for syphilis justified? A) if the mother is not married B) if the symptoms of the father suggest syphilis C) if the family history contains any offspring born with anomalies suggesting congenital syphilis D) in case of intrauterine fetal death of unknown etiology E) in all cases without exception

OBSTETRICS & GYNECOLOGY

Answer: D

Answer: D

Answer: E

Answer: D

Answer: C

Answer: E

DrRognerud@www.medfever.com

107 / 343

Board Exam Question (Smiley + Star) + 2007 OBG-192. In pregnancy, gonococcal infection becomes established in: A) the urethra B) Skene's glands C) the cervical canal D) an obstruction of the ducts of Bartholini's glands E) all of the above OBG-193. The therapy of gonorrhea during pregnancy includes the administration of A) sulfonamides B) penicillins C) tetracyclines OBG-194. Which of the following demonstrate the diabetogenic effect of pregnancy? A) estrogen, prolactin and cortisol are insulin-antagonists B) the blood glucose level is elevated by growth hormone, prolactin and TSH C) the placenta metabolizes a portion of circulating insulin D) all of the above E) only answers (A) and (B) are true OBG-196. Characteristic features of latent (gestational) diabetes include? A) the presence of symptoms; high blood-glucose levels B) the absence of clinical symptoms; normal blood-glucose levels, impaired glucose tolerance C) the absence of clinical symptoms; normal blood-glucose levels and glucose tolerance, however, the enhanced glucose tolerance test yields abnormal results D) clinical symptoms are absent; blood-glucose levels and the results of both the glucose tolerance test and .the enhanced glucose tolerance tests are normal OBG-197. Which of the following may suggest potential diabetes? A) the delivery of a neonate of over 4,000 g body weight B) either parent or both of them are diabetics C) obesity, sudden weight-gain D) all of the above E) only answers (A) and (B) are true OBG-198. Which of the following jeopardize women with gestational diabetes? A) the derangement of carbohydrate metabolism B) the derangement of water-and electrolyte balance C) toxemia, urinary tract infection and vaginitis develop frequently D) all of the above complications E) only answers (A) and (B) are true DrRognerud@www.medfever.com

OBSTETRICS & GYNECOLOGY

Answer: E

Answer: B

Answer: D

Answer: C

Answer: D

Answer: D

108 / 343

Board Exam Question (Smiley + Star) + 2007 OBG-199. Which of the following is more prevalent in pregnant diabetics? A) toxemia B) polyhydramnios C) abortion D) intrauterine fetal death E) all of the above OBG-200. Which of the following is suggested by the delivery of a neonate of over 4,000 g of body weight? A) an improper diet during pregnancy B) a latent diabetes C) Rh isoimmunization D) toxemia OBG-201. During the first 24 hours following delivery, the blood glucose level of a neonate born to a diabetic mother is: A) normal B) hyperglycemia is common C) hypoglycemia is common D) no characteristic changes occur OBG-202. Which of the following fetal consequences should be considered in pregnant diabetics? A) congenital malformations and polyhydramnios are common B) premature birth is more prevalent in these patients C) macrosomia D) all of the above E) only answers (A) and (C) are true OBG-206. Which of the following is a possible complication of an uterine myoma during pregnancy? A) abortion, premature delivery B) impairment of placental separation C) myomatous foci may obstruct delivery D) all of the above E) only answers (A) and (C) are true OBG-209. The term "hypersalivatio gravidarum" means: A) permanent salivation that impairs normal feeding B) ordinary morning sickness with nausea, vomiting and salivation C) increased gastric secretion following meals D) frequent vomiting unrelated to meals and the fullness of the stomach DrRognerud@www.medfever.com

OBSTETRICS & GYNECOLOGY

Answer: E

Answer: B

Answer: C

Answer: D

Answer: D

Answer: A

109 / 343

Board Exam Question (Smiley + Star) + 2007 E) nausea precipitated by strange odors OBG-210. The term "vomitus matutinus" means: A) permanent salivation that impairs normal feeding B) ordinary morning sickness with nausea, vomiting and salivation C) vomiting occurring following meals D) frequent vomiting unrelated to meals and the fullness of the stomach E) vomiting precipitated by strange odors or flavors OBG-211. The term "emesis gravidarum" means: A) permanent salivation that impairs normal feeding B) ordinary morning sickness with nausea, vomiting C) vomiting occurring 2-3 times a day, following meals D) starvation and consequent toxicosis resulting from a malignant vomiting syndrome unrelated to meals E) vomiting precipitated by strange odors or flavors OBG-212. The term "hyperemesis gravidarum" means: A) permanent salivation that impairs and precludes normal feeding B) ordinary morning sickness with nausea, vomiting C) vomiting occurring 2-3 times a day, following meals D) starvation and consequent toxicosis resulting from a malignant vomiting syndrome unrelated to meals E) vomiting precipitated by strange odors or flavors OBG-214. Which of the following belong to the pathomechanism of hyperemesis in pregnancy? A) hormonal factors B) neural factors C) metabolic factors D) hormonal and neural factors E) hormonal, neural and metabolic factors OBG-215. Which of the following hormonal changes is responsible for the development of hyperemesis in pregnancy? A) excessive progesterone production B) high hCG levels C) high hCG and progesterone levels D) excessive production of adrenal corticosteroids E) prolactin, produced only during pregnancy OBG-217. The principal sign of hyperemesis of pregnancy is: A) considerable weight-loss B) significant exsiccosis (fluid depletion) DrRognerud@www.medfever.com

OBSTETRICS & GYNECOLOGY

Answer: B

Answer: C

Answer: D

Answer: E

Answer: B

Answer: E

110 / 343

Board Exam Question (Smiley + Star) + 2007 C) acetone positivity and increased urobilinogen levels in the urine as well as the appearance of casts and leucine-or tyrosine crystals D) weight-loss, alkalosis E) only answers (A), (B) and (C) are true OBG-219. Which of the following laboratory tests should be performed in hyperemesis of pregnancy? A) urine volume; specific gravity; protein, acetone and urobilinogen content B) urinary sediment examination C) measurement of the hematocrit and hemoglobin levels D) measurement of the serum bilirubin level E) all of the above OBG-220. In hyperemesis gravidarum, hematocrit and hemoglobin values are: A) elevated B) reduced C) unchanged OBG-221. The therapy of hyperemesis gravidarum includes: A) bed rest B) parenteral fluid therapy, and nutrition C) administration of antiemetics D) administration of sedatives E) all of the above OBG-223. What is the ranking of toxemia of pregnancy among the causes of maternal mortality? A) first B) second C) third D) fourth E) fifth OBG-225. The cause of generalized vasoconstriction developing in late occurring toxemia of pregnancy is: A) vasopressor substances produced in the placenta B) increased sensitivity of small arterioles to pressor agents C) only answers (A) and (B) are true D) adrenal hyperfunction E) altered sensitivity of the blood-pressure regulating centre OBG-226. Which of the following conditions predisposes to toxemia of pregnancy? DrRognerud@www.medfever.com

OBSTETRICS & GYNECOLOGY

Answer: E

Answer: A

Answer: E

Answer: A

Answer: C

Answer: D 111 / 343

Board Exam Question (Smiley + Star) + 2007 A) hypertension B) diabetes mellitus C) chronic glomerulonephritis B) all of the above E) only answers (A) and (C) are true OBG-228. Pathophysiologic features of late occurring toxemia of pregnancy include: A) generalized vasoconstriction B) increased capillary permeability C) increased retention of water and sodium in the tissues D) all of the above E) only answers (A) and (B) are true OBG-230. Which of the following factors contribute to the development of edema in toxemia of pregnancy? A) increased capillary permeability B) vasoconstriction of arterioles C) tissue hypoxia D) increased effusion of plasmaproteins into the interstitial space E) all of the above OBG-231. What are the consequences of generalized vasoconstriction in toxemia occurring late in pregnancy? A) hypertension B) tissue ischemia C) hypoxia D) all of the above E) only answers (A) and (C) are true OBG-235. Symptoms of late toxemia of pregnancy include: A) hypertension B) proteinuria C) edema D) all of the above E) only answers (B) and (C) are true OBG-237. Which of the following is an appropriate method for detecting latent edema in pregnancy? A) monitoring the balance of fluid intake and loss B) monitoring the changes in body weight C) pressing the skin over the tibia D) determination of fluid compartment volumes by radionuclide studies E) only answers (A) and (B) are true

OBSTETRICS & GYNECOLOGY

Answer: D

Answer: E

Answer: D

Answer: D

Answer: E

DrRognerud@www.medfever.com

112 / 343

Board Exam Question (Smiley + Star) + 2007 OBG-239. Which of the following conditions should eclamptic seizures be differentiated from? A) epilepsy B) hysteric attack C) uremic seizures D) all of the above E) only answers (A) and (B) are true OBG-242. The term "primary (genuine) toxemia of pregnancy" means: A) condition with cumulative occurrence within the family B) that no organic disease can be detected C) toxemic symptoms associated with hypertension as well as renal and vascular disease D) symptoms developing in the first half of pregnancy E) symptoms associated with.multiple pregnancy OBG-245. Which of the following methods is appropriate for evaluating the severity of late occurring toxemia of pregnancy? A) the measurement of blood pressure B) the measurement of urinary protein concentration C) assessing the severity of edema as well as the patient's subjective symptoms D) all of the above E) only answers (B) and (C) are true OBG-248. Which of the following methods is appropriate for the diagnosis of late occurring toxemia of pregnancy? A) the measurement of blood pressure B) urinalysis C) body weight monitoring D) all of the above E) only answers (A) and (B) are true OBG-258. Which of the following drugs is inappropriate for the alleviation of eclamptic seizures? A) hypnotics B) magnesium sulphate C) diazepam D) dextran (Rheomacrodex) E) lytic cocktail OBG-260. The term "spontaneous abortion" means: A) one or more subsequent pregnancies terminating spontaneously B) the spontaneous termination of a pregnancy DrRognerud@www.medfever.com

OBSTETRICS & GYNECOLOGY

Answer: D

Answer: B

Answer: D

Answer: D

Answer: D

Answer: B

113 / 343

Board Exam Question (Smiley + Star) + 2007 C) that an intact pregnancy.is terminated by artificial instrumentation D) that an intact pregnancy is terminated by an illegal, prohibited procedure E) that fetal death is not followed by an abortion OBG-261. The term "habitual abortion" means: A) one or more subsequent pregnancies terminating spontaneously B) three or more subsequent pregnancies terminating spontaneously C) an intact pregnancy is terminated by artificial instrumentation D) an intact pregnancy is terminated by an illegal, prohibited procedure E) fetal death is not followed by an abortion OBG-262. The term "artificial abortion" means: A) one or more subsequent pregnancies terminating spontaneously B) the pregnancy terminates without any intervention C) an intact pregnancy is terminated by artificial instrumentation D) an intact pregnancy is terminated by an illegal, prohibited procedure E) fetal death is not followed by an abortion OBG-263. The term "criminal abortion" means: A) one or more subsequent pregnancies terminating spontaneously B) an intact pregnancy is terminated by a legal procedure C) an intact pregnancy is terminated by an illegal, prohibited procedure D) fetal death is not followed by an abortion OBG-264. The term "missed abortion" means: A) one or more subsequent pregnancies terminating spontaneously B) an intact pregnancy is terminated by a legal procedure C) an intact pregnancy is terminated by an illegal, prohibited procedure D) fetal death is not followed by an abortion OBG-265. Which of the following may cause spontaneous abortion? A) maternal diseases B) anomalies of the ovum C) diseases of the father D) all of the above E) only answers (A) and (B) are true OBG-268. Which of the following forms of abortion represents the most advanced stage of this condition? A) imminent abortion B) incomplete abortion C) complete abortion DrRognerud@www.medfever.com

OBSTETRICS & GYNECOLOGY

Answer: B

Answer: C

Answer: C

Answer: D

Answer: D

Answer: C

114 / 343

Board Exam Question (Smiley + Star) + 2007 D) incipient abortion OBG-269. Which type of abortion is characterized by cramping lower abdominal pain, vaginal bleeding and a closed cervix? A) incipient abortion B) imminent abortion C) incomplete abortion D) missed abortion E) post-abortion residue OBG-270. Which type of abortion is characterized by cramping lower abdominal pain, vaginal bleeding, an open cervix but no passage of the products of conception? A) incipient abortion B) imminent abortion C) incomplete abortion D) missed abortion E) post-abortion residue OBG-271. Which type of abortion is characterized by cramping lower abdominal pain, vaginal bleeding, an open cervix and the passage of the products of conception? A) incipient abortion B) imminent abortion C) incomplete abortion D) missed abortion E) habitual abortion OBG-272. Which of the following is the most important feature for distinguishing between imminent and incipient abortion? A) the volume of blood loss B) cramps C) the bore of the dilated cervix D) the results of the biological pregnancy test E) serum progesterone level OBG-273. Which of the following is the most important feature for distinguishing between incipient and incomplete abortion? A) the volume of blood loss B) the bore of the dilated cervix C) the passage of the products of conception D) the results of the biological pregnancy test E) serum progesterone level

OBSTETRICS & GYNECOLOGY

Answer: C

Answer: A

Answer: C

Answer: C

Answer: C

DrRognerud@www.medfever.com

115 / 343

Board Exam Question (Smiley + Star) + 2007 OBG-274. In which of the following conditions can the administration of progestogenic agents be considered reasonable for the therapy of imminent abortion? A) if the history contains spontaneous abortion B) if lower abdominal cramps are present C) if bleeding is present D) if the serum progesterone level is low E) in all patients with lower abdominal pain OBG-275. The risks of terminating spontaneous abortions by curettage include: A) infection B) bleeding resulting from intrauterine residue C) uterine perforation and surgical injury D) all of the above E) only answers (A) and (B) are true OBG-281. In obstetrical terms, premature delivery means the termination of pregnancy: A) between weeks 12-16 of gestation B) between weeks 16-28 of gestation C) between weeks 28-37 of gestation D) between weeks 38-40 of gestation E) when the weight of the fetus is 2499 grams or less OBG-283. Is active immunization against mumps and varicella permitted during pregnancy? A) yes B) no C) it is permitted under certain conditions OBG-284. Is passive immunization against mumps and varicella permitted during pregnancy? A) yes B) no C) it is permitted under certain conditions OBG-285. Is passive immunization against rubella permitted during pregnancy? A) yes B) no C) it is permitted under certain conditions OBG-286. When does labour start? A) at the time of full dilation and effacement of the cervix DrRognerud@www.medfever.com

OBSTETRICS & GYNECOLOGY Answer: D

Answer: D

Answer: C

Answer: B

Answer: A

Answer: B

Answer: C

116 / 343

Board Exam Question (Smiley + Star) + 2007 B) at the time when the maximum circumference of the fetal head has descended below the pelvic rim C) at the time when rhythmic, expulsive uterine contractions resulting in the dilation of the cervix begin D) at the time of complete cervical dilation when the patient begins to feel the urge to bear down OBG-287. When does the 1st stage of labor start and end? A) from the time of full cervical effacement to the delivery of the fetus B) from the delivery of the fetus to the expulsion of the placenta C) from the start of uterine contractions to the effacement of the cervix D) this period corresponds to the first 2 hours following the delivery of the placenta E) from the start of uterine contractions to the delivery of the fetus OBG-288. When does the 2nd stage of labor start and end? A) from the time of full cervical effacement to the delivery of the fetus B) from the delivery of the fetus to the expulsion of the placenta C) from the start of uterine contractions to the effacement of the cervix D) this period corresponds to the first 2 hours following the delivery of the placenta E) from the start of uterine contractions to the delivery of the fetus OBG-289. When does the 3rd (placental) stage of labor start and end? A) from the time of full cervical effacement to the delivery of the fetus B) from the delivery of the fetus to the expulsion of the placenta C) from the delivery of the fetus to the end of a 2-hour period following the expulsion of the placenta D) this period corresponds to the first 2 hours following the delivery of the placenta E) from the start of uterine contractions to the delivery of the fetus OBG-290. When does the postplacental stage of labor start and end? A) from the time of full cervical effacement to the delivery of the fetus B) from the delivery of the fetus to the expulsion of the placenta C) from the delivery of the fetus to the end of a 2-hour period following the expulsion of the placenta D) this period corresponds to the first 2 hours following the delivery of the placenta E) from the start of uterine contractions to the delivery of the fetus DrRognerud@www.medfever.com

OBSTETRICS & GYNECOLOGY

Answer: C

Answer: A

Answer: B

Answer: C

117 / 343

Board Exam Question (Smiley + Star) + 2007 OBG-291. "Predictory contractions": A) propel the fetus along the osseous and soft tissue passage after the complete effacement of the cervix B) are uterine contractions resulting in the effacement of the cervix C) induce the separation and delivery of the placenta D) are contractions occurring during the first days of the puerperium E) brief uterine contractions occurring at irregular intervals during the last weeks of pregnancy OBG-303. The term "uterine tone" during labor means: A) the rise of intrauterine pressure during contractions B) the lowest intrauterine pressure measured between contractions C) the frequency of contractions D) the product of multiplying the intensity and the frequency of contractions E) the rise of pressure generated by bearing down OBG-304. The term "intensity of uterine contraction" means: A) the rise of intrauterine pressure during contractions B) the lowest intrauterine pressure measured between contractions C) the frequency of contractions D) the product of multiplying the intensity and the frequency of contractions E) the rise of pressure generated by bearing down OBG-307. The optimal frequency of uterine contractions during the 1st stage of labor is: A) 0-1 contraction/ 10 minutes B) 3-4 contractions/ 10 minutes C) 6-8 contractions/ 10 minutes D) 10-12 contractions/ 10 minutes E) 15-20 contractions/ 10 minutes OBG-308. The average intensity of uterine contractions during the 1 st stage of labor is: A) 10-12 mmHg B) 14-16 mmHg C) 50-55 mmHg D) 70-80 mmHg E) 81-90 mmHg OBG-316. Factors contributing to the development of caput succedaneum include: A) the effect of negative pressure exerted by the cervix on the fetal DrRognerud@www.medfever.com

OBSTETRICS & GYNECOLOGY

Answer: E

Answer: B

Answer: A

Answer: B

Answer: C

Answer: D

118 / 343

Board Exam Question (Smiley + Star) + 2007 head B) the strangulation caused by the contact ring C) fetal hypoxia D) all of the above E) only answers (A) and (B) are true OBG-320. Placental contractions: A) propel the fetus along the osseous and soft tissue passage after the complete effacement of the cervix B) are uterine contractions resulting in the effacement of the cervix C) induce the separation and delivery of the placenta D) are contractions occurring during the first days of the puerperium E) brief uterine contractions occurring at irregular intervals during the last weeks of pregnancy OBG-322. The term "early rupture of membranes" means: A) that membranes rupture after the effacement of the cervix B) that membranes rupture before the beginning of uterine contractions C) that membranes rupture after the start of uterine contractions but before the effacement of the cervix D) that membranes rupture before the fetal head has engaged the pelvic brim OBG-325. What is the average duration of the 1 st stage of labor in nulliparous women? A) 1-2 hours B) 3-4 hours C) 6-8 hours D) 16-20 hours E) 20-24 hours OBG-326. What is the average duration of the 1st stage of labor in multiparous women? A) 1-2 hours B) 4-6 hours C) 6-8 hours D) 8-10 hours E) 10-12 hours OBG-332. In which section of the birth canal does the fetal skull perform its second rotation during cephalic position delivery? A) at the pelvic brim B) in the cavity of the pelvis C) at the pelvic outlet DrRognerud@www.medfever.com

OBSTETRICS & GYNECOLOGY

Answer: C

Answer: C

Answer: C

Answer: B

Answer: B

119 / 343

Board Exam Question (Smiley + Star) + 2007 D) outside the vulva OBG-333. In which section of the birth canal does the fetal skull perform its third rotation during cephalic position delivery? A) at the pelvic brim B) in the cavity of the pelvis C) at the pelvic outlet D) outside the vulva OBG-334. In which section of the birth canal does the fetal skull perform its fourth rotation during cephalic position delivery? A) at the pelvic brim B) in the cavity of the pelvis C) at the pelvic outlet D) outside the vulva OBG-336. The signs of complete placental separation include: A) ridging of the uterus B) the umiblical cord is not retracted by pressure applied to the lower abdomen above the symphysis C) the umbilical cord is not retracted after bearing down D) all of the above suggest separation of the placenta E) only answers (A) and (B) are true OBG-337. The Tsukhaloff-Kiistner's sign A) is elicited by pressing the lower abdomen while observing the retraction of the umbilical cord B) means that the uterus loses its globoid shape and becomes flattened and flaccid C) means that the umbilical cord is not retracted after bearing down D) means that the separated placenta is expressed from the uterine cavity like a seed of a plum by applying pressure to the uterine fundus E) means rubbing the fundus in order to elicit uterine contraction OBG-338. Klein's sign A) means that the umbilical cord is not retracted when pressure is applied to the lower abdomen above the symphysis B) means that the uterus loses its globoid shape and becomes flattened and flaccid C) means that the umbilical cord is not retracted after bearing down if the separation of the placenta is complete D) means that the separated placenta is expressed from the uterine cavity like a seed of a plum by applying pressure to the DrRognerud@www.medfever.com

OBSTETRICS & GYNECOLOGY

Answer: C

Answer: D

Answer: D

Answer: A

Answer: C

120 / 343

Board Exam Question (Smiley + Star) + 2007 uterine fundus OBG-339. Where can the upper pole of the uterine fundus be found after delivery of the placenta? A) at the umbilical level B) about 3 centimetres above the symphysis C) about 6 centimetres above the symphysis D) at the level of the symphysis E) in the pelvic cavity OBG-340. Where can the upper pole of the uterine fundus be found on the first day of the puerpuerium? A) about 3 centimetres above the umbilicus B) at the umbilical level C) about 6 centimetres above the symphysis D) at the level of the symphysis E) in the pelvic cavity OBG-341. Which of the following should be determined on admission to the delivery room? A) the presenting part should be identified and its relative location to the pelvic brim should be determined B) the integrity of the fetal membranes and the color of the amniotic fluid C) fetal cardiac function and the dilation of the cervix should be assessed D) uterine contractions should be evaluated E) all of the above OBG-342. Which of the following features of expulsive contractions can be assessed by palpation? A) frequency B) duration C) intensity D) basal tone E) all of the above OBG-343. . Which of the following cannot be determined by vaginal examination performed during labor? A) the degree of dilation and effacement of the cervix B) the integrity of fetal membranes C) neither the presenting part nor its position in the birth canal can be determined D) the risk of fetal hypoxia DrRognerud@www.medfever.com

OBSTETRICS & GYNECOLOGY

Answer: C

Answer: B

Answer: E

Answer: E

Answer: D

121 / 343

Board Exam Question (Smiley + Star) + 2007 E) the location of the governing point and path OBG-345. The essentials of active management of the 3rd stage of labor include: A) after the delivery of the fetus, the uterine fundus is rubbed in order to aid in the firm contraction of the uterus B) after the delivery of the fetus, the placenta is removed by Crede's maneuver C) an oxytocic drug is given immediately after the delivery of the fetus D) after the delivery of the fetus, the separation and expulsion of the placenta is aided by exerting traction on the umibilical cord E) the separation of the placenta should be patiently waited for, without rubbing or massaging the uterus OBG-347. In which of the following cases should the delivered placenta be examined? A) if the separation of the placenta was difficult B) if the 3rd stage of labor was abnormal during a previous pregnancy C) if the delivered placenta is fragmented D) if there are multiple abortions in the past medical history E) meticulous examination is mandatory in all cases OBG-349. Pain associated with labor is caused by: A) the dilation of the cervix B) traction of the uterine ligaments and the peritoneum C) compression of blood vessels and associated ischemia of uterine tissues D) all of the above E) only answers and (B) are true OBG-367. The consequences of prolonged labor include: A) ascending uterine infection B) intrauterine asphyxia C) bleeding resulting from uterine atony D) all of the above E) only answers (A) and (B) are true OBG-368. The prevalence of breech presentation in premature labor is: A) 1-2% B) 3-4% C) 10-12% D) 30-40% OBG-369. The prevalence of breech presentation in all deliveries is: DrRognerud@www.medfever.com

OBSTETRICS & GYNECOLOGY

Answer: C

Answer: E

Answer: D

Answer: D

Answer: C

Answer: B 122 / 343

Board Exam Question (Smiley + Star) + 2007 A) 1-2% B) 4-5% C) 15-20% D) 30-40% OBG-373. Which of the following is the presenting part in single feet presentation? A) the buttocks and one of the feet B) both knees C) one of the knees D) one of the legs E) both legs OBG-374. Which of the following is the presenting part in double feet presentation? A) the buttocks and one of the feet B) both knees C) one of the knees D) both legs E) one of the legs OBG-377. Which of the following is the presenting part in incomplete breechfeet presentation? A) the buttocks B) the buttocks and both feet C) the buttocks and one of the feet D) both knees E) one of the knees OBG-378. Which of the following is the presenting part in double knee presentation? A) the buttocks and both knees B) both knees C) one of the knees only D) one of the knees and the contralateral foot OBG-379. Which of the following is the presenting part in single knee presentation? A) the buttocks and both knees B) . both knees C) one of the knees only D) one of the knees and the contralateral foot OBG-380. Which of the following methods is appropriate for the detection of breech presentation? A) vaginal examination B) ultrasonography C) radiography DrRognerud@www.medfever.com

OBSTETRICS & GYNECOLOGY

Answer: D

Answer: D

Answer: C

Answer: B

Answer: C

Answer: E

123 / 343

Board Exam Question (Smiley + Star) + 2007 D) fetal ECG E) all of the above OBG-381. Compared to cephalic presentation delivery, which of the following represent an increased disadvantage during breech delivery? A) the fetal head compresses the umbbilical cord B) the fetal head has no chance to mold to fit pelvic size C) upward displacement of the fetals arms may cause fracture D) all of the above E) breech presentation carries no additional disadvantage over cephalic presentation delivery OBG-382. Where does venous and arterious blood mix in the fetal circulation? A) the umbilical vein supplies the liver and coalesces to form the branches of the hepatic vein then joins the inferior vena cava B) considered an extension of the umbilical vein the ductus venosus of Arandi joins the inferior vena cava C) both the inferior and the superior vena cava empty into the right atrium and their blood is mixed there D) the ductus arteriosus (Botallo's duct), a branch of the pulmonary artery, empties int6 the aorta OBG-384. How many days does the postnatal obliteration of the ductus venosus take? A) it is obliterated immediately after birth, during the first breath B) 1-2 days C) 2-3 days D) 5-10 days E) 20-30 days OBG-385. What is the cause of the increased heat loss off neonates and their susceptibility to hypothermia? A) the ratio of body surface area to body mass is significantly higher in neonates than in adults B) the subcutaneous fat layer is relatively thin C) the activity of the thermoregulation centre is unstable D) all of the above have a role in the development of hypothermia E) only answers (A) and (B) are true OBG-393. The term "embryopathy" means: A) fetal damage sustained between week 13 of gestation and the time of birth B) fetal damage susteined between weeks 2 and 13 of gestation C) fetal damage sustained between the time of conception and day 14 of gestation DrRognerud@www.medfever.com

OBSTETRICS & GYNECOLOGY

Answer: D

Answer: B

Answer: D

Answer: D

Answer: B

124 / 343

Board Exam Question (Smiley + Star) + 2007 D) damage to the ovum before fertilization resulting in the development of various congenital malformations OBG-394. The term "fetopathy" means: A) fetal damage sustained between week 13 of gestation and the time of birth B) fetal damage susteined between weeks 2 and 13 of gestation C) fetal damage sustained between the time of conception and day 14 of gestation D) damage to the ovum before fertilization resulting in the development of various congenital malformations OBG-405. Which of the following conditions should be considered if meconium ileus develops? A) duodenal atresia B) intestinal atresia C) cystic fibrosis D) phenylketonuria E) biliary atresia OBG-406. Congenital dysplasia of the hip is more prevalent in: A) boys B) girls C) there is no gender-specific difference OBG-407. The chromosomal abnormality characteristic of Down's syndrome includes: A) trisomy G21 B) D/G translocation C) G/G translocation D) all of the above E) only answers (A) and (B) are true OBG-408. The chromosomal abnormality characteristic of Patau's. syndrome includes: A) trisomy G21 B) trisomy D C) D/G translocation D) G/G translocation E) 45,X0 caryotype OBG-415. What is the prevalence of congenital malformations in neonates born to diabetic mothers? A) 1% DrRognerud@www.medfever.com

OBSTETRICS & GYNECOLOGY

Answer: A

Answer: C

Answer: B

Answer: D

Answer: B

Answer: C

125 / 343

Board Exam Question (Smiley + Star) + 2007 B) 2% C) 10% D) 30% E) 60% OBG-416. In Rh-isoimmunization, exhange transfusion is indicated if the serum bilirubin level is higher in the first 24 hours than: A) 10 mmol/1 B) 20 mmol/1 C) 50 mmol/1 D) 137 mmol/1 E) 250 mmol/1 OBG-418. What is the time limit for effective anti-D IgG administration to Rhnegative women after delivery? A) 1 day B) 2 days C) 3 days D) 4 days E) 5 days OBG-420. Which of the following factors induces the differentiation of the gonads? A) testosterone B) estrogens C) androgens produced by the adrenal cortex D) sex chromosomes E) pituitary hormones OBG-421. Which of the following factors induces the deveopment of the genital tract and external genitalia in male fetuses? A) testosterone B) the combined effect of estrogen and testosterone C) sex chromosomes D) pituitary hormones E) neither hormonal nor chromosomal effects are needed as male sexual differentation is always the default OBG-422. Which of the following factors induces the deveopment of the genital tract and external genitalia in female fetuses? A) testosterone BY the combined effect of estrogen and testosterone C) sex chromosomes D) pituitary hormones E) neither hormonal nor chromosomal effects are needed as female sexual differentation is always the default DrRognerud@www.medfever.com

OBSTETRICS & GYNECOLOGY

Answer: D

Answer: C

Answer: D

Answer: A

Answer: E

126 / 343

Board Exam Question (Smiley + Star) + 2007 OBG-423. Which of the following features of gender develop during intrauterine life in humans? A) chromosomal and gonadal gender B) gonadal and genital gender C) chromosomal, gonadal and genital gender D) chromosomal, gonadal, genital and somatic gender E) gonadal, genital and somatic gender OBG-424. Which of the following features of gender develop during extrauterine life in humans? A) genital and somatic gender B) gonadal, genital and somatic gender C) genital, somatic and psychosexual gender D) somatic and psychosexual gender OBG-432. The karyotype characteristic of testicular feminisation is: A) 46, XX B) 46, XY C) 45, XO D) 47, XXY E) 47, XYY OBG-444. The karyotype characteristic of Turners syndrome is: A) 46, XX B) 46, XY C) 45, XO D) 47, XXY E) 47, XYY OBG-445. Which of the following gonads can be found in a patient with Turner's syndrome? A) the ovaries B) the testes C) ovotestis D) "streak" gonad OBG-447. The karyotype characteristic of pure gonadal dysgenesis is: A) 46, XX B) 46, XY C) 45, XO D) 47, XXY E) 47, XYY

OBSTETRICS & GYNECOLOGY

Answer: C

Answer: D

Answer: B

Answer: C

Answer: D

Answer: A

DrRognerud@www.medfever.com

127 / 343

Board Exam Question (Smiley + Star) + 2007 OBG-450. The most frequent cause of precocious puberty is: A) hormone producing ovarian neoplasm B) adrenal neoplasm C) brain tumor D) dysfunction of the adrenal cortex E) hypothyroidism OBG-451. In delayed puberty, no signs of sexual maturation manifest themselves until the age of: A) 12 B) 13 C) 14 D) 15 E) 16 OBG-456. Which of the following therapeutic methods should be applied for controlling bleeding in juvenile metropathia? A) curettage B) hysteroscopy C) hormonal (chemical) abrasion D) suction curettage E) no therapy is necessary, as this condition is transitory and selflimiting OBG-457. Which of the following drugs is inappropriate for (chemical) hormonal abrasion? A) ethinylestradiol (Mikrofollin) B) norethisteron (Norcolut) C) lynestrenol (Orgametril) D) methylestrenolon (Orgasteron) E) progesterone (Glanducorpin) OBG-459. Which of the following may cause vaginal discharge of non-infectious origin in puberty? A) estrogen deficiency B) psychogenic factors C) sideropenia D) all of the above E) only answers (A) and (B) are true OBG-462. In which of the following cases is amenorrhea considered normal? A) in pregnancy and during lactation B) in postmenopausal women C) before the onset of menarche D) in all of the above cases DrRognerud@www.medfever.com

OBSTETRICS & GYNECOLOGY

Answer: A

Answer: D

Answer: C

Answer: A

Answer: D

Answer: D

128 / 343

Board Exam Question (Smiley + Star) + 2007 E) only answers (A) and (B) are true OBG-473. What is the likely cause of vaginal bleeding if the estrogen and estrogen-progesteron challenge tests as well as gonadotropin levels are normal and the absence of pregnancy is certain? A) hypothalamic dysfunction B) pituitary dysfunction C) ovarian dysfunction D) uterine dysfunction E) dysregulatory dysfunction OBG-477. A hyperprolactinoma of the pituitary causes: A) gigantism and acromegalia B) Cushing's disease C) hyperprolactinemia D) Sheehan's syndrome E) Cushing's syndrome OBG-507. In which phase of the menstrual cycle should hysterosalpingography be performed? A) in the early follicular phase B) at the time of ovulation C) in the early secretory phase D) in the late secretaory phase OBG-508. Which of the following methods is appropriate for the detection of uterine malformations? A) hysterosalpingography B) hysteroscopy C) laparotomy (laparoscopy) D) all of the above E) only answers (A) and (B) are true OBG-512. When should curettage and endometrial histology be performed in order to verify the occurrence of ovulation? A) at the time of menstruation B) on the week following menstruation C) at midcycle D) on the week preceeding menstruation E) this method is not applicable in the diagnostics of the menstrual cycle OBG-516. Which of the following drugs is not effective for ovulation induction? A) human menopausal gonadotropin DrRognerud@www.medfever.com

OBSTETRICS & GYNECOLOGY

Answer: D

Answer: C

Answer: A

Answer: D

Answer: D

Answer: D

129 / 343

Board Exam Question (Smiley + Star) + 2007 B) clomiphene citrate C) LH-releasing hormone D) synthetic oxytocin E) human pituitary gonadotropin OBG-518. Which of the following methods is appropriate for the detection of the anatomical abnormalities of the uterus and Faloppian tubes? A) hysterosalpingography B) ultrasonography C) laparotomy (laparoscopy) D) all of the above E) only answers (A) and (C) are true OBG-519. The term "birth rate" means: A) the number of live.births per 100 inhabitants B) the number of live births per 1,000 inhabitants C) the number of live births per 10,000 inhabitants D) the percentage of pregnancies ending with delivery E) the percentage of pregnancies where live, healthy neonates are delivered OBG-529. What are the components of combined oral contraceptive preparations? A) estrogen only B) progesterone only C) all tablets contain both estrogen and progesterone D) the first 14 pills contain estrogen, the next 7 pills contain estrogen and progesterone OBG-531. What are the components of the "minipill"? A) estrogen only B) progesterone only C) all tablets contain both estrogen and progesterone in small doses D) the first 14 pills contain estrogen, the next 7 pills contain estrogen and progesterone OBG-532. Which of the following Hungarian preparations is a combined contraceptive? A) Rigeuidon B) Tri-Regol C) Ovidon D) all of the above OBG-533. Which of the following is a minipill contraceptive containing progesterone only? A) Tri-Regol DrRognerud@www.medfever.com

OBSTETRICS & GYNECOLOGY

Answer: D

Answer: B

Answer: C

Answer: B

Answer: D

Answer: E

130 / 343

Board Exam Question (Smiley + Star) + 2007 B) Marvelon C) Ovidon D) Rigevidon E) Continum OBG-534. How does the menses change under the effect of combined oral contraceptives? A) it becomes more copious B) it becomes reduced in volume C) it remains unchanged D) it becomes reduced in volume, and the intensity of cramping is also reduced E) the discharge of menstrual secretions becomes prolonged OBG-535. What should be done if breakthrough bleeding occurs dining the use of oral contraceptives? A) nothing, as this condition resolves spontaneously B) the patient should suspend the use of contraceptives for 7 days then restart taking the tablets C) the number of tablets taken should be increased; perhaps the actual regimen should be supplemented by an estrogen preparation D) switch to another contraceptive preparation E) oral contraceptives should be stopped and an IUD should be Inserted OBG-536. The most appropriate protocol for oral contraceptive use is: A) to always start from the first day of menstruation and continue taking the tablets for the next 21 days B) 21 days on the pill followed by a 7-day pause C) the tablets should be taken continuously, without interruption D) depending on the length of the menstrual cycle, 18-35 days on the pill followed by a 7-day pause E) the tablets should be taken according to the calendar, from the first day of the month to the 21 st day OBG-537. The most appropriate protocol for the minipill is: A) to start always from the first day of menstruation and continue taking the tablets for the next 21 days B) 21 days on tablets followed by a 7-day pause C) the tablets should be taken continuously, without interruption D) the tablets should be taken according to the calendar, from the first day of the month to the 21 st day OBG-538. The contraindications to oral contraceptive use include: A) thromboembolism B) liver disease DrRognerud@www.medfever.com

OBSTETRICS & GYNECOLOGY

Answer: D

Answer: C

Answer: B

Answer: C

Answer: D

131 / 343

Board Exam Question (Smiley + Star) + 2007 C) endocrine disorders D) all of the above E) only answers (A) and (B) are true OBG-539. The contraindications to oral contraceptive use include: A) malignant neoplasms of the breast or the genitals B) diabetes C) hypertension and advanced renal disease D) pregnancy E) all of the above OBG-540. Which of the following contraceptives are appropriate for nursing mothers? A) ethinyl estradiol / norgestrel B) ethinyl estradiol/ desogestrel C) ethynodiol diacetate D) none of the above OBG-544. The adverse effects of intrauterine devices include: A) bleeding B) pain C) pelvic inflammatory disease D) all of the above E) only answers (A) and (C) are true OBG-545. What type of contraception is appropriate for nulliparous women? A) oral contraceptives B) intrauterine device C) conventional methods D) only answers (A) and (C) are true E) there is no effective method OBG-547. Which of the following belongs to the requirements for inserting IUDs? A) all licensed family practitioners are allowed to insert IUDs B) all specialists are allowed to insert IUDs at a polyclinic C) IUDs should be inserted only at the hospital D) IUDs should be inserted by the professionals at university . clinic gynecologic care and consulting services E) family and gynecologic care services with an institutional background are allowed to insert IUDs OBG-548. In women of reproductive age, the optimal period for inserting an IUD is: DrRognerud@www.medfever.com

OBSTETRICS & GYNECOLOGY

Answer: E

Answer: C

Answer: D

Answer: D

Answer: E

Answer: B

132 / 343

Board Exam Question (Smiley + Star) + 2007 A) the first day of menstruation B) between days 4 and 6 of the menses C) 2-3 days before the expected time of menstruation D) the IUD can be inserted at any time OBG-549. When should the IUD be inserted during lactation? A) only if regular menstruation has already returned B) if at least one menstruation has occurred since delivery C) 6-8 weeks after the delivery, if the possibility of pregnany can be excluded D) only 5 months after the delivery E) the use of IUDs is contraindicated in the period of lactation OBG-550. Contraindications to IUD insertion inlcude: A) pelvic inflammatory disease B) genital malignancies C) pregnancy D) all of the above E) only answers (A) and (C) are true OBG-554. The onset of menopause is premature before the age of: A) 40 years-old B) 43 years-old C) 50 years-old D) 52 years-old E) 55 years-old OBG-555. The menopause is delayed if uterine bleeding due to the cyclic changes of ovarian hormones occurs before the age of: A) 45 years-old B) 48 years-old C) 50 years-old D) 52 years-old E) 55 years-old OBG-558. Case Study: A 45-year-old female presents with metrorrhagia. The proper therapy of this condition includes: A) no intervention is necessary as failing ovarian function normally results in metrorrhagia at this age B) oxytocic agents should be administered to control bleeding C) chemical curettage by hormone therapy, followed by cyclic administration of estrogen and progesterone D) fractional curettage should be performed to exclude malignancy E) hysterectomy is indicated as the incidence of uterine malignancies DrRognerud@www.medfever.com

OBSTETRICS & GYNECOLOGY

Answer: C

Answer: D

Answer: C

Answer: E

Answer: D

133 / 343

Board Exam Question (Smiley + Star) + 2007 is extremely high at this age OBG-559. Which of the following hormones has the greatest importance in the atrophization of the genitals in climacteric women? A) estrogens B) progesterone C) androgens D) follicle stimulating hormone E) luteinizing hormone OBG-562. Which of the following drugs is the most appropriate for the treatment of climacteric symtpoms (e.g. hot flushes, perspiration, palpitation, etc.)? A) conjugated estrogens B) gestogens C) synthetic estrogens D) androgens E) the combination of androgens and estrogens OBG-564. Which of the following estrogen preparations acts on the vaginal epithelium primarily? A) ethinyl estradiol (Mikrofollin) B) estradiol (Akrofollin) C) estriol (Ovestin) D) dienestrol (Dienoestrol) OBG-565. Case Study: A 25-year-old woman with premature menopause would like to have a child. Which of the following would you recommend? A) pregnancy would be feasible by in vitro fertilization B) therapy with clomiphene citrate as this is successful in most cases C) Pergonal (FSH+LH) therapy is the only chance D) there is no remedy for this condition as it represents irreversible infertility E) laparotomy and wedge resection of the ovaries OBG-571. Which of the following infections is associated with vaginal discharge and itching? A) gonococcal infection B) Trichomonas vaginalis infection C) fungal infections D) bacterial infections E) viral infections OBG-572. Which of the following is the pathogen of condylomata acuminata DrRognerud@www.medfever.com

OBSTETRICS & GYNECOLOGY

Answer: A

Answer: A

Answer: C

Answer: D

Answer: C

Answer: B 134 / 343

Board Exam Question (Smiley + Star) + 2007 (moist warts)? A) bacteria B) viruses C) fungi D) Trichomonas vaginalis E) Treponema pallidum OBG-575. Characteristic macroscopic features of vaginal discharge in Trichomonas vaginalis infection include: A) thin and milky discharge B) frothy, greenish and purulent discharge C) the vaginal wall is covered by easily removable, whitefish-grey material D) thin, whitefish-grey discharge from the cervix E) there are no characteristic macroscopic features OBG-576. Characteristic macroscopic features of vaginal discharge in fungal infections include: A) thin and milky discharge B) frothy, greenish and purulent discharge C) the vaginal wall is covered by easily removable, whitefish-grey material D) thin, whitefish-grey discharge from the cervix E) there are no characteristic macroscopic features OBG-577. Which of the following drugs is used for the treatment of vaginal trichomoniasis? A) metronidazol (Klion) tablets and vaginal suppositories B) natamycine (Pimafucin) vaginal tablets C) clotrimazole (Canesten) tablets D) all of the above E) only answers (A) and (B) are true OBG-578. Which of the following lesions is considered a precancerous stage of chorionic carcinoma? A) cystic adenomatous hyperplasia of the endometrium B) leukoplakia of the portio vaginalis of the cervix C) hydatidiform mole D) chronic cervicitis E) placenta residues OBG-579. Which of the following hormones is secreted by chorionic carcinomas? A) estrogen B) progesterone C) androgens DrRognerud@www.medfever.com

OBSTETRICS & GYNECOLOGY

Answer: B

Answer: C

Answer: A

Answer: C

Answer: D

135 / 343

Board Exam Question (Smiley + Star) + 2007 D) human chorionic gonadotropin E) follicle stimulating hormone OBG-580. Which of the following gynecological malignancies are associated with the positivity of pregnancy tests? A) endometrial carcinoma B) cervical carcinoma C) chorionic carcinoma D) hormonally active ovarian tumors E) ovarian cystadenocarcinomas OBG-581. Which of the following obstetrical events are associated with the subsequent development of chorionic carcinoma most frequently? A) delivery B) abortion C) hydatidiform mole D) ectopic pregnancy E) missed abortion OBG-583. Which of the following tests is the most reliable for diagnosing chorionic carcinoma at the earliest possible time? A) Rana reaction B) Gravimun test C) Menotest D) detection of hCG b-subunits by RIA E) Ascheim-Zondek reaction OBG-584. Which of the following organs is involved in the metastatic spread of chorionic carcinoma most fregently? A) the vagina B) the lung and brain C) the liver and the kidneys D) all of the above E) the vagina, lung and brain only OBG-587. Cytotoxic chemotherapy gives the best results in the therapy of: A) ovarian cystadenocarcinoma B) endometrial adenocarcinoma C) mesonephroid tumors D) chorionic carcinoma E) sarcoma OBG-589. Which of the following is a complication of benign ovarian tumors? A) torsion of the pedicle of the cyst DrRognerud@www.medfever.com

OBSTETRICS & GYNECOLOGY

Answer: C

Answer: C

Answer: D

Answer: D

Answer: D

Answer: D

136 / 343

Board Exam Question (Smiley + Star) + 2007 B) infection, abscess formation C) malignant transformation D) all of the above E) benign ovarian tumors cause no complications OBG-591. Characteristic signs of Meigs' syndrome include: A) pleural effusion B) ascites C) ovarian fibromyoma D) all of the above E) bilateral, smooth and firm mobile ovarian tumors OBG-596. Stage I ovarian carcinoma neoplastic disease involves: A) one of the ovaries and fallopian tubes only B) one or both ovaries C) one or both ovaries and the pelvis D) one or both ovaries and various intraabdominal organs E) one or both ovaries and gives metastases to distant sites OBG-597. Stage II ovarian carcinoma neoplastic disease involves: A) one of the ovaries and fallopian tubes only B) one or both ovaries C) one or both ovaries and the pelvis D) one or both ovaries and various intraabdominal organs E) one or both ovaries and gives metastases to distant sites OBG-598. Stage III ovarian carcinoma neoplastic disease involves: A) one of the ovaries and fallopian tubes only B) one or both ovaries C) one or both ovaries and the pelvis D) one or both ovaries and various intraabdominal organs E) one or both ovaries and gives metastases to distant sites OBG-599. Stage IV ovarian carcinoma neoplastic disease involves: A) one of the ovaries and fallopian tubes only B) one or both ovaries C) one or both ovaries and the pelvis D) one or both ovaries and various intraabdominal organs E) one or both ovaries and gives metastases to distant sites OBG-600. Carcinoma of the ovary is most frequently treated by: A) cytotoxic chemotherapy B) surgery only C) surgery followed by irradiation DrRognerud@www.medfever.com

OBSTETRICS & GYNECOLOGY

Answer: D

Answer: B

Answer: C

Answer: C

Answer: D

Answer: D

137 / 343

Board Exam Question (Smiley + Star) + 2007 D) surgery followed by cytotoxic chemotherapy E) irradiation only OBG-601. In metastatic tumors of the ovary, the primary neoplasm is found most frequently in the: A) breast B) pancreas C) gastrointestinal tract D) lung E) trachea OBG-602. Dysgerminoma of the ovary develops from: A) cells of the ovarian stroma B) hilar Leydig-cells of the ovary C) undifferentiated cells of the germinal epithelium D) granulosa cells of atretic follicles E) undifferentiated chorionic cells OBG-603. Ovarian calcification is visible on anteroposterior radiographs of the pelvis in: A) endometriosis of the ovary B) parovarian cyst C) cystic ovarian adenocarcinoma D) dermoid tumors of the ovary E) sarcoma OBG-607. Which of the following screening methods is appropriate for the detection of ovarian neoplasms? A) a gynecologic examination every 6-12 months B) ultrasonography every 6-12 months C) lavage and aspiration cytology of the cul-de-sac every 12 months D) methods (A) and (B) are used primarily E) there is` no reliable method for the detection of ovarian neoplasms OBG-608. The diganosis of endometriosis refers to: A) fibrosis resulting from chronic endometritis B) the presence of extrauterine endometrium implants C) the precancerous stage of endometrial carcinoma D) viral endometritis E) endometrial lesions caused by IUDs OBG-609. The diganosis of endometriosis refers to: A) fibrosis resulting from chronic endometritis B) the presence of extrauterine endometrium implants DrRognerud@www.medfever.com

OBSTETRICS & GYNECOLOGY

Answer: C

Answer: C

Answer: D

Answer: D

Answer: B

Answer: B

138 / 343

Board Exam Question (Smiley + Star) + 2007 C) the precancerous stage of endometrial carcinoma D) viral endometritis E) endometrial lesions caused by IUDs OBG-611. In which of the following periods of life is endometriosis of significance? A) in the neonatal age B) in infancy C) in the reproductive age D) in the postmenopausal age E) in advanced age OBG-612. The characteristic signs of intrauterine endometriosis include: A) dysmenorrhea B) hypermenorrhea C) sterility D) a firm, uniformly enlarged uterus E) all of the above F) only answers (A) and (C). are true OBG-614. The contents of the ovarian cyst resembles chocolate or tar in: A) fibromyoma B) adenocarcinoma C) endometriosis D) necrotized dysgerminoma E) hemorrhagic corpus luteum cyst OBG-615. Which ofthe following drugs is inapprorpiate for the therapy of endometriosis? A) danazol (DTIC-DOME) (a synthetic androgen) B) lynestrenol (Orgametril) C) norethisteron (Norcolut) D) ethinylestradiol (Mikrofollin forte) E) hydroxyprogesterone acetate OBG-616. Which ofthe following drugs is recommended for the therapy of endometriosis? A) stilbene derviatives B) natural estrogens C) progestogenic norsteroid compounds (19-nortestosterorone derivatives) D) natural progesterone derivatives E) natural androgens

OBSTETRICS & GYNECOLOGY

Answer: C

Answer: E

Answer: C

Answer: D

Answer: C

DrRognerud@www.medfever.com

139 / 343

Board Exam Question (Smiley + Star) + 2007 OBG-619. The ratio between the length of the uterine corpus and the cervix is normally: A) 1:1 B) 2:1 C) 3:1 D) 1:2 E) 1:3 OBG-620. In uterine hypoplasia, the ratio between the length of the uterine corpus and the cervix is normally: A) 1:1 B) 2:1 C) 3:1 D) 1:2 E) 1:3 OBG-627. In total uterine prolapse: A) the uterus descends beneath the level of the hymenal ring and the portio vaginalis of the cervix appears in the vaginal introitus B) despite the descensus of the uterus, the portio vaginalis of the cervix does not sink beneath the level of the hymenal ring C) the uterus is found before the introitus, in the prolapsed vaginal sac D) the position of the uterine corpus is normal but the cervix is significantly elongated OBG-628. Congenital malformations of the genitals are freqently associated with congenital abnormalities of the: A) urinary tract B) kidneys C) rectum D) all of the above E) only answers (A) and (B) are true OBG-648. Case Study: The history of a 28-year-old patient contains freqently recurring adnexitis. Her last regular menses occurred 37 days before admission. Complaints: slight, brownish vaginal discharge occurred 10 days be* fore, at the time of the last menses. On the morning of her admission the patient "felt ill" and lost her consciousness for a few minutes. The pregnancy test performed several days earlier had been positive. On admission: her face is pale, sweating, her extremities are cold and she is complaining about dizzines, weakness, severe pain in the left-lower abdomen associated with a slight urge to defecate. Heart rate: 120/ DrRognerud@www.medfever.com

OBSTETRICS & GYNECOLOGY Answer: B

Answer: B

Answer: C

Answer: E

Answer: D

140 / 343

Board Exam Question (Smiley + Star) + 2007 min, blood pressure 90/60 mmHg, body temperature 36.8 C; WBC: 7,300/.l, Hb: 5,9 mmol/l, Hct: 28%. Gynecologic examination: local bulging of the abdomen, moderate tenderness up to the umbilical level; small-volume brownish vaginal discharge, the portio vaginalis of the cervix is cyanotic, tender to motion, the uteris is in anteflexionanteversion, it is slightly enlarged and softened. The right adnexum is not palpable; a vague, moderately tender adnexal mass of a size of a plum is palpable on the right side. The cul-de-sac is bulging and yields a large volume of clotted blood on diagnostic puncture. What is the most likely diagnosis? A) perforation of the gall-bladder B) acute appendicitis C) nephrolithiasis D) ectopic pregnancy E) acute adnexitis

OBSTETRICS & GYNECOLOGY

MULTIPLE CHOICE QUESTIONS WITH KEY ANSWERS / TYPE II Every question or incomplete statement has only one answer in the following combinations: A) if the answers 1, 2, and 3 are true B) if the answers 1 and 3 are true C) if the answers 2 and 4 are true D) if only the answer 4 is true E) if all the four answers are true Select one of these key combinations!!! OBG-650. Certain signs of pregnancy include: 1) the detection of active fetal motions by the examiner 2) Hegar's sign 3) detection of the fetus by ultrasonography 4) a blowing murmur from the uterus OBG-652. Sexually transmitted diseases include: 1) type II Herpesvirus infection 2) Trichomonas infection 3) non-gonococcal urethritis 4) condylomata acuminata OBG-654. Premature delivery starting without cervical dilation or early rupture of the fetal membranes can probably be stopped by the administration of: 1) morphine sulphate 2) intravenous alcohol 3) barbiturates 4) ritodrine

Answer: B

Answer: E

Answer: C

DrRognerud@www.medfever.com

141 / 343

Board Exam Question (Smiley + Star) + 2007 OBG-658. Methods appropriate for the study of fetal chromosomes include: 1) amniocentesis 2) cordocentesis 3) chorionic villous sampling 4) echo-Doppler duplex flowmetry OBG-677. Which problems of postmenopeusal women deserve special attention? 1) vaginitis 2) depression 3) osteoporosis 4) sexual dysfunction OBG-679. Valid statements regarding menopause include: 1) it usually starts between the age of 40 to 50 years 2) is characterized by the absence of menstruation for 12 months in women older than 45 years 3) menopause is always preceded by hot flushes 4) FSH and LH levels are elevated OBG-681. The pharmacological effects of oral contraceptives containing the combination of estrogen and progesterone include: 1) the inhibition of the maturation of the oocyte 2) the prevention of the penetration of sperm into the cervical mucus 3) the inhibition of implantation by the induction of atrophic changes of the endometrium 4) the induction of uterotubal hypermotility inhibits sperm motility OBG-687. Which of the following laboratory parameters is expected to yield elevated values during pregnancy? 1) the serum albumin 2) the plasma fibrinogen 3) the blood urea nitrogen 4) the erythrocyte sedimentation rate OBG-689. Which of the following factors may induce ureteral dilation during pregnancy? 1) compression exerted by the pregnant uterus 2) external compression due to the dilated right ovarian vein 3) the effect of progesterone 4) increasesd glomerular filtration rate OBG-704. Permanent estrogen replacement may be dangerous or explicitly contraindicated in women with: DrRognerud@www.medfever.com

OBSTETRICS & GYNECOLOGY Answer: A

Answer: E

Answer: C

Answer: A

Answer: C

Answer: A

Answer: A

142 / 343

Board Exam Question (Smiley + Star) + 2007 1) liver dysfunction 2) thromboembolic disorders 3) estrogen dependent neoplasms 4) have a mother or siblings with osteoporosis OBG-706. Absolute contraindications to the use of oral contraceptives include: 1) thromboembolisc disorders 2) congenital hyperlipidemia 3) obesity and smoking for 35 years 4) ectopic pregnancy in the patient's history OBG-707. Contraindications to the insertion of an IUD include: 1) pelvic inflammatory disease in the patient's history 2) previous conception despite IUD use 3) abnormal genital bleeding 4) previous wedge resection of the cervix OBG-712. Valid statements regarding identical twins include: 1) identical twins often result after the use of intrauterine devices 2) identical twins often result after ovulation induction with clomiphene citrate 3) the development of identical twins is more common than that of fraternal twins 4) the incidence of this condition is 1:250 pregnancies OBG-714. Ultrasonography performed in the third trimester detects: 1) anencephaly and major neural tube defects 2) fetal death 3) polyhydramnios 4) the accurate age of pregnancy OBG-716. Risks associated with smoking during pregnancy and the puerperium include: 1) the delivery of a low birth-weight neonate 2) spontaneous abortion 3) premature delivery 4) sudden infant death syndrome OBG-717. Severe fetal or neonatal disease may result from maternal infection by which of the following viruses? 1) Coxsackie B virus 2) Rubellavirus 3) Smallpox virus 4) type 2 Herpesvirus hominis DrRognerud@www.medfever.com

OBSTETRICS & GYNECOLOGY

Answer: A

Answer: B

Answer: D

Answer: A

Answer: E

Answer: E

143 / 343

Board Exam Question (Smiley + Star) + 2007 OBG-718. The fetal or neonatal consequences of maternal diabetes include: 1) macrosomia 2) delayed pulmonary maturation 3) hypoglycemia 4) hypocalcemia OBG-719. Valid statements regarding toxoplasmosis developing in pregnancy include: 1) the infection is contracted by the consumption of raw meat 2) the infection is contracted by contact with feline feces 3) infection occurring in the early stage of pregnancy may result in abortion 4) the incidence of this condition is 1:2,000-2,500 pregnancies OBG-720. Hormones produced by the human placenta include: 1) gonadotropin 2) somatomammotropin (hCS) 3) progesterone 4) hydrocortisone OBG-721. The risk of fetal morbidity and mortality is maternal diabetes is increased by: 1) maternal ketoacidosis 2) maternal ketonuria occurring without diabetic ketoacidosis 3) maternal hyperglycemia 4) maternal hypoglycemia OBG-722. An increased risk of postpartum bleeding should be expected after/ in: 1) prolonged delivery 2) rapid delivery 3) the stimulation of uterine contractions with oxytocin 4) multiple pregnancy OBG-723. Which of the following drugs are appropriate for the therapy of postpartum bleeding? 1) ergometrine 2) oxytocin injection 3) ergotamine 4) prostaglandins OBG-724. Antibiotics contraindicated during pregnancy include: DrRognerud@www.medfever.com

OBSTETRICS & GYNECOLOGY

Answer: E

Answer: A

Answer: A

Answer: E

Answer: E

Answer: E

Answer: B 144 / 343

Board Exam Question (Smiley + Star) + 2007 1) tetracyclines 2) penicillin 3) chloramphenicol 4) ampicillin OBG-740. The imperforate hymen may result in the development of: 1) hematocolpos 2) dysuria 3) hematometra 4) periodic lower abdominal pain OBG-741. Acute urinary retention may result from: 1) retroflexion of the pregnant uterus 2) uterine obstruction caused by a myoma 3) hematocolpos 4) pelvic hematocele OBG-755. Hormonal replacement is recommended in climacteric women: 1) for the prevention of atherosclerotic heart disease 2) for the alleviation of vasomotor symptoms 3) to reverse osteoporosis 4) for the treatment of the atrophy of the vaginal mucosa OBG-760. Characteristic features of Turner syndrome (45, XO) include: 1) a low stature 2) a female gender with bilateral inguinal hernia 3) hypergonadotropic amenorrhea with low estrogen levels 4) elevated gonadotropin levels, the presence of ovarian follicles and amenorrhea OBG-764. Hemodynamic changes associated with the shifting from fetal to neonatal circulation result in: 1) the constriction of umbilical vessels and the gradual obliteration of the foramen ovale as well as the ductus venosus 2) the fall of systemic blood pressure 3) expansion of the fetal lung 4) reversal of the direction of blood flow in the ductus arteriosus

OBSTETRICS & GYNECOLOGY

Answer: E

Answer: E

Answer: C

Answer: B

Answer: E

ASSOCIATION QUESTIONS Associate the following terms/statements marked by the letters A, B, C... with the corresponding statements/terms marked by and in the order given by the figures 1, 2, 3... ...for example: 1-C, 2-B, 3-A, 4-D. Put the answer as C, B, A, D! (Note: Different statements can be associated with the same terms!!!)

DrRognerud@www.medfever.com

145 / 343

Board Exam Question (Smiley + Star) + 2007 OBG-767. Associate the following term(s) with their corresponding statement(s)! A) Genital tubercle B) Genital bud C) Urogenital sinus D) Urethral folds E) Mllerian ducts 1) labia minora 2) labia majora 3) clitoris 4) lower third of the vagina 5) oviducts OBG-768. Associate the following term(s) with their corresponding statement(s)! A) Uterine vein B) B) Right ovarian vein C) Left ovarian vein D) Uterine artery E) Ovarian artery 1) hypogastric artery (emerges from the internal iliac artery) 2) joins the internal iliac veins 3) joins the inferior vena cava 4) emerges from the abdominal aorta 5) joins the left renal vein OBG-769. Associate the following term(s) with their corresponding statement(s)! A) Spontaneous abortion B) Threatened abortion C) Habitual abortion D) Therapeutic abortion E) Elective abortion 1) termination of the pregnancy on maternal indication before the fetus attains viability 2) termination of the pregnancy before the fetus attains viability on indication other than the protection of maternal health or from causes other than fetal disease 3) spontaneous termination of gravidity occurring in about 10% of all pregnancies 4) spontaneous termination of gravidity associated with chromosomal abnormalities in 50-60% of cases 5) spontaneous termination of gravidity that is unfeasible in the case of the first pregnancy . OBG-770. Associate the following statement(s) with their corresponding term(s)! A) oral contraceptive use should be suspended for 7 days then reinstituted B) oral contraceptives should be continued as ususal DrRognerud@www.medfever.com

OBSTETRICS & GYNECOLOGY Answer: DBACE

Answer: DABEC

Answer: DEAAC

Answer: BBCCBE

146 / 343

Board Exam Question (Smiley + Star) + 2007 C) oral contraceptives should be continued and a supplemental contraceptive method should be applied in addition D) an extra tablet should be taken E) oral contraceptive use should be abandoned and a diagnostic workup is necessary 1) Nausea occurring in the first cycle during oral contraceptive use 2) Menstruation is absent during the 7 days following the 21-day long period of proper oral contraceptive use 3) The patient has forgotten to take one tablet 4) The patient has forgotten to take oral contraceptives for 10 consecutive days 5) Slight bleeding at midcycle during the first month of oral contraceptive use 6) Hemoptysis OBG-771. Associate the following term(s) with their corresponding statement(s)! A) Ectopic pregnancy B) Cortisol excess C) 21-Hydroxylase deficiency D) 19-Nortestosterone-progestins E) Toxic inflammatory endometritis 1) centripetal obesity, moon-face, purple striae 2) 10% incidence of ectopic tubal pregnancy 3) congenital adrenal hyperplasia 4) supresses luteinizing hormone-releasing factor secretion 5) Arias-Stella phenomenon OBG-772. Associate the following statement(s) with their corresponding term(s)! A) 47 XXY B) 45 XO C) trisomy 21 D) Tay-Sachs disease E) testicular feminization 1) Down's syndrome 2) Klinefelter syndrome 3) Turner's syndrome 4) The most prevalent XY female-syndrome 5) Amniocentesis

OBSTETRICS & GYNECOLOGY

Answer: BECDA

Answer: CABED

CASE STUDIES Answer the multiple task questions (simple choice and multiple choice with/without key answers; relation analysis etc.) as they are related to each case study!!! OBG-776. Case Study A 24-year-old primigravida presents in the 28th week of her pregnancy with spider nevi, palmar erythema and diffuse pruritus. Liver function test results: alkaline phosphatase: 190 IU/l (normal DrRognerud@www.medfever.com Answer: CB

147 / 343

Board Exam Question (Smiley + Star) + 2007 value: 29-91 IU/1); SGOT: 38 IU/1 (normal value: 6-18 IU/1); total bilirubin: 1.8 mg% (normal value: 0.3-1.0 mg%); direct bilirubin: 1.0 mg% (normal value 0.1-0.3 mg%). 5.776/ 1. The most likely diagnosis is: A) liver cirrhosis B) infectious hepatitis C) cholestasis D) acute pancreatitis E) cholecystitis 5.776/2. After delivery, the doctor should recommend to the patient: A) not to have any more babies B) to abstain from food with a high fat content and not to take oral contraceptives C) to avoid exertion D) to undergo a cholecystectomy E) none of the above OBG-777. Case Study A 20-year-old female presents at the clinic with lower abdominal pain. Her menstruation cycle is regular and she has not been pregnant yet. At present, she is taking an oral contraceptive. Colposcopy: ectopic tissue on the portio vaginalis of the cervix. Pelvic examination: average vaginal capacity, smooth, firm portio vaginalis, regular, large uterine corpus in avf. The uterus is mobile, palpation is normal on the right side. On the left side, ventrally, a semisolid, clearly delineate mobile mass is palpated. 5.777/ 1. The most likely diagnosis is: A) ovarian endometriosis B) malignant ovarian neoplasm C) uterine myoma D) dermoid cystoma E) paraovarian cyst 5.777/2. What can be seen on an anteroposterior radiograph of the pelvis? A) calcification B) phleboliths C) psammoma bodies D) a soft-tissue shadow E) calcification within a soft-tissue shadow 5.777/3. What is the percentage of neoplasms occurring in both ovaries simultaneously? A) 1% B) 15% C) 25% D) 40% E) 60% 5.777/4. DrRognerud@www.medfever.com

OBSTETRICS & GYNECOLOGY

Answer: DEAEAC

148 / 343

Board Exam Question (Smiley + Star) + 2007 Which of the following is a possible complication of this neoplasm? A) torsion of the pedicle of the cyst B) suppuration and peritonitis C) malignant transformation D) all of the above E) only answers (A) and (C) are true 5.777/5. What is the prognosis of this tumor? A) good, as malignant transformation seldom occurs B) extremely good, as malignant transformation does not occur C) malignant transformation is common, thus it cannot be detected at an early stage D) poor, beacuse it is a malignant lesion 5.777/6. The appropriate therapy is: A) puncture and aspiration of the contents via the vaginal route B) laparoscopic aspiration of the contents C) laparotomy to remove the tumor selectively with the preservation of functional ovarian remnants D) laparotomy with oophorectomy in all cases E) laparotomy with bilateral oophorectomy as bilateral occurrence is common

OBSTETRICS & GYNECOLOGY

OBG-789. Case Study A 55-year-old nulliparous, postmenopausal woman presents with bloody vaginal discharge present for the last 7 days. Status: The portio vaginalis of the cervix and the vaginal wall is covered ered by atrophic epithelium that bleeds easily. The uterus is hypoplastic, smaller than normal, no adjacent abnormality is palpated. 5.789/ 1. The most likely cause of the bleeding is: A) cervical polyp B) senile vaginitis C) cervical carcinoma D) endometrial carcinoma E) hormone secreting ovarian neoplasm 5.789/2. What should be done next? A) the bleeding should be controlled by estrogen administration B) fractional curettage should be performed C) oncocytology testing is recommended D) chemical curettage is necessary E) nothing should be done 5.789/3. Curettage yields copious medullary tissue from the uterine cavity. The most likely diagnosis is: A) senile endometritis B) submucous myoma C) endometrial carcinoma D ervical DrRognerud@www.medfever.com

Answer: DBCEB

149 / 343

Board Exam Question (Smiley + Star) + 2007 5.789/4. Which of the following conditions are associated with an increased' risk of endometrial carcinoma? A) obesity B) hypertension C) diabetes D) functional sterility in the history (anovulation) E) all of the above 5.789/5. Which of the following methods should be applied as the first choice therapy of endometrial carcinoma? A) irradiation B) surgery (hysterectomy) C) norsteroid therapy D) cytotoxic chemotherapy E) the combination of (A), (B) and (C)

OBSTETRICS & GYNECOLOGY

DrRognerud@www.medfever.com

150 / 343

Board Exam Question (Smiley + Star) + 2007

NEUROLOGY

NEUROLOGY
(426) TRUE-FALSE TYPE QUESTIONS Put T for true statements and F for false statements!!! NEU-2. At high doses all barbiturates exhibit an anticonvulsive effect. NEU-3. The electric stimulation of certain areas of the brain can mimic normal sleep. NEU-4. Phenytoin strongly inhibits the post-tetanic increase of excitation. NEU-5. Local anesthetics inhibit the depolarization and propagation of physiological stimuli. NEU-6. D-tubocurarine is a non-specific inhibitor of the motor end-plate. NEU-8. impaired liver function affects the metabolism of phenytoin leading to the subsequent accumulation of this component. NEU-9. Phenothiazines and Rauwolfia alkaloids induce Parkinson-like symptoms. NEU-12. It is generally accepted that the dopaminergic pathways of the extrapyramidal system are damaged in Parkinson's disease. NEU-18. Tumors never induce spike activity. NEU-23. In tuberculous meningitis symptoms other than ophthalmoplegia are observed. NEU-24. In tuberculous meningitisthe tuberculin test is usually negative. NEU-30. A cerebellar abscess can occur in chronic mastoididtis. DrRognerud@www.medfever.com Answer: T Answer: T Answer: T Answer: T Answer: F Answer: T

Answer: T

Answer: T Answer: F Answer: T Answer: F Answer: T 151 / 343

Board Exam Question (Smiley + Star) + 2007 NEU-45. A decreased serum ceruloplasmin level is typical of Wilson's disease. NEU-56. Clinical symptoms alone are not sufficient to differentiate Friedreich's ataxia from multiple sclerosis. NEU-83. Mark the prognosis of the following pediatric tumors as favorable (F) or unfavorable (U): 1. cerebellar astrocytoma 2. cerebral astrocytoma 3. medulloblastoma 4. tumor of the medulla NEU-84. Mark the following as causing hydrocephalus of a communicating (C) or non-communicating type (NC): 1. subdural hematoma 2. post-inflammatory hydrocephalus 3. basilar impression 4. thrombosis of the lateral sinus NEU-85. Mark the following alterations as either of muscular (A) or NEUrogenic (B) origin: 1. frequent fasciculations 2. predominantly proximal weakness 3. predominantly distal weakness 4. lack of fasciculations 5. fibrillations 6. muscle irritability at percussion NEU-92. Herpes infection of the geniculate ganglion usually causes a facial paralysis. NEU-93. In some cases herpes zoster spares the nuclei and causes paralysis of only the nerve. NEU-100. Parenteral histamine is the therapy of choice in migraine fits. NEU-102. In most cases the site of pain helps to differentiate a migraine from trigeminal neuralgia.

NEUROLOGY Answer: T Answer: F

Answer: FUUU

Answer: CCCC

Answer: NMNMNN

Answer: T

Answer: T Answer: F Answer: T

DrRognerud@www.medfever.com

152 / 343

Board Exam Question (Smiley + Star) + 2007 NEU-103. A Parkinsonian tremor is more pronounced during voluntary motions. NEU-105. Sensory disturbances can quickly develop in arsenic and alcoholinduced polyneuritis. NEU-108. Micturition symptoms develop early in cervical spondylosis. NEU-118. In case of a well localized cerebral abscess the body temperature is normal or subnormal. SINGLE CHOICE QUESTIONS Select the single best response to each of the following questions!!! NEU-126. The cells of the fasciculus cuneatus are localized in the: A) gelatinous substance B) ganglia of the dorsal horns C) nucleus proprius D) all of the above. E) none of the above NEU-127. Fibers enter the gelatinous substance from: A) the lateral spinothalamic tract B) the dorsal roots associated with sensation of pain and heat C) the Clarke nucleus D) all of the above E) none of the above NEU-129. The following two lobes constitute the inferior part of the parietal lobe: A) triangular and opercular lobes B) supermarginal and angular lobes C) triangular and angular lobes D) all of the above E) none of the above NEU-130. Pigment granules can be observed in: A) the cells of the substantia nigra B) the cells of the locus ceruleus C) the dorsal efferent nuclei of the vagus D) all of the above E) none of the above DrRognerud@www.medfever.com

NEUROLOGY Answer: F Answer: T Answer: F Answer: T

Answer: B

Answer: B

Answer: B

Answer: D

153 / 343

Board Exam Question (Smiley + Star) + 2007 NEU-131. Which of the following is a parasympathetic nucleus? A) Edinger-Westphal nucleus B) Inferior salivatory nucleus C) Dorsal efferent nucleus of the vagus D) all of the above E) none of the above NEU-133. Which types of cells can be found in the brain cortex? A) pyramidal cells B) stellate cells C) Cajal's cells D) all of the above E) none of the above NEU-134. The recurrent artery (Heubneri) originates from: A) the internal carotid artery B) the anterior communicating artery C) the anterior choroid artery D) all of the above E) none of the above NEU-140. Cerebral posterior arteries: A) originate from the basilar artery B) can join with the superior cerebellar artery C) supply the occipital lobe D) all of the above E) none of the above NEU-142. The "genu" of the internal capsule carries the following fibers: A) fibers from the anterior thalamic nuclei B) frontoparietal fibers C) corticobulbar fibers D) all of the above E) none of the above NEU-145. Following the occlusion of the saggital sinus: A) the ventricles dilate B) the CSF pressure increases C) both of the above D) none of the above NEU-149. The blood-brain barrier is composed of. DrRognerud@www.medfever.com

NEUROLOGY

Answer: D

Answer: -

Answer: E

Answer: D

Answer: C

Answer: C

Answer: D 154 / 343

Board Exam Question (Smiley + Star) + 2007 A) capillary endothelium B) basal membrane C) podocytes of the astrocytes D) all of the above NEU-164. Which of the following is a neurotransmitter in the parasympathetic postganglionic synapses? A) muscarine B) norepinephrine C) epinephrine D) acetylcholine E) buterylcholine NEU-166. Which of the following is not a side-effect of antihistamine drugs? A) supression of sea-sickness B) alleviation of some symptoms of Parkinson's disease C) alleviation of symptoms of tertiary syphilis D) suppression of petit mal epilepsy NEU-167. The typical memory loss occurring in Korsakoffs syndrome is due to a lesion of which of the following structures? A) parietal associative cortex B) frontal associative cortex C) mammilary body, limbic cortex D) habenula NEU-168. Which fibers convey nociceptive stimuli? A) unmyelinated C fibers B) myelinated fibers of small diameter C) both of the above D) none of the above NEU-170. Mark the false statement -the pyramidal tract: A) a part of it arises from Brodmann's area 4 B) most of it originates from neocortical areas C) 80-90% of the pathways cross in the inferior part of medulla NEU-173. The conduction velocity of an axon: A) increases as the diameter increases B) is less in axons with lower threshold values C) both of the above D) none of the above

NEUROLOGY

Answer: D

Answer: C

Answer: C

Answer: C

Answer: B

Answer: A

DrRognerud@www.medfever.com

155 / 343

Board Exam Question (Smiley + Star) + 2007 NEU-174. The motor unit: A) consists of the alpha-motor neurons of a given muscle B) consists of the alpha and gamma-motor neurons of a given muscle C) consists of an alpha-motor neuron and the muscle fibres supplied by this neuron. D) consists of the muscle fibers supplied by one motor neuron and a nerve. NEU-175. The retinal representation of the eye is observed in: A) the lateral geniculate body B) the primary visual cortex C) both of the above D) none of the above NEU-176. The membrane potential is created by a: A) concentration difference on both sides of the membrane and different permeabilities for individual ions B) Na+-pump C) resting potential D) reduced activity of the ions in the cytoplasm NEU-177. Typical electric signs of a paradox sleeping behavior are: A) low amplitude, high frequency B) high amplitude, high frequency C) low amplitude, low frequency D) high amplitude, low frequency NEU-179. After transection of the, spinal cord in humans: A) flexion reflexes reappear earlier than the extension reflexes B) extension reflexes reappear earlier than the flexor reflexes C) flexor and extensor reflexes reappear at the same time D) after trauma sweating is a common response to strong stimuli NEU-181. Loss of which of the following functions develops in spinal shock caused by transection of the spinal cord? A) voluntary movement B) sensory function C) spinal reflexes D) all of the above NEU-183. The following substrate is mainly metabolized by the human DrRognerud@www.medfever.com

NEUROLOGY Answer: D

Answer: C

Answer: A

Answer: A

Answer: A

Answer: D

Answer: C 156 / 343

Board Exam Question (Smiley + Star) + 2007 brain: A) glutamate B) albumin C) glucose D) none of the above NEU-195. The frequency of the alpha rhythm is: A) 4-6 cycles/sec B) 6-8 cycles/sec C) 8-12 cycles/sec D) none of the above NEU-204. Blood in a CSF sample gained with lumbar puncture quickly disappears: A) if it was caused by subarachnoid bleeding B) if bleeding was caused by the puncture itself C) if it was caused by thrombosis of the cerebral vessels D) if it was caused by an embolism of the cerebral vessels E) suggests rupture of an intracranial aneurysm NEU-208. In acute purulent meningitis, the CSF is NEVER: A) opalescent B) clear C) turbid ("cloudy") D) purulent E) flowing at increased pressure NEU-217. Which symptom occurs only rarely in infants with a subdural hematoma? A) fever B) fontanelle protrusion C) hemiparesis D) papilledema E) retinal hemorrhage NEU-219. Which of the following are the most common symptoms of an epidural hematoma in a child? A) papilledema and stupor B) retinal bleeding and coma C) ataxia and hemiparesis D) hemiparesis and contralateral pupil dilation E) hyperreflexia and contralateral paresis of cranial nerve VI. NEU-220. If the case history of an infant mentions trauma, vomiting, seizures, and an enlarged head. What should be done? DrRognerud@www.medfever.com

NEUROLOGY

Answer: C

Answer: B

Answer: B

Answer: D

Answer: D

Answer: D

157 / 343

Board Exam Question (Smiley + Star) + 2007 A) an EEG B) a lumbar puncture C) an X-ray of the skull D) a CT of the skull E) a radionuclide scan of the brain NEU-221. The most common symptom of increased intracranial pressure in children is: A) a loss of visual acuity B) stupor C) vomiting D) ataxia E) diplopia NEU-230. The best diagnostic method in pediatric head trauma is: A) X-ray of the skull B) pneumoencephalography (PEG) C) skull CT D) EEG E) radionuclide brain scan. NEU-231. Seizures in pediatric patients: A) occur most frequently in tumors B) occur most frequently with high fever C) rarely occur after the introduction of "Sabin's vaccination D) rarely occur in subdural hematoma E) rarely occur in hypocalcemia NEU-232. Case Study: Shortly after a penetrating foot injury a pediatric patient developed trismus, risus sardonicus, opisthotonus, abdominal rigidity. The most probable diagnosis is: A) drug allergy B) hysteria C) tetanus D) gas gangrene E) meningitis NEU-233. Subdural hematomas in infants are: A) bilateral and occipital B) unilateral and frontoparietal C) bilateral and frontoparietal D) unilateral and basal E) unilateral and temporal

NEUROLOGY

Answer: C

Answer: -

Answer: B

Answer: C

Answer: C

DrRognerud@www.medfever.com

158 / 343

Board Exam Question (Smiley + Star) + 2007 NEU-234. Which of the following CSF findings occur in acute purulent meningitis? A) normal pressure, clear and xanthochromic B) normal pressure, clear and colorless C) increased pressure, hemorrhagic D) increased pressure, turbid E) increased pressure, clear NEU-235. Which of the following pediatric tumors spreads (is disseminated) in the CSF prior to surgery? A) cerebellar astrocytoma B) medulloblastoma C) craniopharyngioma D) glioma of the optic nerve E) teratoma NEU-257. After an intramuscular penicillin injection an immediate sciatic nerve neuropathy develops. What is suspected? A) an allergic reaction B) direct injury to the sciatic nerve C) a toxic effect of penicillin on the nerve tissue D) none of the above NEU-264. Duchenne's dystrophy: A) the symptoms develop at the age of 3-5 B) sometimes mental retardation occurs C) can be best diagnosed with muscle biopsy D) the serum creatine-phosphokinase is elevated E) all of the above NEU-271. Case Study: A 45-year-old male patient complains of lumbar pain radiating to his leg.Ipsilateral Achilles areflexia and hypesthesia in the small toe were found. At which spinal root level does the patient have a disc hernia compression? A) L2 root B) L3 root C) L4 root D) L5 root E) S1 root NEU-283. Occlusion of the main trunk of the middle cerebral artery results in: A) hemiplegia and hemianesthesia B) hemiparesis DrRognerud@www.medfever.com

NEUROLOGY

Answer: D

Answer: B

Answer: B

Answer: E

Answer: E

Answer: C

159 / 343

Board Exam Question (Smiley + Star) + 2007 C) hemiplegia, hemianesthesia and homonymous hemianopsia D) the thalamus syndrome and choreoathetosis NEU-284. Thrombosis of which artery is the most common vascular lesion of the brain stem? A) the anterior chorioid artery B) the paramedian pontine artery C) the anterior inferior cerebellar artery D) the posterior inferior cerebellar artery E) the superior cerebellar artery NEU-285. Define the following: emotional lability, dysarthria, dysphagia, hyperactive mandibular reflex: A) Wallenberg's syndrome B) Weber's syndrome C) Pseudobulbar syndrome D) Millard-Gubler's syndrome E) Benedict's syndrome NEU-286. A sudden development of hemiplegia and contralateral ophthalmoplegia is most probably due to alterations in the: A) frontal lobe B) lateral medulla C) mesencephalon D) pons E) paracentrallobe NEU-289. The symptoms of a left-sided epidural hematoma in comatous patients include: A) contralateral hemiplegia B) ipsilateral hemiplegia C) contralateral hemiplegia and ipsilateral paralysis of CN III D) contralateral hemiplegia and paralysis of CN III E) ipsilateral hemiplegia and paralysis of CN III NEU-290. Case Study: A male patient complains of diplopia and numbness of one side of his body and face. Later he became comatous, developed quadriplegia and narrow pupils not sensitive to light. The most probable site of thrombosis is the: A) anterior cerebral artery B) basilar artery C) superior cerebellar artery D) inferior cerebellar artery E) lenticulostriatal artery DrRognerud@www.medfever.com

NEUROLOGY

Answer: D

Answer: C

Answer: C

Answer: C

Answer: B

160 / 343

Board Exam Question (Smiley + Star) + 2007 NEU-291. Case Study: A 64-year-old male patient has mental disorders, abnormal gait and incontinence. The CT study revealed expanded ventricles and compression of the subarachnoid space. The most probable diagnosis is: A) Alzheimer's disease B) Pick's disease C) a normal pressure hydrocephalus D) Jakob-Creutzfeldt disease E) Quentin's disease NEU-292. The worst prognosis of the functional restitution of the bladder is expected in: A) a thoracic spinal injury B) a cervical spinal injury C) a lumbar spinal injury D) an injury of the sacral plexus E) a cervical plexus injury NEU-293. The best diagnostic approach in familial periodic paralysis is: A) a muscle biopsy B) an EMG C) the serum K+ concentration D) an oral glucose tolerance test E) the nerve conductance velocity NEU-294. Nasal discharge of cerebrospinal fluid occurs in: A) cranial fractures B) brain tumors C) congenital defects D) pseudotumor E) all of the above NEU-296. Which structure remains unaffected in multiple sclerosis? A) the cerebellar tract B) the pyramidal tract C) the motoneurons of the ventral horn D) the spinal dorsal horn E) the visual tract NEU-297. Lumbar and low extremity pain, peripheral low extremity paresis and urination disorders occur in: A) lumbar spondylosis B) lumbar disc hernia DrRognerud@www.medfever.com

NEUROLOGY

Answer: C

Answer: D

Answer: C

Answer: A

Answer: C

Answer: D

161 / 343

Board Exam Question (Smiley + Star) + 2007 C) a tumor of the cauda equina D) all of the above NEU-298. Cataplexy occurs in: A) grand mal epilepsy B) Kleine-Levin's syndrome C) narcolepsy D) psychomotor epilepsy E) "status epilepticus" NEU-299. Which diseases are accompanied by paralysis of the external ocular muscles? A) myasthenia gravis B) multiple sclerosis C) Tolosa-Hunt's syndrome D) Wernicke's encephalopathy E) migraine headache F) ischemic injury of the brain stem G) all of the above NEU-300. The therapy of choice in trigeminal neuralgia is: A) clonezepam (Rivotril) B) alcohol infiltration C) carbamazepine D) retroganglionic neurotomy E) none of the above NEU-301. Occlusion of the anterior cerebral artery has the following symptoms: A) contralateral homonymous hemianopsia B) contralateral hemihypesthesia C) Gerstmann's syndrome D) ipsilateral paresis and sensory disturbances of the lower extremity. E) contralateral paresis and sensory disturbances of the lower extremity NEU-302. Atrophy of the optic nerve can be caused by: A) glioma of the optic nerve B) severe congestion of the fundus C) Kennedy-Gowers's syndrome D) optochiasmatic arachnoiditis E) all of the above F) none of the above

NEUROLOGY

Answer: C

Answer: G

Answer: C

Answer: E

Answer: E

DrRognerud@www.medfever.com

162 / 343

Board Exam Question (Smiley + Star) + 2007 NEU-303. Atrophy of the optic nerve can occur: A) as a sequel of glaucoma B) after rupture of an intracranial aneurysm C) after papillitis D) after cerebral commotion E) none of the above NEU-304. Which of the following can occur after an ipsilateral occlusion of the internal carotid artery? A) no neurologic signs B) ipsilateral amaurosis and contralateral hemiparesis C) contralateral hemiparesis and hemihypesthesia D) contralateral hemiparesis, hemihypesthesia, and homonymous hemianopsia E) all of the above NEU-305. A sudden visual disorder can occur in: A) temporal arteritis B) disease of the ipsilateral internal carotid artery C) ipsilateral embolism of the central retinal artery D) migraine headaches E) all of the above NEU-306. Which of the following is typical of the retrobulbar NEUritis occurring in multiple sclerosis? A) it is usually unilateral B) marked visual disorders C) later it is accompanied by temporal pallor D) all of the above E) none of the above NEU-307. Embolization of the cerebral vessels occurs: A) as a sequel of mitral valve prolapse B) as a seqel of atrial fibrillation C) if a lateral thrombus develops D) as a sequel of subacute bacterial endocarditis E) after cardiac surgery F) all of the above NEU-308. Which of thefollowing should be considered in the differential diagnosis of papilledema? A) pseudoedema of the papilla B) papillitis C) thrombosis of the central vein DrRognerud@www.medfever.com

NEUROLOGY Answer: C

Answer: E

Answer: E

Answer: D

Answer: F

Answer: D

163 / 343

Board Exam Question (Smiley + Star) + 2007 D) all of the above E) none of the above NEU-309. Which of the following diseases is accompanied by papilledema? A) Gullain-Barr syndrome B) lung emphysema C) anemia D) hypoparathyroidism in children E) hypervitaminosis -A F) all of the above NEU-310. The most common cause of cerebral vascular thrombosis is: A) hypertension B) arteriosclerosis C) diabetes D) syphilis E) collagen disease NEU-311. When do the symptoms of parainfectional encephalomyelitis develop? A) 2 weeks before the appearance of exanthemas B) 1 week before the appearance of exanthemas C) simultaneously with the exanthemas D) 2 weeks after the appearance of exanthemas E) none of the above NEU-314. A brain abscess: A) is a sequel of bronchiectasis B) is a sequel of frontal sinusitis C) is similar to other intracranial space occupying processes as it causes neurologic symptoms D) is fatal if left untreated E) all of the above NEU-318. Nneurologic symptoms accompanying cardiac diseases are: A) thrombosis and embolism B) abscess and aneurysm C) seizures and syncope D) all of the above NEU-321. Hemiplegia developing after a subarachnoid hemorrhage can be caused by: A) softening due to vascular spasm B) an intracerebral hematoma C) both of the above DrRognerud@www.medfever.com

NEUROLOGY

Answer: F

Answer: B

Answer: D

Answer: E

Answer: D

Answer: C

164 / 343

Board Exam Question (Smiley + Star) + 2007 D) none of the above NEU-322. Case Study: During physical work a 50-year-old male patient had a sudden headache, neck rigidity, seizures, paralysis of the 3rd cranial nerve and a rapidly developing coma The most probable diagnosis is: A) bacterial meningitis spontaneous subarachnoid hemorrhage due to an aneurysm C) rupture of a brain abscess D) glioma hemorrhage E) brain metastasis NEU-324. Which process is usually followed by hematomyelitis? A) a blood dyscrasia B) congenital aneurysm C) trauma D) a demyelinizating process E) all of the above NEU-326. Which pathological process is usually accompanied by an epidural hematoma? A) brain contusion B) rupture of the medial meningeal artery C) rupture of the lateral sinus D) rupture of the pontine veins E) none of the above NEU-327. If a head injury is followed by unconsciousness. Which pathological process is assumed? A) brain contusion B) brain commotion C) an epidural hematoma D) a subdural hematoma E) abrain abscess NEU-328. The most common neurologic symptoms in amyotrophic lateral sclerosis are: A) organic mental syndromes B) urological symptoms C) aphasia; agnosia D) spinal muscle atrophy, spasticity, pyramidal symptoms E) visual field disturbances NEU-329. Case Study: A 10-year-old girl with normal gait complains of proximal muscle weakness. DrRognerud@www.medfever.com

NEUROLOGY

Answer: B

Answer: C

Answer: B

Answer: -

Answer: D

Answer: C

165 / 343

Board Exam Question (Smiley + Star) + 2007 This, as well as atrophy of the proximal muscles of the lower extremities has been verified during the examination. A weak patellar reflex and pseudohyperlrophy of the leg muscles are also observed. Muscle biopsy indicated neurogenic atrophy. The most probable diagnosis is: A) myasthenia gravis B) hereditary amyotrophic lateral sclerosis C) Wolfhart-Kugelberg-Welander's disease D) Duchenne's dystrophy E) polymyositis NEU-330. The most common sequel of a cerebrospinal fluid fistula is: A) headache B) meningitis or brain abscess C) a decrease of the CSF pressure D) cortical atrophy E) development of a hygroma NEU-331. The diagnosis of syringomyelia is primarily based on: A) myelography B) the CSF findings C) localized muscle atrophy, sensory disorders, and impaired development of the bones D) an EMG E) any remissions in the course of the disease NEU-333. Which test is of no (less) value in the diagnosis of progressive muscle dystrophy? A) a muscle biopsy B) the serum creatinine phosphokinase level C) the EMG D) the muscle histochemistry E) the EEG NEU-335. A less typical symptom of myotonic dystrophy is: A) pseudohypertrophy B) cataract C) baldness D) testis atrophy E) mental disorders F) atrophy of the facial muscles NEU-336. One of the following is not characteristic of Parkinson's syndrome: A) degeneration of the substantia nigra B) dopamine deficiency C) cholinergic substances provoke parkinsonism DrRognerud@www.medfever.com

NEUROLOGY

Answer: B

Answer: -

Answer: E

Answer: A

Answer: D

166 / 343

Board Exam Question (Smiley + Star) + 2007 D) decreased muscle tone E) mental symptoms NEU-341. Which of the following is typical of an alcoholic polyneuropathy? A) a piercing pain is always present B) sensory disorders in-the lower extremities C) absence of autonomic symptoms D) normal conducting velocity in the peroneal nerve E) hyperthermia NEU-344. One of the following is less typical of multiple sclerosis: A) spastic paraparalysis B) internuclear ophthalmoplegia C) nystagmus D) a concomitantpregnancy should be interrupted E) retrobulbar neuritis NEU-346. Which of the following is not true for epilepsy? A) the diagnosis of epilepsy does not solely depends on the EEG B) the CT or MRI can be helpful if focal EEG alterations are observed C) after cessation of a seizure the medication used for its therapy can be discontinued immediately D) pathogenic causes can be ruled out in symptomatic epilepsy E) the dose of the effective drug should be continuously increased NEU-348. Case Study: A 45-year-old male patient is being examined for epilepsy. A mild left facial and left-sided hemiparesis were found. A circumscribed bone thinning and bone spicules on the right side were also visualized on the X-ray. What is the most probable diagnosis? A) occlusion of the right carotid artery B) medulloblastoma C) cerebellar astrocytoma D) withdrawal symptoms E) meningioma NEU-349. The most frequent cause of retrobulbar neuritis is: A) a paranasal sinus infection B) temporal arteritis C) multiple sclerosis D) aneurysm E) alcohol; smoking NEU-350. Case Study: DrRognerud@www.medfever.com

NEUROLOGY

Answer: B

Answer: D

Answer: C

Answer: E

Answer: C

Answer: C 167 / 343

Board Exam Question (Smiley + Star) + 2007 A 30-year-old woman complains of diplopia, disturbances of speech and swallowing. The symptoms show daily variations. Which of the following tests is the most relevant? A) an oral glucose tolerance test B) the EEG C) an Edrophonium (Tensilon) test D) an oligoclonal gamma test E) the VEP NEU-352. Case Study: After sustaining a skull injury a 50-year-old male patient complains of progressive pulsating exophthalmus and diplopia What could have happened? A) a subtemporal hematoma B) an epidural hematoma C) a carotid-cavernous fistula D) an intracerebral hematoma E) an aneurysm of the basilar artery NEU-353. Select a lumbar CSF finding typically occurring in viral meningitis: A) 2 mononuclear cells/mm3, protein 20 mg%, glucose 20 mg% B) 2 mononuclear cells/mm3, protein 50 mg%, glucose 20 mg% C) 20 mononuclear cells/mm3, protein 100 mg%, glucose 20 mg% D) 200 mononuclear cells/mm3, protein 100 mg%, glucose 20 mg% E) 100-1000 mononuclear cells/mm3, protein 60 mg%, glucose 50 mg% NEU-354. A suddenly developing nystagmus, vertigo, Horner's syndrome, ataxia without paralysis, and alternating sensory disorder suggest occlusion of which artery? A) anterior chorioidal artery B) posterior chorioidal artery C) basilar artery D) inferior posterior cerebellar artery E) superior cerebellar artery NEU-355. Case Study: An 80-year-old alcoholic male patient complains of a headache probably due to skull injury. He has had left-sided hemiparesis for a month. The right carotidangiography showed detachment of the cerebral vessels from the internal surface of the skull. The most probable diagnosis is: A) cerebral contusion B) right-sided glioblastoma of the frontal lobe C) right-sided subdural hematoma D) right-sided epidural hematoma E) occlusion of the right middle cerebral artery DrRognerud@www.medfever.com

NEUROLOGY

Answer: C

Answer: E

Answer: D

Answer: C

168 / 343

Board Exam Question (Smiley + Star) + 2007 NEU-357. One of the following is not characteristic of the cauda syndrome: A) it is commonly caused by traumatic alterations in Ll-2 B) it can impair all sensory modalities C) sphincter paralysis D) no pyramidal symptoms E) spastic paralysis NEU-368. Which part of the nervous system is the least affected in multiple sclerosis? A) the spinal cord B) the cerebral cortex C) the cerebellar tracts D) the brain stem E) the visual tract NEU-371. In optic atrophy which of the following alterations are accompanied by a bitemporal visual field defect? A) parachiasmallesion B) retinal vascular disease C) chiasmatic lesion D) tumor of the optic nerve E) none of the above NEU-372. The most common intracranial tumor is: A) meningoma B) acoustic neuroma C) glioma D) pituitary adenoma E) angioma F) metastasis NEU-374. Which disease does not accompany alcoholism? A) central pontine myelinolysis B) Pick's psychosis C) primary degeneration of the corpus callosum D) Wernicke's syndrome E) Korsakoffs psychosis F) parenchymal cerebellar degeneration NEU-375. Which of the following diseases shows the greatest coincidence with myasthenia gravis? A) thymus tumor B) thymus hyperplasia DrRognerud@www.medfever.com

NEUROLOGY

Answer: E

Answer: B

Answer: C

Answer: C

Answer: B

Answer: E

169 / 343

Board Exam Question (Smiley + Star) + 2007 C) hyperthyroidism D) hypothyroidism E) all of the above NEU-376. Select the main anastomoses of the cerebral circulation: A) The circle of Willis B) supracortical anastomoses of the main cerebral arteries C) anastomosis between internal and external carotid arteries D) all of the above E) none of the above NEU-378. Continous therapy with phenytoin can cause: A) peripheral neuropathy B) visual impairment, ataxia C) gingival hyperplasia D) all of the above E) none of the above NEU-382. Coma frequently occurs in: A) thrombosis B) embolism C) hemorrhage NEU-383. Headache frequently occurs in: A) thrombosis B) embolism C) hemorrhage NEU-384. Blood pressure is usually normal in: A) thrombosis B) embolism C) hemorrhage NEU-385. Atrial fibrillation frequently occurs in: A) thrombosis B) embolism C) hemorrhage NEU-386. Anisocoria is frequently observed in: A) thrombosis B) embolism C) hemorrhage

NEUROLOGY

Answer: D

Answer: D

Answer: C

Answer: C

Answer: B

Answer: B

Answer: C

DrRognerud@www.medfever.com

170 / 343

Board Exam Question (Smiley + Star) + 2007 NEU-387. Case Study: A 54-year-old hypertensive male patient suddenly develops vertigo, headache, and vomiting. At examination the patient is conscious and has unilateral ataxia without weakness. Conjugated eye movements are disturbed. What is suspected? A) pontine hemorrhage B) capsule hemorrhage C) cerebellar hemorrhage D) subarachnoid hemorrhage E) aneurysm of the basilar artery NEU-388. Which symptom is not typical of tabes dorsalis? A) Argyll-Robertson pupils B) ataxia C) hypertonic muscles D) lack of vibration sensation E) root pain NEU-389. The typical course of herpes zoster is: A) pain-blisters-pigmentation B) blisters-pigmentation-pain C) pain-pigmentation-blisters D) pigmentation-blisters-pain E) blisters-pain-pigmentation NEU-391. The most common cause of acute meningitis in adults is: A) PNEUmococcus B) Streptococcus C) Hemophilus influenza D) Meningococcus E) none of the above NEU-393. The onset of bacterial meningitis can be best diagnosed by the following: A) the general symptoms of the patient B) a murmur over the skull C) central scotoma D) bitemporal hemianopsia E) fasciculations NEU-394. The early symptoms of bacterial meningitis include: A) occipital stiffness and hypertension B) Kernig and Brudzinsky's signs DrRognerud@www.medfever.com

NEUROLOGY

Answer: C

Answer: C

Answer: A

Answer: D

Answer: A

Answer: B

171 / 343

Board Exam Question (Smiley + Star) + 2007 C) photophobia and papilledema D) bradycardia E) hypothermia NEU-395. The CSF in case of meningococcal meningitis: A) is clear and colorless B) has a normal protein content C) has a normal pressure D) has a normal cell count E) none of the above NEU-405. Spina bifida means a congenital anomaly of which of the following structures? A) the spine and brain stem B) the cerebellum, but not of the cerebrum C) the cerebrum but not of the cerebellum D) the spine but not of the brain stem E) the brain stem but not of the spine NEU-406. The typical onset of syringomyelia is: A) a focal atrophy of the upper extremities, weakness and a lack of pain sensation to burning B) pain in the upper extremity C) sphincter disorders D) fasciculations E) none of the above NEU-416. The recommended therapy of "paralysis agitans" (a form of parkinsonism) is: A) L-dopa therapy combined with a peripheral decarboxylase inhibitor B) surgical treatment in elderly patients with bilateral disease C) trihexyphenidyl, 20-25 mg/day in split doses D) atropine and prochlorperazine E) diphenhydramine, 50 mg daily NEU-426. Congenital sacular (berry) aneurysm: A) such unruptured aneurysms occur in 2% of postmortem findings B) its wall is comprised only of intima and adventitia C) both of the above D) none of the above NEU-428. Post-traumatic epilepsy: A) rarely develops following car accidents DrRognerud@www.medfever.com

NEUROLOGY

Answer: E

Answer: A

Answer: A

Answer: A

Answer: C

Answer: B

172 / 343

Board Exam Question (Smiley + Star) + 2007 B) usually develops 6-15 months following injury C) usually develops 1 week after injury D) none of the above NEU-429. Case Study: A hemorrhagic CSF which rapidly cleared up was obtained after a lumbar puncture. The first portion of the CSF contained 2000 RBC/mm3. The RBC content of the third portion was only 10 RBC/ mm3. The most probable diagnosis is: A) cerebral embolization B) rupture of an intracranial aneurysm C) subarachnoid hemorrhage D) artificial CSF hemorrhage E) cerebral thrombosis NEU-430. Case Study: A lumbar puncture revealed an opalescent CSF with increased pressure. The most probable diagnosis is: A) the patient is healthy but at strain B) a brain tumor C) the alterations are due to trauma caused by the puncture D) subarachnoid hemorrhage E) tuberculous meningitis NEU-431. Case Study: An infant has lost his appetite, vomits, has seizures and fever. What should be done? A) a cultivation of nasal and pharyngeal discharge B) try to find emotional problems in the family C) obtaining a urine sample with a catheter D) a lumbar puncture E) intravenous pyelography NEU-432. Case Study: A 7-year-old child complains of a sore throat, joint pain, and headache. He has fever and occipital stiffness. What should be done? A) an ECG B) an X-ray of the wrists and knees C) a lumbar puncture D) a cultivation of nasal and pharyngeal discharge E) the RBC sedimentation rate NEU-433. After smallpox vacination any postvaccination encephalomyelitis ocurs within: A) 30-40 days DrRognerud@www.medfever.com

NEUROLOGY

Answer: D

Answer: E

Answer: D

Answer: C

Answer: -

173 / 343

Board Exam Question (Smiley + Star) + 2007 B) 20-25 days C) 10-12 days D) 4-6 days E) 2-3 days NEU-434. One of the following occurs in a subdural hematoma. A) an increased intracranial pressure B) a xanthochromic CSF C) an elevated CSF protein level D) all of the above E) none of the above NEU-435. Which of the following rarely occurs in infants in case of a subdural hematoma? A) papilledema B) fever C) protrusion of the fontanelle D) a hemorrhage in the retina E) hemiparesis NEU-436. When do the symptoms of an epidural hemorrhage in children develop? A) weeks after trauma B) days after trauma C) seconds after trauma D) hours after trauma E) minutes after trauma NEU-437. Common symptoms of an epidural hemorrhage in a child include: A) ataxia and hemiparesis B) papilledema and stupor C) hyperreflexia and contralateral paralysis of the abducent nerve D) retinal hemorrhage and coma E) hemiparesis and contralateral pupil dilatation NEU-438. The most common symptom of increased intracranial pressure in children is: A) ataxia B) diplopia C) stupor D) vomiting E) impaired visual acuity NEU-439. A common cause of intracranial hemorrhage in children is: DrRognerud@www.medfever.com

NEUROLOGY

Answer: D

Answer: A

Answer: D

Answer: E

Answer: D

Answer: E 174 / 343

Board Exam Question (Smiley + Star) + 2007 A) hepatic disease B) a blood dyscrasia C) an intracranial aneurysm D) glomerulonephritis E) trauma NEU-440. Case Study: 3 days after a tibial fracture the child became confused, had fever, blood in the sputum, and developed hemiparesis. The most probable diagnosis is: A) cerebral fat embolism B) subdural hematoma C) metastatic cerebral abscess D) cortical contusion E) traumatic cerebral thrombosis NEU-441. Case Study: 2 weeks after a penetrating hand injury a child developed abdominal muscle rigidity, trismus, risus sardonicus, and opisthotonus. Diagnosis: A) meningitis B) hysteria C) drug allergy D) intercurrent peritonsillar abscess E) tetanus NEU-442. The CSF in acute purulent meningitis: A) clear; colorless; and has normal pressure B) increased pressure; clear C) increased pressure; opaque D) increased pressure; hemorrhagic E) normal pressure; clear; xanthochromic NEU-444. If a child has recurrent otitis the following is assumed: A) mastoiditis B) deficient alimentation C) a dermal sinus duct D) all of the above E) none of the above NEU-446. A postinfection encephalomyelitis can be due to the following: A) measles B) varicella C) mumps D) rubella E) all of the above

NEUROLOGY

Answer: A

Answer: E

Answer: C

Answer: A

Answer: E

DrRognerud@www.medfever.com

175 / 343

Board Exam Question (Smiley + Star) + 2007 NEU-447. Which of the following is frequently observed in children after head injuries? A) vertigo B) seizures C) headaches D) all of the above E) none of the above NEU-449. The first symptoms of a brain tumor in children are: A) seizures and coma B) behavior disturbances C) headaches and vomiting D) hemiparesis and hyperreflexia E) none of the above NEU-451. An X-ray study is a valuable tool in the diagnosis of lead intoxication because: A) it reveals lead lines along the long bones B) it reveals suture separation C) an abdominal image shows lead incorporation D) all of the above E) none of the above NEU-453. A lucid period following a head injury indicates a: A) subdural hematoma B) cerebral laceration C) cerebral contusion D) cerebral commotion E) epidural hematoma NEU-454. Secondary Parkinsonism suggests one of the following intoxications: A) manganese B) phenothiazine C) carbon monoxide D) all of the above E) none of the above NEU-455. Case Study: A 31-year-old male patient complains of a bifrontal headache and impaired visual acuity lasting for about 4 weeks. During the previous 4 months the patient had a mild, intermittant headache. At present the patient is irritable and difficult to live with. In the previous months he felt sleepy, sometimes sleeping for 20-30 hours. 8-9 months earlier the patient had an accident, when he had fallen out DrRognerud@www.medfever.com

NEUROLOGY

Answer: D

Answer: C

Answer: D

Answer: E

Answer: D

Answer: B

176 / 343

Board Exam Question (Smiley + Star) + 2007 from a moving car causing a laceration of the skull skin. Examination revealed papilledema, a dilated right pupil and leftsided hemiparesis. The assumed diagnosis is: A) paralytic dementia B) a chronic subdural hematoma C) bromide intoxication D) a space-occupying process in the brain NEU-456. Case Study: A 38-year-old female patient complains of intermitting episodes of tinnitus, vertigo, and feelings that the auditory passage on the rightside was closed, which have been persisting for about a year. These episodes usually last for 3-5 hours. Recently the patient complained of a hearing impairment on the right side which always got worse during the fits. Examination performed during one of these fits revealed right nystagmus while looking to the right and a right-sided perception hearing impairment. The patient was instable in Romberg's position. The assumed diagnosis is: A) petrositis B) a tumor of the cerebellar-pontine angle C) Mnire's disease D) vertebrobasilar insufficiency NEU-457. Which of the following can be observed in a healthy new-born? A) patellar clonus and Babinski reflex B) optikinetic nystagmus C) suckling and Moro's reflex D) all of the above E) none of the above NEU-460. Case Study: A patient has a sudden vertigo, swallowing disturbances and subsequently vomits. Examination reveals Horner's syndrome, analgesia, thermanesthesia on one side of his face, and sensory disturbances on the contralateral side. The most likely cause is: A) thrombosis of the middle or anterior cerebral artery B) thrombosis of the inferior posterior cerebellar or vertebral artery C) hemorrhage affecting the internal capsule D) embolization of the posterior cerebral artery E) occlusion of the carotid artery NEU-461. Acute paralysis of the left lower extremity with only a mild involvement of the upper extremity suggests: A) occlusion of the anterior cerebral artery B) occlusion of the middle cerebral artery DrRognerud@www.medfever.com

NEUROLOGY

Answer: C

Answer: D

Answer: B

Answer: A

177 / 343

Board Exam Question (Smiley + Star) + 2007 C) occlusion of the posterior cerebral artery D) occlusion of the inferior posterior cerebellar artery E) occlusion of the anterior spinal artery NEU-462. Occlusion of which artery is accompanied by a typical symptom of homonymous hemianopsia? A) lenticulostriatal artery B) Heubner's artery C) posterior cerebral artery D) superior cerebellar artery E) none of the above NEU-463. Transient unilateral amblyopia with unilateral motor and sensory deficits is most characteristic of which of the following? A) internal carotid artery disease B) vertebrobasilar artery disease C) middle cerebral artery disease D) posterior chorioid artery disease E) the (A) and (C) answers are true NEU-464. The most common cause of a subarachnoid hemorrhage of nontraumatic origin is: A) aneurysm B) arterio-venous malformation C) intracranial tumor D) blood dyscrasia E) none of the above NEU-465. The most important factor in the development of cerebral aneurysms is: A) trauma B) congenital anomaly C) syphilis D) septic embolism E) none of the above NEU-466. The most common site of an intracranial aneurysm in adults is between: A) the anterior communicans artery and the internal carotid artery B) the basilar artery and the internal carotid artery C) the middle cerebral artery and the vertebrobasilar artery D) the posterior cerebral artery and the basilar artery E) none of the above

NEUROLOGY

Answer: C

Answer: A

Answer: A

Answer: B

Answer: A

DrRognerud@www.medfever.com

178 / 343

Board Exam Question (Smiley + Star) + 2007 NEU-470. A common cause of cerebral embolization in adults is: A) cardiac disease B) air embolism C) tumor thrombus D) fat embolism due to fracture E) septic lung disease NEU-471. Occlusion of the internal carotid artery: A) if the internal carotid artery is affected, a murmur is auscultated on the neck; in case of external carotid artery disease this finding is absent B) a hypersensitive carotid sinus reflex is nearly always absent C) transient unilateral amblyopia usually develops; homonymous hemianopsia occurs in about 50% of the cases D) a transient ischemic attack lasting for 5-30 min. can occur E) pathological ophthalmic findings are as common as in vertebrobasilar disease NEU-472. Cardiac disturbances accompanying cerebrovascular disease: A) are rare B) are unknown C) sometimes occur D) are common E) occur in a ratio of 1:1 NEU-474. The rupture of an intracranial aneurysm usually does not occur together with the following: A) severe headache B) nausea and vomiting C) unconsciousness D) occipital stiffness E). none of the above NEU-481. The course of a progressive stroke DOES NOT: A) show stepwise development B) show a constant development C) show similarity to a subdural hematoma D) show similarity to a brain tumor E) none of the above NEU-482. Stroking the sole of a foot with a pointed object is a technique for eliciting: A) Hoffman's sign B) Babinski's sign DrRognerud@www.medfever.com

NEUROLOGY Answer: A

Answer: D

Answer: D

Answer: E

Answer: E

Answer: B

179 / 343

Board Exam Question (Smiley + Star) + 2007 C) Chaddock's sign D) Romberg's sign E) Gordon's sign NEU-484. Delirium or confusion occur in all of the following EXCEPT: A) pneumonia B) thyrotoxicosis C) congested cardiac failure D) thiamine deficiency E) homocystinuria NEU-488. All of the following are true for amyotrophic lateral sclerosis EXCEPT: A) it starts at the age of 50-60-years-old B) it is symmetric C) it has symptoms of impaired sensory function D) fasciculations in the upper extremities E) it should be differentiated from a cervical spine compression F) there are no signs of a neurogenic bladder or incontinence NEU-489. Case Study: A 40-year-old male patient, -occupation: technician. His motorcycle smashed into a car and the patient was attended to at a traumatology unit. Commotio cerebri and twisted neck were diagnosed. The patient was discharged after a 5 day observation period. After a two week rest period he returned to work. Subsequently 2 months later, at home, the patient had a left-sided headache, vertigo, transient impairment of speech, and numbness on the right side of his face and tongue. His family doctor failed to find any external injures on his body. A two-dimensional cranial and cervical X-ray did not reveal any traumatic alterations. Blood pressure 140/90, heart rate 82/min. Examination: the left pupil was somewhat larger than the right one. The grip in his right hand was slightly weaker than the left (the patient is right-handed). Ophthalmic examination: 0.5D congestion in the left fundus. What is the suggested diagnosis? A) hypertensive disease B) brain tumor C) post-commotion syndrome D) chronic subdural hematoma E) vertebrobasilar insufficiency NEU-490. Complete stroke: A) in 80-90% of the cases seizure fits will develop in patients with both embolic or atheromatous infarction B) a sudden onset is typical of an embolic infarction, while the atheromatous form is characterized by a gradual onset DrRognerud@www.medfever.com

NEUROLOGY

Answer: B

Answer: E

Answer: D

Answer: B

180 / 343

Board Exam Question (Smiley + Star) + 2007 C) alternating symptoms suggest supratentorial ischemia D) anosognosia usually develops in a right-sided paralysis E) none of the above NEU-492. The circulatory disturbances of the occipital cerebral regions: A) comprise about 80% of the cases of cerebral ischemia B) do not affect head movements (passive moving) C) a bilateral occipital infarction can be followed by cortical blindness D) occipital infarction is always accompanied by the Weber's syndrome E) none of the above NEU-497. Diagnosis of cerebral embolism: A) a complex examination (EEG, carotid and heart ultrasound study, complete blood count including hemostasis) is useful B) an examination of the cerebrospinal fluid is of crucial importance C) a normal sinus rhythm excludes embolism D) MRI is the method of choice E) amnesia is typical NEU-498. In cerebral vein thrombosis: A) epileptic fits rarely occur B) it is usually of a septic or focal origin C) headache and delirium are common, but confusion and seizures are not typical D) headache and vomiting never develop E) focal neurologic symptoms are rare NEU-499. If a murmur is detected during the examination of a possible extracranial vascular obstruction: A) a murmur below the mandibular angle suggests a disease of the carotid artery B) a murmur directly under the clavicle suggests a disease of the vertebral artery C) the murmur always accurately reflects the severity of stenosis D) a loud murmur indicates obstruction E) none of the above NEU-500. In a patient with cerebral infarction: A) the CT study has fully replaced the pneumoencephalography in radiologicexaminations B) a CT study does not substitute static brain scintigraphy in the evaluation C) a CT study does not substitute echoencephalography in the DrRognerud@www.medfever.com

NEUROLOGY

Answer: C

Answer: A

Answer: B

Answer: A

Answer: -

181 / 343

Board Exam Question (Smiley + Star) + 2007 evaluation D) during the first 1-2 hours, a CT study can differentiate between the intact and infarction area E) In 50% of the cases a CT study can help to detect any complications such as hemorrhage into the brain matter NEU-501. An ECG is not required in cerebral infarction because: A) a CT can determine the diagnosis and the cause of this disease B) this is a false statement becausethe ECG can reveal arrhythmia, coronary disease, and infarction C) an isoenzyme study is more important D) only the ECG under a physical stress test is informative E) none of the above NEU-502. Which of the following special examinations should be done in case of cerebral infarction? A) angiography, which reveals different vascular changes and hemodynamic disturbances B) a brain radionuclide study, which shows a marked isotope accumulation during the first day followed by a subsequent decrease C) echoencephalography, which can differentiate between cerebral edema and tumor, as well as between tumor and subdural hematoma D) angiography because mortality due to this intervention at present is only 10% E) none of the above NEU-503. In the differential diagnosis of a cerebral infarction: A) a hemorrhagic CSF excludes a cerebral infarction B) an angiographic study visualizes a brain abscess as an avascular region; and lymphocytes are frequently found in the CSF C) if the state of the patient worsens, angiography should be postponed until an exploration bore is made D) the CT study helps in the differential diagnosis of infarction, and subdural or epidural hematoma E) none of the above NEU-504. In the prognosis of cerebral infarction: A) the age of the patient, but not the extent of the nervous system damage is an important predictive factor of early mortality B) after the development of a stroke, hypertension and diabetes do not affect the outcome C) the severity of a concomitant cardiovascular disease is an important factor of the survival DrRognerud@www.medfever.com

NEUROLOGY

Answer: B

Answer: A

Answer: D

Answer: C

182 / 343

Board Exam Question (Smiley + Star) + 2007 D) about 3/4 of the patients die during the first attack E) About one-half of the patients surviving the first ischemic infarction will have a hemorrhage within 1-7 years. NEU-505. Antiedematous therapy in cerebral infarction: A) hypertonic urea is not effective B) mannitol is effective and has no rebound effect C) glycerine is not effective D) dexamethasone is mainly used for the therapy of cytotoxic edema E) none of the above NEU-506. In cerebral infarction the cerebral blood flow can be increased with the use of: A) aminophylline, tolazoline, histamine B) papaverine and nicotinic acid C) a blockade of the stellate ganglion D) carbon dioxide E) none of the above NEU-507. In repeated Transient Ischemic Attacks (TIAs), the following therapy is recommended: A) lithium carbonate if neutropenia develops B) aspirin if an ulcer develops C) aspirin, 2-3 g/day D) anticoagulants if cardiogenic stroke develops E) anticoagulants if endocarditis underlies the ischemia NEU-508. The therapy of progressive stroke: A) anticoagulant therapy is of no value B) multifactorial therapy by itself or if combined with anticoagulant therapy is of no value C) surgical correction of arterial obstruction has good results D) inhibition of platelet aggregation is never effective E) none of the above NEU-509. The general treatment of completed stroke, including long-term therapy: A) multifactorial therapy is of no value B) surgical intervention in patients with pronounced neurologic defects alleviates the symptoms C) long-term anticoagulant therapy is beneficial and has no significant complications D) carotid surgery in patients with pronounced neurologic defects decreases the morbidity in cases of fresh softening E) none of the above DrRognerud@www.medfever.com

NEUROLOGY

Answer: E

Answer: E

Answer: D

Answer: E

Answer: E

183 / 343

Board Exam Question (Smiley + Star) + 2007 NEU-510. The therapy of a cerebral embolism of cardiac origin: A) anticoagulants decrease morbidity and mortality due to the recurrent embolization B) anticoagulants are given for 6-12 months after valve prosthesis surgery C) cardioversion is always performed in atrial fibrillation to restore the normal sinus rhythm; this intervention reduces the danger of re-embolization D) the incidence of myocardial ischemia is not increased after myocardial infarction E) none of the above NEU-511. In considering contraindications to anticoagulant therapy the following is NEGLIGIBLE: A) an inadequate laboratory background B) unsatisfactory patient compliance C) peptic ulcer, hemorrhagic diathesis, hypertension, sever liver or renal disease D) hemorrhagic CSF E) none of the above NEU-512. Anticoagulant therapy: A) can be started even in case of hemorrhagic softening B) as a rule does not cause bleeding, unless the prothrombin time is 4-5x longer compared to the control C) can be started in cases of severe ischemia (vast area of softening) D) can be started in endocarditis lenta (subacute infectious endocarditis) E) can be very effective in cases of severe ischemic insult if initiated early NEU-513. In hypertensive encephalopathy: A) hypotensive therapy does not markedly affect the outcome B) visual impairment is rare C) confusion, seizures and papilledema are typical D) papilledema is a rare finding E) the CSF fmding is normal NEU-516. Which of the following is typical of the rupture of an aneurysm? A) a sudden headache, vomiting, sometimes confusion B) it typically starts in the morning C) a CSF sample has no diagnostic value D) it has no typical neurologic symptoms E) the CSF obtained immediately after the hemorrhage is xanthochromic DrRognerud@www.medfever.com

NEUROLOGY

Answer: A

Answer: E

Answer: B

Answer: C

Answer: A

184 / 343

Board Exam Question (Smiley + Star) + 2007 NEU-517. Which of the following is typical of brain tumors? A) usually progressive symptoms, frequent headaches, increased intracranial pressure B) the CSF finding is always abnormal C) metastases are the most common finding D) multiform glioblastoma is a malignant, well circumscribed tumor E) brain tumors comprise about 1/3 of all occurring tumors NEU-518. Which of the following possibilities should not be considered in the differential diagnosis of an intracranial hemorrhage? A) an overdose of anticoagulants B) the hemorrhage fully destroying a minor malformation C) a hemorrhagic infarction with marked softening, hence differing from a primary hemorrhage D) leukemia or thrombocytopenia E) none of the above NEU-521. Which of the following is typical of an intracerebral hemorrhage? A) the CT visualizes hyperdense regions even after several months B) the CSF is always hemorrhagic C) it usually. occurs due to a hypertensive vascular disease or a minor a-v malformation D) a fresh hemorrhage is not always detectable E) these patients always have hypnoid confusion NEU-522. After subarachnoid hemorrhage: A) seizures and headache occur at the onset B) hypertonic hemiplegia or hemiparesis with Babinski's sign is the most common neurologic symptom C) papilledema is the most common symptom D) no signs of meningeal excitement are observed E) a CT is of no diagnostic value NEU-524. Which of the following is important in the diagnosis of hemorrhages caused by the rupture of arteriovenous malformations? A) focal seizures in the history, focal neurologic symptoms, hemilateral headaches, nausea B) clinical symptoms markedly differ from those occurring in a ruptured aneurysm C) murmurs equally occur in aneurysms and arteriovenous malformations D) seizures developing at the onset of hemorrhage rather suggest DrRognerud@www.medfever.com

NEUROLOGY

Answer: A

Answer: E

Answer: C

Answer: A

Answer: A

185 / 343

Board Exam Question (Smiley + Star) + 2007 a traumatic origin E) the CT is of no value NEU-525. In the diagnosis of an intracerebral hemorrhage: A) a lumbar puncture nearly always helps to differentiate it from a hemorrhagic infarction B) the onset is usually gradual C) unconsciousness and hypertension are common after the onset D) headache is as frequent as in cerebral infarction E) a CT can differentiate the hemorrhagic area from the intact tissue only several hours after hemorrhage NEU-528. In cases of intracerebral bleeding: A) surgery is the method of choice B) independently of the applied therapy a high ratio of patients in deep coma will die C) surgical removal of intracerebral hematoma due to a ruptured aneurysm has better results compared to those, in which hemorrhage was due to a hypertensive vascular event D) in most cases drug therapy gives satisfactory results E) patients in coma usually survive if treated by calcium-channel blockers NEU-530. Metstatic intracranial tumors: A) comprise about 80% of all intracranial tumors B) primarily originate from the lung or breast C) a supratentorial localization has poor prognosis D) are common in the cerebrum, but rare in the cerebellum NEU-533. The two main causes of cerebral metastases are the: A) breast and lung B) lung and colon C) colon and rectum D) colon and nasal sinuses E) uterus and ovaries NEU-542. Which of the following is typical of a tumor or a disc-compression of the cauda equina? A) the processes are above the L1-2 level B) pain; flaccid paralysis of the lower extremities C) spastic paralysis D) Babinski's sign; hyperreflexia E) none of the above

NEUROLOGY

Answer: C

Answer: B

Answer: -

Answer: A

Answer: B

DrRognerud@www.medfever.com

186 / 343

Board Exam Question (Smiley + Star) + 2007 NEU-543. After cranial injury: A) an epileptic fit indicates at least a cerebral contusion B) brain commotion is always accompanied by morphological symptoms C) headaches following brain commotion last only for several hours D) a CSF sample helps to confirm the diagnosis E) lucid intervals always occur in epidural hematoma NEU-547. Which of the following is typical of diabetic polyneuropathy? A) it develops only in severe diabetes B) the CSF is always normal C) a determination of the nerve conduction velocity is not relevant in the diagnosis D) a symmetric, distal sensomotor diabetic polyneuropathy is the most common disorder E) all of the above F) none of the above NEU-548. A subdural hematoma: A) is not accompanied by epileptic symptoms B) a xanthochromic CSF is not typical C) the CSF is always Clear D) is never isodense on the CT E) all of the above F) none of the above NEU-549. Bell's palsy: A) Bell's palsy (idiopathic form) comprises about 75% of all cases of facial palsy B) rarely occurs in diabetes C) only 5-10% of the patients will recover D) hyperacusis does not occur E) all of the above F) none of the above NEU-553. Full recovery after Bell's palsy: A) occurs in 20% of the cases B) does not occur in elderly patients, hyperacusis, and cases of severe muscle weakness at the onset of the disease C) develops within 10-14 days D) does not depend on the severity of the lesion E) surgery should be immediately performed

NEUROLOGY

Answer: A

Answer: D

Answer: F

Answer: A

Answer: B

DrRognerud@www.medfever.com

187 / 343

Board Exam Question (Smiley + Star) + 2007 NEU-556. Epilepsy after head injury: A) is always accompanied by the same type of seizures B) usually occurs following closed skull injuries C) develops within several minutes or hours following the injury D) the correct therapy indicates a good prognosis E) all of the above F) none of the above NEU-558. An extradural hematoma: A) affects the middle cerebral artery B) is usually bilateral C) the patient always has a lucid interval which is the essential sign for the correct diagnosis D) usually accompanies fractures of the ethmoid bone E) all of the above F) none of the above NEU-559. An epidural hemorrhage: A) is usually localized in the scala media over the hemispheral convexity B) the CSF is usually hemorrhagic C) is usually accompanied by a lucid interval of several days D) usually elicits a contralateral pupil dilation E) usually causes ipsilateral hemiplegia NEU-563. In tuberculous meningitis: A) the upper cranial nerves are damaged B) the CSF glucose level is usually normal C) the WBC count in the CSF usually exceeds several thousand D) the CSF glucose level is elevated E) the CSF usually contains several hundred lymphocytes NEU-564. In the acute phase of bacterial meningitis: A) the glucose level is normal B) the cell count is between 50-100 C) the cell count (mainly polymorphonuclear cells) is between 1000-10,000 D) therapy can be started only after evaluation of the antibiogram E) the CSF pressure is low NEU-565. In acute bacterial meningitis the CSF shows: A) lymphocytes B) polymorphonuclear leukocytes comprising 80-90% of the cells C) a low protein content DrRognerud@www.medfever.com

NEUROLOGY Answer: D

Answer: F

Answer: A

Answer: E

Answer: C

Answer: B

188 / 343

Board Exam Question (Smiley + Star) + 2007 D) positive bacteriologic cultivation results E) all of the above F) none of the above NEU-566. In acute bacterial meningitis: A) clinical symptoms develop over several weeks B) occipital stiffness and Brudzinsky's sign are not always present (like in infants, elderly patients) C) therapy cannot be started before evaluation of the antibiogram D) a daily dose of 10,000 U of penicillin should be administered E) all of the above F) none of the above NEU-567. The most frequent cause of meningitis: A) Leptospira; Brucella B) Salmonella; Listeria C) E.Coli; Shigella D) Cocci; Hemophilus influenzae E) none of the above NEU-570. Viral meningitis: A) causes a typical CSF pressure elevation B) the cell count is usually several thousand C) polymorphonuclear cells are found in the CSF D) the pleocytosis in mumps is milder E) all of the above F) none of the above NEU-571. Viral encephalitis: A) usually accompanies viral meningitis B) rarely causes headache C) usually does not cause focal symptoms, seizures D) the EEG changes are not typical E) the CT is always normal NEU-574. Which symptom is not a sign of meningeal excitement? A) occipital stiffness B) "foxhound" posture C) Kernig's sign D) Brudzinsky's sign E) Chaddock's sign NEU-576. Which pathological process does not elicit any meningeal symptoms? A) a rupture of an intracranial aneurysm DrRognerud@www.medfever.com

NEUROLOGY

Answer: B

Answer: D

Answer: F

Answer: A

Answer: E

Answer: C

189 / 343

Board Exam Question (Smiley + Star) + 2007 B) an increase of the intracranial pressure C) any circumscribed softening of brain tissue D) leptomeningeal inflammation NEU-578. Which symptom is not typical of multiple sclerosis? A) a scanning speech B) a lack of the abdominal cutaneous reflex C) flaccid paraparalysis D) optic atrophy -temporal pallor E) pyramidal signs NEU-583. In which of the following can retrobulbar neuritis occur? A) a tumor of the optic nerve B) multiple sclerosis C) a tumor of the occipital lobe D) Foster-Kennedy's syndrome NEU-584. Which of the following pathological processes cannot be diagnosed on the basis of the fundus finding? A) the presence of an intracranial space occupying process B) whether the patient had an intracranial space occupying process at an earlier time C) an alteration of the vascular system D) any brain atrophy NEU-585. Which of the following is typical of multiple sclerosis? A) it mainly develops in elderly patients B) it is related to an infection caused by ticks C) any oligoclonal y-antibidies in the CSF is of diagnostic value D) a relapsing course with accumulating disability E) it improves after steroid or cytostatic therapy F) only (A), (B), and (C) are true G) only (A), (B), (C), and (D) are true H) only (C), (D), and (E) are true NEU-593. Which of the following causes ipsilateral blindness? A) ipsilateral damage of the optic tract B) ipsilateral damage of the optic nerve C) ipsilateral damage of the chiasma D) contralateral damage of the optic radiation NEU-606. The accomodation reaction: A) accompanies horizontal vision B) is important for acute vision DrRognerud@www.medfever.com

NEUROLOGY

Answer: C

Answer: B

Answer: D

Answer: H

Answer: B

Answer: B

190 / 343

Board Exam Question (Smiley + Star) + 2007 C) is important for vertical vision NEU-610. After a central type of damage to eye movements one of the following symptoms develop: A) conjugated deviation B) diplopia C) indistinct images NEU-611. Which of the following is true for a peripheral visual impairment? A) no diplopia occurs B) diplopia develops C) conjugated deviation develops NEU-622. Which of the following types of trigeminal neuralgia does not exist? A) symptomatic B) psychogenic C) genuine NEU-624. The symptoms of a peripheral facial nerve lesion include: A) the angle of the mouth on the ipsilateral side is localized deeper, and ptosis develops B) the angle of the mouth on the ipsilateral side is localized deeper, the patient has lagophthalmus and cannot wrinkle his forehead C) the angle of the mouth on the contralateral side is localized deeper. D) the angle of the mouth on the contralateral side is localized deeper, the patient has lagophthalmus and cannot wrinkle his forehead NEU-625. Which of the following is typical of a peripheral facial nerve lesion? A) ptosis on the ipsilateral side B) ptosis on the contralateral side C) lagophthalmus on the ipsilateral side D) lagophthalmus on the contralateral side NEU-626. Which of the following is true for a peripheral facial nerve paralysis? A) steroid therapy is recommended B) cytostatic therapy is recommended C) surgery depends on the EMG diagnostic findings D) the patient usually recovers in 5-10 days E) all of the above F) only (A) and (B) are'true G) only (B) and (C) are true DrRognerud@www.medfever.com

NEUROLOGY

Answer: A

Answer: B

Answer: B

Answer: B

Answer: C

Answer: -

191 / 343

Board Exam Question (Smiley + Star) + 2007 H) only (C) and (D) are true I) only (A) and (C) are true NEU-630. Which of the following are symptoms of a peripheral facial nerve paralysis? A) ipsilateral paralysis of the facial muscles of the eye, mouth, and forehead B) ipsilateral tinnitus C) ipsilateral lacrimation disorders; hyperacusis D) flavor sensation disorders on the whole surface of the tongue E) all of the above F) only (A), (B), and (C) are true G) only (B), (C) and (D) are true H) only (A), (B), and (D) are true. NEU-635. Which nerve is not localized in the cavernous sinus? A) the trochlear nerve B) the oculomotor nerve C) the abducent nerve D) the ophthalmic nerve E) the maxillary nerve NEU-636. Which symptom does not belong to those compromising a vestibular lesion? A) ipsilateral tilting of the body B) nystagmus directed towards the lesion C) nystagmus directed away from the lesion D) ipsilaterallateralization NEU-637. The direction of nystagmus is defined: A) according to its slow component B) according to its fast component C) according to the side of the vestibular lesion NEU-638. Tinnitus is usually caused by: A) arteriosclerosis B) hypertension C) medication D) a tumor of the cerebello-pontine angle E) multiple sclerosis F) all of the above G) only (A), (B), (C), and (D) are true H) only (B), (C), (D), and (E) are true

NEUROLOGY

Answer: -

Answer: E

Answer: B

Answer: B

Answer: G

DrRognerud@www.medfever.com

192 / 343

Board Exam Question (Smiley + Star) + 2007 NEU-640. Which of the following is not typical of Mnire's disease? A) confusion B) rotation vertigo C) a feeling of dissolution D) vegetative symptoms NEU-642. The result of the tuning fork test in a perception-type hearing impairment is: A) Weber test: lateralization to the left B) Rinn test: negative on the right side C) Weber test: lateralization to the right NEU-643. The result of the tuning fork test in a right-sided hearing impairment due to a disease in the middle ear is: A) Weber test: lateralization to the right B) Weber test: lateralization to the left C) Rinn test: positive on the right side NEU-652. Which of the following statements is false for a central lesion of the hypoglossal nerve? A) the tongue deviates towards the side of the cerebral lesion B) the tongue deviates to the side opposite to that of the cerebral lesion C) no fasciculations develop in the tongue D) no atrophy is observed in the tongue NEU-661. What does paraneoplasia mean? A) cerebellar atrophy B) polyneuropathy C) encephalitis D) abscess E) polymyositis F) all of the above G) only (A), (B), and (D) are true H) only (A), (B), and (E) are true NEU-667. Tremor at rest is typical of: A) multiple sclerosis B) epilepsy C) parkinsonism D) neurasthenia NEU-670. The examination of headaches includes: DrRognerud@www.medfever.com

NEUROLOGY Answer: A

Answer: A

Answer: A

Answer: A

Answer: H

Answer: C

Answer: H 193 / 343

Board Exam Question (Smiley + Star) + 2007 A) CT and skull X-ray studies B) focus examination and psychological studies C) provocative tests D) a CSF sample is always taken E) WBC count, RBC sedimentation rate F) all of the above G) only (A), (B), (C), and (D) are correct H) only (A), (B), (C), and (E) are correct I ) only (B), (C), (D), and (E) are correct NEU-671. A patient is considered as epileptic if: A) an epileptic seizure can be proved B) a patient had the seizure following a head trauma C) a patient had a seizure and epilepsy was noted in the family history D) a patient has systematic and EEG verified seizures E) all of the above F) only (A) and (B) are correct G) only (C) and (D) are correct NEU-673. Which spinal segment should remain intact for the patellar reflex? A) S 1 B) L4 C) S2 D) L5 and S1 NEU-674. Pseudobulbar syndrome is caused by: A) bilateral damage of the nuclei of the lower cranial nerves B) bilateral damage of the lower cranial nerves C) bilateral damage of the pyramidal tracts projecting towards the nuclei of the lower cranial nerves D) ipsilateral damage of the pyramidal tracts projecting towards the nuclei of the lower cranial nerves NEU-676. The distribution of spastic hypertonia includes: A) flexors of the upper extremities and extensors of the lower extremities B) flexors of the upper and lower extremities C) extensors of the upper and lower extremities D) all groups of muscles NEU-681. Which of the following symptoms does not occur in a lesion of the pyramidal tract? A) spastic hypertonia B) hyperreflexia of the deep reflexes DrRognerud@www.medfever.com

NEUROLOGY

Answer: D

Answer: B

Answer: C

Answer: A

Answer: D

194 / 343

Board Exam Question (Smiley + Star) + 2007 C) pyramidal symptoms D) fasciculation NEU-682. Which of the following symptoms does not belong to those of a transverse lesion? A) all sensory modalities are altered beneath the level of injury B) spasticity or paraparesis of the lower extremities C) urinary disorders D) segmental deficiencies (lack or alteration of certain reflexes, individual atrophy) E) dissociated sensory disorders NEU-683. Which of the following does not occur in central motor neuron damage? A) global atrophy B) hyperreflexia of the deep reflexes C) individual atrophy D) pyramidal signs E) decreased surface reflexes NEU-684. Select a pyramidal sign for the lower extremities: A) Mayer's reflex of the proximal joints B) Juster's sign C) Oppenheim's sign D) Wartenberg's sign NEU-685. Select a pyramidal sign for the upper extremities: A) Chaddock's sign B) Rossolimo's sign C) Tr6mner's sign D) Schaefer's sign NEU-686. Which of the following is not a pyramidal sign? A) Babinski's sign B) Brudzinsky's sign C) Rossolimo's sign D) Chaddock's sign E) Gordon's sign F) Tr.mner's sign NEU-687. Select the definition of an increased deep reflex: A) hyperreflexia B) an enlarged reflexogenic zone C) hyperreflexia accompanied with pyramidal signs DrRognerud@www.medfever.com

NEUROLOGY

Answer: E

Answer: C

Answer: C

Answer: C

Answer: B

Answer: C

195 / 343

Board Exam Question (Smiley + Star) + 2007 D) hyperreflexia accompanied with a grasping reflex NEU-689. Paralysis means: A) cessation of muscle strength B) decreased muscle strength C) flaccid muscles D) coordination disorders E) decreased muscular mass F) complete paraplegia NEU-691. The primary motor cortex: A) is localized in the parietal lobe B) is localized behind the central sulcus C) is localized just before the central sulcus D) is localized in the temporal lobe NEU-694. If a patient with a disc hernia suddenly develops paralysis and autonomic symptom's, then: A) the patient should be examined and operated within 6 weeks B) the patient should be examined and operated within 1 month C) bed rest, with the patient being examined and operated within 1 week D) the patient should be examined and operated immediately E) bed rest, with the patient being examined and operated within 6 weeks NEU-696. Which symptom does not belong to the cauda syndrome? A) flaccid paralysis B) hyporeflexia C) pyramidal signs D) assymmetric symptoms NEU-698. A typical representative of hypertonic hypokinetic disorders includes: A) Alzheimer's disease B) Pick's lobe atrophy C) Korsakoffs syndrome D) parkinsonism NEU-699. A typical representative of hypotonic hyperkinetic disorders includes: A) Wernicke's encephalopathy B) spastic torticollis C) Huntington's chorea D) Foster-Kennedy's syndrome

NEUROLOGY

Answer: B

Answer: C

Answer: D

Answer: C

Answer: D

Answer: C

DrRognerud@www.medfever.com

196 / 343

Board Exam Question (Smiley + Star) + 2007 NEU-701. Which of the following is not typical of polyneuropathy? A) decreased deep reflexes B) autonomic symptoms C) pyramidal signs D) sensory disorders E) paralysis is common NEU-702. Cerebellar hemorrhage: A) is always fatal B) surgery can be performed depending on the size of the hemorrhage C) should always be treated conservatively D) is easy to diagnose because the CSF is always hemorrhagic in these cases NEU-706. Which of the following is not an exteroreflex? A) the plantar reflex B) the abdominal cutaneous reflex C) the conjunctival reflex D) the cremaster reflex E) the masseter reflex NEU-708. Which statement is not true for the therapy of epilepsy? A) the importance of carbamazepine therapy has grown B) monotherapy is a method of choice C) therapeutic surgery should be considered in drug-resistant epilepsy D) the drug blood level evaluation is of growing importance E) the EEG changes are the most important signs for therapy F) certain antiepileptic drugs interact, decreasing each other's efficiency and increasing toxicity NEU-712. Which of the following is not a deep reflex? A) the conjunctival reflex B) the masseter reflex C) the ulnar reflex D) the biceps reflex NEU-713. Which reflex has its receptors in the corresponding muscle? A) the abdominal cutaneous reflex B) the grasping reflex C) the mucosal reflexes D) the plantar reflex E) the cremaster reflex DrRognerud@www.medfever.com

NEUROLOGY

Answer: C

Answer: B

Answer: E

Answer: E

Answer: A

Answer: B

197 / 343

Board Exam Question (Smiley + Star) + 2007 NEU-714. Which of the following is not considered as hyperkinesis? A) ballism B) intention tremor C) chorea D) athetosis E) myoclonus NEU-717. Which of the following is not a sign of psychic deterioration? A) oral exploration B) a tendency to pronate the distal extremities C) a grasping reflex D) a loss of initiation NEU-719. Where should the damaging focus be localized in order to elicit a central-type paralysis in the upper extremities? A) a parasaggital localization B) the precentral gyrus is damaged over the Sylvian fissure C) the postcentral gyrus is damaged over the Sylvian fissure D) a damage of the premotor cortex NEU-722. In which of the following do the deep reflexes remain unchanged? A) damage of a peripheral nerve B) damage of the motor neuron of the ventral horn C) damage of the parietal lobe D) cerebellar damage E) damage of the occipital lobe NEU-724. In which muscle groups is hemiparesis of the upper extremeties more pronounced? A) the flexors of the elbow B) the extensors of the elbow C) the abductors NEU-725. Assign Parkinson's disease to one of the following: A) a hypertonic hypokinetic disorder B) a hypotonic hyperkinetic disorder C) a hypotonic hypokinetic disorder D) a hypertonic hyperkinetic disorder NEU-727. Which of the following is not characteristic of a pyramidal tract lesion? A) hyporeflexia of the surface reflexes B) spastic hypertonia DrRognerud@www.medfever.com

NEUROLOGY

Answer: B

Answer: B

Answer: B

Answer: E

Answer: B

Answer: A

Answer: E

198 / 343

Board Exam Question (Smiley + Star) + 2007 C) Achilles clonus D) patellar clonus E) myoclonus in the muscles of the upper extremities NEU-728. Which spinal segment is required for the Achilles reflex? A) I B) L3 C) S 1 D) L2 NEU-729. Which of the following is not a pyramidal sign of the upper extremities? A) Hoffman's sign B) Oppenheim's sign C) Tr6mner's sign D) Juster's sign NEU-731. In which of the following does cogwheel rigidity develop? A) Foster-Kennedy's syndrome B) Parkinson's disease C) Wernicke's encephalopathy D) Marchiafava-Bignami's, disease NEU-734. At what level does the spinal cord end? A) L2 B) L5 C) S 1 D) S2 NEU-736. A "cerebellar fit" is caused by: A) an elevated intracranial pressure B) a congenital cerebellar anomaly C) alcohol-induced cerebellar atrophy D) vertobrobasilar insufficiency NEU-739. Which of the following symptoms is not typical of a hypertonichypokinetic disorder? A) rigor B) tremor C) akinesis D) chorea NEU-741. How is Oppenheim's reflex elicited? A) by pressing the Achilles tendon DrRognerud@www.medfever.com

NEUROLOGY

Answer: C

Answer: B

Answer: B

Answer: A

Answer: A

Answer: D

Answer: C

199 / 343

Board Exam Question (Smiley + Star) + 2007 B) by pressing the triceps muscle of the calf C) by drawing a forger along the edge of the tibia D) by drawing a pointless object along the lateral edge of the upper surface of the foot NEU-742. Which of the following is not typical of Lrfs reflex? A) an asymmetric reflex indicates a lesion of the pyramidal tract B) bilateral hyperreflexia indicates a lesion of the pyramidal tract C) it is a negative supporting reflex NEU-743. The phases of disorientation are: A) somnolence-stupor-coma B) stupor-somnolence-coma C) coma-stupor-somnolence NEU-745. Which of the following is typical of multiple sclerosis? A) the vision is usually involved B) diplopia is common C) a lack of abdominal skin reflexes D) intention tremor E) rigid hypertonia F) only (A), (B), (C), and (D) are true G) only (A), (B), and (E) are true H) only (B), (C), (D), and (E) are true NEU-747. The cause of deep sensation disorders is: A) a lesion of the spinal ventral horn B) a lesion of the spinal posterior column C) a lesion of the area around the central canal D) a lesion of the ventral root NEU-751. Where is the primary sensory cortex localized? A) just behind the central sulcus B) just before the central sulcus C) at the opercular area D) in the temporal lobe NEU-753. Where does the spinothalamic tract cross? A) in the spinal cord B) in the medulla C) in the pons D) in the mesencephalon

NEUROLOGY

Answer: B

Answer: A

Answer: F

Answer: B

Answer: A

Answer: A

DrRognerud@www.medfever.com

200 / 343

Board Exam Question (Smiley + Star) + 2007 NEU-755. Which of the following is typical of cervical spondylotic headaches? A) they are usually ipsilateral B) they are usually occipital and/or the pain radiates forward C) there are pathological findings on the X-ray of the neck D) they can be caused by abnormal (pathological) positions of the head E) all of the above F) none of the above NEU-757. Which of the following can be used for migraine therapy? A) hydergine (Imigran) B) Aspirin C) dihydroergotamine D) all of the above NEU-760. How is Chaddock's reflex elicited? A) by pressing the Achilles tendon B) by pressing the triceps surae muscle of the calf C) by drawing a finger along the edge of the tibia D) by drawing a pointless object along the lateral edge of the upper surface of foot NEU-761. How is Gordon's reflex elicited? A) by pressing the Achilles tendon B) by pressing the triceps surae muscle of the calf C) by drawing a finger along the edge of the tibia D) by drawing a pointless object along the lateral edge of the upper surface of foot NEU-762. Side effects of phenytoin therapy include: A) gingival hyperplasia B) anemia C) alterations in the bones D) enlarged lymph nodes E) sleep disorders F) all of the above G) only (A), (B), and (E) are true H) only (A), (C), and (E) are true I) only (A), (B), (C) and (D) are true NEU-763. The daily dose of phenytoin is: A) 0.03-0.07 mg/kg B) 3-7 mg/kg C) 30-70 mg/kg DrRognerud@www.medfever.com

NEUROLOGY Answer: E

Answer: D

Answer: D

Answer: B

Answer: I

Answer: B

201 / 343

Board Exam Question (Smiley + Star) + 2007 D) 100-150 mg/kg NEU-764. Side-effects of carbamazepine therapy include: A) allergy B) leukopenia C) polycythemia D) liver disorders E) all of the above F) only (A), (B), and (D) are true G) only (A), (B), and (C) are true H) only (B), (C), and (D) are true NEU-765. Which of the following is not a pyramidal sign? A) hard-palate reflex B) clonus of the foot C) tonic dorsal flexion of the big toe D) corneal hyperreflexia NEU-767. Which of the following is not a complication of chronic alcoholism? A) superior hemorrhagic polyencephalitis B) a lesion of the optic nerves C) central pontine myelinolysis D) syringomyelia NEU-768. The therapy of status epilepticus is: A) phenytoin (Diphedan) iv. B) diazepam iv. C) clonazepam iv. D) disulfiram E) all of the above F) only (B), (C), and (D) are true G) only (C) and (D) are true H) only (A), (B), and (C) are true NEU-769. The medication of primary choice in a grand mal attack is: A) phenytoin (Diphedan) B) carbamazepine C) dipropylacetate D) nitrazepam E) only (A), (B), and (C) are true F) only (A) and (D) are true G) only (C) and (D) are true H) all of the above

NEUROLOGY

Answer: F

Answer: D

Answer: D

Answer: H

Answer: E

DrRognerud@www.medfever.com

202 / 343

Board Exam Question (Smiley + Star) + 2007 NEU-778. Which of the following cannot develop as a consequence of alcoholism? A), polyneuropathy B) cerebral atrophy C) Wernicke's encephalopathy D) central pontine myelinolysis E) Foster-Kennedy's syndrome F) Marchiafava-Bignami's syndrome NEU-782. The causes of aphasia include: A) damage to the nuclei of the lower cranial nerves B) damage to the lower cranial nerves C) a cerebellar lesion D) a lesion of the dominant hemisphere NEU-796. Which pathological processes cannot be visualized by a CT? A) identification of an infarction during the early hours B) identification of any demyelinization disorders C) identification of abscesses D) identification of hemorrhages NEU-797. Which of the following is not applicable in myelography? A) the intravenous administration of contrast substance B) the intrathecal administration of contrast substance C) the intrathecal administration of air NEU-798. A simple spinal X-ray image cannot reveal: A) any degenerative alteration of the spine B) any osteolytic vertebral metastases C) any osteoplastic vertebral metastases D) any intramedullary processes NEU-799. A Doppler study can be used for the identification of. A) the cause of cerebral circulatory disorders B) the diagnosis of any space-occupying process in the brain C) Marchiafava-Bignami's disease D) Foster-Kennedy's syndrome NEU-804. Which of the following is commonly used in myasthenia gravis? A) domperidone (Motilium) B) papaverine (Meristin) C) pyridostigmine (Mestinon) D) mexiletine (Mexitil) E) moroxydine hydrochloride (Morgalin) DrRognerud@www.medfever.com

NEUROLOGY Answer: E

Answer: D

Answer: A

Answer: A

Answer: D

Answer: A

Answer: C

203 / 343

Board Exam Question (Smiley + Star) + 2007 NEU-806. A congested fundus: A) always indicates a brain tumor B) is frequently caused by inflammation C) is common in multiple sclerosis D) frequently occurs in brain tumors, but can also accompany other processes (for example obscure encephalopathies or lymphostatic encephalopathies) E) is always accompanied by severely impaired vision NEU-807. The normal cell count in the lumbar CSF is: A) 20-30/mm3 B) 30-40/mm3 C) 40-50/mm3 D) maximum 8-10/mm3 NEU-808. Which pathologic process is not detectable with a simple skull X-ray? A) progressive staging of intrasellar space-occupying processes B) tumors showing calcification C) a cranial fracture D) cerebral atrophy NEU-809. Cerebral angiography cannot be used for the detection of: A) vascular occlusions B) malformations C) the cause of cranial polyneuropathy D) a subdural hematoma NEU-811. The EMG is used for the following purpose: A) a study of the electric activity of the brain B) evoked potentials due to sensory stimuli C) the state of the muscles, activity of the motor neurons and peripheral nerves D) only intracellular leads are used in clinical practice NEU-815. Which of the following is the most common complication of chronic alcoholism? A) polyneuropathy B) Marchiafava-Bignami disease C) central pontine myelinolysis D) cerebellar vermis atrophy E) alcoholic hallucinosis

NEUROLOGY

Answer: D

Answer: D

Answer: D

Answer: C

Answer: C

Answer: A

DrRognerud@www.medfever.com

204 / 343

Board Exam Question (Smiley + Star) + 2007 NEU-817. Which symptom is not typical of delirium tremens? A) tremor B) increased vegetative symptoms C) visual and tactile hallucinations D) olfactory hallucinations NEU-819. An epileptic patient can drive a car if: A) the drug level is properly set B) if the patient is not a professional driver and has attacks only at night C) if the patient experienced no attacks for 6 weeks and the epilepsy is not due to a progressive disease D) if the patient does not take drugs and is attack-free for at least 2 years and epilepsy is not a sign of a progressive disease E) if the patient takes drugs and has no attacks F) if the patients takes drugs and has fits without unconsciousness NEU-821. During which period of the day is thrombosis of the cerebral arteries manifested for the first time? A) any time B) at day time during physical exercise C) at dawn D) when the patient goes to bed NEU-823. Damage of which lobe causes sensory aphasia? A) the frontal lobe B) the temporal lobe C) the parietal lobe D) the occipital lobe NEU-824. Which of the following is typical of borreliosis? A) it can be accompanied by arthralgia and cutaneous symptoms B) it can be followed by a bilateral paralysis of the peripheral facial nerve C) the CSF content is always normal D) it is caused by a virus E) only (A), (B), and (D) are true F) only (A) and (B) are true NEU-825. How is cerebral hemorrhage differentiated from "brain softening"? A) by the clinical symptoms B) by the course of the disease C) with the EEG D) by the case history DrRognerud@www.medfever.com

NEUROLOGY

Answer: D

Answer: D

Answer: C

Answer: B

Answer: F

Answer: E

205 / 343

Board Exam Question (Smiley + Star) + 2007 E) with a CT study NEU-827. Which of the following plays no role in the diagnosis of cerebral hemorrhage? A) the taking of blood pressure B) an EEG study C) a percussion of cardiac dullness D) the case history E) the determination of the immunoglobulin level NEU-828. Which of the following causes brain atrophy? A) perinatal damage B) alcoholism C) arteriosclerosis D) all of the above E) none of the above NEU-829. Which of the following does not cause polyneuropathy? A) alcoholism B) diabetes mellitus C) uremia D) impaired absorption E) parkinsonism NEU-831. Which of the following studies should be primarily performed if a brain tumor is suspected? A) CT B) angiography C) myelography D) electromyography E) transcranial Doppler NEU-832. Posttraumatic epilepsy frequently occurs in: A) damage to the parietal lobe B) damage to the occipital lobe C) cerebral commotion D) damage to the frontal lobe E) a contusion of the temporal lobe NEU-835. Which of the following is used to study the electrical activity of the brain? A) electromyography B) electromyelography C) electroencephalography DrRognerud@www.medfever.com

NEUROLOGY

Answer: -

Answer: D

Answer: E

Answer: A

Answer: E

Answer: C

206 / 343

Board Exam Question (Smiley + Star) + 2007 D) electroneuronography NEU-836. In which disease is the CSF measles antibody titer always high? A) Hurst encephalitis B) subacute sclerotizing panencephalitis C) diffuse periaxial encephalitis D) concentric periaxial encephalitis NEU-837. Which study is unnecessary in dementia? A) EEG B) EMG C) determination of the serum fat level D) IQ test E) CT NEU-838. Where is the CSF localized? A) the subdural space B) the subarachnoid space . C) under the pia mater D) above the arachnoid NEU-839. Which of the following studies is used to check medicated epileptic patients? A) the CSF and a blood smear B) a blood smear and CT (systematically) C) the blood level of the drug and a systematic CT D) the blood level of the drug and a blood smear E) the blood level of the drug and urinalysis NEU-844. In the acute phase of purulent meningitis: A) the lymphocyte count in the CSF is high B) the leukocyte count in the CSF is high C) the eosinophil count in the CSF is high NEU-845. A transcranial Doppler study cannot be used: A) for the identification of intracranial collaterals B) for the identification of stenosis of the intracranial arteries C) for the identification of intracranial tumors D) for the identification of any occlusions in the intracranial arteries NEU-850. Which of the following is not typical of alcoholic polyneuropathy? A) the symptoms are usually symmetric B) the symptoms are more pronounced in the distal parts of the DrRognerud@www.medfever.com

NEUROLOGY

Answer: B

Answer: B

Answer: B

Answer: D

Answer: B

Answer: C

Answer: D

207 / 343

Board Exam Question (Smiley + Star) + 2007 limbs C) the symptoms first occur in the lower limbs D) micturition disorders develop NEU-856. Which of the following describes the prognosis of Parkinsonism? A) Parkinsonism usually has a good prognosis B) the disease has a progressive character even despite effective medication C) a good prognosis of patients susceptible to therapy can be maintained for decades D) only the prognosis of patients resistant to therapy is poor NEU-858. Which drug is used for the treatment of parkinsonism? A) amantadine B) aminopyrine C) allopurinol D) phenytoin E) buformin NEU-859. The starting Madopar (L-Dopa + benserazide) dose in parkinsonism is: A) 0.06-0.1 mg B) 0.6-1 mg C) 60-100 mg D) 0.6-1 g E) 1-6 g NEU-861. Hemiparalysis with primary lower limb involvement and pyramidal symptoms develop after: A) an occlusion of the anterior cerebral artery B) an occlusion of the middle cerebral artery C) an occlusion of the posterior cerebral artery NEU-866. Which of the following is not an excitation sign? A) epilepsy B) micropsia C) a dreamy state D) miosis E) ansognosia NEU-868. Which area is damaged in Parkinson's disease? A) the dentate nucleus B) the substantia nigra C) the subthalamic nucleus DrRognerud@www.medfever.com

NEUROLOGY

Answer: B

Answer: A

Answer: C

Answer: A

Answer: E

Answer: B

208 / 343

Board Exam Question (Smiley + Star) + 2007 D) the putamen NEU-869. Which of the following is typical of a cholinergic crisis? A) sweating and salivation B) enhanced intestinal function C) both of the above D) none of the above MULTIPLE CHOICE QUESTIONS / TYPE I Select the correct answers to the following questions!!! ...each qestion may have more than one correct answer. NEU-871. Multiple Choice Question The two tests best in revealing changes developing in postvaccination encephalomyelitis are: A) the peripheral blood cell count B) the RBC sedimentation rate C) a study of the acute and convalescent serum D) the EEG E) a CSF study NEU-872. Leukemic involvement of the nervous system means: A) involvement of the meninges and the roots B) the presence of leukemic cells in the CSF C) the CSF glucose level is normal or elevated D) that CSF pleocytosis is rare E) none of the above NEU-873. The typical signs of a basal lesion of the frontal lobe include: A) astereognosis B) a positive grasping reflex C) homonymous superior quadrantanopia D) ipsilateral optic atrophy E) anosmia NEU-881. Typical sequels of a cauda equina lesion include: A) hyperreflexia in the lower limbs B) impotence C) a lack of abdominal reflexes D) a sensory loss in the perineal area E) a loss of the anal sphincter tone NEU-882. A lesion of the lateral medulla causes the following symptoms: A) ipsilateral Horner's syndrome DrRognerud@www.medfever.com

NEUROLOGY

Answer: C

Answer: DE

Answer: AB

Answer: BDE

Answer: BDE

Answer: ACE

209 / 343

Board Exam Question (Smiley + Star) + 2007 B) a loss of contralateral reflexes C) nystagmus D) paralysis of the ipsilateral hypoglossal nerve E) dysphagia MULTIPLE CHOICE QUESTIONS WITH KEY ANSWERS / TYPE II Every question or incomplete statement has only one answer in the following combinations: A) if the answers 1, 2, and 3 are true B) if the answers 1 and 3 are true C) if the answers 2 and 4 are true D) if only the answer 4 is true E) if all the four answers are true Select one of these key combinations!!! NEU-886. In the acute phase of a suspected cerebrovascular disease: 1) the CT shows no immediate changes if a patient has ischemia 2) the CT shows no changes for about 48 hours if a patient has had an infarction 3) the CT reveals immediate pathological signs in case of hemorrhage 4) a radiolucent CT image indicates hemorrhagic areas NEU-887. If a cerebrovascular disease is suspected: 1) the CT is a method of choice in cases of an intracranial hemorrhage 2) nearly all aneurysms can be visualized with a CT 3) the same CT image can usually visualize an old and a fresh infarction 4) a CT-guided lumbar puncture is not obligatory in the diagnosis of an intracranial hemorrhage NEU-890. In which disease does vertigo occur? 1) Foster-Kennedy's syndrome 2) trigeminal neuralgia 3) amyotrophic lateral sclerosis 4) Menire's disease NEU-893. In Which diseases is the cochlear nerve damaged? 1) trauma 2) herpes zoster 3) salicylate-induced damages 4) uremia

NEUROLOGY

Answer: -

Answer: B

Answer: D

Answer: E

DrRognerud@www.medfever.com

210 / 343

Board Exam Question (Smiley + Star) + 2007 NEU-899. Factors contributing to the development of congested papilla include: 1) that an elevation of the CSF pressure is directly transmitted to the CSF of the subarachnoid space surrounding the optic nerve 2) that an elevated CSF pressure disturbs venous circulation 3) edema of the optic fibers 4) a direct compression of the optic nerve NEU-902. In which diseases does paralysis of the eye muscles develop? 1) a sinus thrombosis 2) in multiple sclerosis 3) in Wernicke's encephalopathy 4) an aneurysm NEU-911. A lesion of the facial nucleus occurs in: 1) syringobulbia 2) bulbar paralysis 3) polioencephalitis 4) pontine tumors NEU-912. The acoustic system includes: 1) the otic ganglion 2) the geniculate ganglion 3) the stellate ganglion 4) the spiral ganglion NEU-914. The primary acoustic center is located in: 1) Brodmann's area 38 2) Brodmann's area 39 3) Brodmann's area 40 4) Brodmann's area 41 NEU-915. Which symptoms are typical of circulatory disorders in the brain stem? 1) they are usually provoked by head movement 2) visual disorders 3) swallowing disorder; hoarseness 4) typical pronounced rotatory nystagmus, vomiting NEU-923. Which of the following is typical of Parinaud's syndrome? 1) vertical nystagmus 2) weakness of upward gaze 3) it can be caused by a tumor of the pineal gland 4) Bell's effect is present DrRognerud@www.medfever.com

NEUROLOGY Answer: A

Answer: E

Answer: E

Answer: D

Answer: D

Answer: E

Answer: E

211 / 343

Board Exam Question (Smiley + Star) + 2007 NEU-924. The nuclei of the cerebellum include: 1) the emboliform nucleus 2) the globose nucleus 3) the dentate nucleus 4) the fastigial nucleus NEU-925. The cerebellum is involved in the following functions: 1) vision 2) gnostic performance 3) hearing 4) coordination of muscle activity NEU-926. Which of the following is typical of the thalamus? 1) it is here that subcortical processing of the environmental and internal stimuli occurs 2) it is connected with the cerebellum 3) it is a relay station for specific and aspecific afferent pathways 4) non-specific cortical stimulation occurs via the thalamus NEU-927. Which of the following structures are part of the limbic system? 1) the cingulate gyrus 2) the hippocampus 3) the fornix 4) the mammilary bodies NEU-929. The components of the vestibular system include: 1) the utricule 2) the saccule 3) the semicircular canals 4) Corti's organ NEU-930. Vertigo occurs in: 1) Meniere's disease 2) it can be of a psychogenic origin 3) intoxication 4) vestibular neuritis NEU-933. In which syndrome does the oculomotor nerve remain UNDAMAGED? 1) in Nothnagel's syndrome 2) in Benedikt's syndrome 3) in Weber's syndrome 4) in Fovill's syndrome DrRognerud@www.medfever.com

NEUROLOGY

Answer: E

Answer: D

Answer: E

Answer: E

Answer: A

Answer: E

Answer: D

212 / 343

Board Exam Question (Smiley + Star) + 2007 NEU-935. In wich diseases does a lesion of the facial nerve occur? 1) in otitis media 2) inflammation of the parotid gland 3) a viral infection 4) an acoustic neuroma NEU-937. Which symptoms can be caused by circulatory disorders of the vertebrobasilar area? 1) vertigo 2) confusion 3) drop attack 4) imbalance NEU-938. Which of the following is typical of sclerozing panencephalitis? 1) there is a typical onset in childhood 2) the child's school performance gets worse 3) there are high gamma globulin values 4) epileptic grand mal seizures and myoclonus NEU-939. Medication used,in multiple sclerosis includes: 1) levodopa + benserazide (Madopar) 2) pyridostigmine (Mestinon) 3) methyldopa (Dopegyt) 4) steroids NEU-941. Which of the following is damaged in poliomyelitis? 1) the spinal dorsal horn 2) the pontine tegmentum . 3) the thalamus 4) the spinal ventral horn NEU-943. Which of the following does not cause a peroneal lesion? 1) a fibular fracture 2) a dislocation of the knee joint 3) a compression-induced peroneal lesion 4) an occlusion of the posterior spinal artery NEU-944. In which cases of severe carotid stenosis is a carotid endarterectomy NOT PERFORMED? 1) a transient ischemic attack 2) in symptom-free carotid stenosis 3) a crescendo transient ischemic attack (TIA) DrRognerud@www.medfever.com

NEUROLOGY

Answer: E

Answer: E

Answer: E

Answer: D

Answer: D

Answer: -

Answer: -

213 / 343

Board Exam Question (Smiley + Star) + 2007 4) a hemorrhage in the brain matter NEU-945. Which diseases are usually accompanied by epileptic fits? 1) intracranial tumors 2) alcoholism 3) head injuries 4) multiple sclerosis NEU-946. Which pathological processes can cause unconsciousness? 1) a thrombosis of the basilar artery 2) "grand mal" epilepsy 3) an intracranial hemorrhage 4) syncope NEU-947. Which of the following medications is used in cerebral infarction? 1) pentoxifylline (Trental ) 2) vinpocetine (Cavinton) 3) mannitol (Mannisol) 4) xantinol (Xavin) NEU-949. Plasmapheresis is used in: 1) myasthenia gravis 2) polmyositis 3) Guillain-Barr's syndrome 4) Foster-Kennedy's syndrome NEU-951. Which disease(s) is (are) suspected if a patient complains of a slowly developing left spastic hemiparalysis? 1) a primary tumor of the right hemisphere 2) a right hemisphere hemorrhage 3) a right hemisphere metastasis 4) hernia of a cervical disc NEU-953. What are the sequellae of AIDS? 1) an occurrance of opportunistic infections 2) dementia 3) malignant intracranial tumors can develop 4) polyneuritis can develop NEU-955. What are the typical features of borreliosis? 1) it develops after a tick bite 2) it is a viral disease 3) arthralgia and polyneuropathy can develop DrRognerud@www.medfever.com

NEUROLOGY

Answer: -

Answer: E

Answer: E

Answer: A

Answer: B

Answer: E

Answer: B

214 / 343

Board Exam Question (Smiley + Star) + 2007 4) the CSF finding is always normal NEU-956. Which of the following is typical of Guillain-Barr's syndrome? 1) there is a dissociation of cell proteins 2) it has a poor prognosis 3) flaccid para-and tetraparalysis, and a loss of reflexes 4) a sudden onset NEU-957. Which of the following medications is used in parkinsonism? 1) levodopa + benserazide (Madopar) 2) bromocriptine 3) amantadine (Viregyt K) 4) pyridostigmine (Mestinon) NEU-958. Which of the following is typical of parkinsonism? 1) hypo-or akinesia 2) no mental symptoms 3) rigidity and tremor 4) spasticity NEU-959. Which of the following is typical of an intracranial hemorrhage? 1) a lobular hemorrhage is the most common form 2) the hemorrhage usually occurs in the area of the basal ganglia 3) it usually occurs in the basin of the posterior cerebral artery 4) it usually occurs in the basin of the lenticulostriate aartery NEU-960. Which of the following is typical of an intracranial aneurysm? 1) it can cause compression symptoms 2) it usually develops in the vertebrobasilar region 3) a subarachnoid hemorrhage can be the first sign 4) the first signs usually appear in children NEU-961. Common complications of subarachnoid hemorrhage include: 1) a vasospasm-induced softening 2) a relapse 3) a communicating hydrocephalus 3) secondary suppuration NEU-962. The criteria of brain death are: 1) coma 2) a loss of brain stem reflexes 3) a lack of spontaneous respiration 4) the exclusion of the effects of drugs and other toxic agents DrRognerud@www.medfever.com

NEUROLOGY

Answer: B

Answer: A

Answer: B

Answer: C

Answer: B

Answer: A

Answer: -

215 / 343

Board Exam Question (Smiley + Star) + 2007 NEU-965. Which of the following is typical of Wilson's disease? 1) a Kayser-Fleischer's ring on the cornea 2) autosomal recessive inheritance 3) it leads to dementia 4) extrapyramidal symptoms NEU-967. The local administration of botullinum toxin decreases: 1) athetosis 2) blepharospasm 3) chorea 4) spastic torticollis NEU-969. Which drug reduces increased tone spasticity? 1) Baclofen 2) clozapine (Leponex) 3) tolperisone (Mydeton) 4) levodopa + benserazide (Madopar) NEU-971. The medication of choice in a cholinergic crisis is: 1) neostigmine (Stigmosan) 2) pyridostigmine (Mestinon) 3) edrophonium (Tensilon) 4) atropine NEU-972. Which disease develops after a tick-bite? 1) tick-encephalitis 2) borreliosis 3) tick-meningoencephalitis 4) subacute sclerozing panencephalitis NEU-973. Malignant tumors can be accompanied by: 1) cerebellar atrophy 2) progressive multifocal leukoencephalopathy 3) polyneuropathy 4) dermatomyositis

NEUROLOGY

Answer: E

Answer: C

Answer: B

Answer: D

Answer: A

Answer: -

MULTIPLE CHOICE QUESTIONS WITH KEY ANSWERS/TYPE III Every question or incomplete statement has only one answer in the following combinations: A) if the answers 1, 2, and 3 are true B) if the answers 2, 3, and 4 are true C) if the answers 1, 3, and 5 are true D) if the answers 1 and 5 are true E) if the answers 2 and 5 are true DrRognerud@www.medfever.com 216 / 343

Board Exam Question (Smiley + Star) + 2007 F) if the answers 3 and 4 are true Select one of these key combinations!!! NEU-975. Which of the following factors contribute to the development of a cerebral venous thrombosis? 1) ovulation inhibitors 2) the postpuerperal period 3) an infectional, septic focus 4) thrombocytopenia NEU-976. Which of the following is typical of a subdural hematoma? 1) a cranial fracture is an important prerequisiste 2) the CSF finding is of diagnostic value 3) it more frequently develops in alcoholics 4) it can be typically diagnosed with a CT NEU-978. Which of the following frequently causes spastic paralysis of the lower limbs? 1) multiple sclerosis 2) a parasagittal meningioma 3) the non-acute phase of an anterior spinal artery occlusion 4) Guillain-Barr's syndrome 5) spastic paralysis of the lower limbs is never caused by any spinal space-occupying processes NEU-979. Which of the following structures is usually damaged in amyotrophic lateral sclerosis? 1) the pyramidal system 2) the spinothalamic tract 3) the spinal ventral motor neurons 4) the thalamus 5) the visual system NEU-980. In subarachnoid hemorrhage: 1) surgery is the method of choice 2) surgery should be optimally performed between days 2-10 3) a dull, not severe headache can be present 4) conservative therapy is the method of choice 5) meningeal excitation signs develop

NEUROLOGY

Answer: A

Answer: F

Answer: A

Answer: C

Answer: D

ASSOCIATION QUESTIONS Associate the following terms/statements marked by the letters A, B, C... with the corresponding statements/terms marked by and in the order given by the figures 1, 2, 3... ...for example: 1-C, 2-B, 3-A, 4-D. Put the answer as C, B, A, D! (Note: Different statements can be associated with the same terms!!!) DrRognerud@www.medfever.com 217 / 343

Board Exam Question (Smiley + Star) + 2007 NEU-981. Associate the following term(s) with their corresponding statement(s)! A) Brodmann's area 39, 40 B) Brodmann's area 18, 19 C) Brodmann's area 4 D) Brodmann's area 3, 2, 1 E) Brodmann's area 6, 8 F) Brodmann's area 17 G) Brodmann's area 41, 42 1) somatic sensory cortex 2) speech association area 3) perception of vision 4) visual association 5) acoustic cortex 6) primary motor cortex 7) supplementary motor cortex NEU-982. Associate the following term(s) with their corresponding statement(s)! A) Acetylcholine B) Norepinephrine C) Epinephrine D) Dopamine 1) a precursor of noradrenaline 2) it is synthesized by the spinal motor neurons 3) the main amine produced by the adrenal chromaffin cells 4) the main amine produced by postganglionic sympathetic neurons NEU-984. Associate the following statements(s) with their corresponding term(s)! A) enlarged optic foramen B) suprasellar calcification C) forward protrusion of the aqueduct D) backward protrusion of the aqueduct 1) Medulloblastoma 2) Glioma of the optic nerve 3) Craniopharyngioma 4) Brain stem glioma NEU-985. Associate the following statements(s) with their corresponding term(s)! A) rarely occurs in children but occurs frequently in adults B) frequently occured in the past but is rare nowadays C) surgical treatment D) a paralysis of forward gaze E) the most frequent cerebral glioma 1) Tuberculoma 2) Meningioma DrRognerud@www.medfever.com

NEUROLOGY

Answer: -

Answer: DACB

Answer: CABD

Answer: BADEC

218 / 343

Board Exam Question (Smiley + Star) + 2007 3) Pinealoma 4) Ependymoma 5) Cerebellar astrocytoma NEU-990. Associate the following term(s) with their corresponding statement(s)! A) Ethosuccimide (Suxilep) B) Ergotamine C) Diphenylhydantoin (Diphedan) D) Carbamazepine E) Dramamine (Daedalon) 1) Cluster headache 2) migraine with aura 3) "grand mal" seizures 4) "petit mal" seizures 5) neuralgia 6) Menire's syndrome NEU-993. Associate the following statement(s) with their corresponding term(s)! A) brain stem tumor B) pediatric cerebellar hemisphere tumors C) tumor of the IV ventricle D) medial line cerebellar tumor E) the most common primary cerebellar tumor in adults 1) Astrocytoma 2) Polar spongioblastoma 3) Hemangioblastoma 4) Medulloblastoma 5) Ependydoma NEU-997. Associate the following statement(s) with their corresponding term(s)! A) a loss of voluntary movements without paralysis B) a cortical or subcortical lesion C) an ipsilateral loss of vibration and position sensation, and a contralateral loss of pain and temperature sensation D) damage to the pyramidal system and involvement of the oculomotor nerve 1) Aphasia 2) Apraxia 3) Brown-Sequard's syndrome 4) Weber's syndrome NEU-1006. Associate the following term(s) with their corresponding statement(s)! A) Motor aphasia B) Sensory aphasia C) Conduction aphasia D) Amnestic aphasia DrRognerud@www.medfever.com

NEUROLOGY

Answer: BBCADE

Answer: BAEDC

Answer: BACD

Answer: DACBE

219 / 343

Board Exam Question (Smiley + Star) + 2007 E) None of the above 1) the center is localized in the left lower parietal lobe 2) the center is localized in the distal part of the third frontal gyrus 3) an intact insula is important 4) the center is localized in the left upper temporal gyrus 5) the center is localized in the occipital lobe NEU-1008. Associate the following term(s) with their corresponding statement(s)! A) Guillain-Barr syndrome B) Multiple sclerosis C) Both (A) and (B) D) None of the above 1) it has a good prognosis 2) it has a poor prognosis 3) it is accompanied by spastic paraparalysis 4) it is accompanied by flaccid paraparalysis, hypo-or areflexia 5) it is accompanied with signs of meningeal excitation 6) usually spontaneous improvement occurs 7) usually requires hospital treatment 8) there is damage of the pyramidal tract 9) usually cerebellar symptoms develop NEU-1009. Associate the following term(s) with their corresponding statement(s)! A) Cisternal puncture B) Lumbar puncture C) Both (A) and (B) D) None of the above 1) it is less dangerous than the other noted procedure 2) it does not (or scarcely) causes any postpuncture symptoms 3) if applied the CSF flows spontaneously 4) the normal CSF sample contains 8-l0mm3 cells 5) the CSF sample contains 0.1-0.2 g/L protein 6) the CSF sample contains 1-2 g/L protein 7) in optimal cases the puncture needle should be introduced between cervical vertebra C1 -C2 8) the normal CSF sample is colorless, odorless and water-clear 9) the CSF is obtained from the interpeduncular cistern NEU-1010. Associate the following term(s) with their corresponding statement(s)! A) Mannisol + Vasodilators B) Corticosteroids + Cytostatics C) Surgery D) Myorelaxant E) B complex vitamins F) Carbamazepine (Tegretol + Stazepin) G) None of the above DrRognerud@www.medfever.com

NEUROLOGY

Answer: ABBADACBB

Answer: BABBADDCD

Answer: ACCECFGB

220 / 343

Board Exam Question (Smiley + Star) + 2007 1) . brain softening 2) subarachnoid hemorrhage 3) disc hernia with foot paralysis and urine incontinence 4) alcoholic polyneuropathy 5) cerebellar hemorrhage 6) epilepsy (idiopathic) 7) basilar meningitis 8) polymyositis

NEUROLOGY

CASE STUDIES Answer the multiple task questions (simple choice and multiple choice with/without key answers; relation analysis etc.) as they are related to each case study!!! NEU-1011. A 39-year-old male patient was treated for renal disease in the past. Since that time his blood pressure has been about 160 mmHg and he has complained of regular unilateral headaches. Two days ago he had suffered a severe occipital headache after trying to lift a heavy object. Further the patient experienced sweating, turned pale and complained of vertigo; he later vomited several times, but had no nausea before vomiting. He also complained of impaired vision and that light disturbs him. Present status: blood presure: 185/85 mmHg; heart rate 60/min; temperature: 37.3 C. A systolic murmur at the apex can be recognized. The liver is enlarged by two fingers. Mild occipital stiffness. Positive Brudzinsky's and Kernig's signs. Latent paralysis of the right limbs. A positive Babinski's reflex. 6.1011/1. Single Choice Question The most probable diagnosis is: A) uremia B) meningitis C) subarachnoid hemorrhage D) myalgia due to cervical diskopathy E) migraine headache F) cerebral tumor 6.1011/2. Single Choice Question The most important immediate intervention is: A) suboccipital lidocaine infiltration plus analgetics B) immediate admission to a NEUrologic ward C) an X-ray of the patient's spine D) further, more intensive history taking E) a complete ophthalmologic study F) a renal function study 6.1011/3. Single Choice Question If a CSF study is performed which CSF findings would you expect? A) a water clear CSF B) a greenish-yellow CSF C) a xanthochromic CSF D) an opaque CSF E) a hemorrhagic CSF

Answer: -

DrRognerud@www.medfever.com

221 / 343

Board Exam Question (Smiley + Star) + 2007 NEU-1012. A 21-year-old male patient had left-sided otologic surgery performed at the age of six. At the age of eleven he had mumps and hepatitis. A month before admission the patient complained of malaise and vertigo. 2-3 weeks later he had a pulsating headache, fever (38C) and nausea. At admission: no internal symptoms were found. Occipital stiffness was present but no focal neurologic symptoms could be observed. A lumbar puncture revealed 760/mm3 cells (predominantly lymphocytes). 6.1012/1. Single Choice Question The most probable diagnosis is: A) cerebral tumor B) basilar tuberculous meningitis C) purulent meningitis D) lymphocytic meningitis 6.1012/2. Single Choice Question The typical symptom of this disease is: A) a lack of neurologic focal symptoms B) confusion C) a pulsating headache D) an increased lymphocyte count E) meningeal symptoms and an increased lymphocyte count 6.1012/3. Single Choice Question The most important diagnostic examination is: A) CT B) carotid angiography C) pneumoencephalography D) a detailed CSF study E) electroencephalography 6.1012/4. Single Choice Question Characteristic dynamics of this disease include: A) ictal onset B) a steady progression for many years C) an intermittant progression D) it can develop within several days or weeks E) remissions can occur NEU-1013. A 58-year-old doctor suffering for many years from hypertension collapses at work. At admission: confusion, aphasia and right-sided weakness are found. BP: 230/160 mmHg. A lumbar puncture reveals a hemorrhagic CSF with a xanthochromic supernatant. 6.1013/ l. Single Choice Question The most probable etiology is: A) a hypertensive intracerebral hemorrhage B) a cerebral embolism C) a cerebral thrombosis 6.1013/2. Single Choice Question Which other pathological changes can develop in the course of this disease? A) gastric erosion DrRognerud@www.medfever.com

NEUROLOGY

Answer: -

Answer: -

222 / 343

Board Exam Question (Smiley + Star) + 2007 B) myocardial infarction C) fat atrophy of the liver 6.1013/3. Single Choice Question Which therapy would you suggest? A) steroids B) antihypertensive therapy C) hydantoin iv D) all of the above E) none of the above NEU-1014. A 40-year-old female patient presented with severe back pain. It began at her work place when she bent down and suddenly had a piercing back pain, so that she could not stand upright. The pain radiated along the dorsal surface of her lower limbs down to her ankle. Coughing and sneezing increased the pain. At examination: no cranial nerve symptoms, decreased Achilles reflex and hypesthesia of the S1 dermatome on the right side. Paralumbar defense, a flattening of the normal lumbar lordosis, forced posture, and a positive Lasegue's sign were also found. Lumbosacral X-ray: flattened L5 with elongated ventral and dorsal edges of the vertebra. Lumbar CSF: cell count: 2 mm3, total protein: 0.95 g/L. 6.1014/ 1. Single Choice Question The most probable diagnosis is: A) spondylolisthesis B) a lumbosacral herniated disc C) a compressive fracture of the L5 vertebra D) an intramedullary tumor at the lumbar level E) lumbar spondylosis 6.1014/2. Single Choice Question The least typical symptom is: A) paravertebral defense B) segmental sensory disorders C) a flattening of the normal lumbar lordosis D) elongated vertebral edges E) an increase of the acute root pain when coughing or sneezing 6.1014/3. Select One Of The Key Combinations Which of the following is not important in this diagnosis? 1) myelography 2) spinal X-ray 3) a CSF study 4) EMG 5) a muscle biopsy A) only (1) and (3) are true B) only (3) and (4) are true C) only (2) and (4) are true D) only (2) and (5) are true E) only (4) and (5) are true 6.1014/4. Single Choice Question Immediate surgery is indicated because of: A) foot paralysis DrRognerud@www.medfever.com

NEUROLOGY

Answer: BDEA

223 / 343

Board Exam Question (Smiley + Star) + 2007 B) a further increase of pain C) relapses D) a loss of the Achilles reflex E) restricted movements of the lumbar vertebra NEU-1015. Emergency admission by ambulance: a 48-year-old female patient and her husband left their apartment in the morning. On the way to work the patient's husband observed jerks in the left hand and the left corner of the mouth of the patient. The patient became unconscious, urinated, and the jerks continued for about a minute in all four limbs.So her husband immediately called for an ambulance. In the ambulance the patient regained consciousness and could only talk slowly and with difficulty. Weakness in the left hand developed. At examination: left sided hemiparesis with a slight facio-brachial prevalence. Hyperreflexia of the tendon reflexes. BP: 160/ 100 mmHg; heart rate 58/min; no fever. History: Occupation: administrator, consumes 4 cups of coffee and smokes 10 cigarettes daily. The patient complains of a periodic occipital headache that has been present for about 5 years. A year ago hypertension (160/100 mmHg) was diagnosed. Since then the patient has been on 3x1 tabl. of methyldopa daily. Three weeks ago a periodic headache turned into a severe permanent one and the patient regularly vomited in the morning. Her blood pressure increased to 180/120 mmHg and therefore additional dihydroergotoxin (Redergam) therapy was prescribed (3x20 drops). The patient complains of fatigue, memory disorders and a loss of concentration at her work. Sometimes she has vision disorders (darkness for several seconds) and feels that she has no control over her left hand. 6.1015/ 1. Single Choice Question The most probable diagnosis is: A) a destructive pontine hemorrhage B) epilepsy C) meningitis D) cerebral tumor E) thrombosis of the right internal carotid artery 6.1015/2. Single Choice Question The most relevant study for this diagnosis is: A) a CSF sample B) an EEG study C) consultation with an internist D) right-.sided carotid angiography E) lumbar pneumoencephalography 6.1015/3. Single Choice Question What should be done? A) immediate admission to an internal ward B) after examination, a neurologic follow-up on an outpatient basis C) recommend antiepileptic and antihypertensive therapy which should be controlled by the family doctor D) immediate admission to a neurologic ward E) bed rest at home DrRognerud@www.medfever.com

NEUROLOGY

Answer: -

224 / 343

Board Exam Question (Smiley + Star) + 2007 6.1015/4. Single Choice Question The ideal therapy would be: A) intravenous vasodilation therapy B) dehydration C) antiepileptic medication D) antihypertensive drugs E) anticoagulation therapy 6.1015/5. Single Choice Question Some recommendations on how to alter her way of life are to: A) avoid coffee, cigarettes, and stress B) same as above plus antihypertensive medication C) ask the patient for informed consent for surgical therapy D) prepare documents which would allow the patient to retire NEU-1016. A 39-year-old male patient was treated for renal disease in the past. Since that time his blood pressure has been about 160 mmHg and he has complained of regular unilateral headaches. Two days ago he had suffered a severe occipital headache after trying to lift a heavy object. Further the patient experienced sweating, turned pale and complained of vertigo; he later vomited several times, but had no nausea before vomiting. He also complained of impaired vision and that light disturbs him. Present status: blood presure: 185/85 mmHg; heart rate 60/min; temperature: 37.3 C. A systolic murmur at the apex can be recognized. The liver is enlarged by two fingers. Mild occipital stiffness. Positive Brudzinsky's and Kernig's signs. Latent paralysis of the right limbs. A positive Babinski's's reflex. 6.1016/ 1. Single Choice Question The most probable diagnosis is: A) uremia B) meningitis C) subarachnoid hemorrhage D) myalgia due to cervical diskopathy E) migraine F) a cerebral tumor 6.1016/2. Single Choice Question The most important immediate intervention is: A) suboccipital lidocaine infiltration plus analgetics . B) immediate admission to a neurologic ward C) an X-ray of the patient's spine D) further, more intensive history taking E) a complete ophthalmologic study F) a renal function study 6.1016/3. Single Choice Question The most important diagnostic study is: A) angiography B) hemostatic study C) X-ray D) a study of the eye fundus and a lumber puncture E) EEG F) echo-encephalography DrRognerud@www.medfever.com

NEUROLOGY

Answer: CBDE

225 / 343

Board Exam Question (Smiley + Star) + 2007 6.1016/4. Single Choice Question If a CSF study is performed which CSF findings would you expect? A) a water clear CSF B) a greenish-yellow CSF C) a xanthochromic CSF D) a opaque CSF E) a hemorrhagic CSF NEU-1017. A 42-year-old male patient who has never had any systematic disease in the past. Ten years ago the patient had a head injury and became unconscious. An X-ray at that time study revealed a cranial fracture line and the CSF was hemorrhagic. For five years the patient has been having regular (every 2-3 months) attacks of unconsciousness, limb jerks, biting of the tongue and urination. He never remembers his attacks. At examination: no neurologic symptoms were found. Routine laboratory findings were normal. EEG: bilateral irritation signs without lateralizationerve pneumoencephalography (PEG): symmetric wide ventricles and marked subarachnoid air filling. 6.1017/1. Single Choice Question The most probable diagnosis is: A) proneness to orthostatic collapse B) hypoglycemic fits C) a cerebral tumor D) posttraumatic epilepsy E) Adams-Stokes syncope 6.1017/2. Single Choice Question The most typical symptom of this disease is: A) disorientation B) memory disorders C) "grand mal" type seizure attacks D) dementia E) changes of personality 6.1017/3. Single Choice Question The typical CSF finding is: A) a normal CSF B) an elevated total protein level C) pleocytosis D) cell protein dissociation E) hemorrhagic CSF NEU-1018. A 59-year-old female patient has been hypertensive for years. She had a tonsillectomy and an appendectomy during her childhood. The patient regulary has swollen legs. One month before admission the patient's behaviour had changed. She became silent. unmotivated, could not do her job and neglected herself. The patient became absent-minded and several days before admission urinated and defecated in bed. At examination: BP: 140/80 mmHg; heart rate: 64/min. Bilateral mild anasarca and induration of the limbs. Neurologic status: fundus: DrRognerud@www.medfever.com

NEUROLOGY

Answer: -

Answer: -

226 / 343

Board Exam Question (Smiley + Star) + 2007 blurred left papilla. The left pupil is slightly narrowed. A right-sided central facial paralysis is present. A latent paresis in the right extremities. Babinski's sign, hyperreflexia of the deep reflexes on the right side. Bilateral oral reflexes and an increased grasping reflex. Psychic condition: temporal and spatial disorientation; slow cognition; pronounced dysarthria; impaired memory. Routine laboratory studies: within normal limits. Skull X-ray: no pathological alterations. EEG: slow temporal activity on the left side. Left carotid angiography: the anterior cerebral artery is displaced 2 cm to the right. 6.1018/ 1. Single Choice Question The most probable diagnosis is: A) hypertonic encephalopathy B) a cerebral tumor C) presenile dementia D) cerebral atrophy E) encephalomalacia of the left hemisphere 6.1018/2. Single Choice Question The typical clinical symptoms: A) absent-mindness B) incontinence C) central paralysis D) signs of psychic deterioration E) early signs of fundal congestion 6.1018/3. Single Choice Question Typical dynamics of this disease includes: A) a steady progression B) slow improvement C) a fast onset followed by slow deterioration D) deterioration caused by acute attacks E) transient remissions 1 6.1018/4. Single Choice Question Which studies play no role in this diagnosis? A) angiography B) EEG C) computer tomography D) cisternal puncture E) lumbar puncture NEU-1019. A 2-year-old male child with no history of any previous diseases was brought to the hospital by his friends. While running uphill the patient had experienced severe frontal and occipital headaches and collapsed. His friend had observed a transient disorder of speech and the patient had vomited several times in the ambulance. At admission: occipital stiffness; Kernig's sign positive; multiple poollike hemorrhages in the fundus; a central facial paralysis on the right side; right hand tends to move back when lifted; dorsal flexion tendency on the right side; dysarthria; somnolence. Lumbar puncture: CSF with massive uniform hemorrhage flowing at increased pressure. 6.1019/ 1. Single Choice Question DrRognerud@www.medfever.com

NEUROLOGY

Answer: DC

227 / 343

Board Exam Question (Smiley + Star) + 2007 The most probable diagnosis is: A) cerebral hemorrhage B) hemorrhagic encephalitis C) hemorrhagic infarction D) subarachnoid hemorrhage E) cerebral contusion 6.1019/2. Single Choice Question The pathomechanisms of the disease include: A) rupture of the vascular wall due to a sudden increase of blood pressure B) secondary hemorrhage in a hypoxic area C) angioma or aneurysm rupture D) rupture of a pontine vein E) capillary hemorrhage NEU-1020. Case Study 45-year-old miner with no history of any previous diseases presented to the hospital. About a year ago he started complaining of frontal headaches, though did not consider those a serious problemuscle Recently the headaches appeared more frequently. Before admission the patient had collapsed while watching TV. His family told that his whole body became rigid, his eyes became wide open and his limbs were jerking. The attack lasted for 3-4 minutes, during which the patient bit the left side of his mouth and urinated. After the attack he was very tired and slept. At admission: a central facial and hypoglossal paralysis on the right side. Slight hyperreflexia of the proprioreflexes in the right upper limb. The latent paralysis test showed pronation of the right upper limb. EEG: local temporal cortical disorder of organic character. CSF: cell count: 0/mm3; total protein: 95 mg%; CSF protein level (P.ndy): ++; benzoic: normal. 6.1020/ 1. Single Choice Question Define the character of the attack: A) a functional fit B) a traumatic ischemic attack C) an epileptic attack D) hyperesthesia of the carotid sinus 6.1020/2. Single Choice Question Assign the above disease to one of the following groups: A) psychogenic background -neurosis, exhaustion B) disorders of cerebral circulation, hypoxia C) a space occupying process D) chronic toxicity (for example alcoholism) concealed by the patient 6.1020/3. Single Choice Question What should be done? A) observation at home B) immediate hospitalization C) delayed hospitalization

NEUROLOGY

Answer: CCB

DrRognerud@www.medfever.com

228 / 343

Board Exam Question (Smiley + Star) + 2007 NEU-1021. Case Study A 56-year-old male patient, (occupation: unskilled construction worker) complains of back pain that has persisted for 3 years and of gastric pain that has lasted for about 2 years. The patient had no other diseases in his history. His wife said that for several weeks her husband has had a bad disposition and that he had complained of dull headaches, and fatigue. Three weeks before admission the patient was on a sick-allowance due to general malaise. At home he became more and more silent, inactive, stayed in bed all day long and lost his usual interests. Two weeks ago his gait became unsteady;. the patient drags his left leg and uses his left hand less than his right hand because he drops objects gripped by the left hand. During the last 3-4 days he has complained of nausea and vomiting often. He has become somnolent and could walk only with help. He had no fever. The patient has smoked 10-15 cigarettes daily for about 30 years; he has also consumed 50 ml liquor twice a day for 4-5 years. Once or twice a week he consumes even more and gets drunk. The patient never had surgery. Two months ago when the patient was drunk, his neighbour had seen him collapse and hit his head against a stone For about half a minute he remained motionless, then regained his consciousness and went home. He vomitted twice but thought it was due to alcohol. Since he scarcely had any injures he did not consult a doctor, but during the weekend the patient stayed in bed complaining of headache and nausea. On Monday he went to work and remained complaint-free till the present pre-admission complaints. At admission: heart rate 52/min.; BP: 170/90 mmHg; temperature: 36.6oC; dry, coated tongue/ fetor. Slight epigastric hypersensitivity to compression. No other internal disorders. The patient is slightly somnolent, and cannot maintain his attention. The patient has a degree of temporal disorientation, is indifferent, and has no interest in his surroundings. His speech and movement are slow. He does not cooperate well during the examination. Neurologic status: mild occipital stifness; the right pupil is wider than the left one. He barely feels a needle prick or a touch of cotton wool on the left side of his face and body. When showing his teeth the left corner of his mouth does not follow the right one. Hyperreflexia of the proprioreflexes of the left limbs is present. The grip of his left hand is weaker. The patient's elevated extremities tend to fall down. The left extremities are more clumsy in performing pointing tests. Instability in the Romberg's position. 6.1021 / 1. Select One Of The Key Combinations Which of the following pathological processes is the most probable? 1) superior hemorrhagic polioencephalitis 2) chronic subdural hematoma 3) multiform glioblastoma 4) Cerebral arteriosclerosis 5) Alzheimer's or Pick's disease A) only (1) and (2) are true B) only (4) and (5) are true C) only (2) and (3) are true D) only (1), 2) and (4) are true DrRognerud@www.medfever.com

NEUROLOGY Answer: -

229 / 343

Board Exam Question (Smiley + Star) + 2007 E) only (3) and (4) are true 6.1021/2. Select One Of The Key Combinations Which of the following processes are the least probable? 1) cerebral embolism 2) metastatic tumor 3) medulloblastoma 4) astrocytoma 5) encephalitis virus A) only (1), (2) and (3) are true B) only (2), (4) and (5) are true C) only (1), (3) and (4) are true D) only (1), (3) and (5) are true E) only (2) and (4) are true 6.1021/3. Single Choice Question Correct measures taken by the family doctor includes: A) close observation at home: prescription of analgetics, antiemetics, sedatives, vasodilators; (The following steps will depend on the course of the disease). B) admission to find any suspected primary tumor C) emergency admission to an internal medicine ward D) emergency admission to a neurologic -neurosurgery ward E) the patient should first be attended by the family doctor and should be examined (fundus study) by an ophthalmologist the next day; then the patient should be sent to a neurologist for further study. 6.1021/4. Single Choice Question The possible localization of this pathological process is: A) supratentorial B) the posterior cranial fossa C) the craniospinal area D) the spinal cord E) the extracranial part of a large artery supplying the brain 6.1021/5. Single Choice Question If the process is due to a malignant glioma, which outcome would you expect after a successful operation? A) death within several weeks B) relapse after 1/2 -1 year C) a 10-12 year remission D) a full recovery is expected E) extrecerbral metastases can develop NEU-1022. Case Study A 41-year-old female patient, (occupation -usher), with no history of previous illnesses was admitted to the hospital. The day before admission she complained of blurred vision, nausea, and a severe, pulsating tempo-occipital headache on the right side which lasted for about 4 hours, then ceased. The patient felt that the left extremity became "clumsy", but it lasted only till the next morning. The patient reported that she had similar severe, pulsating, left-or right-sided headaches with nausea during menses for about 20 years. During two pregnancies DrRognerud@www.medfever.com

NEUROLOGY

Answer: CA

230 / 343

Board Exam Question (Smiley + Star) + 2007 the headaches never occurred. At admission: her neurologic status was negative, blood pressure 120/80 mmHg. 6.1022/ 1. Single Choice Question Which disease would you suspect? A) a focal epileptic attack B) a multiple sclerosis attack C) migraine with complications D) glaucoma E) malaise due to increased intracranial pressure 6.1022/2. Single Choice Question What should be done? A) examination and medication B) ophthalmologic consultation C) immediate admission to a neurosurgical ward D) refer the patient to a neurologic ward NEU-1023. Case Study A 74-year-old male patient, (occupation -tailor) has a 20 year history of hypertension. For 15 years he systematically takes reserpine (Rausedyl) -occasionally. with thiazide diuretics. Three years ago the patient noticed that his movement had slowed down, he could only walk in small steps, had difficulties in initiating movement or turning around. He has also aquired a stooping posture. He felt that his extremities became rigid and he could talk in a low voice. During several months his gait became uncertain and he reported falling down when turning around. At admission: slight occipital rigor, stone face, monotonous speech, low voice. Cranial nerves normal. Inertia, uniform proprioreflexes, loss of extroreflexes, no pathological reflexes. All extremities exhibit a medium rigorous tone increase with predominance in the lower limbs and on the right side. Positive pulsion tests; semiflexion posture. Small-paced, shuffling, hypokinetic gait with difficulties at starting and turning. Postural instability. 6.1023/1. Single Choice Question Which disease would you suspect? A) cerebral arteriosclerosis B) depression C) myasthenia gravis D) Parkinson's syndrome NEU-1024. A 57-year-old metal worker has been treated for gastric disorders for 15 years. Eight days before admission the patient was digging, when he observed numbness and weakness of his toes and both feet. His family doctor told him to have a rest and consult him again if the symptoms persisted. The next day the weakness had spread up to his knees; the upper limbs also became numb and clumsy and small objects slipped from the patient's hands.At admission: neurologic status: intact cranial nerves; hyporeflexia of the proprioreflexes in both DrRognerud@www.medfever.com

NEUROLOGY

Answer: D

Answer: CD

231 / 343

Board Exam Question (Smiley + Star) + 2007 upper limbs; no pathological reflexes in the upper limbs. Proprioreflexes of the lower limbs cannot be elicited. No pathological reflexes in the lower limbs. Hypotonia of all the muscle groups of the body. A latent paralysis test reveals difficulties in keeping his upper limbs in position; pronation occurs on the left side. Severe paralysis of the lower limbs increasing toward the distal parts, with left predominance. Sensibility: hypesthesia to all modalities in all the extremities with left and lower limb predominance increasing in the distal directionerve The patient can stand with help but cannot walk. Lumbar puncture: CSF: cell count: 1 /mm3; total protein: 120 mg; CSF protein level (P.ndy): +++; benzoic: normal. EEG: negative. 6.1024/ 1. Single Choice Question Which disease would you suspect?: A) multiple sclerosis B) a spinal tumor C) Guillain-Barr's syndrome D) leukoencephalitis 6.1024/2. Single Choice Question What therapy would you recommend? A) bed rest, observation at home B) physical therapy C) rheumatological consultation D) emergency admission to a neurologic ward, where a respirator is available. NEU-1025. A 23-year-old female with no history of any major illnesses other than a tonsillectomy and appendectomy. Two months ago she delivered a healthy child. Two weeks ago the patient experienced severe psychological trauma, when she learned from her neighbour that her husband had been unfaithful. A week ago she complained of increasing numbness and a "dull" feeling in the lower limbs. Though the patient does not complain of vertigo, her gait became unsteady. Two days ago she also felt numbness on the right side of her face and complains of diplopia while looking to the right. Furthermore, she remembered, that these visual disorders had started about 1.5 year ago when she had blurred vision for about one month. When she finally decided to consult a doctor the complaints ceased. Her general condition is normal, she never complained of headaches or nausea. The patient is afebrile. At examination: blood pressure 120/70 mmHg; no internal alterations; vision: right: 0.5, left: 0.8. At the fundus clear papillary borders, with more pale temporal areas. The patient complains of diplopia while looking to the right. During this performance the right bulbus scarcely turns outward. At gazing upward and to the left a nystagmus with a large wave rotatory component, corresponding to the direction of the gaze develops. When showing her teeth the left corner of her mouth does not quite follow the right one. The rest of the cranial nerves are normal. Marked hyperreflexia of the proprioreflexes in the upper extremities, and sustained increased proprioreflexes in the lower limbs. Bilateral Babinski's and Chaddock's signs. Knee and ankle clonus on the right side can be visualized. Abdominal cutaneous reflexes cannot DrRognerud@www.medfever.com

NEUROLOGY

Answer: -

232 / 343

Board Exam Question (Smiley + Star) + 2007 be elicited. Bilateral tactile and painful hypesthesia of spotty, uncertain character in both lower limbs. Decreased sense of position and movement in the tendons of the toes. Fatigue of the lower limbs with a right side prevalence; weakness of dorsal flexion. Sustained bilateral increase of the extension tone in the lower limbs. Uncertainty, ataxia in the knee-heel test. Broad-based, clumsy gait. Psychic condition: hypothymic, emotionally sensitive, several times cried during the examination -she is convinced that her husband has "ruined her". 6.1025/ 1. Single Choice Question Select the most probable diagnosis: A) polyneuropathy B) a depressive hysteric reaction C) spastic spinal paralysis D) multiple sclerosis E) funicular myelosis 6.1025/2. Select One Of The Key Combinations Select two most relevant studies: 1) carotid and vertebral angiography 2) pneumoencephalography 3) blood smear and Schilling's test 4) EEG 5) CSF electrophoresis 5) myelography 7) ophthalmological study (object-and color visual field, CFF) A) only (1), (3), and (7) are true B) only ( 2), (4), and (5) are true C) only (1), (3), and (4) are true D) only (5) and (7) are true E) only (1), (2), (3), and 4) are true 6.1025/3. Single answer question: The most probable prognosis is: A) fatal outcome in several months B) complete recovery C) a slow steady progression for many years D) improvement, but a new attack-like relapse is expected E) complete recovery with residual symptoms, relapses are not expected. NEU-1026. A 37-year-old female patient (occupation: administrator) complains that the day before, she had a transient right-sided weakness which lasted for about 15 min. She further complained that for several minutes she couldn't talk, although she was in touch with her surroundings. The only other time she had felt similar complaints was three weeks ago when she had felt numbness in her right limbs which had lasted for several minutes -she did not consider it so serious illness. Apart from an appendectomy the patient has not had any serious ilness. She has taken oral contraceptives for 10 years. At admission: her neurologic status was normal, blood pressure 130/90 mmHg. Ophthalmologic and CSF studies gave negative results. A marked slow-wave activity was observed on the EEG in the left hemisphere. DrRognerud@www.medfever.com

NEUROLOGY

Answer: CC

233 / 343

Board Exam Question (Smiley + Star) + 2007 6.1026/ 1. Single Choice Question Select the most probable diagnosis: A) myoclonic seizure B) functional disorder C) TIA (transient ischemic attack) D) "grand mal" seizure 6.1026/2. Single Choice Question Select the most important diagnostic study: A) native skull X-ray B) a detailed psychiatric evaluation C) ultrasound and CT D) carotid angiography

NEUROLOGY

DrRognerud@www.medfever.com

234 / 343

Board Exam Question (Smiley + Star) + 2007

INTERNAL MEDICINE

INTERNAL MEDICINE
(286) TRUE-FALSE TYPE QUESTIONS Put T for true statements and F for false statements!!! INT-1. True-False Type Question Distension is the only natural stimulus which can increase gastric emptying. INT-2. True-False Type Questions Fats are not effective in slowing down gastric emptying. SINGLE CHOICE QUESTIONS Select the single best response to each of the following questions!!! INT-11. The following statements are all true for Exanthematous Typhus, EXCEPT: A) a severe drug-resistant headache typically occurs following the 7-to l4-day incubation period B) hypotension occurs in the most seriously ill patients C) exanthema develops at the end of the second week D) splenomegaly occurs in some cases E) small pink macules appear on the 4th to 6th day F) severe diarrhea occurs at the onset of the disease INT-12. All of the following are true for coccidioidomycosis, EXCEPT: A) acute bronchitis B) erythema nodosum C) leukopenia D) arthritis E) a positive complement-binding test and a negative skin-test in cases of a disseminated infection INT-16. All of the following can cause osteoporosis, EXCEPT: A) a calcium deficiency B) a phosphate deficiency C) hypercortisonism D) hypogonadism INT-17. Which of the following can cause demineralization of the bones? DrRognerud@www.medfever.com

Answer: T

Answer: F

Answer: F

Answer: C

Answer: B

Answer: E 235 / 343

Board Exam Question (Smiley + Star) + 2007 A) tumors B) bone marrow diseases C) hyperparathyroidism D) a vitamin D deficiency E) all of the above INT-18. All of the following are used to treat osteoporosis, EXCEPT: A) calcium B) vitamin D C) sodium fluoride D) calcitonin when estrogen therapy is contraindicated E) potassium INT-20. All of the following are typical of Wiskott-Aldrich syndrome EXCEPT: A) chronic eczema B) thrombocytopenic purpura C) a sex-dependent recessive inheritance pattern D) anemia E) thymus aplasia INT-21. Rheumatoid factor usually occurs together with the following symptoms EXCEPT: A) subcutaneous nodules B) splenomegaly C) vasculitis D) neuropathy E) juvenile rheumatoid arthritis F) Sjorgen's syndrome INT-26. Select the most important restricting side-effect of chloroquine, when used for systemic lupus erythematosus (SLE) therapy: A) cardiotoxicity B) hepatotoxicity C) ocular toxicity D) peripheral neuropathy E) drug fever INT-30. Impetigo is a superficial infection caused by one of the following bacteria: A) hemolytic staphylococcus B) hemolytic streptococcus group A C) both of the above D) none of the above INT-32. Streptococcus infections can cause the following diseases EXCEPT: DrRognerud@www.medfever.com

INTERNAL MEDICINE

Answer: E

Answer: E

Answer: E

Answer: C

Answer: C

Answer: D 236 / 343

Board Exam Question (Smiley + Star) + 2007 A) lymphangitis B) puerperal infections C) erysipelas D) pyelonephritis INT-33. Which site (excluding the lung) is mainly affected by childhood pulmonary tuberculosis? A) the mediastinal lymph nodes B) the kidney C) the spine D) the pericardium E) the peritoneum INT-34. All of the following alterations occur in syphilis, EXCEPT: A) condyloma latum B) mucocutaneous pustular lesions C) gumma D) meningitis E) uveitis F) condyloma acuminatum INT-41. All of the following are opiate withdrawal symptoms, EXCEPT: A) yawning and rhinorrhea B) sweating and lacrimation C) dilated pupils D) piloerection E) myalgia F) hypothermia INT-47. If the mother has measles during the first trimester of pregnancy all of the following can occur in the newborn, EXCEPT. A) cardiac disorders B) cataracts C) thrombocytopenic purpura D) deafness E) the measles virus cannot be identified in newborns INT-48. All of the following are contraindications of the viable measles vaccine, EXCEPT: A) pregnancy B) leukemia C) active tuberculosis D) simultaneous steroid therapy E) polio vaccination performed 4 weeks before

INTERNAL MEDICINE

Answer: A

Answer: F

Answer: F

Answer: E

Answer: E

DrRognerud@www.medfever.com

237 / 343

Board Exam Question (Smiley + Star) + 2007 INT-49. All of the following are useful symptoms which help to differentiated acute pancreatitis from a perforated peptic ulcer, EXCEPT: A) hypertension develops in acute pancreatitis B) a perforated ulcer is more pressure sensitive C) intestinal sounds are typical of acute pancreatitis D) in perforated peptic ulcer air is visible under the diaphragm, while in acute pancreatitis pleural effusion is typical E) an elevated serum amylase level INT-53. Which of the following can cause postoperative hypokalemic, hypochloremic alkalosis? A) dehydration due to artificial nutrition B) acute renal insufficiency C) unrecognized diarrhea D) nasogastric catheter E) overnutrition INT-54. Select the pCO2 value characteristic of pure metabolic alkalosis if the serum bicarbonate value is 34 mmol/L? A) 20 mmHg B) 30 mmHg C) 40 mmHg D) 60 mmHg E) 70 mmHg INT-55. In gastric fluid loss, all of the following can occur, EXCEPT: A) an increased urine pH B) a decreased urine potassium level C) an increased urine bicarbonate level D) a decreased urine chloride level E) increased renin secretion INT-56. Metabolic alkalosis occurs in all of the following, EXCEPT: A) diabetes mellitus B) pyloric stenosis C) Bartter's syndrome D) after administration of calcium carbonate E) as a side-effect of ethacrynic acid therapy INT-61. Case Study: A patient with chronic metabolic acidosis treated with alkalic equivalents to correct the acidosis kept hyperventilating for several days. Which of the following is the most typical finding in this case? A) pH 7.4; pCO2 40; HCO3 24 DrRognerud@www.medfever.com

INTERNAL MEDICINE

Answer: E

Answer: D

Answer: C

Answer: B

Answer: A

Answer: C

238 / 343

Board Exam Question (Smiley + Star) + 2007 B) pH 7.5; pCO2 40; HCO3 30 C) pH 7.5; pCO2 20; HCO3 16 D) pH 7.3; pCO2 30; HCO3 14 E) pH 7.5; pCO2 50; HCO3 36 INT-62. Which of the following can occur if the pCO2 is 70 mmHg and the bicarbonate level is 33 mmol/L? A) chronic respiratory acidosis B) acute respiratory acidosis C) respiratory and metabolic acute acidosis D) respiratory acidosis and metabolic alkalosis E) none of the above INT-70. Which of the following statements about uric acid transport is FALSE? A) uric acid is reabsorbed and secreted by the renal tubules B) transport mainly occurs in the proximal part of the nephron C) changes in glomerular filtration do not significantly regulate the urate clearance D) urate clearance increases with the increase of urine flow E) urate is mainly transported by diffusion INT-71. According to data obtained in many hospital patients, hyperuricemia is primarily caused by: A) chronic renal insufficiency B) poorly treated hypertension C) chronic diuretic therapy D) obesity and an increased consumption of purines E) diabetes mellitus in adults INT-87. In diabetic ketoacidosis, the normalization of dehydration affects glucosuria in the following way: A) it increases glucosuria B) it decreases glucosuria C) the glucosuria remains unchanged D) the glucosuria decreases if the blood glucose level decreases E) the glucosuria disappears INT-93. Which of the following statements about the antihypertensive effect of diuretics is FALSE? A) a decreased plasma renin level B) . a decreased cardiac output C) decreased extracellular fluid D) decreased peripheral resistance E) a decreased sensitivity to vasoconstrictor agents

INTERNAL MEDICINE

Answer: A

Answer: E

Answer: C

Answer: A

Answer: A

DrRognerud@www.medfever.com

239 / 343

Board Exam Question (Smiley + Star) + 2007 INT-103. Which of the following agents is not contraindicated in severe renal insufficiency? A) furosemide B) spironolactone C) triamterene D) amiloride E) potassium tablets (with a long-term effect) INT-106. Chronic thiazide therapy can cause all of the following symptoms, EXCEPT: A) an increased excretion of bicarbonate B) a decreased excretion of calcium C) a decreased excretion of uric acid D) a decreased excretion of potassium E) an increased excretion of chloride INT-108. Which of the following statements about furosemide is FALSE? A) in a maximum dose furosemide is more effective than the thiazides B) furosemide is efficient even at very low filtration rates C) it increases RBF and GFR if severe hypovolemia is not present D) it maintains the concentrating capacity of the kidney E) other diuretics potentiate it's effect INT-110. The chronic administration of furosemide can cause all of the following symptoms, EXCEPT: A) hypokalemia B) hypochloremia C) hypomagnesiemia D) hypocalcemia E) hyperchloremia INT-114. Spironolactone-induced full-scale sodium excretion occurs: A) within 10-30 min. B) within 2-4 hours C) within 12-20 hours D) within several days E) none of the above INT-117. Case Study: A 25-year-old unconscious female patient was admitted to the hospital. No friends or witnesses could give any further information on what had happened Centrifugation of the urine obtained by catheter revealed fat drops in the sample. Which of the following diagnoses is DrRognerud@www.medfever.com

INTERNAL MEDICINE

Answer: A

Answer: D

Answer: D

Answer: E

Answer: D

Answer: D

240 / 343

Board Exam Question (Smiley + Star) + 2007 possible? A) eclampsia B) bone fracture C) CO intoxication D) all of the above E) none of the above INT-121. Case Study: A 26-year-old female patient is admitted due to a suspected nephrotic syndrome. The determination of the different molecular mass protein clearance indicated an albumin predominance. High molecular-mass globulins were not found in the urine. Select a possible diagnosis: A) membranous glomerulopathy B) proliferative glomerulonephritis C) a "minimal change" process D) membranoproliferative glomerulonephritis E) focal glomerulonephritis INT-129. Case Study: A 47-year-old diabetic patient developed oliguria after intravenous urography. The urine osmolarity was 480 mOsm. The most probable diagnosis is: A) acute renal insufficiency B) prerenal azotemia C) papillary necrosis D) diabetic nephropathy E) obstructive uropathy INT-136. Case Study: In a patient with severe hypertension a normal creatinine level and a 4 g/24h proteinuria were found. Urography revealed an enlarged (16 cm) right kidney and a smaller (7 cm) left kidney with a blurred border. Results of arteriography suggest stenosis of the right renal artery. Which of the following should be the next adequate study? A) a bilateral retrograde urography B) a right-sided percutaneous renal biopsy C) left-sided percutaneous renal biopsy D) a right-side biopsy with surgical approach E) the determination of the renin concentration in the renal veins INT-148. Which of the following statements about hypertension is FALSE? A) if the diastolic pressure is over 90 mmHg, life expectancy is always reduced in all patients B) the official WHO definition of normotension is: systolic pressure 140-160 mmHg; diastolic pressure 90-95 mmHg C) the rate of pressure increase is higher in pre-existing hypertension DrRognerud@www.medfever.com

INTERNAL MEDICINE

Answer: C

Answer: A

Answer: E

Answer: B

241 / 343

Board Exam Question (Smiley + Star) + 2007 D) females tolerate hypertension better than males E) 15-20% of the adult population in the USA is hypertensive INT-149. Which of the following statements is true for a "borderline" hypertension? A) the cardiac output is usually increased B) the blood volume is usually increased C) the peripheral vascular resistance is usually increased D) there is an increased role of the alpha-adrenergic nervous system E) these patients are usually resistant to beta-blockers INT-151. Which of the following pressure ranges is considered "borderline hypertension" in young patients? A) 140/90-160/95 mmHg B) 120/80-140/90 mmHg C) 120/80-165/90 mmHg D) 130/85-140/95 mmHg E) 130/85-139/89 mmHg INT-152. The blood pressure can increase: A) in obesity B) after consumption of substantial amounts of alcohol C) due to smoking D) in gout E) all of the above INT-162. Which of the following statements about the baroreceptors of the carotid sinus and the aortic arch is FALSE? A) in animals full denervation causes constant hypertension B) the baroreceptor sensitivity slightly decreases in essential hypertension C) the baroreceptor activity can increase in young individuals with "borderline hypertension" D) at decreased baroreceptor activity, the peripheral resistance and diastolic pressure can markedly increase in the orthostatic position E) a higher pressure is necessary to activate baroreceptors in hypertension INT-163. All of the following hormones participate in the regulation of blood pressure in humans, EXCEPT: A) adrenaline B) noradrenaline C) renin D) vasopressin E) aldosterone

INTERNAL MEDICINE

Answer: A

Answer: A

Answer: E

Answer: D

Answer: C

DrRognerud@www.medfever.com

242 / 343

Board Exam Question (Smiley + Star) + 2007 INT-164. Catecholamines exhibit all of the following effects, EXCEPT: A) anxiety followed by hyperventilation, palpitation and tremor B) a long-term elevation of blood pressure C) an increased cardiac output D) an increased muscle blood flow E) an increased renal blood flow INT-168. Which of the following statements is true for angiotensin I? A) it is the most potent vasoconstrictor known B) it is activated by the liver converting enzymes C) it consists of 10 aminoacids D) it stimulates aldosterone release E) it is formed from the "big renin" INT-170. Which of the following statements about angiotensinogen (renin substrate) is FALSE? A) it is mainly produced in the liver B) its concentration is markedly increased after bilateral adrenalectomy C) its concentration remains unchanged after bilateral adrenalectomy D) its concentration decreases in liver cirrhosis E) estrogens increase its level INT-176. The aldosterone level is always increased in: A) essential hypertension B) malignant hypertension C) pheochromocytoma D) Liddle's syndrome E) polycystic kidney disease INT-185. In which of the following diseases does the plasma renin level always remain low? A) essential hypertension B) aortic coarctation C) pheochromocytoma D) Conn's syndrome E) Cushing's syndrome INT-191. Case Study: A 32-year-old male patient complains of various fits of hypertension and headaches. The repeatedly determined urine and plasma vanillylmandelic acid, catecholamine, and metanephrine values were normal. Select a possible explanation: A) cerebral astrocytoma DrRognerud@www.medfever.com

INTERNAL MEDICINE Answer: E

Answer: C

Answer: B

Answer: B

Answer: D

Answer: A

243 / 343

Board Exam Question (Smiley + Star) + 2007 B) a non-functioning adrenal tumor C) aortic coarctation D) aberrant adrenal tissue E) an inadequate sampling of the urine INT-192. Case Study: The blood pressure of a pregnant woman (24th week of gestation) was 170/ 100 mmHg. Select a possible explanation: A) borderline or mild hypertension B) pseudohypertension of pregnancy C) hyperkinetic syndrome D) severe hypertension E) sustained hypertension INT-193. Which drugs or drug combinations should be avoided in the therapy of pheochromocytoma? A) dibenzyline B) beta-blockade followed by alpha blockade C) alpha-blockade followed by beta blockade D) prazosin E) clonidine INT-194. Which of the following studies will yield pathological values in neurofibromatosis with hypertension? A) the determination of the concentration of metanephrine in a 24-hour urine sample B) the plasma renin activity C) the serum cortisol level D) the determination of the concentration of aldosterone in a 24hour urine sample E) the fractional potassium excretion INT-195. In which of the following diseases, (which are accompanied with orthostatic hypertension), does hyperglycemia, glucosuria, hypermetabolism, and type-II multiple endocrine neoplasm occur? A) Cushing's disease B) essential hypertension C) malignant hypertension D) pheochromocytoma E) cerebellar hemangioblastoma INT-203. All of the following symptoms occur in renal vein thrombosis, EXCEPT: A) severe lumbar pain B) ureter incisure on the intravenous pyelogram C) hematuria DrRognerud@www.medfever.com

INTERNAL MEDICINE

Answer: D

Answer: B

Answer: A

Answer: D

Answer: E

244 / 343

Board Exam Question (Smiley + Star) + 2007 D) proteinuria E) medial deviation of the ureter INT-204. Single Choice Question Renal vein thrombosis can be a complication of all of the following EXCEPT: A) hypernephroma B) congestive cardiac failure C) in a patient in the terminal state of papillary necrosis D) nephrosclerosis E) nephrosis syndrome INT-206. Primary renal impairment in disseminated intravascular thrombosis is due to: A) a hyaline thrombus B) swelling of the endothelial cells C) the formation of "half-moon" lesions D) the rupture of basal membrane E) the proliferation of mesangial cells INT-207. Which of the following changes is considered a late complication of the hemolytic-uremic syndrome of infants? A) anemia B) thrombocytopenia C) hypertension D) intravascular coagulation E) none of the above INT-215. Which of the following substances does not occur in the urine in Hartnup disease? A) histidine B) tryptophan C) phenylalanine D) methionine E) arginine INT-216. Which of the following remarks about proximal tubular acidosis is FALSE? A) a decreased proximal maximum bicarbonate reabsorption B) an intact distal acidifying system C) after administration of ammonium chloride the urine pH value becomes less than 5.4 D) an increased bicarbonate excretion in the urine E) an impaired distal acidifying system

INTERNAL MEDICINE

Answer: D

Answer: B

Answer: C

Answer: E

Answer: E

DrRognerud@www.medfever.com

245 / 343

Board Exam Question (Smiley + Star) + 2007 INT-221. Case Study: A 55-year-old patient has a primary carcinoma of unknown origin. Which of the following is totally resistant to therapy? A) adenocarcinima of the prostate B) adenocarcinoma of the lung C) adenocarcinoma of the breast D) germinal cell carcinoma E) carcinoma of the thyroid gland INT-222. Eosinophilia primarily occurs in: A) enterobiasis B) diarrhea due to Giardiasis (lambliasis) C) Schistosomiasis D) measles E) corticosteroid therapy INT-223. All of the following can occur as a complication of chronic obstructive pulmonary disease EXCEPT: A) cor pulmonarye B) polycythemia C) respiratory insufficiency D) left ventricular insufficiency E) bronchogenic carcinoma INT-224. Chronic obstructive pulmonary disease (COPD) can be either emphysematous or bronchitic, depending on the character of the pathological alterations in the lung. Though they rarely occur as autonomous forms, the two diseases can be differentiated on the basis of their clinical manifestation. Select the most common feature characteristic of emphysematous and bronchitic COPD: A) polycythemia B) bronchodilators can improve the airflow C) dyspnea D) a chronic cough E) hypercapnia INT-228. Case Study: A 50-year-old female patient complains of redness, swelling and rigidity of the distal interphalangeal joints. She has no other arthralgias. The most probable diagnosis is: A) erosion osteoarthritis B) rheumatoid arthritis C) systemic lupus erythematosus (SLE) D) ankylosing spondylitis E) systemic sclerosis DrRognerud@www.medfever.com

INTERNAL MEDICINE Answer: B

Answer: C

Answer: D

Answer: C

Answer: A

246 / 343

Board Exam Question (Smiley + Star) + 2007 INT-232. Which of the following statements is the most typical feature of an amebic liver abscess? A) that surgical drainage is necessary B) it can be caused by one of the six ameba types C) in the USA it commonly occurs among homosexuals and hospital patients D) it usually indicates an intestinal amebic infection E) it usually occurs in patients with normal serum alkalic phosphatase levels but elevated transaminase levels INT-235. Case Study: A patient with chronic renal insufficiency due to severe chronic hypertension is examined for chest pain. For the last 2 years the patient has been undergoing hemodialysis twice a week. Hypotensive episodes occurred several times during dialysis. The chest pain is localized above the trapezius muscle; in an orthostatic position it decreases, and during deep inspiration it increases. Select the most probable cause of this chest pain: A) pericarditis B) coronary disease C) diffuse esophageal spasm D) pulmonary embolism E) costochondritis INT-245. All of the following can cause mechanical ileus EXCEPT: A) cholera B) hernia C) carcinoma D) volvulus E) gallstones INT-246. All of the following can be caused by a gallstone EXCEPT: A) biliary cirrhosis B) acute pancreatic necrosis C) atrophic cirrhosis D) chronic cholecystitis INT-251. Select the correct localization of a concretion in case of hydrops vesica felleae: A) the hepatic duct B) the cystic duct C) the common bile duct D) Vater's papilla E) Wirsungian's duct DrRognerud@www.medfever.com

INTERNAL MEDICINE

Answer: C

Answer: A

Answer: A

Answer: C

Answer: B

247 / 343

Board Exam Question (Smiley + Star) + 2007 INT-252. Combined tumors of the salivary gland: A) usually have a submaxillary localization B) are usually malignant C) usually develop in the parotid gland D) usually cause facial nerve paralysis E) typically cause sialolithiasis INT-253. Select the correct term for transplantation of tissue-friendly foreign materials: A) replantation B) alloplasty C) homologous transplantation D) autogenous transplantation E) isologous transplantation INT-254. Select the single correct statement: A) preblastomatosis is a pathological alteration preceding skin cancer B) preblastomatosis is a pathological alteration which leads to the development of a tumour C) preblastomatosis is a long-term pathological alteration which does not lead to malignancy D) preblastomatosis means pre-invasive cancer E) hematologic myeloid tumors are defined as preblastomatosis INT-255. Pneumothorax can be caused by all of the following EXCEPT: A) trauma B) bullous emphysema C) damage of the thoracic duct D) intersitial emphysema E) positive pressure artificial ventilation INT-256. Case Study: A round-shaped shadow of 3 cm in diameterwas found on an X-ray screening of a 65-year-old, complaint-flee, smoking male patient. 15 years ago the patient had undergone a successful operation for colon carcinoma. Select the most probable diagnosis: A) tuberculosis B) carcinoma metastasis C) aspiration pneumonia D) bronchial carcinoma E) pulmonary abscess

INTERNAL MEDICINE

Answer: C

Answer: B

Answer: B

Answer: C

Answer: D

DrRognerud@www.medfever.com

248 / 343

Board Exam Question (Smiley + Star) + 2007 INT-257. All of the following are typical of Graves' disease, EXCEPT: A) obesity B) goiter C) left ventricular hypertrophy D) exophthalmus INT-258. The most common cause of Addison's disease is: A) autoimuune adrenal atrophy B) amyloid accumulation in the adrenals C) tumor metastasis in the adrenals D) bilateral adrenal apoplexia E) adrenal tuberculosis INT-259. Which of the following hormones is increased in Conn's syndrome? A) cortisol B) adrenaline C) noradrenaline D) aldosterone E) cortisone INT-260. Case Study: Marked hypertension and its sequels, as well as polyuria and pplydipsia were found in a middle-aged patient. serum Na: 152 mmol/L; serum K: 2,2mmo1/L. Select the correct diagnosis: A) Cushing's syndrome B) diabetes mellitus C) Addison's disease D) Conn's syndrome E) phaeochromocytoma INT-261. Xanthoma is common in: A) diabetes insipidus B) acute hemorrhagic pancreatitis C) diabetes mellitus D) diffuse acute glomerulonephritis E) liver cirrhosis (Laennec's) INT-262. Case Study: A patient's auricular cartillage has a brownish color, the urine sample turns brown after a while, and the patient complains of arthralgia. Select a correct diagnosis: A) erythropoietic porphyria B) gout C) ochronosis DrRognerud@www.medfever.com

INTERNAL MEDICINE Answer: A

Answer: A

Answer: D

Answer: D

Answer: C

Answer: C

249 / 343

Board Exam Question (Smiley + Star) + 2007 D) cystinosis E) hemoglobinuria INT-263. Which of the following statements is not typical of post-hepatic jaundice? A) acholic stool B) direct positive diazo-reaction C) cholemia D) increased bilirubin in the urine E) increased urobilinogen in the urine INT-264. In all of the following melanine is accumulated in the skin, EXCEPT: A) Mongolian spot B) vitiligo C) nevus coeruleus (blue nevus) D) ephelis (freckle) E) chloasma (melasma) INT-265. Define the term vitiligo: A) a generalized lack of melanin in the skin B) a circumscribed lack of melanin in the skin C) lipopigment D) reaction in the peritraumatic areas E) a degeneration of trophoblasts INT-270. Which of the following can occur in chronic systemic congestion? A) thickening of the alveolar walls B) esophagus varicosity C) pulmonary edema D) spot-like hemorrhages of the gastric mucosa E) kidney micro-infarctions INT-277. In which of the following should generalized mycosis be suspected? ?r) hypertensive disease B) cardiac decompensation C) gastric ulcer D) long-term antibiotic therapy E) diabetes insipidus INT-279. In which of the following vascular beds does Buerger's disease commonly occur? A) basilar artery B) arteries of the limbs C) coronary artery D) renal artery DrRognerud@www.medfever.com

INTERNAL MEDICINE

Answer: E

Answer: B

Answer: B

Answer: D

Answer: D

Answer: B

250 / 343

Board Exam Question (Smiley + Star) + 2007 E) branches of the mesenteric artery INT-281. Hematuria is typical of all of the following diseases, EXCEPT: A) acute glomerulonephritis B)acute pyelonephritis C) renal tuberculosis D) renal calculus E) renal cancer INT-289. In all of the following renal calculi can be formed, EXCEPT: A) in urinary tract obstruction B) in congenital metabolic disorders C) in constant substantial fluid loss D) along with gallstones E) in pyelitis INT-290. The most pronounced enlargement of the lymph nodes occurs in one of the following types of leukemia: A) acute lymphoid leukemia B) chronic myeloid leukemia C) chronic lymphoid leukemia D) acute myeloid leukemia E) erythroleukemia INT-291. Which of the following red blood cells alterations is typical of irondeficient anemia? A) poikilocytosis B) anisocytosis C) hypochromia D) fragility E) polychromasia INT-294. Which of the following is typical of Hodgkin's disease stage II? A) the involvement of lymph nodes of a single region and the spleen B) the involvement lymph nodes of a single region on both sides of the diaphragm C) the involvement of lymph nodes of several regions on both sides of the diaphragm D) the involvement of lymph nodes of several regions on both sides of the diaphragm, and alterations in the spleen E) the involvement of lymph nodes of several regions on one side of the diaphragm INT-297. Pulmonary emboli originate from: DrRognerud@www.medfever.com

INTERNAL MEDICINE

Answer: B

Answer: D

Answer: B

Answer: C

Answer: E

Answer: B 251 / 343

Board Exam Question (Smiley + Star) + 2007 A) the femoral artery B) the femoral vein. C) one of the pulmonary veins D) the brachiocephalic trunk E) none of the above INT-301. Which of the following can lead to the development of chronic cor pulmonale? A) lobar pneumonia B) thromboembolism of the pulmonary vein C) lung edema D) chronic pulmonary emphysema E) bronchopneumonia INT-302. All of the following occur in Goodpasture's syndrome, EXCEPT: A) focal glomerulonephritis B) septic spleen C) pulmonary fibrosis; induration D) cutaneous purpura E) positive berlin-blue staining in the lung parenchyma INT-304. In which of the following parts of the gastrointestinal tract does diverticulosis primarily develop? A) in the esophagus B) in the duodenum C) in the jejunum and ileum D) in the ascending colon E) in the sigmoid colon INT-306. Define Crohn's disease: A) regional enteritis B) tabes mesaraica (tuberculosis of the mesenteric glands) C) stercoral abscess D) intestinal cystoid pneumatosis E) necrotizing enteritis INT-307. Select the correct name for a gastroduodenal ulcer developing together with a gastrin-secreting pancreatic tumor: A) Zollinger-Ellison's syndrome B) Mallory-Weiss's syndrome C) Waterhouse-Fridrichsen's syndrome D) Stein-Leventhal's syndrome E) Sheehan's syndrome

INTERNAL MEDICINE

Answer: D

Answer: B

Answer: E

Answer: A

Answer: A

DrRognerud@www.medfever.com

252 / 343

Board Exam Question (Smiley + Star) + 2007 INT-308. Which of the following substances causes coagulation necrosis of the gastric mucosa? A) sodium hydroxide (NaOH) B) phosphorus C) lead D) mercury E) arsenic INT-309. Case Study: A patient complains of frequent blushing fits. Asthmatic fits and profuse diarrhea are also frequent. A physical examination revealed a harsh murmur above the heart. Select the correct diagnosis: A) Basedow's disease B) Carcinoid syndrome C) Tetralogy of Fallot D) Polycythemia rubra vera E) Peutz-Jeghers syndrome INT-310. The Zollinger-Ellison syndrome is caused by: A) an adenoma of the pancreatic beta cells B) an adenoma of the pancreatic alpha cells C) an adenoma of the pancreatic gamma cells D) a carcinoma of the exocrine pancreas E) mucoviscidosis INT-311. A dilated, rigid, painless gallbladder (Courvoisier's s sign) and jaundice is typical of one of the following: A) infectious hepatitis B) hepatocellular carcinoma C) cancer of the pancreas head D) cancer of the pancreas tail E) calculus in the Wirsungian's duct INT-312. Which of the following cells primarily infiltrate the portal tract in acute viral hepatitis? A) neutrophils B) eosinophils C) lymphocytes D) plasma cells E) giant cells INT-313. Coagulopathy developing in obstructive jaundice is due to: A) secondary thrombocytopenia B) thrombocytopenia due to decreased vitamin K absorption DrRognerud@www.medfever.com

INTERNAL MEDICINE Answer: D

Answer: B

Answer: C

Answer: C

Answer: C

Answer: B

253 / 343

Board Exam Question (Smiley + Star) + 2007 C) increased capillary fragility D) thrombasthenia E) a low activity of tissue thromboplastin INT-314. Which of the following can cause priapism? A) syphilis B) urethritis C) induratio penis plastica (penal prosthesis) D) increased libido E) epispadiasis INT-315. Define the term priapism: A) an extrapyramidal disorder B) a mental disease C) pathological erection D) penis inflammation E) penis gangrene INT-319. Asbestos causes cancer in which of the following organs? A) the kidneys B) the pleura C) the brain D) the adrenals INT-327. All of the following are criteria and symptoms of Mixed Connective Tissue Disease (MCTD), EXCEPT: A) Raynaud's symptom B) swollen hands and fingers C) myositis D) esophageal dysmotility E) xerostomia INT-328. All of the following polysystemic autoimmune diseases contribute to the development of Mixed Connective Tissue Disease (MCTD), EXCEPT A) systemic lupus erythematosus (SLE) B) rheumatoid arthritis C) progressive systemic sclerosis (scleroderma) D) Sj.rgen's syndrome E) polymyositis/dermatomyositis INT-329. All of the following are among the glandular symptoms of Sj.rgen's syndrome, EXCEPT: A) pharyngitis -bronchitis B) vulvitis DrRognerud@www.medfever.com

INTERNAL MEDICINE

Answer: B

Answer: C

Answer: B

Answer: E

Answer: D

Answer: D

254 / 343

Board Exam Question (Smiley + Star) + 2007 C) vaginitis D) pyelitis E) conjunctivitis INT-337. All of the following are atopic diseases, EXCEPT: A) allergic rhinitis B) allergic bronchial asthma C) gastrointestinal allergy D) allergic conjunctivitis E) hypertensive pneumonitis INT-343. Which study should be performed if infectious arthritis is suspected? A) X-ray study B) scintigraphy C) articular puncture D) antibiotic therapy INT-344. All of the following occur in progressive systemic sclerosis (scleroderma), EXCEPT: A) pulmonary fibrosis B) the incidence of antinuclear autoantibodies in the serum is about 90% C) chronic active hepatitis D) dysmotility of the lower esophageal segment E) Raynaud's syndrome INT-346. All of the following diseases can occur in the neonatal period, EXCEPT: A) thrombopenia (ITP) B) myasthenia gravis C) Basedow's disease D) pernicious anemia E) systemic lupus erythematosus (SLE) INT-350. The first heart sound is loud in: A) first degree AV block B) the Wolff-Parkinson-White (WPW) syndrome C) mitral stenosis due to calcification D) aortic stenosis E) hypertrophic obstructive cardiomyopathy INT-352. Which of the following ECG changes are typical of Printzmetal's angina during a fit? A) ST depression in a localized area DrRognerud@www.medfever.com

INTERNAL MEDICINE

Answer: E

Answer: C

Answer: C

Answer: D

Answer: B

Answer: B

255 / 343

Board Exam Question (Smiley + Star) + 2007 B) ST elevation in a localized area C) diffuse ST depression D) diffuse ST elevation E) none of the above INT-353. Select the most important effect of a normal dose of nitroglycerin in angina pectoris (apart from coronary dilation!): A) relaxation of bronchial smooth muscles B) dilation of the peripheral vessels C) constriction of the peripheral arterioles D) constriction of the peripheral venules E) none of the above INT-354. Select the drug of primary choice in supraventricular tachycardia with narrow ventricular QRS complexes: A) digoxin B) procainamide C) dopamine D) verapamil E) propranolol INT-356. The typical symptom of ischemic colitis in its acute phase is: A) steatorrhea B) a normal X-ray picture after a barium meal C) nausea and vomiting D) symptoms of generalized peritonitis E) hemorrhagic diarrhea INT-357. Which of the following statements is typical of chronic granulomatosis? A) neutrophils cannot phagocyte the bacteria B) chronically enlarged lymph nodes C) recurrent candidiasis D) a dominant inheritance pattern E) hypogammaglobulinemia INT-358. All of the following can increase the normal insulin requirement, EXCEPT: A) pregnancy B) infections, fever, sepsis C) idiopathic spontaneous exacerbations D) hypothyroidosis E) burns due to irradiation and ultraviolet damage of the deep tissues

INTERNAL MEDICINE

Answer: B

Answer: D

Answer: E

Answer: B

Answer: D

DrRognerud@www.medfever.com

256 / 343

Board Exam Question (Smiley + Star) + 2007 INT-359. All of the following can induce systemic lupus erythematosus (SLE), EXCEPT: A) chlorpromazine B) phenytoin C) aspirin D) procainamide INT-361. Which of the following antibiotics can form unsoluble chelates with the aluminium of antacids, which would then impair their absorption? A) penicillins B) tetracyclines C) erythromycin D) sulfonamides E) none of the above INT-363. Case Study: Select the drug of choice in .a 2-year-old girl with fever and polyarthritis (diagnosis: juvenile rheumathoid arthritis): A) cytozan B) prednisolone C) aspirin D) chloroquine E) penicillinamine INT-365. All of the following increase the blood glucose level, EXCEPT: A) corticosteroids B) clofibrate C) diazoxide D) lithium carbonate INT-376. Which of the following diseases does not usually lead to the development of the nephrotic syndrome? A) glomerulonephritis B) polycystic kidney C) renal vein thrombosis D) -lupus nephritis E) Kimmensteil-Wilson syndrome INT-391. Which of the following findings indicates analgetic-induced nephropathy? A) red blood cells-casts B) oval adipose bodies C) sterile pyuria D) phenacetine crystals E) pigmented hyaline casts DrRognerud@www.medfever.com

INTERNAL MEDICINE Answer: C

Answer: B

Answer: C

Answer: B

Answer: B

Answer: C

257 / 343

Board Exam Question (Smiley + Star) + 2007 INT-392. Which of the following drugs does not cause renal disease? A) penicillin B) gentamycin C) erythromycin D) sulfonamides E) cephalosporins INT-393. Plasma or tissue erythropoietin-like substances are accumulated in all of the following diseases, EXCEPT: A) hypernephroma B) cerebellar hemangioma C) hepatoma D) solitary renal cyst E) gastric carcinoma INT-394. Case Study: A 50-year-old agricultural worker was admitted with nausea, vomiting with a peculiar aftertaste of metal, and diarrhea. He had been working with insecticides and disinfectants. The serum urea nitrogen value was 25 mmol/L. The urine volume excreted in 24 h. was 300 ml. Proteinuria, red blood cellsand epithelial cell casts were observed. Select the correct therapy: A) monitoring of fluid and electrolyte balance with fluid and electrolyte replacement therapy if needed B) high dose steroid therapy C) a dimercaptol (BAL) injection (3 mg/kg/24 h.) D) gastric lavage E) dimercaptol (BAL) injection 3 mg/kg followed by hemodyalisis INT-401. Case Study: A 30-year-old male patient suffers from recurrent renal calculi. An abdominal X-ray study revealed bilateral nephrocalcinosis. Which of the following is the least probable? A) hyperparathyroidism B) sarcoidosis C) primary hyperoxaluria (oxalosis) D) renal tubular acidosis E) cystinuria INT-402. Which of the following renal calculi are the most common? A) urate B) calcium-phosphate C) mixed calcium-oxalate and calcium-phosphate D) magnesium ammonium phosphate E) calcium oxalate DrRognerud@www.medfever.com

INTERNAL MEDICINE

Answer: C

Answer: E

Answer: E

Answer: E

Answer: C

258 / 343

Board Exam Question (Smiley + Star) + 2007 INT-403. Which of the following diseases commonly occurs in pregnant women with renal calculi? A) pre-eclampsia B) urinary tract infections C) spontaneous abortions D) congenital anomalies E) renal insufficiency INT-405. Which of the following drugs causes neuropathy in uremic patients? A) vibramycin B) chloramphenicol C) trimethoprim D) nitrofurantoin E) cephalosporins INT-407. Which of the following gastrointestinal symptoms of renal insufficiency is not improved by hemodialysis? A) uremic gastritis B) uremic fetor C) peptic ulcer D) uremic colitis E) nausea and vomitus INT-409. Which of the following drugs should be markedly decreased in uremia? A) gentamycin B) lincomycin C) nafcillin D) oxacillin E) novobiocin INT-410. All of the following occur in pregnancy, EXCEPT: A) an increased body water content B) a decreased blood volume C) an increased cardiac output D) an increased renal blood flow E) an increased GFR INT-411. In pregnancy, limb edema is due to one of the following: A) salt retention B) hypoalbuminemia C) an increased capillary permeability D) an increased venous pressure DrRognerud@www.medfever.com

INTERNAL MEDICINE

Answer: B

Answer: B

Answer: C

Answer: A

Answer: B

Answer: D

259 / 343

Board Exam Question (Smiley + Star) + 2007 E) an increased blood pressure INT-412. Which of the following usually causes symptom-free bacteriuria during pregnancy? A) anemia B) a premature delivery C) a congenital anomaly D) pyelonephritis E) toxemia INT-415. Which of the following causes cessation of toxemia after delivery? A) delivery of the placenta B) delivery of the fetus C) uterus decompression D) urether decompression E) the loss of amniotic fluid INT-419. Which of the following intra-abdominal organs does not move with respiration? A) the kidney B) the pancreas C) the spleen D) the transverse colon E) the stomach INT-422. Which of the following substances is the most potent stimulant of gastric acid secretion? A) proteins B) fats C) carbohydrates INT-423. A 5-fold elevation of serum amylase level strongly suggests: A) parotitis B) pancreatitis C) intestinal obstruction D) pancreatic carcinoma E) penetrating ulcer INT-424. A characteristic symptom of pyloric obstruction is: A) bile vomiting B) abdominal murmurs C) resonance above Traube's space D) succussion (splashing sound) E) visible peristalsis DrRognerud@www.medfever.com

INTERNAL MEDICINE

Answer: D

Answer: A

Answer: B

Answer: A

Answer: B

Answer: D

260 / 343

Board Exam Question (Smiley + Star) + 2007 INT-432. The diagnosis of malaria is based on: A) a stained blood smear for the identification of the pathogenic agent B) a hemoculture C) identification of the pathogenic agent on the skin D) a fluorescent antibody study INT-435. Liver transplantation is a new method for the therapy of fatal liver diseases. Which of the following markedly improves the post-transplantation survival rate? A) a better selection of patients B) the early recognition of malignant diseases C) a better understanding of the pathomechanism of liver insufficiency D) cyclosporine A INT-436. Which part of the colon is the largest feces reservoir? A) the cecum B) the transverse colon C) the descending colon D) the sigmoid colon E) the rectum INT-437. The defecation stimulus is triggered by: A) contraction of the external anal sphincter B) contraction of the internal anal sphincter C) distension of the sigmoid colon D) distension of the rectum E) contraction of the rectum INT-438. The anatomic anomaly which causes congenital megacolon (Hirschsprung's disease) is: A) hypertrophy of the descending colon B) lack of colon peristalsis C) absence of the autonomic plexuses in the colon D) rectal atresia E) lack of the internal rectal sphincter INT-440. How often (% incidence) does ulcerative colitis affect the rectosigmoidal mucosa? A) 10-20% B) 25-30% C) 50-70% D) 70-85% DrRognerud@www.medfever.com

INTERNAL MEDICINE

Answer: A

Answer: D

Answer: D

Answer: D

Answer: C

Answer: E

261 / 343

Board Exam Question (Smiley + Star) + 2007 E) 85-100% INT-442. The most common site of diverticulosis in the colon is: A) the rectum B) the sigmoid colon C) the descending colon D) the transverse colon E) the cecum INT-445. The most common cause of chronic relapsing pancreatitis is: A) gallstones B) alcohol consumption C) Whipple's disease D) trauma E) infection INT-446. Case Study: A 40-year-old worker had a massive upper gastrointestinal hemorrhage accompanied by splenomegaly. He never consumed alcohol. Which of the following could have caused this gastrointestinal bleeding? A) a peptic ulcer B) gastritis C) esophageal varices D) gastric carcinoma E) esophagitis INT-452. If a patient with classic reflux esophagitis shows no reaction to cimetidine or a therapeutic elevation of his bed then therapy must be supplemented with metoclopramide because: A) it stimulates gastric evacuation, which is impaired in 50% of these patients B) it increases the lower esophageal sphincter tone C) it improves the acid clearance D) it improves gastritis which occurs in 75% of these patients E) all of the above INT-453. The enzyme metabolizing alcohol is: A) alcohol reductase B) alcohol oxidase C) alcohol dehydrogenase D) alcohol synthetase E) glucose-6-phosphatase INT-455. How does the vagal tone affect the basal gastrin level? DrRognerud@www.medfever.com

INTERNAL MEDICINE

Answer: B

Answer: B

Answer: C

Answer: E

Answer: C

Answer: A 262 / 343

Board Exam Question (Smiley + Star) + 2007 A) it elevates the gastrin level B) it lowers the gastrin level C) it has no effect on the gastrin level INT-461. Which of the following statements supports the diagnosis of hemolytic jaundice? A) an elevated non-conjugated bilirubin level in the plasma B) the presence of bilirubin in the urine C) an elevated reticulocyte count D) both (A) and (C) are true E) none of the above INT-462. Case Study: In a patient with primary biliary cirrhosis, polyclonal gammopathy was established by protein-electrpphoresis. The immunoglobulin found is predominantly: A) IgA B) IgM C) IgG D) IgD E) IgE INT-464. Which of the following agents inhibits vitamin B12 absorption? A) phenytoin B) methotrexate C) cycloserine D) trimethoprim E) all of the above., INT-465. A manifestation of Crohn s disease can occur in: A) the oral cavity B) the esophagus C) the stomach D) the ileum E) all of the above INT-467. Which of the following endocrine alterations can be accompanied by an exudative ascites? A) hyperparathyroidism B) hyperthyroidism C) hypothyroidism D) Addison's disease E) acromegaly

INTERNAL MEDICINE

Answer: D

Answer: B

Answer: E

Answer: E

Answer: C

DrRognerud@www.medfever.com

263 / 343

Board Exam Question (Smiley + Star) + 2007 INT-471. Case Study: 2 weeks after a coronary bypass operation the patient's SGOT/SGPT was 30/350 with a normal liver function. 3 months later the patient complained of fatigue and weakness, his SGPT level increased over 300. Which of the following is the most probable diagnosis? A) delta-hepatitis B) B-type hepatits C) non-A non-B hepatitis D) CMV hepatitis E) activation of a chronic hepatitis INT-473. All of the following are clinico-pathological symptoms of lead intoxication, EXCEPT: A) anemia with basophilic granules in the red blood cells B) membranous nephropathy with nephrotic syndrome C) increased renal excretion of aminolevulinic acid and coproporphyrin D) lethargy, stupor, mental retardation and encephalopathy INT-477. Which of the following alterations is not characteristic of immunodeficient diseases (AIDS)? A) lymphoid depletion in the cortical and paracortical areas of lymph nodes B) viral inclusions in the histiocytes of lymph nodes C) gammaglobulinemia D) an inverted ratio of T-helper/T-inductor cells E) common pneumocystis carni infections INT-491. Case Study: Which of the following is the most probable diagnosis in a 24-year-old male patient suffering from arthritis, conjunctivitis and urethritis? A) rheumatoid arthritis B) Reiter's syndrome C) pseudo-gout D) gonorrhea E) ankylosing spondylitis INT-496. Which of the following is a dead vaccine? A) pertussis immunization B) BCG immunization C) measles immunization D) rubella immunization E) yellow fever immunization

INTERNAL MEDICINE Answer: C

Answer: B

Answer: C

Answer: B

Answer: A

DrRognerud@www.medfever.com

264 / 343

Board Exam Question (Smiley + Star) + 2007 INT-501. Case Study: A 30-year-old patient with chronic active hepatitis (type B) converted from hepatitis E-antigen+ to hepatitis E antibody+. His transaminase values became normal and a liver biopsy showed inactive postnecrotic cirrhosis. What should be done next? A) comfort the patient; inform him that he is in complete remission B) inform the patient that no relationship exists between the E antibodies and the transaminase values C) observe the patient, since spontaneous seroconversion to Eantigen would reactivate the disease D) none of the above INT-505. The adequate therapy of anemia developing in the cecal loop syndrome is: A) surgery B) folic acid supplementation C) vitamin B12 supplementation D) wide-spectrum antibiotics E) iron supplementation INT-512. Case Study: A 24-year-old female patient has a 2-year history of mild non-erosive arthritis. The patient also suffers from severe depression, which developed when she had started taking oral contraceptive pills. The patient is penicillin-sensitive. At examination a pericardial friction sound and proteinuria (over 3.5g) were found. Select the most probable diagnosis: A) rheumatoid arthritis B) rheumatic fever C) mixed connective tissue disease D) systemic lupus erythematosus (SLE) E) polyarteritis nodosa INT-527. Which of the following statements concerning Goodpasture's syndrome is FALSE? A) a synonym for glomerulonephritis caused by antiglomerular basement membrane antibodies B) pulmonary hemorrhage always disappears after a bilateral nephrectomy C) antiglomerular basement membrane antibodies show crossreactivity with alveolar membrane D) pulmonary hemorrhage is sometimes not recognized E) agressive plasma replacement therapy can stop the pathological process in the lung INT-532. Which type of renal calculi is the most common? DrRognerud@www.medfever.com

INTERNAL MEDICINE Answer: C

Answer: D

Answer: D

Answer: B

Answer: E 265 / 343

Board Exam Question (Smiley + Star) + 2007 A) apatite concrement B) magnesium-ammonium-phosphate concrement C) urate concrement D) cystine concrement E) calcium oxalate with or without calcium phosphate INT-533. 70-80% of a renal calculus is comprised of: A) calcium B) phosphate C) uric acid D) oxalic acid E) ammonium INT-535. Which of the following should be the drug of choice in idiopathic hypercalciuria? A) ascorbic acid B) trichlormethiazide C) oral orthophosphate D) cortisone E) cellulose phosphate INT-537. Cardiovascular complications comprise about 10% of the annual mortality of dialysed patients. Which of the following is another substantial mortality factor in this group of patients? A) sepsis B) neuropathy C) cerebral complications D) metabolic disorders E) anemia INT-538. If a patient with ulcerative colitis develops thromboembolic complications, which of the following statements concerning such related coagulation disorders is true? A) an increased platelet count is a causative factor B) an elevated fibrinogen level is a causative factor C) a decreased factor V is a causative factor D) an elevated factor VII is a causative factor INT-539. Lyme disease is caused by: A) Streptobacillus moniformis B) Borrelia recurrentis C) Borrelia burgdorferi D) Bartonelle baciliformis E) none of the above

INTERNAL MEDICINE

Answer: A

Answer: B

Answer: C

Answer: C

Answer: C

DrRognerud@www.medfever.com

266 / 343

Board Exam Question (Smiley + Star) + 2007 INT-541. Case Study: A 22-year-old male patient has a painless swollen ankle. Swelling developed during a 2 month period. The patient also reported a 9 year history of bilateral pleural pain, and recurrent, 3-9 day-periods of a swollen painful right knee and fever. Select the cause of the edema: A) amyloidosis B) phlebitis C) cor pulmonale D) rheumatic cardiac disease E) chronic glomerulonephritis INT-542. Cerebral metastatic carcinomas usually originate from the: A) prostate gland B) esophagus C) bones D) lungs E) salivatory glands MULTIPLE CHOICE QUESTIONS / TYPE I Select the correct answers to the following questions!!! ...each qestion may have more than one correct answer. INT-549. Hemochromatosis: A) is more common in women than in men B) is rare before middle age C) results from an autosomal recessive trait D) hepatomas occur with increased frequency in patients with longstanding hemochromatosis E) none of the above INT-559. The fourth heart sound: A) can be physiological if it occurs in young individuals under 30yearsold B) starts simultaneously with the "a" wave of the jugular vein pulse curve C) occurs on the sudden opening of the mitral valve D) is related to ventricular distensibility E) is a sign of systemic hypertension INT-560. The intensity of cardiac murmurs: A) increases on the right side during expiration B) amyl nitrate increases the murmur in aortic stenosis C) increases during Valsalva's maNEUver in hypertrophic obstructive cardiomyopathy D) in mitral stenosis the intensity of the murmur depends on the DrRognerud@www.medfever.com

INTERNAL MEDICINE

Answer: A

Answer: D

Answer: BCD

Answer: BDE

Answer: BCE

267 / 343

Board Exam Question (Smiley + Star) + 2007 extent of the stenosis E) increases during a physical load in aortic insufficiency INT-561. A protruding pulmonary artery on the chest X-ray indicates: A) a ventricular septal defect B) valvular pulmonary stenosis C) Tetralogy of Fallot D) severe mitral stenosis E) a massive pulmonary embolism INT-562. Under normal conditions, the stroke volume increases: A) during increased sympathetic activity B) during increased parasympathetic activity C) in an increased ventricular end-diastolic volume D) in changing from the recumbent position to the upright position E) in high ambient temperatures INT-563. A left ventricular aneurysm: A) is a common sequel of rheumatic fever B) causes recurrent ventricular tachycardia C) causes ST depression on the ECG D) leads to cardiac insufficiency E) correlates with a higher incidence of systemic embolization INT-565. Which of the following is characteristic of mitral stenosis? A) an "opening snap" B) a shifted apical beat C) it is a complication of infective endocarditis D) it is more common in women E) a history of rheumatic fever is frequently found in this disorder INT-567. Typical features of severe aortic stenosis include: A) syncope induced by physical load B) a louder second sound over the aorta C) a blood pressure of 180/120 mmHg D) left ventricular hypertrophy on the ECG/Echocardiography E) the systolic gradient between the left ventricle and the aorta exceeds 60 mmHg INT-574. Which of the following ECG changes develop in the early stage of an acute transmural myocardial infarction? A) a dome-like ST-elevation B) a pathological Q wave C) negative T waves DrRognerud@www.medfever.com

INTERNAL MEDICINE

Answer: ABDE

Answer: ACE

Answer: BDE

Answer: ADE

Answer: ADE

Answer: AB

268 / 343

Board Exam Question (Smiley + Star) + 2007 D) ST depression in the infarcted area E) none of the above INT-577. Drugs inducing goiter include: A) aminosalicylic acid B) digoxin C) lithium D) propylthiouracil E) streptomycin INT-580. Typical alterations in Paget's disease include: A) increased bone resorption and bone formation B) stronger than normal bones C) an increased serum parathormone level D) high-output cardiac insufficiency is the usual cause of death E) a high level of urinary hydroxyproline excretion INT-585. Which of the following statements are characteristic of pheochromocytoma? A) the intravenous administration of beta-mimetic agents is necessary in an acute hypertensive crisis B) remote metastases have already developed by the time of diagnosis C) the diagnosis is usually based on paroxysmal hypertensive fits D) an excessive renal excretion of 5-hydroxy-indolacetic acid E) the maximum incidence is between the third and fifth decades INT-591. Which of the following is observed in severe pyloric stenosis? A) hypokalemia B) an elevated urea concentration C) a low urine pH value D) hypochloremia E) tetany INT-600. Characteristic findings in polycythemia vera rubra include: A) a decreased plasma volume B) a tendency to hemorrhage easily C) an increased RBC sedimentation rate D) itching and pruritis E) an increased neutrophilic activity of alkaline phosphatase INT-613. Which of the following drugs can cause hirsutism? A) spironolactone B) phenytoin C) minoxidil DrRognerud@www.medfever.com

INTERNAL MEDICINE

Answer: ACD

Answer: AE

Answer: CE

Answer: ABCDE

Answer: BDE

Answer: BC

269 / 343

Board Exam Question (Smiley + Star) + 2007 D) digoxin E) clomiphene INT-623. Which pattern of regional enteritis (Crohn's disease) is most often recognized? A)obstruction B) diffuse jejunoileitis C) abscess formation D) inflammation E) fistulas INT-624. Mechanical intestinal obstruction is assumed if the physical examination reveals: A) visible peristalsis B) a lack of intestinal sounds C) a "tympanitic" abdomen D) loud peristaltic sounds E) decreased liver dullness INT-627. Which of the following methods are used in the differential diagnosis of congenital megacolon? A) sigmoidoscopy B) study of the distal rectum C) rectum biopsy D) barium enema E) stool cultivation INT-629. A lesion of which of the following parts of the gastrointestinal tract can cause tarry stool? A) the esophagus B) the stomach C) the jejunum D) the cecum E) the sigmoid colon INT-638. An elevated SGOT value can occur in which of the following cases? A) liver necrosis B) myocardial disease C) opiate administration D) muscle lesions E) only (A) and (B) are true INT-642. Typical endoscopic findings in a benign gastric ulcer show: A) sharp regular edges DrRognerud@www.medfever.com

INTERNAL MEDICINE

Answer: ABCDE

Answer: ACD

Answer: CD

Answer: ABCD

Answer: ABCD

Answer: AB

270 / 343

Board Exam Question (Smiley + Star) + 2007 B) a clear, smooth base C) a nodular base D) rigid folds E) none of the above INT-657. In which of the following does erythema nodosum occur? A) lymphoma B) sarcoidosis C) Streptococcal infections D) ulcerative colitis E) when oral contraceptives are taken over an extended period INT-663. Which of the following are true for a 0 blood group patient? A) the patient has anti-A antibodies B) the patient has anti-B antibodies C) the patient's parents have blood group A or B D) the patient has a higher risk of developing a duodenal ulcer E) the patient has a higher risk of developing a gastric carcinoma INT-664. Which of the following indicate hemolysis due to an incompatible blood transfusion? A) pulmonary edema B) lumbar pain C) headache D) urticaria E) hemoglobinuria INT-665. Which of the following statements about hemolytic anemia of, newborns are true? A) it occurs if the father is Rh-positive and the mother is Rhnegative B) it is more probable if ABO incompatibility occurs C) it never occurs in the first newborn D) the most frequent pairing resulting in sensitivity is maternal blood group O and fetal blood group A E) it can be prevented by injecting a high-titer anti-Rh gammaglobulin preparation within 72 hours after delivery INT-666. Which of the following causes Coombs-positive hemolysis? A) dapsone B) methyldopa C) penicillin D) Clostridium perfringens sepsis E) cephalosporins

INTERNAL MEDICINE

Answer: BCDE

Answer: ABCD

Answer: BCE

Answer: ABDE

Answer: BCE

DrRognerud@www.medfever.com

271 / 343

Board Exam Question (Smiley + Star) + 2007 INT-670. When are specific human immunoglobulins used? A) for hepatitis A B) for hepatitis B C) for rabies D) for varicella zoster E) for diphteria INT-677. Which of the following are associated with pulmonary eosinophilia? A) Ascaris lumbricoides B) Strongyloides stercoralis C) Ancylostoma braziliense D) nickel carbonate E) chlorpropamide INT-683. Though endoscopic study-induced infections are rare, such a possibility must be considered if postendoscopic fever occurs. Which of the following infections can develop following gastroscopy? A) Salmonella typhi B) Mycobacterium tuberculosis C) Hepatitis B D) Stercoralis E) Pseudomonas aeruginosa INT-714. Which of the following features are characteristic of Bence-Jones protein? A) it is found in the urine of 70% of patients with myeloma B) it precipitates at 37 0C and is redissolved at higher temperatures C) it is comprised of both kappa and lambda chains D) the "Albustix" test is positive E) when it is present the risk of renal symptoms is increased

INTERNAL MEDICINE Answer: BCD

Answer: ABCDE

Answer: ACDE

Answer: AE

MULTIPLE CHOICE QUESTIONS WITH KEY ANSWERS / TYPE IV Every question or incomplete statement has only one answer in the combinations specified after each and every question: Select one of these mentioned key combinations!!! INT-715. Secondary osteoporosis can develop in: 1) steroid therapy 2) hyperthyroidism 3) acromegaly 4) postmenopause 5) Cushing's syndrome 6) starvation due to an absorption disorder A) 1, 3, and 4 are true B) 2 and 5 aretrue DrRognerud@www.medfever.com Answer: E

272 / 343

Board Exam Question (Smiley + Star) + 2007 C) 4 and 5 aretrue D) only 3 is true E) all of the above INT-719. Schistosomiasis has the following characteristic clinical symptoms: 1) fever and allergic symptoms 2) abdominal pain and diarrhea 3) portal hypertension 4) cor pulmonale 5) myocarditis A) 1, 3, and 4 are true B) 2 and 4 aretrue C) 1 and 5 aretrue D) only 5 is true E) all of the above INT-721. Which of the following characterizes the cerebrospinal fluid in aseptic meningitis? 1) a turbid fluid 2) a low glucose content 3) a polymorphonuclear reaction at the onset of the disease 4) an elevated protein level A) 1 and 4 aretrue B) 2 and 4 aretrue C) 1 and 3 aretrue D) 3 and 4 aretrue E) all of the above F) none of the above INT-722. Hypothermia occurs in: 1) myxedema 2) high-dose sedatives 3) alcohol intoxication 4) hyperthyroidism A) 1, 2, and 3 are true B) 1 and 3 aretrue C) 2 and 4 aretrue D) only 4 is true E) all of the above INT-725. Case Study: An elderly patient with severe airway obstruction and chronic bronchitis is treated with digitalis for circulatory insufficiency. During therapy he suddenly develops a supraventricular paroxysmal tachycardia (160/min) which was terminated with physical interventions. Which of the following medications should be used for the DrRognerud@www.medfever.com

INTERNAL MEDICINE

Answer: E

Answer: D

Answer: A

Answer: B

273 / 343

Board Exam Question (Smiley + Star) + 2007 therapy of this arrhythmia? 1) digoxin iv. 2) pindolol iv. 3) metoprolol (Betaloc, cardioselective beta-blocker) iv. 4) quinidine p.o. 5) verapamil iv. A) 1, 3, and 4 are true B) 3 and 5 aretrue C) 3 and 4 aretrue D) 1 and 4 aretrue E) 2, 4, and 5 are true INT-729. Which of the following methods help in the diagnosis of lung cancer? 1) bronchoscopy 2) a scalene lymph node biopsy 3) a cytological smear from the sputum 4) angiography 5) ultrasound A) 1, 2, and 3 are true B) 1 and 3 aretrue C) 2 and 4 aretrue D) only 4 is true E) all of the above INT-732. Case Study: A blood pressure value of 170/120 was found on four occasions in a 25-year-old female patient who takes no antihypertensives. A physical study showed no alterations. What should be done next? 1) nothing, because women are less susceptible to the sequels of hypertension 2) to prescribe a high potassium diet and recheck the patient after 6 weeks 3) measure the sodium content of the red blood cells 4) check for secondary hypertension A) 1, 2, and 3 are true B) 1 and 3 aretrue C) 2 and 4 aretrue D) only 4 is true E) all of the above INT-733. Case Study: In a weak male patient with sinus bradycardia one doctor repeatedly measured blood pressure values of 170/ 100 mmHg, while an other doctor always obtained 160/ 100 mmHg. Which of the following might have caused this difference? 1) the cuff is too wide 2) digital devices have different accuracies ("silent" range) DrRognerud@www.medfever.com

INTERNAL MEDICINE

Answer: A

Answer: D

Answer: D

274 / 343

Board Exam Question (Smiley + Star) + 2007 3) the emotional liability of the patient 4) the two doctors released the air with different speeds A) 1, 2, and 3 are true B) 1 and 3 aretrue C) 2 and 4 aretrue D) only 3 and 4 is true E) all of the above INT-734. Which of the following alterations can be found in the fundus in both hypertension and arteriosclerosis? 1) blurred disk margins 2) narrowed vessels 3) crossing of the vessels 4) copper-wire arteries A) 1, 2, and 3 are true B) 1 and 3 aretrue C) 2 and 4 aretrue D) only4 is true , E) all of the above INT-736. Select the characteristic features of hypertension in end-stage renal disease: 1) the hypertension is usually volume-dependent 2) the plasma renin level is usually low 3) the hypertension can be controlled with adequate dialysis 4) the hypertension is of a low-volume type A) 1, 2, and 3 are true B) 1 and 3 aretrue C) 2 and 4 aretrue D) only 4 is true E) all of the above INT-737. Case Study: A 14-year-old boys blood pressure is 140/ 100 mmHg. Two weeks ago he complained of a sore throat Which ofthe following studies should be performed next? 1) a urine sediment study 2) a urine cultivation for the presence of any microbes 3) a urine study with sulfosalicylic acid 4) the determination of the urine pH value A) 1, 2, and 3 are true B) 1 and 3 aretrue C) 2 and 4 aretrue D) only 4 is true E) all of the above

INTERNAL MEDICINE

Answer: E

Answer: A

Answer: B

DrRognerud@www.medfever.com

275 / 343

Board Exam Question (Smiley + Star) + 2007 INT-747. Case Study: A 32-year-old male patient has paroxysmal hypertension and headaches, but the repeated values of the urine and plasma levels of vanilinmandelic acid, catacholamines and metanephrine are always normal. Which of the following can be assumed? 1) inadequate urine sampling techniques 2) aberrant adrenal tissue 3) an inactive adrenal tumor 4) a cerebral astrocytoma A) 1, 2, and 3 are true B) 1 and 3 aretrue C) 2 and 4 aretrue D) only 4 is true E) all of the above INT-748. Select the differential criteria of renal and non-renal progressive hypertension: 1) proteinuria 2) red blood cells casts 3) a high serum creatinine level 4) none of the above A) 1, 2, and 3 are true B) 1 and 3 aretrue C) 2 and 4 aretrue D) only 4 is true E) all of the above INT-752. Case Study: A 55-year-old patient on antihypertensive therapy complains of palpitation and headache. He also has accompanying signs of fluid retention. Which of the followIng antihypertensive agents can cause these symptoms? 1) hydralazine 2) phentolamine 3) minoxidil 4) bumetanide A) 1, 2, and 3 are true B) 1 and 3 aretrue C) 2 and 4 aretrue D) only 4 is true E) all of the above INT-758. Which of the following statements about calcium channel blocking agents is true? 1) verapamil and diltiazem block the intracellular calcium influx 2) diltiazem enhances calcium outflux from the cell 3) a combination of calcium entry blockers with beta blockers can lead to cardiac insufficiency DrRognerud@www.medfever.com

INTERNAL MEDICINE Answer: D

Answer: D

Answer: B

Answer: E

276 / 343

Board Exam Question (Smiley + Star) + 2007 4) the direct effect of sublingual nifedipine administration lasts for 3 hours A) 1, 2, and 3 are true B) 1 and 3 aretrue C) 2 and 4 aretrue D) only 4 is true E) all of the above INT-762. Which of the following statements about antihypertensive drugs is tunie? 1)chlorthalidone, metolazone, and triamterene are ali diuretics and have a similar effect on the potassium balance 2) captopril and saralasin are ACE inhibitors 3) propranolol, timolol, and atenolol are all lipid-soluble betablockers 4) minoxidil, hydralazine, diazoxide, and nitroprusside are all vasodilators and elevate the plasma renin activity A) 1,2, and 3 are true B) l and 3 are true C) 2 and 4 aretrue D) only 4 is true E) all of the above INT-771. Dynamic ileus can be caused by the following: 1) chronic lead intoxication 2) acute diffuse peritonitis 3) biliary colic 4) renal colic 5) intermittant acute porphyria A) 1, 2, and 4 are true B) 2, 3, and 4 are true C) all of the above D) 2, 3, 4, and 5 aretrue E) 1, 2, 3, and 4 aretrue INT-772. Early metastases of gastric cancer can develop in the following organs: 1) the axillary lymph nodes 2) the supraclavicular lymph nodes 3) the inguinal lymph nodes 4) the liver 5) the testis A) 1, 3, and 5 are true B) 2, 4, and 5 are true C) 1 and 3 aretrue D) 2 and 4 aretrue E) 2 and 5 aretrue INT-773. Prostatitis can be caused by the following: DrRognerud@www.medfever.com

INTERNAL MEDICINE

Answer: D

Answer: C

Answer: D

Answer: B 277 / 343

Board Exam Question (Smiley + Star) + 2007 1) gonorrhea 2) syphilis 3) tularemia 4) tuberculosis 5) brucellosis A) 1, 3, and 5 are true B) 1 and 4 aretrue C) 2, 3, and 5 are true D) only 3 is true E) all of the above INT-784. In which of the following does hypercalcemia occur? 1) in steatorrhea 2) in an overdose of vitamin D 3) in hypoparathyroidism 4) in hyperparathyroidism 5) in multiple myeloma A) 1, 2, 4. and 5 aretrue B) all of the above C) 2 and 4 aretrue D) 1, 3, and 4 are true E) 2, 4, and 5 are true INT-787. Which of the following can cause an air embolism? 1) damage of the pulmonary veins 2) damage of the systemic veins 3) inhalation of hyperbaric oxygen 4) opened veins following the abruption of the placenta 5) damage of the lymphatic vessels A) 1, 2, and 4 are true B) 3 and 5 aretrue C) only 3 is true D) 1 and 5 aretrue E) all of the above INT-791. Select the dangers of immunosuppressive therapy: 1) the occurrence of intercurrent infections 2) the development of a malignant tumor 3) a decreased titer of auto-antibodies 4) anemia 5) prolonged wound healing A) 1, 3, and 5 are true B) 2 and 3 aretrue C) 3 and 4 aretrue D) 1, 2, 4, and 5 aretrue E) only 3 is true

INTERNAL MEDICINE

Answer: E

Answer: A

Answer: D

DrRognerud@www.medfever.com

278 / 343

Board Exam Question (Smiley + Star) + 2007 INT-794. Which of the following diseases can lead to the development of chronic cor pulmonale? 1) aortic coarctation 2) mitral insufficiency 3) patent foramen ovale 4) emphysema 5) silicosis A) 1, 3, and 5 are true B) 2, 4, and 5 are true C) 3 and 4 aretrue D) only 3 is true E) all of the above INT-796. Secondary cardiomyopathy can be caused by: 1) hyperthyroidism 2) pheochromocytoma 3) beriberi 4) amyloidosis 5) glycogenosis A) 1, 3, and 5 are true B) 2 and 4 aretrue C) 4 and 5 aretrue D) only 3 is true E) all of the above INT-797. Right-sided heart hypertrophy can be caused by: 1) emphysema 2) silicosis 3) mitral stenosis 4) carcinoid syndrome 5) Adams-Stokes syndrome A) 1 and 2 aretrue B) 1, 2 and 3 are true C) 1, 2 and 4 are true D) 1, 2, 3, and 4 aretrue E) all of the above INT-801. Hydronephrosis can occur in: 1) urethral stricture 2) prostatic hypertrophy 3) diabetes insipidus 4) spinal damage 5) the lodging of a concrement in the urinary tract A) 1, and 2 are true B) 1, 2, and 3 are true C) all of the above DrRognerud@www.medfever.com

INTERNAL MEDICINE

Answer: B

Answer: E

Answer: D

Answer: D

279 / 343

Board Exam Question (Smiley + Star) + 2007 D) 1, 2, 4, and 5 aretrue E) 2 and 5 aretrue INT-806. Where do ulcers typically occur in the Zollinger-Ellison syndrome? 1) in the limbs 2) in the soft palate 3) in the stomach 4) in the duodenum 5) in the rectum A) 1, 2, 3, and 4 aretrue B) 2, 3, and 4 are true C) 3 and 4 aretrue D) 3, 4, and 5 are true E) 2 and 3 aretrue INT-807. Obstructive jaundice can be caused by: l) carcinoma of Vater's papilla 2) a concrement in the common bile duct 3) obstruction of the Wirsungian duct 4) a carcinoma of the gallbladder 5) a carcinoma of the pancreatic tail A) 1, 2, and 5 are true B) 1 and 2 aretrue C) 1, 2, 3, and 5 aretrue D) all of the above E) 1, 2, 4, and 5 aretrue INT-811. Which of the following factors markedly contributes to the development of head and neck cancer? 1) alcohol 2) smoking 3) the Epstein-Barr virus 4) the hepatitis B virus A) 1, 2, and 3 are true B) 1 and 3 aretrue C) 2 and 4 aretrue D) only 4 is true E) all of the above INT-823. Which of the following histocompatibility antigens predisposes to rheumatoid arthritis? 1) HLA/DRI 2) HLA/DR2 3) HLA/DR3 4) HLA/DR4 A) 1 and 2 aretrue DrRognerud@www.medfever.com

INTERNAL MEDICINE

Answer: C

Answer: B

Answer: A

Answer: C

280 / 343

Board Exam Question (Smiley + Star) + 2007 B) 1 and 3 aretrue C) 1 and 4 aretrue D) 2 and 3 aretrue E) 2 and 4 aretrue F) 3 and 4 aretrue

INTERNAL MEDICINE

ASSOCIATION QUESTIONS Associate the following terms/statements marked by the letters A, B, C... with the corresponding statements/terms marked by and in the order given by the figures 1, 2, 3... ...for example: 1-C, 2-B, 3-A, 4-D. Put the answer as C, B, A, D! (Note: Different statements can be associated with the same terms!!!) INT-839. Associate the following statement(s) with their corresponding term(s)! A) potassium excreting kidney B) difficulties in the initiation of movements; slow relaxation C) during such a fit the serum potassium level decreases D) athletes 1) Familial hypokalemic paralysis 2) Thomsen's syndrome 3) Primary hyperaldosteroneism 4) Ossifying myositis INT-840. Associate the following statement(s) with their corresponding term(s)! A) vitamin A intoxication B) it is complicated by sarcoma C) neurologic complications; fractures D) skin pigmentation E) a saddle-nose deformity F) uneven cartilage G) in the differential diagnosis angina pectoris 1) Paget's disease 2) Hyperostosis 3) Osteoporosis 4) Fibrotic dysplasia 5) Ollier's syndrome 6) Relapsing polychondritis 7) Tietze's syndrome INT-846. Associate the following statement(s) with their corresponding term(s)! A) hypertension B) hypotension C) both of the above D) none of the above 1) Vasopressin 2) Prostaglandins 3) Renin DrRognerud@www.medfever.com

Answer: CBAD

Answer: BACDFEG

Answer: ACDADB

281 / 343

Board Exam Question (Smiley + Star) + 2007 4) Aldosterone 5) Glucagon 6) Bradykinin INT-847. Associate the following statement(s) with their corresponding term(s)! A) an increased venous capacity B) sympathetic blockade C) an inhibition of the converting enzyme D) a low plasma volume E) short-term alpha blockade 1) Nitroprusside 2) Captopril 3) Methyldopa 4) Amiloride 5) Phentolamine 6) Chlorthalidone INT-849. Associate the following term(s) with their corresponding statement(s)! A) Nephrosclerosis B) Nephrosis C) Chronic pyelonephritis D) Renovascular disease E) Conn's syndrome 1) the 24 hour protein excretion is 3.8 g; normal iv. urogram 2) the 24 hour protein excretion is 400 mg; normal iv. urogram 3) the 24 hour protein excretion is 400 mg; iv. urogram reveals the symmetrically shortened, deformed calyx 4) the 24 hour protein excretion is 800 mg; iv. urogram shows the left kidney to be 7cm long and the right kidney to be 10 cm long INT-850. Associate the following term(s) with their corresponding statement(s)! A) 11-hydroxylase defect B) 3-beta-dehydrogenase defect C) Both of the above D) None of the above 1) hypertension 2) hypokalemia 3) hyperkalemia 4) glucocorticoid therapy 5) metyrapone therapy 6) hypogonadism 7) virilisation 8) usually associated with fatal outcome in pediatric patients INT-855. Associate the following statement(s) with their corresponding term(s)! A) cholestyramine has a beneficial effect DrRognerud@www.medfever.com

INTERNAL MEDICINE

Answer: ACBDED

Answer: BACD

Answer: CCDCDAA

Answer: DBAEC

282 / 343

Board Exam Question (Smiley + Star) + 2007 B) a good response to a gluten-free diet C) a good response to wide-spectrum antibiotics D) a good response to the withdrawal of milk E) the administration of digestive enzymes decreases steatorrhea 1) Lactase deficiency 2) Celiac disease 3) Enteropathy caused by bile acid salts 4) Chronic pancreatitis 5) Whipple's disease INT-856. Associate the following term(s) with their corresponding statement(s)! A) Carcinoma of the pancreas body B) Mild, chronic pancreatitis C) Pancreatitis related to alimentary and metabolic factors D) Carcinoma of the pancreas head with complete obstruction E) Pancreatic disease related to hemochromatosis 1) a lack or markedly decreased amount of fluid, low bicarbonate and amylase level 2) a decreased amount of fluid, with normal bicarbonate and amylase level 3) a normal amount of fluid, with low bicarbonate and normal or low amylase level 4) a normal amount of fluid, with normal bicarbonate and low amylase level 5) an increased amount of fluid, low bicarbonate and normal amylase level INT-858. Associate the following term(s) with their corresponding statement(s)! A) Pancreas B) Lung C) Kidney D) Muscle E) Breast 1) Zenker's waxy necrosis 2) enzymatic fatty necrosis 3) traumatic fatty necrosis 4) anemic infarction 5) hemorrhagic infarction INT-864. Associate the following term(s) with their corresponding statement(s)! A) Digoxin B) Lidocaine C) Propranolol D) Albuterol 1) a sudden withdrawal in patients with angina pectoris can cause angina 2) it is an efficient bronchodilator DrRognerud@www.medfever.com

INTERNAL MEDICINE

Answer: DABCE

Answer: DAECB

Answer: CDCAB

283 / 343

Board Exam Question (Smiley + Star) + 2007 3) it should not be given to asthmatic patients 4) it lowers the increased ventricular rate in atrial fibrillation 5) it affects only ventricular arrhythmias

INTERNAL MEDICINE

RELATION ANALYSIS In the following questions determine if the statement in the first half of the sentence and the explanation in the second half of the sentence are true and if a causal relationship exists between them. Select the single correct version from the five possible combinations: A) both the statement and the explanation are true and a causal relationship exists between them; B) both the statement and the explanation are true but there is no causal relationship between them; C) the statement is true, but the explanation is false; D) the statement is false, but the explanation itself is true; E) both the statement and the explanation are false. INT-876. Prostate hyperplasia leads to the retention of urine, therefore a bacterial infection can easily occur in prostatic hyperplasia. INT-878. An embolus is always solid because fluids and gases cannot cause obstruction of the vascular lumen. INT-879. Anaerobic glycolysis is enhanced in the body's tissue during hypoxia, therefore glycogen accumulates in the hypoxic hepatic tissue. INT-880. Melanin accumulation occurs in Addison's disease because hydrocortisone fails to inhibit the pituitary melanocyte-stimulating hormone (MSH) secretion. INT-885. Aortic stenosis markedly contributes to the left ventricular load, therefore left ventricular hypertrophy is a sequel of aortic stenosis. INT-888. Fluid transudation across the lymphatic vessel walls occurs in lymphedema because in lymphedema the permeability of the lymphatic vessels increases. INT-889. Heparin exhibits an antithrombin effect because the heparin molecule is comprised of acidic mucopolysaccharides. INT-890. Inflammatory hyperemia is an active process because the vascular DrRognerud@www.medfever.com Answer: A

Answer: E

Answer: C

Answer: A

Answer: A

Answer: C

Answer: B Answer: B 284 / 343

Board Exam Question (Smiley + Star) + 2007 permeability is increased in a focus of inflammation. INT-891. Anemic infarction does not always develop after an occlusion of the mesenteric artery because an impairment of the venous outflow is another important factor in the development of hemorrhagic infarction. INT-892. Gelatinous pneumonia is an exudative form of pulmonary tuberculosis because a specific granuloma is the most typical tuberculotic reaction. INT-895. The left ventricular blood supply is inadequate in mitral insufficiency, therefore. the left ventricle can exhibit a slight hypertrophy in mitral insufficiency INT-897. In mitral insufficiency the congestion can spread to the pulmonary circulation, therefore the right atrium and ventricle will undergo dilation and hypertrophy in mitral insufficiency. INT-899. Renal papillary necrosis mostly occurs in diabetic patients because the Kimmelstiel-Wilson's syndrome is a typical renal disease in diabetes. INT-901. A partial or intermittant urethra occlusion can lead to the development of hydronephrosis because urine retention in the pyelon usually leads to a bacterial infection. INT-902. Acute glomerulonephritis is accompanied by albuminuria, therefore the edema typical of acute glomerulonephritis is due to hypoproteinemia. INT-906. The red blood cells are more vulnerable in the spherocytic-type of anemia because the red blood cells' osmotic resistance is decreased in the spherocytic type of anemia. INT-907. Metaplasia of the bronchial epithelium can occur in bronchiectasis, therefore adeno-carcinoma is common in patients with bronchiectasis. INT-909. Dyspnea is one of the leading symptoms of emphysema because the surface of the emphysematous lung is decreased.

INTERNAL MEDICINE

Answer: E

Answer: B

Answer: E

Answer: A

Answer: B

Answer: B

Answer: C

Answer: A

Answer: C

Answer: A

DrRognerud@www.medfever.com

285 / 343

Board Exam Question (Smiley + Star) + 2007 INT-910. Pulmonary alterations elicited by silicosis are irreversible because the lung cannot eliminate the absorbed silicon. INT-914. Male patients with liver cirrhosis frequently develop gynecomastia because the hepatic estrogen inactivation system is progressively destroyed. INT-915. A gallstone can sometimes impact itself in the hepatic duct, therefore acute hemorrhagic pancreatic necrosis can develop as a complication of cholelithiasis. INT-916. Relation Analysis Portal congestion develops due to liver cirrhosis, therefore an intermittant jaundice can occur as a complication of liver cirrhosis.

INTERNAL MEDICINE Answer: A

Answer: A

Answer: B

Answer: B

CASE STUDIES Answer the multiple task questions (simple choice and multiple choice with/without key answers; relation analysis etc.) as they are related to each case study!!! INT-919. A 37-year-old female patient was urgently admitted due to abdominal seizures. During childhood the patient had been treated for jaundice in an infectious diseases ward. She has had 3 pregnancies; one was terminated by abortion. During the first pregnancy the patient complained of abdominal seizures on the right side. She took drotaverin (No-Spa) and was on a diet for a long time. Then the patient became symptom-free. She took oral contraceotive (norgestrol ethynil-estradiol -Ovidon) for three years. Two months before admission the oral contraceptive had been replaced by an IUD. The patient frequently compained of headaches and nausea. Before admission she had fever, vomited and developed abdominal seizures under the right costal arch, in the area of the cardia of the stomach. Spasmolytics relieved the pain, but not completely. The urine of the patient had a slightly darker color than normal. Physical status (most important findings): subicterus, enlarged (2 fingers), tender liver. Pressure tenderness under the right costal arch. Laboratory findings: temperature: 38.9oC. red blood cell sedimentation rate: 39 mm/h. Serum bilirubin: 45 mol/L. SGOT: 210 IU; SGPT: 180 IU. ALP: 99 IU/L; GGT: 130 IU/L. WBC: 12900. Smear: Stab cells:6, Segm.: 83, Lymph.: 11%. Urine: norm. Specific density:. 1024. Prot.: (++). Ubg.: norm., Bilirubin: positive. Precipitate: several WBC/visual field. 7.919/ 1. Single Choice Question The most probable diagnosis is: A) adrug-induced cholestasis B) viral hepatitis DrRognerud@www.medfever.com Answer: DCCCC

286 / 343

Board Exam Question (Smiley + Star) + 2007 C) acute pyelonephritis D) calculous cholecystitis E) adnexitis 7.919/2. Select One Of The Key Combinations Which of the following studies should be performed in the acute phase of the disease to support the diagnosis? 1) an ERCP study 2) a HbsAg determination 3) a serum amylase determination 4) an ultrasound study 5) cholecystography A) (1),( 2), and (5) are true B) (3), (4), and (5) are true C) (2), (3), and (4) are true D) all of the above E) none of the above 7.919/3. Select One Of The Key Combinations Which of the following laboratory parameters should be regularly monitored during the diasease? 1) serum Fe, Cu, and ferritin 2) bacteriological and parasitologic bile tests 3) WBC and serum amylase level 4) serum bilirubin, GOT, GPT, ALP 5) Weber's stool test A) (1), ( 2). (4), and (5) are true B) (3), (4), and (5) are true C) (3) and (4) are true D) all of the above E) none of the above 7.919/4. Select One Of The Key Combinations Which of the following complications is the most probable? 1) coagulation disorders 2) chronic active hepatitis 3) pancreatitis 4) empyema vesicae felleae 5) hydronephrosis A) (2), (3), and (4) are true B) (1), (4), and (5) are true C) (3) and (4) are true D) all of the above E) none of the above 7.919/5. Select One Of The Key Combinations Which of the following studies supporting the diagnosis should be performed? 1) if a liver disease due to a long-term use of oral contraceptive is suspected, a liver biopsy should be done 2) if the fever is further elevated an acute cholecystography should be done 3) an ultrasonic study should always be done first, since this non-invasive method yields important information 4) if a hydrops develops, ERCP should be done DrRognerud@www.medfever.com

INTERNAL MEDICINE

287 / 343

Board Exam Question (Smiley + Star) + 2007 5) palpation of hydrops is an indication for surgery A) (1), (3), and (5) are true B) (2), (4), and (5) are true C) (3) and (5) are true D) all of the above E) none of the above INT-920. Case Study A 42-year-old female patient complained for 3 months of sustained headaches. Mild analgetics she subsequently received were successful for the headaches. No other symptoms were found in the history. The patient had three normal pregnancies, but during the third trimester of the fourth pregnancy the patient had to take diuretics. The only disease she had recently, was a cystitis which was treated with antibiotics. The patient's father and uncle both died of a heart attack at the age of 43 and 46 years old respectively. Physical examination: the patient is slim, her blood pressure taken at rest on both arms is 180/120 mmHg. Vein "notching" was observed during examination of the fundus. No other alterations could be observed. The initial laboratory tests showed trace amounts of protein in the urine. The ECG, and chest X-ray findings were normal. 7.920/ 1. Single Choice Question The most probable cause of the hypertension is: A) pheochromocytoma or renal artery disease B) renal artery disease or brain tumor C) cerebral tumor or Cushing's syndrome D) Cushing's syndrome or primary hypertension E) primary hypertension or chronic pyelonephritis 7.920/2. Single Choice Question All of the following diagnostic studies should be performed, EXCEPT: A) intravenous pyelography B) a bone marrow puncture C) a creatinine clearance test D) the plasma renine activity 7.920/3. Single Choice Question If hypokalemia is found then all of the following can be assumed, EXCEPT: A) pheochromocytoma B) primary aldosteronism C) Cushing's syndrome D) renal perenchymal disease E) antidiuretic drug abuse 7.920/4. Single Choice Question If a patient has primary hypertension, the primary therapeutic interventions comprise all of the following, EXCEPT: A) rest B) intravenous hydralazine C) hydrochlorothiazide D) a low salt diet E) sedatives

INTERNAL MEDICINE

Answer: EBAB

DrRognerud@www.medfever.com

288 / 343

Board Exam Question (Smiley + Star) + 2007 INT-921. Case Study A 28-year-old male patient was admitted to an intensive care unit for chest pain (which had lasted for two hours), cough, and dyspnea. He had had several similar fits in the past. Such fits were usually elicited by emotional factors or strain. The patient(also has hay fever. Furthermore, the members of his family also have similar symptoms and fits. Physical examination: dyspnea, orthopnoea and cyanosis. A whistlingstridorous sound was found on auscultation of the lungs. Heart rate: 130/min, regular. Sputum: sticky, mucous. Arterial P02: 65 mmHg. 7.921 / 1. Single Choice Question The most probable diagnosis is: A) right heart insufficiency B) left heart insufficiency C) intrinsic bronchial asthma D) extrinsic bronchial asthma E) pneumoconiosis 7.921/2. Single Choice Question All of the following can cause this disease, EXCEPT: A) antigens, allergens B) excessive salt intake C) emotional stress D) infection E) analeptics 7.921/3. Single Choice Question The most probable respiratory functional finding during a fit is: A) a decreased FEV 1 B) an increased vital capacity C) a decreased residual volume D) a decreased airway resistance E) an increased arterial P02 7.921/4. Single Choice Question The best therapeutic step is: A) fluid restriction B) cardioversion C) beta-mimetics, bronchodilators D) beta-adrenergic blockers E) diuretics INT-922. Case Study A 64-year-old male patient was admitted complaining of frequent urination urges, anxiety, and nocturia. The symptoms have been persisting for 6 months. Apart from the afore mentioned symptoms, the patient also had two episodes of burning dysuria, which was treated with antibiotics. Furthermore, the patient also has a one-year history of angina pectoris and takes nitroglycerin when needed. Physical examination: blood pressure: 130/90, heart rate 90/min. Rectal examination revealed an enlarged prostate. Laboratory and ECG DrRognerud@www.medfever.com

INTERNAL MEDICINE

Answer: DBAC

Answer: AEEB

289 / 343

Board Exam Question (Smiley + Star) + 2007 findings were not contraindictory for surgery. The prostate was removed by a transurethral resection with minimal blood loss. Six hours following surgery the patient started shivering, his temperature increased to 40 C and his blood pressure fell to 90/60 mmHg. 7.922/1. Single Choice Question The most probable diagnosis is: A) gram-negative sepsis B) a myocardial infarction C) a post-operative hemorrhage D) arrhythmia E) lobular pneumonia 7.922/2. Single Choice Question A further study of the hypotension will reveal all of the followIng, EXCEPT: A) lactic acidosis B) cold, wet skin C) an increased level of fibrin-degradation products D) sinus tachycardia on the ECG E) an increased urine output 7.922/3. Single Choice Question Bacteriemia due to urinary tract infection is caused by: A) Salmonella typhi murium B) Shigella sonnei C) Vibrio cholerae D) Hemophilus influenza E) Escherichia coli 7.922/4. Single Choice Question The most important intervention is: A) cardioversion B) the monitoring of the central venous pressure C) an electric pacemaker D) salt restriction E) potassium infusion INT-924. Case Study A 56-year-old female patient complains of weakness and fatigue that have been persisting persisting for about half a year. The patient sweats at night and is sometimes febrile. During this period she has lost 10 kg. A month ago she complained of pressure under the left costal arch. The patient sometimes has palpitations and pain which radiates to her left arm. Physical examination: the patient's skin and mucosa are pale. Percussion causes sternal pain. The liver is palpable. The spleen is enlarged exceeding the left costal arch by 4 cm. Laboratory findings: Hb: 100 g/L; Hct: 0.29; WBC: 100 g/L; platelets: 350 g/L; blood smear: myeloblasts: 0.01; promyelocytes: 0.03; myelocytes: 0.06; juvenile: 0.08; bands: 0.10; segmented: 0.60; eo: 0.02; ba: 0.06; ly: 0.04; red blood cells: sustained hypochromia and anisocytosis. Normoblasts: 1 / 100 WBC; platelets: anisocytosis. 7.924/ 1. Single Choice Question The most probable diagnosis is: DrRognerud@www.medfever.com

INTERNAL MEDICINE

Answer: CCACB

290 / 343

Board Exam Question (Smiley + Star) + 2007 A) myelofibrosis B) leukemoid reaction C) chronic granulocytic leukemia D) thrombocytemia E) polycythemia vera 7.924/2. Single Choice Question Keeping in mind the above noted diagnosis, select the stage of the disease: A) the initial stage B) the CML accelerated stage C) the CML inactive stage D) the blastic stage 7.924/3. Single Choice Question Which of the following is a characteristic laboratory finding? A) a chromosome analysis will demonstrate the Philadelphia chromosome B) the morphology of the blood cells is abnormal C) monocytosis D) the leukocyte alkaline phosphate score is high E) a low reticulocyte count 7.924/4. Single Choice Question Select the therapeutic drug of first choice: A) tamoxifen (Zitazonium) B) cyclophosphamide (cytoxan) C) busulfan (Myleran) D) melphalan (Alkeran) 7.924/5. Select One Of The Key Combinations Select the most common causes of death in the above noted disease: 1) disorders of pacemaker function 2) blast crisis 3) busulfan-induced bone marrow aplasia 4) pathologic fractures 5) infections A) (1), (2), and (5) are true B) (2},-(3), and (5) are true C) (1) and (4) are true D) only (3) is true E) all the above INT-926. Case Study A 32-year-old female patient felt a sudden retrosternal pain, which radiated to the shoulder and neck. The patient had an asphyxia episode and coughed. The patient takes oral contraceptives for 2 years. Physical examination: dyspnea, cyanosis, tachycardia (130 /min). The left ankle is slightly swollen and tender. No other pathological signs were found. Blood pressure: 110/80 mmHg, red blood cell sedimentation rate: 8 mm/h. WBC: 6200; Ht: 41%. Urine: negative findings. 7.926/ 1. Select One Of The Key Combinations Which of the following studies are the most important in supporting the diagnosis: 1) ECG DrRognerud@www.medfever.com

INTERNAL MEDICINE

Answer: CBC

291 / 343

Board Exam Question (Smiley + Star) + 2007 2) blood smear 3) lung scintigraphy 4) echocardiography 5) heart catheterization 6) coronary angiography 7) urinalysis A) (2), (4), (6), and (7) are true B) (4), (6), and (7) are true C) (1) and (3) are true D) (2) and (5) are true E) all of the above 7.926/2. Single Choice Question Select the most probable correct diagnosis: A) myocardial infarction B) pulmonary embolism C) pericarditis D) lung tuberculosis E) acute pancreatitis 7.926/3. Single Choice Question Select the correct therapy: A) antibiotics B) immune suppression C) anticoagulants D) rest and diet E) antituberculotics INT-928. Case Study A 55-year-old male patient suffers for years from chronic bronchitis which causes obstructive ventilation disorders. He also has diabetes mellitus. The patient was urgently admitted with fever, cough, substantial amount of purulent sputum, dyspnea, and cyanosis. The arterial blood gas analysis was as follows: P02: 51 mmHg; PC02: 54 mmHg; pH: 7.28; St. bicarbonate: 31 mmol/L. 7.928/ 1. Single Choice Question Select the correct acid-base disorder: A) metabolic acidosis B) metabolic alkalosis C) respiratory acidosis D) respiratory alkalosis 7.928/2. Single Choice Question The patient required oxygen therapy and received 4 L 02/min via a nasal catheter. After 24 hours the blood gas analysis showed the following changes: P02: 65 mmHg; PC02: 81 mmHg; pH: 7.20; St. bicarbonate: 36 mmol/L. Select the correct therapy: A) increase the oxygen via the nasal catheter B) decrease the oxygen via the nasal catheter C) artificial respiration is required D) inhalation of 100% (pure) oxygen

INTERNAL MEDICINE

Answer: CC

DrRognerud@www.medfever.com

292 / 343

Board Exam Question (Smiley + Star) + 2007 INT-930. Case Study A 50-year-old alcoholic male patient has complained for several months of fatigue, weakness, and abdominal swelling. The swelling was at first intermittant, later it became constant. The day before, his hindlimbs became swollen too. The patient lost his appetite and his body weight decreased. Physical examination: jaundice, free abdominal fluid, liver: a four -fin-ger enlargement, uneven surface. Laboratory findings: Se-bilirubin: 50; direct bilirubin 40 mol/1; GOT: 50 IU; GPT: 22 IU; gamma-GT: 580 IU; ALP: 40 IU/L. Urine: bilirubin: positive, UBG: increased. 7.930/ 1. Single Choice Question Select the correct diagnosis: A) alcoholic hepatitis B) liver metastases C) hepatic cirrhosis D) right heart insufficiency 7.930/2. Select One Of The Key Combinations Which of the following studies should be done in the acute phase of the disease to verify the diagnosis? 1) duodenal tube 2) liver scintigraphy 3) cholecystography 4) liver biopsy A) (1, (2), and (3) are true B) (1) and (3) are true C) (2) and (4) are true D) only (4) is true E) all of the above 7.930/3. Select One Of The Key Combinations Select the possible complications of the disease: 1) esophageal varicous bleeding 2) hepatoma 3) coagulation disorders 4) empyema vesicae felleae A) (1, (2), and (3) are true B) (1) and (3) are true C) (2) and (4) are true D) only (4) is true E) all of the above INT-934. Case Study A month ago a patient became subfebrile. He also complained of a sore throat, poor general condition, pain in the limbs and in the chest, and cough. At present he complains of dyspnea, sensation of pressure in the liver, tachycardia. He can sleep only with an elevated pillow. Physical examination: mild lip cyanosis. Respiration is accompanied by flaring of the nostrils. Bilateral protruding jugular veins, tachycardia. A third heart sound at the apex was noted; decreased heart sounds. DrRognerud@www.medfever.com

INTERNAL MEDICINE

Answer: CCA

Answer: CAA

293 / 343

Board Exam Question (Smiley + Star) + 2007 The relative dullness of the heart is shifted toward the lateral chest wall. Harsh respiration. The liver is enlarged by 4 flngers. The spleen is not palpable. Adequate pulsation of the peripheral arteries. Blood pressure: 120/70 mmHg. ECG: sinus tachycardia, low-voltage. Left axis deviation. Diffuse, flattened T waves. Chest X-ray (+ 2-D imaging of the heart): "cor bovinum". Low pulsation along the heart contoure. Laboratory findings: We: 30 mm/h. AST: 120 U; SGOT, GPT, and alkaline phosphatase -normal. Pharynx: bacteria + . A high titer of the anti-Coxsackie virus antibodies. WBC count: Se: 59%; Ly: 40%; Mo: 1%. 7.934/ 1. Single Choice Question The most probable diagnosis is: A) a pulmonary embolus B) a left ventricular aneurysm following a myocardial infarction C) an exudative pericarditis with a substantial amount of fluid after a viral infection D) a combined mitral defect E) rheumatic carditis F) viral hepatitis 7.934/2. Single Choice Question Which of the following non-invasive studies should be done to support this diagnosis? A) echocardiography B) lung scintigraphy C) phono-mechanocardiography D) bronchoscopy 7.934/3. Select One Of The Key Combinations Select the correct therapy: 1) salicylates, steroids and rest (if necessary) 2) implantation of an artificial valve 3) digitalis 4) hepatoprotective therapy; diet; bed-rest 5) anticoagulant therapy 6) diuretics; pericardial puncture if necessary A) (1) and (6) are true B) (1), (2), and (5) are true C) (3), (5), and (6) are true D) (1) and (5) are true E) (3) and (6) are true INT-936. Case Study A 26-year-old female patient complained for 3 years of hypogastric, spastic pain accompanied by abdominal swelling. Every year the patient has 5-6 bouts of diarrhea, each lasting for 2-3 months. The stool contains fresh red blood and mucus. The patient also complains of frequent occasions of tenesmus. She has lost weight and complains of arthralgia. The symptoms worsen after consumption of milk. Physical examination: slightly anemic mucosa; hypogastric tenderness. Other DrRognerud@www.medfever.com

INTERNAL MEDICINE

Answer: DACEDBB

294 / 343

Board Exam Question (Smiley + Star) + 2007 findings negative. 7.936/1. Single Choice Question The most probable diagnosis is: A) a rectal tumor B) Crohn's disease C) the irratable bowel syndrome D) a severe form of ulcerative colitis E) salmonellosis 7.936/2. Select One Of The Key Combinations Which of the following should be done to support this diagnosis: 1) a stool cultivation for amebiasis and dysenteria 2) rectoscopy; biopsy; stool cultivation 3) only irrigoscopy 4) a digital rectal exploration 5) stool cultivation; colonoscopy; biopsy A) (2) and (5) are true B) (1), (2), (3), and (4) are true C) (1), (3), and (5) are true D) (2), (3), (4), and (5) are true E) (1), (2), (3), and (5) are true 7.936/3. Single Choice Question Select the etiology and pathogenesis of this disease: A) a viral infection B) a bacterial infection C) can be an autoimmune disorder D) unknown E) an allergy to milk and other food products 7.936/4. Single Choice Question Select the most characteristic complication of this disease: A) liver and pulmonary metastases B) intestinal perforation C) fistula formation D) sooner or later ileus will develop E) since the patient is in a precancerous state, cancer might develop in the future 7.936/5. Single Choice Question . Select the prognosis of this disease: A) uncurable, with a fatal outcome B) it does not affect the life-style of the patient C) cancer will develop D) it can be controlled with conservative therapy E) secondary infections will markedly reduce the life-span of the Patient 7.936/6. Single Choice Question Select the most characteristic form of the disease: A) continuous deterioration B) exacerbations and remissions are typical C) it has a sudden onset with severe toxic state D) spontaneously heals in several weeks E) lasts for years, with constant activity 7.936/7. Single Choice Question DrRognerud@www.medfever.com

INTERNAL MEDICINE

295 / 343

Board Exam Question (Smiley + Star) + 2007 Select the most relevant therapy for this disease: A) diet; spasmolytics B) surgery C) surgery; X-ray therapy D) sulfosalazine; steroids E) antibacterial and symptomatic therapy INT-937. Case Study An 18-year-old female patient complains for about a month of fatigue, dyspnea after exercise, and swollen limbs. Three days before admission the patient had an upper respiratory infection. The patient is pale. A palm-size dullness was detected over the left side of the diaphragm. Chest X-ray: fluid (a 5-finger sized area) over the left diaphragm. Edema of both lower limbs. Clear heart sounds, rhythmic contractions. Pleural puncture: straw-yellow color fluid: 1.5% protein; specific density: 1012. BP: 130/80; sedimentation: 65 mm/h; cholesterol: 6.6 mmol/L; se-creatinine: 96 mmol/L; urine protein excretion: 2.8 g/24h; se total protein: 52 g%. 7.937/1. Single Choice Question The most probable diagnosis is: A) cardiac insufficiency B) lymphedema C) the nephrotic syndrome D) acute glomerulonephritis E) diabetic glomerulosclerosis 7.937/2. Select One Of The Key Combinations Which of the following data support the assumed diagnosis? 1) hypercholesterolemia 2) anuria 3) proteinuria 4) leukemic cells 5) edema A) (1), (2) and (5) are true B) (2) and (4) are true C) (1), (4), and (5) are true D) (2), (3), and (5) are true E) (1) and (5) are true 7.937/3. Select One Of The Key Combinations Which of the following studies should be done to support the diagnosis? 1) ECG 2) a kidney concentrating ability test 3) abdominal lymphography 4) renal ultrasonography 5) renal biopsy A) (1), (3) and (5) are true B) (2) and (4) are true C) (1), (4), and (5) are true D) (1) and (4) are true E) (4) and (5) are true 7.937/4. Select One Of The Key Combinations DrRognerud@www.medfever.com

INTERNAL MEDICINE

Answer: CAECCDA

296 / 343

Board Exam Question (Smiley + Star) + 2007 Select the typical findings of this disease: 1) a decreased glomerular filtration rate 2) sinus arrhythmia 3) metabolic acidosis 4) hypoproteinemia 5) an increased red blood cell sedimentation rate A) (1), (3), and (5) are true B) (2) and (4) are true C) (1), (4), and (5) are true D) (2), (4), and (5) are true E) (1), (2), (4), and (5) are true 7.937/5. Select One Of The Key Combinations Select the correct therapy? 1) cardiac therapy 2) immunosuppressive therapy 3) steroids 4) the patient does not require drug therapy 5) peritoneal dialysis A) (1), (3), and (5) are true ` B) (2) and (5) are true C) (2) and (3) are true D) only (4) is true E) (1), (2), and (3) are true 7.937/6. Select One Of The Key Combinations Which of the following parameters should be checked during therapy? 1) the serum transaminase levels 2) urine protein excretion 3) the red blood cell sedimentation rate 4) the total se protein 5) Weber's test (feces) A) (1), (3), and (4) are true B) (1), (2), (3), and (4) are true C) (2), (3), and (5) are true D) (2), (3), and (4) are true E) (1), (3), and (5) are true 7.937/7. Single Choice Question Select the most probable prognosis of this disease: A) a total cure within two years B) a progression of the process C) the development of diabetic retinopathy D) cor pulmonale INT-939. Case Study A 32-year-old obese female patient was admitted due to an epigastric pain on the right side. After a 5-hour observation the symptoms indicated appendicitis and appendectomy was subsequently performed. The postoperative period and the next day were uneventful. When the patient tried to stand up she complained of pain in the right lower limb accompanied by a period of dyspnea. After rest however, the symptoms disappeared. Physical examination: sustained tachycardia. On DrRognerud@www.medfever.com

INTERNAL MEDICINE

Answer: DFDDDCE

297 / 343

Board Exam Question (Smiley + Star) + 2007 the 3rd day the patient had a severe stabbing pain in the chest during walking with marked dyspnea, cough, and hemorrhagic sputum. The most important observations in the status were: dyspnea, marked cyanosis, protruding neck veins, a circumscribed pleural friction sound above the left diaphragm, subfebrility, several cm difference in the diameter of the lowe limbs, and hypotension. X-ray: lung opacity over the left diaphragm. ECG: P pulmonale and signs of right heart strain. The patient had tuberculosis in the past. The patient never took any medications other than oral contraceptives (for three years). 7.939/1. Single Choice Question The most probable diagnosis is: A) pneumonia B) pulmonary tuberculosis C) superficial thrombophlebitis D) thrombosis of the deep veins with subsequent pulmonary embolism E) cardiac insufficiency 7.939/2. Single Choice Question All of the following studies can verify the diagnosis EXCEPT: A) a chest X-ray B) ECG C) venography D) laboratory studies (hemostasis, enzymes) E) a lung-scan F) lymphography 7.939/3. Single Choice Question Select the most helpful therapy: A) combined antibiotic therapy B) cardiac support C) low-dose heparin therapy D) high-dose intravenous heparin therapy E) diuretics 7.939/4. Single Choice Question How long should the patient be treated after disappearance of the acute symptoms? A) until improvement is noted B) until the patient is complaint-free C) for 2-3 weeks after disappearance of the complaints D) for half a year or a year after disappearance of the complaints 7.939/5. Single Choice Question Select the drug of choice for prolonged therapy: A) aspirin (Colfarit) tabl. B) heparin C) rutoside (Venoruton) D) coumarin (Syncumar) E) antibiotics 7.939/6. Single Choice Question Select the laboratory studies indicated in controlling this patient's state: A) a platelet count B) the bleeding time DrRognerud@www.medfever.com

INTERNAL MEDICINE

298 / 343

Board Exam Question (Smiley + Star) + 2007 C) the prothrombin time D) the coagulation time E) the partial thromboplastin time 7.939/7. Single Choice Question Which of the following does not contribute to the development of this disease? A) immobilization B) obesity C) varicosity D) surgery E) hypertension F) oral contraceptives INT-940. Case Study A 40-year-old female patient lost 6 kg in 3 months. She complains of fatigue, vertigo and headaches. In one month the patient had 4 occasions of orthostatic vertigo and fainted twice. The patient also complains of a constant abdominal pain. She has lost her appetite, frequently vomits, and has had diarrhea for several days. The patient observed an axillary and pubic hair loss and complains of decreased libido despite a normal menstruation cycle. Six weeks ago she sunbathed for several hours and got a tan of a surprisingly intensive color; she still has the tan of the same and even darker color. The patient is dysthymic and excitable. Physical examination: the patient is adynamic and has increased skin pigmentation without cyanosis. Blood pressure is 100/70 mmHg (lower than the usual value mentioned by the patient). Heart and lungs negative; free abdomen; ptosis of the liver (one finger beyond the costal arch). The liver is smooth, not edgy. The spleen is not palpable. No. tenderness in the renal region. No edema. Loss of axillary hair. 7.940/ 1. Single Choice Question The most probable diagnosis is: A) a prolactin-secreting adenoma of the anterior pituitary B) hypothyroidism C) primary adrenal insufficiency D) the first signs of menopause E) post-partum pituitary damage (Sheehan's syndrome) F) anorexia nervosa G) porphyria cutanea tarda (urocoproporphyria) 7.940/2. Select One Of The Key Combinations Select the most relevant clinical study for this diagnosis: 1) "Cold-pressor" test 2) the presence or absence of orthostatic hypotension 3) palpation of the lymphatic nodes 4) a simple equilibrium test (Romberg's) 5) examination of the buccal mucosa 6) examination of the palms A) (1), (2), and (3) are true B) (2), (5), and (6) are true C) (3, (4), and (6) are true D) (1), (3), 5), and (6) are true DrRognerud@www.medfever.com

INTERNAL MEDICINE

Answer: CBDBAC

299 / 343

Board Exam Question (Smiley + Star) + 2007 E) all of the above 7.940/3. Select One Of The Key Combinations Select the relevant questions which can support the diagnosis: 1) Did the patient have sarcoidosis? 2) When was thelast pregnancy, if any? 3) Did the patient have antituberculotic therapy? 4) Did the patient have a cranial injury? 5) Did the patient take hormonal contraceptives? A) (1), (3), and (5) are true B) (2), (3), and (4) are true C) (1), (2), (3), and (4) are true D) (1) and (3) are true E) all of the above 7.940/4. Select One Of The Key Combinations Considering the diagnosis, select the most important accompanying diseases or symptoms: 1) chronic thyroiditis 2) fatty degeneration of the liver 3) polyglobulinemia; an elevated red blood cell count 4) increased susceptibility to vein thrombosis 5) chronic atrophic gastritis; hypoacidity A) (1), (3), (4), and (5) are true B) (1) and (5) are true C) (2), (3), and (5) are true D) (3), (4), and (5) are true E) all of the above 7.940/5. Select One Of The Key Combinations Which of the following should be done: 1) Ask for admittance to an internal ward 2) Define the patient as unfit to work; a control examination should be done after a 6-8 week rest. The following steps depend on the state of the patient (improved -deteriorated) 3) Ask for a consultation with a gynecologist 4) Ask for admittance to a special endocrinologic ward 5) Explain to the patient that her problem is not so serious. Prescribe oxedrine (Sympathomin) drops and appetite improving therapy -cyproheptadine (Peritol). The patient should avoid sunlight, because photosensitivity indicates proneness to autoimmune diseases. A) (1) and (4) are true B) (2), (3), and (4) are true C) (1), (2), and (5) are true D) (2), (4), and (5) are true E) all of the above 7.940/6. Select One Of The Key Combinations If you have a chance to perform certain simple and quick tests, which of the following would confirm your diagnosis? 1) an X-ray of the sella turcica 2) the blood glucose level 3) the red blood cell sedimentation rate 4) a cholesterol and triglycerin test (information on the thyroid DrRognerud@www.medfever.com

INTERNAL MEDICINE

300 / 343

Board Exam Question (Smiley + Star) + 2007 function) 5) determination of the Se Na+, K-, and Cl-levels 6) an ECG A) (1), (4), and (6) are true B) (2), (3), and (5) are true C) (2) and (5) are true D) (1), (2), (3), and (4) are true E) all of the above INT-942. Case Study A 58-year-old male patient complains for 3-4 weeks of heartburn, belching, a burning epigastric pain relieved by sodium bicarbonate or milk; the pain never radiates elsewhere. The patient has no fever. He has lost his appetite and 3 kg. He has stool only every second day. The patient is normacidic. Gastric X-ray: filling excess is observed at the lesser curvature and irregular folds in the adjacent area. Laboratory findings: We: 26 mm/h; WBC: 8.4 G/L; Hb: 8.81 mmol/L; Ht: 0.40. 7.942/1. Single Choice Question Which of the following steps would you recommend: A) immediate admission to an internal ward, because the disease requires hospitalization B) immediate admission to a surgical ward, because the patient must soon undergo immediate surgery C) prescribe an immediate diet for patients with ulcer, as well as a therapy aimed at neutralization and decrease of acid production D) outpatient gastrofiberoscopy, because the clinical signs and the X-ray findings suggest the possibility of a ventricular malignant ulcer. Until the histological data are ready, prescribe a diet for patients with ulcer, as well as a therapy aimed at neutralization and decrease of acid production E) same as in (C); 4-5 weeks later repeat control X-ray study. If the ulcer is healed the therapy can be terminated. If no healing tendency is found perform a gastrofiberoscopy and/or refer the patient to a surgical ward. 7.942/2. Single Choice Question Select the adequate diet if the patient is treated conservatively: A) hunger diet: tea or milk with 1-2 cakes in the morning and evening; some unseasoned, lean soup and mashed potatoes B) the patient should avoid alcohol consumption, seasoned meals, and meat, and should eat three times a day C) the patient should avoid seasoned, fatty, smoked and other heavy meals, alcohol, and coffee. Otherwise the diet should guarantee a normal daily energy requirement, and adequate vitamin, and fat consumption. Between the three main meals recommend to the patient to have milk and dry cakes. D) avoid seasoned food and alcohol, restrict the amount of consumed food; the diet should be low in fiber and fat. Recommend to the patient to eat every hour E) let the patient keep his own diet (it is the recent trend in dietology); recommend him to avoid food which causes him DrRognerud@www.medfever.com

INTERNAL MEDICINE

Answer: DC

301 / 343

Board Exam Question (Smiley + Star) + 2007 complaints and to consume everything that does not cause him symptoms of any kind. INT-943. Case Study A 27-year-old female patient was urgently admitted due to complaints of dyspnea and pleuritic chest pain. Four days before the patient observed swelling and pressure tenderness in the right leg. Clinical signs suggest deep vein thrombosis which might have caused pulmonary embolism. 7.943/ 1. Select One Of The Key Combinations Select data in the history which support the diagnosis of deep vein thrombosis: 1) oral contraceptives 2) long-term immobilization of the lower limbs 3) injury of the lower extremity 4) hypertension A) (1), (2), and (3) are true B) (1) and (3) are true C) (2) and (4) are true D) only (4) is true E) all of the above 7.943/2. Select One Of The Key Combinations Which of the following methods help in the verification of deep vein thrombosis: 1) Doppler's ultrasonography 2) contrast venography 3) impedance plethysmography 4) nuclear scanning with 125-iodine labelled fibrinogen A) (1), (2), and (3) are true B) (1) and (3) are true C) (2) and (4) are true D) only (4) is true E) all of the above INT-945. Case Study A 47-year-old female patient presented with a complaint of arthralgia which started about 6 months ago. The joints of her fingers are red and swollen; in the morning she complains of joint rigidity in the fingers, toes, wrists and knees. The pain decreases after an hour. Physical examination: red, swollen interphalangeal proximal joints and multiple small nodes on the extensor surface of the elbow. 7.945/1. Single Choice Question The most probable diagnosis is: A) osteoarthritis B) gout C) scleroderma D) psoriatic arthritis E) rheumatoid arthritis DrRognerud@www.medfever.com

INTERNAL MEDICINE

Answer: AE

Answer: AE

302 / 343

Board Exam Question (Smiley + Star) + 2007 7.945/2. Single Choice Question All of the following parameters are pathologically altered, EXCEPT. A) bone marrow iron stores B) red blood cell sedimentation rate C) latex agglutination test D) immunoglobulin levels E) X-ray of the hands INT-946. Case Study A 60-year-old female patient complains of progressive weakness in both hindlimbs which started three days ago. Several hours before the examination the patient became incontinent. Physical examination: bilateral plantar reflex (Babinski positive), bilateral weakness in the legs and decreased sensory function up to the middle of the abdomen. 7.946/ 1. Single Choice Question Select the most important urgent study: A) myelography B) CT study of the brain C) cystoscopy D) CT scan of the whole body E) EEG 7.946/2. All of the following diagnoses are possible, EXCEPT: A) cerebral tumor B) spinal meningeoma C) multiple sclerosis D) transverse myelitis E) spinal epidural abscess

INTERNAL MEDICINE

Answer: AA

DrRognerud@www.medfever.com

303 / 343

Board Exam Question (Smiley + Star) + 2007

SURGERY

SURGERY
(157) SINGLE CHOICE QUESTIONS Select the single best response to each of the following questions!!! SUR-1. The most appropriate method for maintaining patent airways in a patient with multiple trauma resulting from a car accident due to drunken driving is: A) oropharyngeal intubation B) endotracheal intubation C) thrusting the jaw and lifting the chin D) intubation with a cuffed endotracheal tube E) tracheostomy SUR-5. Which of the following is the most common site of aortic injury in the thorax? A) at the origin of the aortic arch B) just above the origin of the innominate artery C) at the fusion ofthe thoracal and abdominal segments of the aorta D) just distal to the ligamentum arteriosum E) between the origin of the left common carotid artery and that of the left subclavian artery SUR-6. The most appropriate therapy for clean, fresh lacerations of peripheral nerves is A) debridement of the wound only B) the administration of antibiotics only C) immediate approximation of separated nerve endings D) delayed suture E) immobilization of the limb only SUR-8. Which of the following drugs could cause acute adrenal insufficiency in the postoperative period? A) vitamin C B) aspirin C) heparin D) meperidine / pethidine (Dolargan) E) cephalothin (Keflin) SUR-14. When treating fractures: A) immobilization of adjacent joints is unnecessary B) adjacent joints are occasionally immobilized DrRognerud@www.medfever.com

Answer: D

Answer: D

Answer: C

Answer: C

Answer: E

304 / 343

Board Exam Question (Smiley + Star) + 2007 C) immobilization of the joints results in the development of contractures D) only the proximal joint is immobilized E) both the joint proximal and the one distal to the fracture site should be immobilized SUR-15. Case Study: Two hours following the application of a plaster splint for a supracondylar fracture, of the radius, a patient returns to the clinic complaining of severe pain in his injured hand. The fingers are swollen and cyanotic on examination. The most appropriate solution would be to: A) monitor the patient B) administer vasodilators C) administer analgesics D) cut and loosen the splint at the fingers E) cut the splint in its whole length immediately SUR-17. Clavicular fractures of children under 10 years of age should be treated as follows: A) no therapy is necessary in most cases B) reduction and the application of a "figure-of-eight" splint dressing C) open reduction is appropriate D) the medial third of the clavicle should be removed E) open fixation by a medullary rod is necessary SUR-19. The principle of treatment for radial and ulnar fractures: A) the angulation of the ulna must be reduced B) the angulation of the radius must be reduced C) the overriding of fractured ends must be reduced D) the proper relationship of the radius and the ulna must be restored E) only the elbow should be immobilized SUR-21. The most helpful information in the diagnosis of acute osteomyelitis of 1 week duration in infants is provided by: A) a bone x-ray B) a hemoculture C) a radiologic examination of the adjacent joint D) local tenderness of the bone E) general systemic dysfunction SUR-23. A non-impacted fracture of the femoral neck causes a characteristic deformity of the lower extremity, that is: A) shortening and external rotation B) shortening only DrRognerud@www.medfever.com

SURGERY

Answer: E

Answer: A

Answer: D

Answer: D

Answer: A

305 / 343

Board Exam Question (Smiley + Star) + 2007 C) external rotation only D) flexion, adduction and internal rotation E) flexion, adduction and external rotation SUR-25. What is the usual position of the lower extremity after habitual luxation of the hip? A) the hip is in flexion and adduction B) the hip and the knee are both extended C) the hip is flexed, the knee is extended D) in the neutral position E) the hip is rotated externally SUR-26. The therapy of osteogenic sarcoma is: A) irradiation B) administration of antimetabolites C) amputation D) curettage E) excision SUR-27. Case Study: A 20-year-old football player presents with a "locked up" knee after a torsion injury to his lower extremity. The most likely diagnosis is: A) avulsion of the tibial tuberosity B) fracture of the distal portion of the femur C) meniscus injury D) disruption of the tibial collateral ligament E) disruption of the anterior cruciate ligament SUR-29. The most severe complication of an open fracture is usually: A) hemorrhage B) limb shortening C) infection D) comminuted fracture E) muscle contracture SUR-34. The primary source of metastatic spread of malignancies affecting the spine is most often: A) the kidney B) the prostate C) the breast D) the lung E) some other bone SUR-36. All of the following conditions belong to bone development disorders DrRognerud@www.medfever.com

SURGERY

Answer: A

Answer: C

Answer: C

Answer: C

Answer: C

Answer: B 306 / 343

Board Exam Question (Smiley + Star) + 2007 due to altered cartilage proliferation and calcification, EXCEPT: A) achondroplasia B) osteogenesis imperfecta C) metaphyseal aclasis D) polyostotic fibrous dysplasia E) dyschondroplasia SUR-39. All of the following features are characteristic of bone pathology in rickets, EXCEPT: A) a widened epiphyseal zone B) a wide and abnormal metaphysis C) blood vessel fragility D) cartilage proliferation E) deformed costochondral syndesmoses ('rickety rosary') SUR-43. The most frequent cause of lung a abscess is: A) the aspiration of infective material from the oral cavity or pharynx B) a blood-borne infection C) lymphatic spread from an infective focus D) a penetrating chest injury E) bronchogenic carcinoma SUR-48. A solitary mass in the lung of a middle-aged patient is most likely a: A) granuloma B) malignancy C) adenoma D) tuberculotic lesion E) secondary neoplasm SUR-49. The treatment of choice for hemothorax with an effusion of 500 cm3 or more is by: A) needle aspiration B) intercostal tube thoracostomy C) thoracotomy and ligation of the ruptured blood vessel D) supportive therapy with monitoring E) transfusion of fresh blood SUR-54. All of the following statements are valid regarding lung cancer, EXCEPT: A) lung cancer is never symptom-free B) on roentgenograms, it can occur as a coin lesion (round shadow) on the periphery of the lung C) hemoptysis is common in lung cancer D) a dry, distressing, unproductive cough is often the only symptom of lung cancer E) a partial or complete airway obstruction may predispose to DrRognerud@www.medfever.com

SURGERY

Answer: C

Answer: A

Answer: A

Answer: B

Answer: A

307 / 343

Board Exam Question (Smiley + Star) + 2007 lung infection SUR-55. All of the following are manifestations of the superior vena cava syndrome, EXCEPT: A) increased venous pressure B) edema of the head and neck C) enlarged veins which are visible on the anterior chest wall D) cyanosis E) dyspnea SUR-56. All of the following are diagnostic signs of cardiac tamponade, EXCEPT: A) increased venous pressure B) shock C) a reduced cardiac output D) increased filling volume of the heart during diastole E) a reduced blood pressure SUR-64. The 5-year survival of all lung cancer cases is not higher than: A) 10% B) 20% C) 30% D) 40% E) 50% SUR-65. The most common congenital heart abnormality is: A) a ventricular septal defect B) an atrial septal defect C) a patent ductus arteriosus D) transposition of the great vessels E) Tetralogy of Fallot SUR-66. A small volume/easily compressible radial pulse is characteristic of A) aortic valve stenosis B) coarctation of the aorta C) a patent ductus arteriosus D) stenosis of the mitral valve E) Tetralogy of Fallot SUR-67. All of the following are associated with the formation of aneurysms of the descending aorta, EXCEPT: A) Marfan's syndrome B) injury DrRognerud@www.medfever.com

SURGERY

Answer: E

Answer: D

Answer: A

Answer: A

Answer: A

Answer: B

308 / 343

Board Exam Question (Smiley + Star) + 2007 C) syphilis D) atherosclerosis E) cystic median necrosis (Erdheim's cystic median necrosis) SUR-71. A possible etiologic factor in the development of pleural mesothelioma is: A) pneumoconiosis B) asbestosis C) anthracosis D) hereditary disposition E) peritoneal mesothelioma SUR-73. Which of the following cardiac disorders causes hemoptysis: A) mitral valve stenosis B) tricuspid valve incompetency C) Tetralogy of Fallot D) aortic stenosis E) anomaly of the pulmonary veins SUR-77. What percent of the small intestine can be removed without severe consequences on digestive capacity and the subsequent risk of metabolic disorders? A) 5% B) 10% C) 30% D) 70% E) 90% SUR-79. Intestinal diverticula develop most frequently in the: A) duodenum B) jejunum C) transverse colon D) descending colon E) sigmoid colon SUR-80. All of the following signs are present in mesenteric artery obstruction, EXCEPT: A) a sudden pain around the umbilicus B) that the pain is disproportionally intense compared to the severity of physical signs C) an urge to defecate D) bloody stool E) bowel sounds reflecting permanently hyperactive peristalsis

SURGERY

Answer: B

Answer: A

Answer: D

Answer: E

Answer: E

DrRognerud@www.medfever.com

309 / 343

Board Exam Question (Smiley + Star) + 2007 SUR-82. The single most important sign of appendicitis is: A) vomiting B) an elevated body temperature C) leukocytosis D) tenderness in the right lower abdominal quadrant E) hyperesthesia in the right lower abdominal quadrant SUR-90. In Crohn's disease, the most frequent indication for surgery is: A) an intra-abdominal fistula B) an external fistula C) an intra-abdominal mass (intestinal conglomerate) D) stagnant bowel syndrome E) intestinal obstruction SUR-92. The therapy of intestinal carcinoid tumours is comprised of: A) irradiation B) chemotherapy C) administration of serotonin antagonists D) surgery followed by irradiation E) intestinal resection SUR-97. Gastric carcinoma develops most frequently in the region of the: A) fundus B) cardia B) corpus D) pyloric and antral region E) gastroesophageal junction SUR-102. The principal factor in the etiology of acute appendicitis is: A) bacterial infection B) mechanical obstruction C) local circulatory insufficiency D) the role of chemical substances E) the lymphatic tissue of the processus vermiformis SUR-105. All of the following statements are valid regarding the destruction of erythrocytes in the spleen, EXCEPT: A) a reduction of enzymatic and metabolic activity can be observed in association with the ageing of erythrocytes B) elderly erythrocytes travel through the spleen slowly C) local hypoxia and an acidic environment contribute to the degradation of erythrocytes in the spleen D) the ATP-content of erythrocytes is reduced to an extremely low level E) the ageing of erythrocytes is influenced by the overall functioning DrRognerud@www.medfever.com Answer: D

SURGERY

Answer: E

Answer: E

Answer: D

Answer: B

Answer: E

310 / 343

Board Exam Question (Smiley + Star) + 2007 of the spleen SUR-112. All of the following are present in long-standing ulcerative colitis, EXCEPT: A) a shortening of the gut B) shrinkage and thickening of the mesentery C) enlarged masses comprised of lymph node conglomerates D) a dull, greyish mucosal surface E) perforation and abscesses along the mesenteric margin SUR-115. Characteristic features of ischemic colitis are influenced by all of the following factors, EXCEPT: A) the extent of vascular obstruction B) the duration of obstruction C) the patency of the collateral circulation D) the extent of bacterial invasion E) the intensity of the inflammation SUR-117. Which of the following is the most appropriate therapeutic measure following the resection of a carcinoma of the descending colon? A) colonoscopy which should be repeated every 6 months B) a barium enema repeated every 6 months C) monitoring of the serum CEA (carcinoembryonic antigen) level every 3 months D) sulfasalazine (Salazopyrine) and prednisone therapy E) checking the stool for occult bleeding regularly SUR-118. All of the following drugs can induce gastrointestinal bleeding, EXCEPT: A) salicylates B) corticosteroids C) alcohol D) phenothiazines E) anticoagulants SUR-125. Which of the following compounds is contraindicated following a liver resection? A) carbohydrates B) albumin C) vitamin K D) hypnotic analgesics E) antibiotics SUR-127. Mechanical ileus due to obstruction by a biliary stone develops most frequently in the: DrRognerud@www.medfever.com

SURGERY

Answer: C

Answer: E

Answer: C

Answer: D

Answer: D

Answer: C

311 / 343

Board Exam Question (Smiley + Star) + 2007 A) duodenum B) jejunum C) ileum D) sigmoid colon E) anorectal junction SUR-128. All of the following are characteristic features of acute suppurative cholangitis, EXCEPT: A) jaundice B) fever and chills C) shock D) hemorrhagic diathesis E) CNS depression SUR-129. Which of the following features reflect a poor prognosis in acute pancreatitis? A) high surges of serum amylase levels B) hyperglycemia and glycosuria C) a high urinary amylase level D) a prolonged coagulation time E) a reduced serum calcium level SUR-130. All of the following factors cause acute pancreatitis, EXCEPT: A) alcohol B) hypercalcemia associated with parathyroid dysfunction C) biliary calculi D) hyperlipidemia E) hemochromatosis SUR-131. The most appropriate surgical intervention for a rectal carcinoma located 4 centimetres above the anal orifice is: A) an anterior resection B) a rectal resection with the "pull-through" procedure C) an abdominoperineal amputation of the rectum D) a posterior resection E) a two-stage resection SUR-134. All of the following statements are valid regarding thrombosed hemorrhoids,. EXCEPT: A) the abrupt appearance of a painful external bulge in the anal region B) the appearance of a tense, bluish, tender induration on the anal border C) an untreated lesion has a tendency to ulcerate and bleed D) hemorrhoids usually resolve within 24 hours DrRognerud@www.medfever.com

SURGERY

Answer: D

Answer: E

Answer: E

Answer: C

Answer: D

312 / 343

Board Exam Question (Smiley + Star) + 2007 E) prompt surgical decompression brings symptomatic relief SUR-135. Which of the following procedures is appropriate for the definitive diagnosis of congenital megacolon? A) a stool culture and parasitology B) a rectal biopsy C) a radiographic examination D) pancreatic enzyme activity measurements in stool specimens E) a sweat test SUR-136. The operation of choice for congenital megacolon is: A) a colostomy B) an enterostomy C) a splanchnicectomy D) a total colectomy E) none of the above SUR-138. The anatomical border between the anus and the rectum is the: A) lateral hemorrhoidal groove B) interhemorrhoidal groove C) dentate line D) pecten E) anorectal ring SUR-147. The most frequently occurring gynecologic lesion is: A) a cervical infection B) a vaginal infection C) endometrial cancer D) cervical cancer E) Bartholini-abscess SUR-149. All of the following statements are valid regarding an in situ'carcinoma of the cervix, EXCEPT: A) cellular changes in the squamous epithelial lining of the cervix represent the development of cancer B) this type of carcinoma is asymptomatic C) the diagnosis can be established by the histologic examination of biopsy specimens D) this carcinoma is best treated by a radical hysterectomy E) the 5-year survival rate equals 100% SUR-152. Acute mastitis is prevalent at/in: A) birth B) puberty DrRognerud@www.medfever.com

SURGERY

Answer: B

Answer: E

Answer: C

Answer: A

Answer: D

Answer: D

313 / 343

Board Exam Question (Smiley + Star) + 2007 C) pregnancy D) breast feeding E) menopause SUR-157. The most prevalent type of parathyroid cancer is a: A) follicular carcinoma B) papillary carcinoma C) anaplastic carcinoma D). medullary carcinoma E) secondary carcinoma SUR-158. All of the following belong to skeletal changes occurring in hyperparathyroidism, EXCEPT: A) osteitis fibrosa cystica B) osteopetrosis C) solitary or multilocular bone cysts D) pathologic fractures E) osteoporosis SUR-160. The treatment of choice for Cushing's disease is: A) a hypophysectomy B) irradiation of the pituitary gland C) a unilateral adrenalectomy D) a subtotal adrenalectomy E) a total adrenalectomy SUR-165. The least prevalent symptom(s) of uterine myomas is(are): A) irregular bleeding B) pain C) tenderness of the urinary bladder D) malignant transformation E) infertility and miscarriage SUR-166. All of the following are examples of APUD-cell (amine precursor uptake and decarboxylation) tumors, EXCEPT: A) medullary carcinoma of the thyroid gland B) the Zollinger-Ellison syndrome C) parathyroid adenoma D) pheochromocytoma E) Cushing's syndrome associated with lung cancer SUR-184. The most frequent symptom of a carcinoma of the renal parenchyma in adults is: A) an abdominal mass DrRognerud@www.medfever.com

SURGERY

Answer: B

Answer: B

Answer: B

Answer: D

Answer: V

Answer: B

314 / 343

Board Exam Question (Smiley + Star) + 2007 B) hematuria C) a fever of unknown origin D) anemia E) hemoptysis resulting from lung complications SUR-193. All of the following are indications for a prostatectomy in benign prostatic hyperplasia, EXCEPT: A) a weak urinary stream B) distressing pollakiuria and nocturia C) a considerable volume of infected residual urine D) an enlarged prostate E) secondary diverticula or urinary calculi SUR-203. The most valuable information for the assessment of fluid replacement in burned patients is: A) diuresis is maintained at 30-50 ml/hour B) the patient regains a normal level of consciousness C) an adequate capillary filling time (tested under the fingernails) D) physiologic arterial blood gases measurements E) the development of mild pulmonary edema SUR-207. All of the following statements are valid regarding mandibular fractures, EXCEPT: A) mandibular fractures are often complicated B) mandibular fractures cause numbness in the lip C) malocclusion of the teeth is an ordinary sign D) the masseter muscles prevent further displacement of fracture fragments E) dental fixation is appropriate in most cases SUR-208. All of the following statements are valid regarding syndactylia, EXCEPT: A) males are afflicted more often. than females B) this condition shows an autosomal dominant inheritance in many families C) syndactylia always affects the bones D) a correct surgical plan is a prerequisite to the prevention of contractures E) the operation must be performed at 1-2 years of age SUR-209. All of of the following statements are valid regarding tenosynovitis, EXCEPT: A) poor natural resistance of the region is characteristic B) starting from the wound of the thumb, the infection spreads to the radial bursa and the interinterstitial spaces of the thenar DrRognerud@www.medfever.com

SURGERY

Answer: D

Answer: A

Answer: D

Answer: C

Answer: C

315 / 343

Board Exam Question (Smiley + Star) + 2007 C) the infection of the other four fingers spreads to the ulnar bursa D) early surgical drainage is essential E) postoperative physiotherapy must be started early to preserve tendon functions SUR-214. During an examination of the scalenus anticus syndrome: A) all movements resulting in the relaxation of the anterior scalenus muscle aggravate pain B) pain can be elicited by pressing the shoulder passively down while turning the head excessively to the contralateral side C) pain can be elicited by pressing the shoulder passively down while turning the head excessively to the ipsilateral side D) passive flexion of the biceps muscle evokes severe pain E) pain is aggravated by lifting the shoulder SUR-230. The most frequent cause of an epileptic seizure in a middle-aged patient with an otherwise fair condition is: A) an idiopathic seizure B) arteriosclerosis C) a brain tumor D) Parkinson's disease E) previous cerebral trauma SUR-231. The coincidence of headache, vomiting and papilledema in the same patient is suggestive of. A) a brain concussion B) bulbar paralysis C) a brain tumor D) a migraine headache E) a subarachnoid hemorrhage SUR-241. The primary therapy for pulmonary embolism is: A) anticoagulant therapy B) ligation of the inferior vena cava C) thrombectomy D) pulmonary embolectomy SUR-251. The most frequent cause of arterial occlusion is: A) embolism B) arteriosclerosis C) varicose veins D) all of the above

SURGERY

Answer: C

Answer: C

Answer: C

Answer: A

Answer: B

DrRognerud@www.medfever.com

316 / 343

Board Exam Question (Smiley + Star) + 2007 SUR-252. The leading cause of death during the first three decades of life is: A) inadvertent poisoning B) heart disease C) drug addiction D) trauma SUR-253. The first and most important step in the care of a severely injured patient is: A) the maintenance of patent airways B) antishock therapy by parenteral volume replacement C) the immediate immobilization of fractures SUR-259. Which of the following heart diseases most often leads to hemoptysis? A) mitral regurgitation B) tricuspid regurgitation C) patent ductus arteriosus D) mitral stenosis with pulmonary infarction SUR-273. In which of the following conditions is the presence of SternbergReed cells a characteristic feature? A) leukemia B) multiple myeloma C) liposarcoma D) Hodgkin's disease E) none of the above SUR-274. Metastatic tumours rarely develop at which of the following sites? A) the long bones B) the bones distal to the knee or elbow C) the skull D) the vertebrae E) the pelvis SUR-276. In the case of clean, closed wounds, the most common source of a Staphylococcus infection is: A) the ambient air of the operating theatre B) bedside instruments C) dust D) bedclothes E) bacterial flora of the patient or personnel SUR-278. Mammography is a particularly valuable method for the reduction of DrRognerud@www.medfever.com Answer: D

SURGERY

Answer: A

Answer: D

Answer: D

Answer: B

Answer: E

Answer: B 317 / 343

Board Exam Question (Smiley + Star) + 2007 breast cancer mortality, because: A) it does not frighten female patients away from being examined B) it detects most of the cases with stage I disease C) there are no false positive results D). it is a painless procedure E) it is a particularly specific test in young females SUR-281. The most prevalent cause of mortality due to a duodenal ulcer can be: A) hemorrhage B) peritonitis resulting from acute perforation C) pyloric obstruction and inanition D) an untreatable disease E) an esophageal rupture associated with the regurgitation of the acidic gastric contents SUR-283. Most cases of acute subdeltoid bursitis are associated with which of the following? A) slight subluxation of the joint B) periarticular calcification of the tendons of the rotator muscles C) none of the above SUR-285. Mild infections of the hand can progress to potentially severe infections, as: A) the skin of the hand is a particularly heavily contaminated area of the body B) the hand often sustains injury when it is infected C) there are numerous tendons with a poor blood supply and the maintenance of any required immobilization is difficult SUR-286. The commonly occurring paronychia of the fingers is caused by: A) frequent injury to the delicate skin of the fingers B) extremely heavy contamination of the skin of the hand C) excessive use and exposure of the hand and fingers during work SUR-288. Which of the following must be given a priority in the treatment of head and neck injuries? A) any airway obstruction and hemorrhage B) periorbital injuries with the potential of visual loss C) any head trauma inducing intracranial hemorrhage D) any injuries to the cervical spine SUR-289. Flat T-waves on the ECG are suggestive of: A) hypokalemia B) hyperkalemia DrRognerud@www.medfever.com

SURGERY

Answer: A

Answer: B

Answer: C

Answer: A

Answer: A

Answer: A

318 / 343

Board Exam Question (Smiley + Star) + 2007 C) a normal serum potassium level SUR-290. Cardiac arrest due to atrial fibrillation is more likely in: A) hypokalemia B) an excessively fast intravenous administration of potassium SUR-291. In which of the following cases does incomplete conjoining of fracture fragments frequently occur? A) in comminuted fractures B) in infected fractures C) in insufficient immobilizations D) all of the above E) none of the above SUR-293. All of the following are characteristic features of full-blown traumatic (hypovolemic) shock, EXCEPT: A) oliguria B) peripheral vasoconstriction C) an increased blood viscosity D) stupor E) bradycardia SUR-294. Which of the following diseases can be most likely transmitted by a blood transfusion? A) serum hepatitis B) lymphoblastic leukemia C) myeloblastic leukemia D) erythrocytosis E) Hodgkin's lymphoma SUR-298. Which of the following is the characteristic feature of the MalloryWeiss syndrome? A) mucosal rupture in the esophagus B) rupture of the gastric mucosa C) rupture of the esophageal mucosa and the gastric mucosa in the region of the cardia D) bleeding from a gastric polyp E) portwine stains (naevus flammeus) of the stomach SUR-304. The most common indication for renal transplantation is: A) hydronephrosis B) end-stage glomerulonephritis or pyelonephritis C) renal tuberculosis D) Wilms' tumor DrRognerud@www.medfever.com

SURGERY

Answer: B

Answer: D

Answer: E

Answer: A

Answer: C

Answer: B

319 / 343

Board Exam Question (Smiley + Star) + 2007 E) "staghorn stones" SUR-306. All of the following statements are valid regarding an adamantinoma of the mandible, EXCEPT: A) this is the most prevalent solid, hard neoplasm of the mandible B) it develops at the junction of the mandibular body and the ramus C) tt grows slowly but may invade soft-tissues eventually D) the therapy of choice is a segmental resection of the mandible E) recurrence is likely if the adjacent soft-tissues have not been removed SUR-308. The most common nerve injury associated with trauma to the humerus is damage to the: A) radial nerve B) median nerve C) ulnar nerve D) axillary nerve E) musculocutaneous nerve SUR-310. The primary treatment for osteogenic sarcoma is: A) irradiation B) antimetabolites C) radical amputation D) abrasion E) immunotherapy SUR-312. The therapy of choice for patients with tension pNEUmothorax associated with dyspnea is: A) intravenous volume replacement B) administration of oxygen by mask C) immediate aspiration of the air from the pleural cavity D) administration of analeptic drugs E) intubation SUR-313. All of the following statements are valid regarding lung cancer, EXCEPT: A) lung cancer is never asymptomatic B) lung cancer may be detected on roentgenograms as a coinsized shadow on the periphery of the lung C) hemoptysis is a common sign of lung cancer D) a dry, distressing, improductive cough may be the only symptom of lung cancer E) partial or complete bronchial obstruction due to lung cancer may predispose the patient to lung infections

SURGERY

Answer: E

Answer: A

Answer: C

Answer: C

Answer: A

DrRognerud@www.medfever.com

320 / 343

Board Exam Question (Smiley + Star) + 2007 SUR-321. Acute mastitis is common in: A) neonates B) adolescents C) pregnancy D) lactation E) menopause SUR-324. The immunosuppressive drug that has improved the results of organ transplantation significantly is: A) azathioprine (Imuran) B) pyrmidine derivatives C) corticosteroids D) 0 cyclosporine A E) actinomycin D SUR-327. Which of the following facilitates collagen deposition during the process of wound healing? A) epithelial cells B) endothelial cells C) fibroblasts D) capillary network E) none of the above SUR-331. The treatment of hyperkalemia and its sequelae include all of the following, EXCEPT: A) full restriction of potassium administration B) administration of glucose with insulin C) administration of ion-exchange resins D) magnesium administration E) dialysis SUR-350. All of the following may lead to the thoracic outlet syndrome, EXCEPT: A) a cervical rib B) the scalenic anticus syndrome C) costoclavicular compression D) Raynaud's disease E) a hyperabduction syndrome SUR-359. All of the following methods are appropriate for the diagnosis of deep-vein thrombosis, EXCEPT: A) phlebography B) a radioisotope labelled fibrinogen uptake test C) Doppler -ultrasonography D) measurement of the venous flow-rate DrRognerud@www.medfever.com Answer: D

SURGERY

Answer: D

Answer: C

Answer: D

Answer: D

Answer: D

321 / 343

Board Exam Question (Smiley + Star) + 2007 E) impedance plethysmography SUR-361. The most important cause of the mortality of patients with a tracheoesophageal fistula is/are: A) pulmonary complications B) diarrhea C) vomiting D) malnutrition, feeding incapability E) other congenital abnormalities SUR-362. Which of the following is an etiologic factor of intestinal atresia? A) maternal diabetes B) maternal drug addiction C) intrauterine vascular disorders of the intestine D) birth trauma E) hydramnios SUR-363. Appendicitis developing in small children is different from that occurring in adults, because: A) the symptoms are often caused by mesenteric lymphadenitis B) perforation is a more prevalent complication C) it usually follows a parotid flare D) Meckel's diverticulum is more prevalent E) interval appendectomy' is often feasible (in the d froid phase) SUR-369. Where does Wilms' tumor originate from? A) the renal capsule B) the renal tubules C) the epithelium of the renal pelvis D) the renal cortex E) the embryonal tissues of the kidney TRUE-FALSE TYPE QUESTIONS Put T for true statements and F for false statements!!! SUR-388. In general, the treatment of fractures is less complicated in pediatric patients than in adults. SUR-389. Establishing the diagnosis of an articular "sprain" exclusively by clinical examination imposes a certain risk for the patient. Answer: T

SURGERY

Answer: A

Answer: C

Answer: B

Answer: E

Answer: T

MULTIPLE CHOICE QUESTIONS WITH KEY ANSWERS / TYPE II Every question or incomplete statement has only one answer in the DrRognerud@www.medfever.com 322 / 343

Board Exam Question (Smiley + Star) + 2007 following combinations: A) if the answers 1, 2, and 3 are true B) if the answers 1 and 3 are true C) if the answers 2 and 4 are true D) if only the answer 4 is true E) if all the four answers are true Select one of these key combinations!!! SUR-391. Which of the following parameters is useful for the recognition of mounting intracranial pressure following head trauma? 1) a reduction in the level of consciousness 2) an increasing bradycardia 3) an elevation of the blood pressure 4) cardiac arrhythmia SUR-399. The most valuable proceedures indicated for the diagnosis of an expanding intracranial hematoma include: 1) a spinal tap 2) encephalography 3) electroencephalography (EEG) 4) a CT-scan SUR-414. In polyomyelitis: 1) paralysis ensues on the 3rd or 4th day following the onset of symptoms 2) some patients do not become paralyzed 3) the regeneration of muscle power may take as long as 2 years 4) sensory loss always resolves SUR-416. Surgical treatment of congenital hip dysplasia: 1) is unnecessary under 1 year of age 2) is indicated in all forms of dysplasia 3) is mandatory in cases with soft-tissue interposition in the joint space 4) often consists of open reduction in older children SUR-421. The Pancoast tumor: 1) is a form of lung cancer 2) causes brachial neuralgia 3) causes Horner's syndrome 4) is usually bilateral SUR-422. When multiple ribs sustain double fractures simultaneously, a large region of the chest wall is destabilized, that is, "flail chest" develops. DrRognerud@www.medfever.com

SURGERY

Answer: A

Answer: D

Answer: A

Answer: B

Answer: A

Answer: C

323 / 343

Board Exam Question (Smiley + Star) + 2007 The significance of this condition is that: 1) it is difficult to diagnose 2) the affected side of the chest wall moves paradoxically with ventilation 3) it is often associated with pericardiac tamponade 4) it leads to respiratory insufficiency SUR-427. Factors contributing to an increased risk of developing lung cancer include: 1) smoking 2) exposure to radioactive substances 3) exposure to chromium compounds 4) exposure to asbestos SUR-430. Indications for coronary bypass surgery include: 1) severe angina resisting drug therapy 2) a progressively deteriorating angina 3) a previous myocardial infarction and coronary disease 4) a previous myocardial infarction and coronary disease confined to a single vessel SUR-432. Indications for the surgical treatment of aortic stenosis include: 1) a valvular pressure gradient >50 mmHg 2) stenosis associated with the angina 3) any associated cardiac defects 4) all patients with aortic stenosis SUR-435. Hormones influencing calcium metabolism include: 1) adrenal steroids 2) thyroxine 3) growth hormone 4) parathormone (parathyroid hormone) SUR-438. Which of the following is not contraindicated in a patient with a suspect tetanus wound? 1) toxoid 2) antitoxin 3) debridement of the wound 4) penicillin G SUR-443. The presence of air in the biliary tract is a diagnostic feature of: 1) a choledochoduodenal fistula 2) intestinal obstruction 3) emphysematous cholecystitis DrRognerud@www.medfever.com

SURGERY

Answer: A

Answer: A

Answer: A

Answer: E

Answer: E

Answer: B

324 / 343

Board Exam Question (Smiley + Star) + 2007 4) viral hepatitis SUR-446. Signs and symptoms of a perforated duodenal ulcer include: 1) an abrupt onset 2) severe abdominal pain 3) muscular rigidity (guarding) 4) pneumoperitoneum SUR-454. Indications for the surgical treatment of a duodenal ulcer include: 1) perforation 2) hemorrhage 3) obstruction 4) resistance to adequate and permanent medical (conservative) therapy SUR-458. The characteristic features of Zollinger-Ellison's syndrome include: 1) massive gastric hypersecretion 2) untreatable and recurring peptic ulcer(s) 3) non-beta islet cell neoplasm of the pancreas 4) diarrhea SUR-462. The characteristic features of stress ulcers include: 1) stress ulcers developing most freqently in the stomach 2) flat, shallow lesions 3) multiple lesions 4) perforation SUR-463. The clinical features of choledocholithiasis include: 1) jaundice 2) gallstone 3) fever 4) ascites SUR-468. Causes of extracellular hypovolemia developing in intestinal obstruction include: 1) fluid sequestration in the lumen of the obstructed intestinal loops 2) fluid sequestration in the intestinal wall 3) vomiting and the suction of gastric content through a nasogastric tube 4) fluid loss into the abdominal cavity as peritoneal fluid SUR-477. In carcinoma of the gall bladder: 1) most of the patients are females DrRognerud@www.medfever.com

SURGERY

Answer: E

Answer: E

Answer: E

Answer: A

Answer: A

Answer: E

Answer: B

325 / 343

Board Exam Question (Smiley + Star) + 2007 2) the presenting symptoms are characteristic and of diagnostic value 3) gallstones are detected in 90% of the cases 4) the five-year survival rate is 50% SUR-481. The clinical features of intestinal angina include: 1) postprandial pain 2) weight loss 3) steatorrhea 4) recurrent midline abdominal pain SUR-482. Case Study: A 2.5 cm node was detected in the left lobe of the thyroid gland of a female patient and a partial thyroidectomy was performed. A histological examination demonstrated a papillary carcinoma of the thyroid. The subsequent therapy includes: 1) periodic follow-up examinations 2) a total thyroidectomy 3) external irradiation 4) a modified radical cervical block-dissection SUR-493. After a partial mastectomy or focal excision, the therapy of breast cancer should include: 1) chemotherapy 2) adrenalectomy 3) oophorectomy 4) irradiation SUR-499. What are the effects of parathyroid hormone (PTH)? 1) it has a direct effect on the bones, resulting in bone resorption 2) it increases the absorption of calcium from the small intestine 3) it reduces the tubular resorption of phosphates 4) it increases the tubular resorption of calcium SUR-512. In leukoplakia of the oral cavity: 1) 5% of the lesions eventually undergo malignant transformation 2) the thin, early lesion requires surgical excision 3) patients with early hyperplasia and hyperkeratosis must abstain from smoking and the consumption of alcoholic beverages 4) irradiation is indicated in cases where the size of the lesions precludes total extirpation of the mucosa SUR-516. Second-degree burns: 1) damage the epidermis and corium DrRognerud@www.medfever.com

SURGERY

Answer: A

Answer: C

Answer: D

Answer: E

Answer: B

Answer: E

326 / 343

Board Exam Question (Smiley + Star) + 2007 2) are characterized by blister formation 3) heal by epithelial regeneration starting from the sweat glands 4) heal by scarring SUR-517. In third-degree burns: 1) the skin and subcutaneous tissues are destroyed by coagulation necrosis 2) flushing is a characteristic feature 3) early escharotomy and skin grafting precludes the development of contractures 4) associated physiological abnormalities are minimal SUR-520. Which of the following is an important diagnostic aid in the treatment of a spontaneous subarachnoid hemorrhage? 1) spinal tap 2) emergency craniotomy 3) CT-scan 4) ventriculography SUR-524. Torsion of the testis is suspected: 1) in young patients 2) when sudden pain strikes in the inguinal region 3) if the testis is enlarged and soft 4) if hematuria is present SUR-534. Valid statements about the potassium requirements of postoperative patients include: 1) the serum potassium level must be measured before supplementation 2) normal renal function is a prerequisite to parenteral potassium supplementation 3) the daily potassium requirement of a normal adult is 30-40 mEq/litre 4) the urinary potassium level must be measured before the administration of any potassium containing preparation

SURGERY

Answer: B

Answer: B

Answer: A

Answer: A

ASSOCIATION QUESTIONS Associate the following terms/statements marked by the letters A, B, C... with the corresponding statements/terms marked by and in the order given by the figures 1, 2, 3... ...for example: 1-C, 2-B, 3-A, 4-D. Put the answer as C, B, A, D! (Note: Different statements can be associated with the same terms!!!) SUR-560. Associate the following term(s) with their corresponding statement(s)! A) Atrophy of the muscles of the hand B) Wrist paralysis C) Boutonniere deformity DrRognerud@www.medfever.com Answer: ADCB

327 / 343

Board Exam Question (Smiley + Star) + 2007 D) Hammer toes 1) injury to the ulnar nerve at the level of the wrist 2) injury to the extensor tendon at the terminal phalanx 3) injury to the extensor tendon near its insertion at the middle phalanx 4) injury to the radial nerve in the upper arm SUR-561. Associate the following term(s) with their corresponding statement(s)! A) Ulcerative colitis B) Granulomatous colitis C) Both of the above D) None of the above 1) the small intestine is often involved 2) the risk of cancer is high 3) intestinal fistulas are a complication 4) a proctocolectomy is required SUR-564. Associate the following term(s) with their corresponding statement(s)! A) Peutz-Jeghers' syndrome B) Familial polyposis C) Gardner's syndrome D) Multiple polyps 1) hamartomas 2) malignant transformation in all patients 3) osteomas and fibromas 4) pigmentation of the mucosa is absent SUR-566. Associate the following term(s) with their corresponding statement(s)! A) Ulcerative colitis B) Crohn's disease C) Both of the above D) None of the above 1) bleeding is common 2) malignant transformation is possible 3) it causes intestinal stricture 4) it is caused by an infection 5) it has extraintestinal manifestations 6) it involves both the small and large intestines SUR-568. Associate the following term(s) with their corresponding statement(s)! A) Pseudomucinous cyst of the ovary B) Chocolate cyst of the ovary C) Dermoid cyst of the ovary D) Cyst of the corpus luteum 1) it is a non-neoplastic cyst 2) it can reach an extreme size DrRognerud@www.medfever.com

SURGERY

Answer: BABA

Answer: ABCD

Answer: AABDCB

Answer: DABC

328 / 343

Board Exam Question (Smiley + Star) + 2007 3) associated with pelvic endometriosis 4) a benign neoplasm with a tendency for bilateral development SUR-572. Associate the following term(s) with their corresponding statement(s)! A) Embryonal carcinoma B) Seminoma C) Interstitial cell tumor D) Chorioepithelioma. E) Teratoma 1) elevated urinary gonadotropin levels 2) precocious puberty in boys 3) the most prevalent testicular tumor in childhood 4) it may contain a cartilage component 5) the least malignant testicular neoplasm SUR-573. Associate the following term(s) with their corresponding statement(s)! A) Dislocated shoulder B) Valgus deformity of the elbow C) Laceration of the wrist D) Fracture of the humerus 1) the ulnar nerve is damaged 2) the median nerve is damaged 3) the axillary nerve is damaged 4) the radial nerve is damaged SUR-575. Associate the following term(s) with their corresponding statement(s)! A) A brain tumor in adults B) A brain tumor in children C) Both of the above D) None of the above 1) it is often located supratentorially 2) it resides in the posterior fossa (scala posterior) 3) a malignant glioma 4) it usually develops in the midline 5) a low-grade astrocytoma

SURGERY

Answer: DCAEB

Answer: BCAD

Answer: ABABB

CASE STUDIES Answer the multiple task questions (simple choice and multiple choice with/without key answers; relation analysis etc.) as they are related to each case study!!! SUR-585. Case Study A 22-year-old man is admitted to the hospital with injuries sustained in an automobile accident. A lefet-sided hemothorax is detected on the chest x-ray. 8.585/ l. The emergency treatment is: DrRognerud@www.medfever.com Answer: AA

329 / 343

Board Exam Question (Smiley + Star) + 2007 A) aspiration of the left pleural cavity (thoracocentesis) B) intercostal blockade and fixation with adhesive tape C) bilateral tube thoracostomy D) pericardiocentesis E) endotracheal intubation, mechanical ventilation and drainage of the pleural cavity 8.585/2. An x-ray taken three weeks after the injury denotes no improvement. The next therapeutic step is: A) a thoracotomy and evacuation of the hematoma B) the intrapleural administration of tetracycline C) the intrapleural injection of streptokinase D) an exploration of the mediastinum E) the maintenance of mechanical ventilation for another 2 weeks SUR-586. Case Study There are three victims following a road accident; two of them have sustained moderately severe injuries and one is severely injuried. 8.586/ l. As the first doctor on the scene, what would you do first? A) patients with the least severe injuries are treated first as they have the best chance for survival B) it is best not to'do anything until an ambulance arrives C) you should make a report of what happened D) you transport all three victims to the hospital with the help of bystanders E) provided that neither of the patients with moderate injuries is in life-threatening condition, you start treating the patient with the most severe injury immediately 8.586/2. The skin of the severly injured patient is unusually pale, his breathing is gasping and shallow, the peripheral and carotid pulses are unpalpable. The right pupil is dilated in comparison to the left one. Does this condition satisfy the criteria of clinical death? A) yes, because peripheral pulses are not palpable B) no, beacause the diameter of the pupils is different C) yes, because muscle rigidity is also present D) no, because respiration and circulation, though insufficient, are detectable E) yes, because the patient is hyperventilating 8.586/3. Which of the following combinations represent the optimal sequence of cardiopulmonary resuscitation? A) the maintenance of patent airways, the administration of 100% oxygen through a mask, external cardiac compression, drug therapy, electric defibrillation, volume replacement B) external cardiac compression, the maintenance of patent airways, the administration of oxygen, drug therapy, volume replacement C) volume replacement, external cardiac compression, drug therapy, the maintenance of patent airways DrRognerud@www.medfever.com

SURGERY

Answer: EDEBACEE

330 / 343

Board Exam Question (Smiley + Star) + 2007 D) the administration of 100% oxygen through a mask, the maintenance of patent airways, external cardiac compression, drug therapy, volume replacement E) the maintenance of patent airways, simultaneous mechanical ventilation and external cardiac compression, volume replacement through a large-bore canule, the intravenous administration of 8.4% NaHCO3, drug therapy, defibrillation depending on the ECG findings. 8.586/4. Which of the following agents is unsuitable in influencing the cardiac and circulatory functions? A) atropine B) calcium C) epinephrine D) isoproterenol E) calcium gluconate 8.586/5. Two minutes later the heart of the patient stops functioning, the pupils dilate progressively. Cardiopulmonary resuscitation must be started immediately, because this progressively deteriorating condition leads to death if left untreated. A) both the statement and the explanation are true and a causal relationship exists between them B) both the statement and the explanation are correct but there is no causal relationship between them C) the statement is true, but the explanation is false D) the statement is false, but the explanation itself is correct E) both the statement and the explanation are false 8.586/6. The detection of biologic death renders cardiopulmonary resuscitation unnecessary, because there is no chance of restarting circulatory and respiratory functions 4 minutes after the commencement of clinical death. A) both the statement and the explanation are true and a causal relationship exists between them B) both the statement and the explanation are true but there is no causal relationship between them C) the statement is true, but the explanation is false D) the statement is false, but the explanation itself is true E) both the statement and the explanation are false 8.586/7. Before starting cardiopulmonary resuscitation, it must be decided whether respiratory or cardiac damage has ensued first, because this determines further actions as well as the sequence of measures to be taken. A) both the statement and the explanation are true and a causal relationship exists between them B) both the statement and the explanation are true but there is no causal relationship between them C) the statement is true, but the explanation is false D) the statement is false, but the explanation itself is true DrRognerud@www.medfever.com

SURGERY

331 / 343

Board Exam Question (Smiley + Star) + 2007 E) both the statement and the explanation are false 8.586/8. The patient regains consciousness as a result of undelayed, effective resuscitation, because unconsciousness persisting after successful resuscitation can only be the consequence of brain damage. A) both the statement and the explanation are true and a causal relationship exists between them B) both the statement and the explanation are true but there is no causal relationship between them C) the statement is true, but the explanation is false D) the statement is false, but the explanation itself is true E) both the statement and the explanation are false SUR-588. Case Study A victim of a road accident is brought to the emergency room. On admission, the patient is exhausted, pale, sweating and lip cyanosis is visible. The abdomen is distended; percussion sounds are shortened on the left side; hepatic dullness is intact, bowel sounds are hypoactive. Heart rate: 140/min, the pulse is thready. Blood pressure: 60/0 mmHg. 8.588/1. Which of the following signs contradicts the possibility of circulatory shock? A) tachycardia B) bradycardia C) the abrupt onset of hypotension D) emtpy and collapsed veins 8.588/2. The patient in shock should be transferred to the: A) operating theatre B) patients' ward C) radiology department D) intensive care unit 8.588/3. Single Choice Question What is the essential pathophysiology of a shock syndrome? Mark the incorrect one among the following definitionsl A) an acute disparity between the circulating blood volume and the capacity of the vascular bed B) the fall of the blood pressure and the central venous pressure due to the reduction of the cardiac volume C) the discrepancy between the oxygen supply and the demand of peripheral tissues D) an acute microcirculatory failure 8.588/4. Select the emergency measures required for the hospital treatment of a patient in shock! 1) the administration of laxatives and stimulants of gastrointestinal motility 2) an analysis of the blood gases 3) a measurement of the body temperature DrRognerud@www.medfever.com

SURGERY

Answer: BDDEBDA

332 / 343

Board Exam Question (Smiley + Star) + 2007 4) the emptying of the stomach 5) monitoring of the patient's NEUrologic status 6) monitoring of the patient's blood pressure and heart rate 7) monitoring of the patient's central venous pressure 8) securing a patent vein for volume replacement 9) measurement of the patient's height and weight 10) monitoring the diuresis A) answers (1), (3), (4), (5) and (9) are correct B) answers (2), (4) and (9) are correct C) answers (3), (4) and (5) are correct D) only answer (4) is correct E) answers (2); (6), (7), (8) and (10) are correct F) all the above answers are correct 8.588/5. Which of the following findings is characteristic of circulatory shock? A) metabolic alkalosis B) metabolic acidosis C) respiratory alkalosis D) respiratory acidosis 8.588/6. Which of the following solutions is not used for the treatment of shock? A) Ringer's solution B) dextran C) human plasma D) mannitol 8.588/7. Is circulatory shock a contraindication for surgical intervention? Mark the incorrect answer! A) yes, because anesthetic agents would cause the collapse of the centralized circulation of the patient B) yes, with the exception of cases where the precipitating cause of shock can be eliminated by surgery only C) surgery is appropriate for all patients in shock if volume replacement is adequate D) all patients in shock must be operated upon if the operation is a component of antishock therapy SUR-590. Case Study You are called to attend to a middle-aged male patient found sitting in the driver seat of his pickup van (6-7 minutes before your arrival). The patient is unconscious, his skin is pale, there are no signs of respiration or circulation, the carotid pulse is unpalpable, his pupils are dilated. There are food remnants left in his mouth. Witnesses inform you that the patient has had a seizure, groaned for a while, then fell silent just before your arrival. 8.590/ 1. The tentative diagnosis is: A) the patient has epilepsy and he is in a postconvulsion coma DrRognerud@www.medfever.com Answer: DDC

SURGERY

333 / 343

Board Exam Question (Smiley + Star) + 2007 B) a cerebrovascular crisis C) a syndrome associated with recent brain damage D) clinical death E) anaphylactic shock F) the aspiration of a foreign object 8.590/2. The immediate action to be taken is: A) an EEG examination B) to start volume replacement C) to perform coniotomy D) to perform proper cardiopulmonary resuscitation 8.590/3. Determine the correct sequence of the phases of cardiopulmonary resuscitation) 1) endotracheal intubation 2) external cardiac coompression 3) rescue breathing (mouth-to-mouth) 4) the removal of foreign bodies from the pharynx 5) notifying the rescue service 6) to remove the patient from the vehicle 7) a precordial thump on the chest 8) mechanical ventilation with an appropriate device 9) to set up an intravenous line for infusion A) the correct sequence is (1), (7), and (9) B) the correct sequence is (1), (8), (2), and (5) C) the correct sequence is (6), (4), (3), (2), (5), (1), (8), (9), and D) the correct sequence is (2), (3), (4), and (5) E) the correct sequence is (7), (8), and (9) F) only step (6) should be taken SUR-592. Case Study A 67-year-old male patient is found unconscious by his neighbour, who calls the family practitioner immediately. The history of the patient includes an ulcer disease. The skin of the patient is extremely pale, cold and cyanotic. Skin turgor is poor, the tongue is dry. The capillary refill, assessed by pressing the fingernails, is slow. Auscultation reveals bronchial breath sounds. The abdomen is slightly distended, tender. Blood pressure: 6G/40 mmHg. Heart rate: 140/min. Respiratory rate: 40/min. 8.592/1. What would be your tentative diagnosis? A) pneumonia B) hyperglycemic coma C) shock syndrome D) acute heart failure E) hypoglycemic coma 8.592/2. What is the leading sign that your diagnosis can be based on? A) pale skin; tachypnea; bronchial breath sounds B) bronchial breath sounds; tachycardia DrRognerud@www.medfever.com Answer: CDEA

SURGERY

334 / 343

Board Exam Question (Smiley + Star) + 2007 C) the lack of signs indicating any external trauma D) hypotension; tachycardia; signs of poor peripheral perfusion E) poor skin turgor and a tender abdomen 8.592/3. Which of the following drugs is contraindicated for the adjunctive therapy of the diagnosed disease? A) digitalis B) calcium gluconate C) 8.4% sodium bicarbonate D). methylprednisolone E) insulin 8.592/4. Relation Analysis The insertion of an indwelling bladder catheter is recommended by all means, because the measurement of diuresis is required for the establishment of a more accurate diagnosis. A) both the statement and the explanation are true and a causal relationship exists between them B) both the statement and the explanation are true but there is no causal relationship between them C) the statement is true, but the explanation is false D) the statement is false, but the explanation itself is true E) both the statement and the explanation are false SUR-594. Case Study A 45-year-old female patient is suspected of having colon cancer. 8.594/ 1. Which of the following sets of signs and symptoms is characteristic of a neoplasm of the descending colon? A) occult bleeding; lower abdominal pain B) constipation alternating with diarrhea, blood and mucus containing stools, abdominal cramps C) fever; recurrent diarrhea accompanied by vomiting, colicky abdominal pain D) recurrent tenesmus; blood and mucus containing stools; the patient complains of incomplete emptying of the rectum even by repeated attempts 8.594/2. Which of the following procedures is hazardous in mechanical intestinal obstruction due to a suspected neoplasm of the colon? A) percussion of the abdomen B) colonoscopy C) a GI series of x-rays D) rectoscopy E) a digital rectal examination F) a barium enema SUR-599. Case Study A 36-year-old male patient was found in his home in an extremely DrRognerud@www.medfever.com

SURGERY

Answer: BC

Answer: ABDBDEBBGABB

335 / 343

Board Exam Question (Smiley + Star) + 2007 prostrated condition by his relatives. The patient had been lying on the floor conscious and complained that he felt weak. Half an hour earlier he had vomited about 3 decilitres of fresh blood. He was transported to the medical emergency room of the local hospital by an ambulance. Medical history: the patient disclosed that he had been consuming ardent spirits regularly for 20 years. He had endured an infectious disease associated with jaundice. He had been experiencing heartburn, epigastric pain regularly in the autumn and in the spring the last several years, but did not seek medical advice. Physical status: Pale, clammy skin, a slight jaundice is visible in the sclera. Blood pressure: 90 mmHg, heart-rate: 120/min; the peripheral veins are empty. The patient passed liquid stool with large volumes of fresh blood. 8.599/ 1. What is your first action in the emergency room of the hospital? A) to secure a blood sample for blood-grouping B) to administer plasma expanders C) to administer pethidine D) to perform an emergency gastroscopy E) to order a barium meal study 8.599/2. Provided that the hemorrhage is intense and shock develops, is it mandatory to wait for the official blood-grouping results reported by the local blood bank? A) yes B) no 8.599/3. How can a blood transfusion be performed without the results of the official blood-grouping? A) the attending physician determines the ABO group of the patient himself B) the attending physician determines the ABO and Rh group of the patient himself C) type 0 Rh-negative blood is transfused immediately D) two properly qualified and authorized persons determine the ABO and Rh-group of the patient independently 8.599/4. Determine the correct sequence of measures to be taken during the emergency therapy of the patient with upper gastrointestinal bleeding! 1) angipgraphy 2) gastroscopy 3) insertion of a nasogastric tube 4) antishock therapy 5) blood-grouping 6) barium meal A) the correct sequence is (1), (2), (3), (4), (5), and (6) B) the correct sequence is (5), (4), (3), and (2) C) the correct sequence is (4), (6), (3), and (1) D) the correct sequence is (6), (5), (3), and (4) 8.599/5. All of the following physical signs help the examiner to establish or DrRognerud@www.medfever.com

SURGERY

336 / 343

Board Exam Question (Smiley + Star) + 2007 verify the correct diagnosis, EXCEPT: A) low-grade jaundice B) spider angiomas C) caput medusae D) ascites E) palmar erythema 8.599/6. Which of the following is the most likely source of the patient's hemorrhage? 1) Barrett's ulcer 2) Mallory-Weiss syndrome 3) Peutz-Jeghers syndrome 4) nasal polyp 5) portal hypertension 6) duodenal ulcer 7) jejunal angiodysplasia A) answers (2) and (7) are correct B) answers (1), (3), and (4) are correct C) answers (2) and (5) are correct D) answers (1), (2), (4), and (7) are correct E) answers (5) and (6) are correct F) answers (6) and (7) are correct G) answers (1) and (7) are correct H) answers (3) and (4) are correct 8.599/7. Is the rupture of esophageal varices obligatory in all patients with liver cirrhosis? A) yes B) no 8.599/8. Considering the history of the patient, which type of endoscopy would you prefer? A) esophagoscopy B) esophago-gastroscopy C) colonoscopy D) bronchoscopy 8.599/9. Supposing that the source of the hemorrhage is identified as a ruptured subcardial varix, which of the following procedures would you consider? 1)tamponade by inserting a Sengstaken-Blakemore tube 2) an infusion of vasopressin 3) a transfusion 4) sclerotherapy through a rigid endoscope 5) laser-coagulation 6) percutaneous transhepatic embolization 7) transection of the esophagus 8) emergency bypass-surgery 9) the administration of B-blockers 10) total gastrectomy A) answers (5), (8), and (10) are correct B) answers (3), (5), (7), and (8) are correct DrRognerud@www.medfever.com

SURGERY

337 / 343

Board Exam Question (Smiley + Star) + 2007 C) answers (2), (4), (6), and (10) are correct D) answers (1) and (3) are correct E) answers (2), (3), (8), and (9) are correct F) answers (4), (7), and (10) are correct G) answers (1), (2), and (3) are correct H) answers (3), (5), and (9) are correct 8.599/10. Which therapeutic modalities could be considered if the combined conservative treatment is unsuccesful? 1) percutaneous transhepatic embolization 2) emergency bypass surgery 3) a total gastrectomy 4) sclerotherapy of the esophageal varices 5) ligation of the vena cava 6) axillofemoral bypass surgery 7) none of the above A) answers (1), (2), and (4) are correct B) answers (2), (3), (5), and (6) are correct C) answers (1), (3) and (5) are correct D) answers (3) and (4) are correct E) answers (5) and (6) are correct F) only answer (7) is correct 8.599/11. Which classification is used to categorize patients with liver cirrhosis? A) Dukes' classification B) Child's classification C) TNM-classification 8.599/12. How do the operations performed for portal hypertension influence the spontaneous clinical course of the disease? A) bleeding is controlled but disease progression is enhanced B) bleeding is controlled and disease progression is left unchanged C) the progress of the disease is retarded SUR-614. Case Study A 65-year-old male patient presents with a small white spot detected in the anterolateral region of his tongue. He is a pipe smoker since early adulthood. The lesion was painless for a month, than gradually became tender due to exulceration. 8.614/1. What is the most likely diagnosis? A) benign aspecific ulcer B) benign leukoplakia C) epulis D) lingual cancer E) lingual ranula 8.614/2. Which of the following diagnostic tests is appropriate for the verification of the tentative diagnosis? A) a positive Wassermann-test DrRognerud@www.medfever.com Answer: DCBE

SURGERY

338 / 343

Board Exam Question (Smiley + Star) + 2007 B) a positive LE phenomenon C) a biopsy of the lesion D) radiography of the mandible and the maxilla E) close observation for any progression 8.614/3. The most common sequel of this condition is: A) metastatic spread to the supraclavicular lymph nodes B) propagation to the collar lymph nodes C) metastatic spread to the internal mammary nodes . D) metastatic spread occurs in the terminal phase only E) metastatic spread never occurs 8.614/4. The treatment of this patient includes: A) local radioisotope implantation B) high-voltage radiotherapy C) radical oral and cervical dissection D) therapy is futile if propagation has reached the cervical lymph nodes E) surgical therapy and irradiation SUR-618. Case Study A 3-year-old child is investigated for constipation present since birth. The child experiences frequent attacks associated with abdominal distension. On examination: the abdomen is distended and the colon contains feces and gas in large volumes. The rectum is empty. 8.618/ 1. The most likely diagnosis is: A) enterocolitis B) massive intestinal worm infestation C) congenital megacolon D) idiopathic sprue E) duodenal atresia 8.618/2. This condition is freqently associated with: A) Meckel's diverticulum B) familial polyposis C) abdominal distension D) rectal hemorrhage E) intestinal atresia 8.618/3. The characteristic pathology of this condition includes: A) infarction of the large bowel B) multiple diverticula of the colon C) hypertrophy of the intramural neural plexuses of the colon D) absence of the neural plexuses of the colon E) hypertrophy of the longitudinal muscle layer of the colonic wall SUR-619. DrRognerud@www.medfever.com

SURGERY

Answer: CCD

Answer: 339 / 343

Board Exam Question (Smiley + Star) + 2007 Case Study A 2-year-old infant is admitted for colicky abdominal pain of abrupt onset associated with crying and frequent vomiting. The patient is asymptomatic between the painful attacks. The mother informs you that the child has passed bloody-mucous stools. 8.619/1. The most likely diagnosis is: A) intussusception/invagination B) gastroenteritis C) congenital pyloric stenosis D) gastritis E) renal stone 8.619/2. The characteristic finding detected on physical examination is: A) a sausage-like mass in the right upper abdominal quadrant B) general abdominal tenderness C) virtually no abnormality is found D) epigastric tenderness E) costovertebral angle tenderness 8.619/3. A likely finding on the plain abdominal x-ray is: A) the absence of the cecal "gas cap" in the right iliac region B) a patchy small intestine distended with gas C) nothing in particular can be noted D) a dilated stomach E) radiolucent stone in the kidney 8.619/4. Single Choice Question The most valuable supplemental diagnostic study is: A) a barium enema B) an upper GI series C) intravenous urography D) a CT-scan E) serial radiography of the small intestine SUR-621. Case Study A 27-year-old female patient is brought to the surgery department of the outpatient clinic. According to the rescue personnel, she has fallen down some slippery stairs and knocked her head to the ground. Even though the patient denies the loss of her consciousness she does not remember the preceding events accurately. She has had nausea but no vomiting. Further, none of the acute conditions associated with unconsciousness nor any other type of trauma are present in her history. Her blood pressure has been always normal. Physical status: vital organ functions are normal. Blood pressure: 130/80 mmHg, heartrate: 86/min. A 5 cm subcutaneous hematoma is visible in the middle of the patient's forehead with a 2 cm excoriation at the centre of the injury. The left nostril is covered by crusted blood; thin bloody discharge is leaking from the right nostril. Neurologic examination: reveals intact mental status, nuchal rigidity is absent. Cranial nerves are normal. Olfactory tests are not feasible. Tendon reflexes are normal, DrRognerud@www.medfever.com AAAA

SURGERY

Answer: DDADB

340 / 343

Board Exam Question (Smiley + Star) + 2007 with no other abnormal reflexes. Motor functions are intact, sensory loss is absent. Signs of cerebral lobe injury are absent. Normal psychomotor function. Coordination is intact, tests of cerebellar function reveal instability during the Romberg test. No fractures are seen on bidirectional skull films and roentgenograms of the base of the skull. Air is detected on the 'face-up' film. One day following admission periorbital suffusion (racoon's eyes) develops. 8.621 / 1. What is the most likely diagnosis? 1) cerebral concussion 2) contusion of the head and excoriations of the forehead 3) cerebral contusion 4) basal skull fracture 5) intracerebral hemorrhage 6) suturolysis A) answers (1), (4), and (5) are correct B) answers (4), (5), and (6) are correct C) answers (3), (5), and (6) are correct D) answers (1), (2), and (4) are correct E) none of the answers are correct 8.621/2. What kind of roentgenograms would you order? A) bidirectional skull film B) basal skull film C) face up film D) all of the above 8.621/3. Which of the following tests are suitable for the identification of the source of the discharge leaking from the nostril? 1) pneumoencephalography 2) the appearance of a ring developing around a droplet of the discharge dropped on filter-paper 3) measuring the glucose content of the discharge 4) the determination of the difference between radiation activity of nasal and pharyngeal swabs during isotope cysternography 5) carotid angiography 6) injection of contrast material into the cisterna magna and the observation of the route of elimination by serial radiography A) answers (2), (3), (4), and (6) are correct B) answers (1), (2), and (3) are correct C) answers (3), (4), and (5) are correct D) answers (4), (5), and (6) are correct E) none of the answers are correct 8.621/4. What is the appropriate action to be taken in the case of liquorrhea? A) discharge of the patient after proper wound care has been delivered, strict bed rest, follow-up examination 1 week later B) observation, antibiotic therapy C) observation, antibiotic therapy, repeated spinal taps until the cessation of cerebrospinal fluid leakage DrRognerud@www.medfever.com

SURGERY

341 / 343

Board Exam Question (Smiley + Star) + 2007 D) observation, antibiotic therapy, repeated spinal taps for 8-10 days; surgery if leakage is permanent 8.621/5. What are the possible risks and complications associated with this condition? 1) headaches resulting from the permanent leakage of cerebrospinal fluid 2) meningitis 3) damage to the olfactory nerve 4) brain abscess 5) intracranial hemorrhage associated with increased intracranial pressure SUR-622. Case Study A 58-year-old male patient has felt a sudden crack in his right shoulder while lifting his coat from the rack. He could not move his right arm anymore, the limb was fixed in the elevated position. Passive reposition was unsuccessful. The right shoulder assumed a sloped contour and the axis of the right humerus pointed to the axillary fossa instead of the humeroscapular joint. Peripheral pulses are readily palpable, no signs of damaged innervation to the arm or hand are detected. 8.622/1. What is the most likely diagnosis? A) fracture of the surgical neck of the humerus B) pathologic fracture of the humerus C) dislocation of the shoulder D) acromioclavicular dislocation 8.622/2. Which of the following studies is the most objective in verifying the diagnosis? A) bidirectional roentgenograms of the right humerus B) bidirectional roentgenograms of the right shoulder joint C) physical examination and inspection D) laboratory tests including the determination of the alkaline phosphatase activity 8.622/3. Which of the following is the appropriate therapy for the above diagnosis? A) plaster cast B) reposition and plaster cast C) surgical reduction, stable osteosynthesis D) surgical treatment involving the resection of the proximal end of the humerus SUR-623. Case Study A 58-year-old male patient has been kicked by a horse. The hoof of the animal has hit the patient at the level of the left costal margin. On admission: the patient is complaining about severe abdominal pain in the upper-left quadrant. The hematocrit and the WBC are normal; DrRognerud@www.medfever.com

SURGERY

Answer: CBB

Answer: CCC

342 / 343

Board Exam Question (Smiley + Star) + 2007 the circulation is stable. Continuous monitoring of circulatory parameters are uncharacteristic of internal bleeding. The Hct is stable, however, the WBC count is slightly elevated. On the third day following admission shock develops abruptly during defecation. Blood pressure falls and can not be corrected by infusion or transfusion. 8.623/1. The most likely diagnosis is: A) perforated gastric ulcer B) mechanical intestinal obstruction C) rupture of the spleen D) fractue of the 9-10th ribs associated with injury to the subcostal artery 8.623/2. Which of the following studies inappropriate in verifying the diagnosis? A) a chest x-ray B) plain abdominal radiography C) emergency surgical exploration D) laboratory tests for the monitoring of the Hct and the WBC Count 8.623/3. What is the therapy of choice? A) a massive transfusion B) a laparotomy and closure of the perforation C) a laparotomy and the control of any bleeding D) a gastric resection

SURGERY

DrRognerud@www.medfever.com

343 / 343

You might also like